You are on page 1of 119

CONSTI2_Article IV_CITIZENSHIP (119 pages)

 (1) (Co v. HRET, G.R. Nos. 92191-92, 92202-03, July 30, 1991)
 (16) (In re: Yu v. Defensor-Santiago, G.R. No. 83882, January 24, 1989)
 (18) (Frivaldo v. COMELEC, G.R. No. 87193, June 23, 1989)
 (21) (Republic v. De La Rosa, G.R. No. 104654, 105715, 105735, June 06, 1994)
 (26) (Labo, Jr. v. COMELEC, G.R. No. 86564, August 01, 1989)
 (33) (Aznar v. COMELEC, G.R. No. 83820, May 25, 1990)
 (36) (Mercado v. Manzano, G.R. No. 135083, May 26, 1999)
 (44) (Re: Vicente D. Ching, B.M. No. 914 (Resolution), October 01, 1999)
 (49) (Bengson III v. HRET, G.R. No. 142840, May 07, 2001)
 (52) (Moy Ya Lim Yao v. Commissioner of Immigration, G.R. No. L-21289, October 04, 1971)
 (91) (Tecson v. COMELEC, G.R. No. 161434, 161634, 161824, March 03, 2004)
 (103) (Republic v. Lim, G.R. No. 153883, January 13, 2004)
 (105) (Altarejos v. COMELEC, G.R. No. 163256, November 10, 2004)
 (110) (Reyes v. COMELEC, G.R. No. 207264, June 25, 2013)
 (116) (Republic v. Li Ching Chung, G.R. No. 197450, March 20, 2013)

(Co v. HRET, G.R. Nos. 92191-92, 92202-03, rule, not be reviewed by this Court . . . the power
July 30, 1991) granted to the Electoral Tribunal is full, clear and
complete and excludes the exercise of any
EN BANC authority on the part of this Court that would in
any wise restrict it or curtail it or even affect the
[G.R. Nos. 92191-92. July 30, 1991.] same."

ANTONIO Y. CO, petitioner, vs. ELECTORAL 2. ID.; ID.; JUDGMENTS THEREOF AS A RULE
TRIBUNAL OF THE HOUSE OF REPRESENTATIVES BEYOND JUDICIAL INTERFERENCE; EXCEPTION;
and JOSE ONG, JR., respondents. ARBITRARY AND IMPROVIDENT USE OF POWER
RESULTING TO DENIAL OF DUE PROCESS. — In
[G.R. Nos. 92202-03. July 30, 1991.] the case of Robles vs. HRET (181 SCRA 780
[1980]) the Supreme Court stated that the
SIXTO T. BALANQUIT, JR., petitioner, vs. judgments of the Tribunal are beyond judicial
ELECTORAL TRIBUNAL OF THE HOUSE OF interference save only "in the exercise of this
REPRESENTATIVES and JOSE ONG, JR., Court's so-called extraordinary jurisdiction, . . .
respondents. upon a determination that the Tribunal's decision
or resolution was rendered without or in excess of
Hechanova & Associates for petitioner Co. its jurisdiction, or with grave abuse of discretion
or paraphrasing Morrero, upon a clear showing of
Brillantes, Nachura, Navarro and Arcilla Law such arbitrary and improvident use by the
Offices for respondent Ong, Jr. Tribunal of its power as constitutes a denial of
due process of law, or upon a demonstration of a
SYLLABUS very clear unmitigated ERROR, manifestly
constituting such GRAVE ABUSE OF DISCRETION
1. CONSTITUTIONAL LAW; ELECTORAL TRIBUNAL that there has to be a remedy for such abuse." In
OF THE HOUSE OF REPRESENTATIVES AND HOUSE the leading case of Morrero vs. Bocar (66 Phil.
OF SENATE; SOLE JUDGES OF ALL CONTESTS 429 [1938]) the Court ruled that the power of the
RELATING TO ELECTION, RETURNS AND Electoral Commission "is beyond judicial
QUALIFICATIONS OF THEIR RESPECTIVE interference except, in any event, upon a clear
MEMBERS. — The Constitution explicitly provides showing of such arbitrary and improvident use of
that the House of Representatives Electoral power as will constitute a denial of due process."
Tribunal (HRET) and the Senate Electoral Tribunal The Court does not venture into the perilous area
(SET) shall be the sole judges of all contests of trying to correct perceived errors of
relating to the election, returns, and qualifications independent branches of the Government. It
of their respective members (See Article VI, comes in only when it has to vindicate a denial of
Section 17, Constitution). The authority conferred due process or correct an abuse of discretion so
upon the Electoral Tribunal is full, clear and grave or glaring that no less than the Constitution
complete. The use of the word sole emphasizes calls for remedial action.
the exclusivity of the jurisdiction of these
Tribunals. The Supreme Court in the case of 3. ID.; ID.; ID.; APPLIED IN CASE AT BAR. — In the
Lazatin vs. HRET (168 SCRA 391 [1988]) stated absence of a showing that the HRET has
that under the 1987 Constitution, the jurisdiction committed grave abuse of discretion amounting
of the Electoral Tribunal is original and exclusive. to lack of jurisdiction, there is no occasion for the
And that, " . . . so long as the Constitution grants Court to exercise its corrective power; it will not
the HRET the power to be the sole judge of all decide a matter which by its nature is for the
contests relating to election, returns and HRET alone to decide (See Marcos vs. Manglapus,
qualifications of members of the House of 177 SCRA 668 [1989]). It has no power to look
Representatives, any final action taken by the into what it thinks is apparent error. As
HRET on a matter within its jurisdiction shall, as a constitutional creations invested with necessary
1|CONSTI2_Article IV_CITIZENSHIP
power, the Electoral Tribunals, although not citizenship after February 2, 1987 but also to
powers in the tripartite scheme of the those who, having been born of Filipino mothers,
government, are, in the exercise of their functions elected citizenship before that date. The provision
independent organs — independent of Congress in Paragraph 3 was intended to correct an unfair
and the Supreme Court. The power granted to position which discriminates against Filipino
HRET by the Constitution is intended to be as women. To make the provision prospective from
complete and unimpaired as if it had remained February 3, 1987 is to give a narrow
originally in the legislature (Angara vs. Electoral interpretation resulting in an inequitable
Commission, 63 Phil. 139 [1936]). In passing situation. It must also be retroactive. The
upon petitions, the Court with its traditional and provision in question was enacted to correct the
careful regard for the balance of powers, must anomalous situation where one born of a Filipino
permit this exclusive privilege of the Tribunals to father and an alien mother was automatically
remain where the Sovereign authority has placed granted the status of a natural-born citizen while
it (See Veloso vs. Boards of Canvassers of Leyte one born of a Filipino mother and an alien father
and Samar, 39 Phil. 886 [1919]). would still have to elect Philippine citizenship. If
one so elected, he was not, under earlier laws,
4. ID.; SUPREME COURT; EXPANDED JURISDICTION conferred the status of a natural-born. Under the
UNDER 1987 Constitution. — The Supreme Court 1973 Constitution, those born of Filipino fathers
under the 1987 Constitution, has been given an and those born of Filipino mothers with an alien
expanded jurisdiction, so to speak, to review the father were placed in equal footing. They were
decisions of the other branches and agencies of both considered as natural-born citizens. Hence,
the government to determine whether or not they the bestowment of the status of "natural-born"
have acted within bounds of the Constitution (See cannot be made to depend on the fleeting
Article VIII, Section 1, Constitution). Yet, in the accident of time or result in two kinds of citizens
exercise thereof, the Court is to merely check made up of essentially the same similarly
whether or not the government branch or agency situated members. It is for this reason that the
has gone beyond the Constitutional limits of its amendments were enacted, that is, in order to
jurisdiction, not that it erred or has a different remedy this accidental anomaly, and, therefore,
view. treat equally all those born before the 1973
Constitution and who elected Philippine
5. ID.; CONSTITUTIONAL PROVISIONS; HOW citizenship either before or after the effectivity of
CONSTRUED; SPIRIT AND INTENDMENT MUST that Constitution.
PREVAIL. — In construing the law, the Courts are
not always to be hedged in by the literal meaning 7. ID.; ID.; SECTION 2 OF ARTICLE IV OF THE 1987
of its language. The spirit and intendment Constitution; ELECTION OF CITIZENSHIP; APPLIES
thereof, must prevail over the letter, especially ONLY TO THOSE BORN OF FILIPINO MOTHER AND
where adherence to the latter would result in ALIEN FATHER BUT NOT TO ONE WHOSE FATHER
absurdity and injustice (Casela vs. Court of HAS BEEN NATURALIZED WHEN MINOR WAS ONLY
Appeals, 35 SCRA 279 [1970]). A Constitutional NINE (9) YEARS OF AGE. — There is no dispute
provision should be construed so as to give it that respondent's mother was a natural born
effective operation and suppress the mischief at Filipina at the time of her marriage. Crucial to this
which it is aimed, hence, it is the spirit of the case is the issue of whether or not the
provision which should prevail over the letter respondent elected or chose to be a Filipino
thereof (Jarrolt vs. Mabberly, 103 U.S. 580). In the citizen. Election becomes material because
words of the Court in the case of J.M. Tuazon vs. Section 2 of Article IV of the Constitution accords
LTA (31 SCRA 413 [1970]); "To that primordial natural born status to children born of Filipino
intent, all else is subordinated. Our Constitution, mothers before January 17, 1973, if they elect
any constitution is not to be construed narrowly citizenship upon reaching the age of majority. To
or pedantically, for the prescriptions therein expect the respondent to have formally or in
contained, to paraphrase Justice Holmes, are not writing elected citizenship when he came of age
mathematical formulas having their essence in is to ask for the unnatural and unnecessary. The
their form but are organic living institutions, the reason is obvious. He was already a citizen. Not
significance of which is vital not formal . . . ." only was his mother a natural born citizen but his
father had been naturalized when the respondent
6. ID.; CITIZENSHIP; SECTION 1, PARAGRAPH 3 OF was only nine (9) years old. He could not have
ARTICLE IV OF 1987 Constitution; CONSTRUED. — divined when he came of age that in 1973 and
Article IV of the Constitution provides: "Section 1. 1987 the Constitution would be amended to
The following are citizens of the Philippines: . . . require him to have filed a sworn statement in
(3) Those born before January 17, 1973, of Filipino 1969 electing citizenship in spite of his already
mothers, who elect Philippine citizenship upon having been a citizen since 1957. In 1969,
reaching the age of majority; and . . . Section 2. election through a sworn statement would have
Natural-born Citizens are those who are citizens been an unusual and unnecessary procedure for
of the Philippines from birth without having to one who had been a citizen since he was nine
perform any act to acquire or perfect their years old.
citizenship. Those who elect Philippine citizenship
in accordance with paragraph 3 hereof shall be 8. ID.; ID.; ID.; ID.; CASE OF IN RE: FLORENCIO
deemed natural-born citizens." The Court MALLARE (59 SCRA 45 [1974]) APPLIES IN CASE
interprets Section 1, Paragraph 3 above as AT BAR. — In the case of In Re: Florencio Mallare
applying not only to those who elect Philippine (59 SCRA 45 [1974]), the Court held that the

2|CONSTI2_Article IV_CITIZENSHIP
exercise of the right of suffrage and the habitual residence. This domicile, once
participation in election exercises constitute a established is considered to continue and will not
positive act of election of Philippine citizenship. In be deemed lost until a new one is established
the exact pronouncement of the Court, we held: (Article 50, NCC; Article 40, Civil Code of Spain;
"Esteban's exercise of the right of suffrage when Zuellig vs. Republic, 83 Phil. 768 [1949]). Ong Te
he came of age, constitutes a positive act of became a permanent resident of Laoang, Samar
election of Philippine citizenship." The private around 1895. Correspondingly, a certificate of
respondent did more than merely exercise his residence was then issued to him by virtue of his
right of suffrage. He has established his life here being a resident of Laoang, Samar. The domicile
in the Philippines. For those in the peculiar that Ong Te established in 1895 continued until
situation of the respondent who cannot be April 11, 1899; it even went beyond the turn of
expected to have elected citizenship as they were the 19th century. It is also in this place where Ong
already citizens, we apply the In Re Mallare rule. Te set up his business and acquired his real
The filing of a sworn statement or formal property. Ong Te falls within the meaning of sub-
declaration is a requirement for those who still paragraph 4 of Article 17 of the Civil Code of
have to elect citizenship. For those already Spain. Although Ong Te made brief visits to China,
Filipinos when the time to elect came up, there he, nevertheless, always returned to the
are acts of deliberate choice which cannot be less Philippines. The fact that he died in China, during
binding. Entering a profession open only to one of his visits in said country, was of no
Filipinos, serving in public office where citizenship moment. This will not change the fact that he
is a qualification, voting during election time, already had his domicile fixed in the Philippines
running for public office, and other categorical and pursuant to the Civil Code of Spain, he had
acts of similar nature are themselves formal become a Spanish subject. If Ong Te became a
manifestations of choice for these persons. Spanish subject by virtue of having established
his domicile in a town under the Monarchy of
Spain, necessarily, Ong Te was also an inhabitant
of the Philippines for an inhabitant has been
9. ID.; ID.; AN ATTACK THERETO MAY ONLY BE defined as one who has actual fixed residence in
DONE THROUGH A DIRECT ACTION. — The a place; one who has a domicile in a place
petitioners argue that the respondent's father (Bouvier's Law Dictionary, Vol. II). A priori, there
was not, validly, a naturalized citizen because of can be no other logical conclusion but to educe
his premature taking of the oath of citizenship. that Ong Te qualified as a Filipino citizen under
The Court cannot go into the collateral procedure the provisions of Section 4 of the Philippine Bill of
of stripping Mr. Ong's father of his citizenship 1902.
after his death and at this very late date just so
we can go after the son. The petitioners question 12. ID.; ID.; "RESIDENCE"; MEANING THEREOF
the citizenship of the father through a collateral UNDER THE CONSTITUTION. — Under the
approach. This can not be done. In our Constitution, the term "residence" has been
jurisprudence, an attack on a person's citizenship understood as synonymous with domicile not only
may only be done through a direct action for its under the previous Constitutions but also under
nullity (See Queto vs. Catolico, 31 SCRA 52 the 1987 Constitution. The term "domicile"
[1970]). denotes a fixed permanent residence to which
when absent for business or pleasure, one
10. ID.; ID.; TO DECLARE THE GRANT THEREOF AS intends to return (Ong Huan Tin vs. Republic, 19
NULL AND VOID VIOLATIVE OF THE DUE PROCESS SCRA 966 [1967]). The absence of a person from
CLAUSE WHERE PERSON INVOLVED HAS BEEN said permanent residence, no matter how long,
LAID TO REST. — To ask the Court to declare that notwithstanding, it continues to be the domicile
grant of Philippine citizenship to Jose Ong Chuan of that person. In other words, domicile is
as null and void would run against the principle of characterized by animus revertendi (Ujano vs.
due process. Jose Ong Chuan has already been Republic, 17 SCRA 147 [1966]).
laid to rest. How can he be given a fair
opportunity to defend himself. A dead man 13. ID.; ID.; ID.; ESTABLISHMENT THEREOF;
cannot speak. To quote the words of the HRET: OWNERSHIP OF A HOUSE NOT NECESSARY. — The
"Ong Chuan's lips have long been muted to petitioners' allegation that since the private
perpetuity by his demise and obviously he could respondent owns no property in Laoang, Samar,
not rise beyond where his mortal remains now lie he cannot, therefore, be a resident of said place
to defend himself were this matter to be made a is misplaced. The properties owned by the Ong
central issue in this case." family are in the name of the private
respondent's parents. Upon the demise of his
11. ID.; ID.; ARTICLE 17 OF THE CIVIL CODE OF parents, necessarily, the private respondent,
SPAIN SUB-PARAGRAPH 4 THEREOF IN RELATION pursuant to the laws of succession, became the
TO SECTION 4 OF THE PHILIPPINE BILL OF 1902, co-owner thereof (as a co-heir), notwithstanding
APPLIED IN CASE AT BAR. — Article 17 of the Civil the fact that these were still in the names of his
Code of Spain enumerates those who were parents. Even assuming that the private
considered Spanish Subjects, viz: "ARTICLE 17. respondent does not own any property in Samar,
The following are Spaniards: . . . (4). Those the Supreme Court in the case of De los Reyes vs.
without such papers, who may have acquired Solidum (61 Phil. 893 [1935]) held that it is not
domicile in any town in the Monarchy." The required that a person should have a house in
domicile of a natural person is the place of his order to establish his residence and domicile. It is

3|CONSTI2_Article IV_CITIZENSHIP
enough that he should live in the municipality or transcript of the proceedings of the body upon
in a rented house or in that of a friend or relative. which the resolution of the 1971 Constitutional
Convention was predicated. On the contrary, the
14. ID.; ID.; ID.; TEMPORARY ABSENCE DOES NOT documents presented by the private respondent
NECESSARILY CONNOTE CHANGE THEREOF; fall under the exceptions to the best evidence
"ANIMUS REVERTENDI" ESTABLISHED IN CASE AT rule. It was established in the proceedings before
BAR. — It has also been settled that absence from the HRET that the originals of the Committee
residence to pursue studies or practice a Report No. 12, the minutes of the plenary session
profession or registration as a voter other than in of the 1971 Constitutional Convention held on
the place where one is elected, does not November 28, 1972 cannot be found. This was
constitute loss of residence (Faypon vs. Quirino, affirmed by Atty. Ricafrente, Assistant Secretary
96 Phil. 294 [1954]). The private respondent of the 1971 Constitutional Convention; by Atty.
stayed in Manila for the purpose of finishing his Nolledo, Delegate to the 1971 Constitutional
studies and later to practice his profession. There Convention; and by Atty. Antonio Santos, Chief
was no intention to abandon the residence in Librarian of the U.P. Law Center, in their
Laoang, Samar. On the contrary, the periodical respective testimonies given before the HRET to
journeys made to his home province reveal that the effect that there is no governmental agency
he always had the animus revertendi. which is the official custodian of the records of
the 1971 Constitutional Convention. The
15. ID.; ID.; PROSPECTIVE JUDICIAL execution of the originals was established by Atty.
RECOMMENDATION; MORE HUMANE AND LESS Ricafrente, who as the Assistant Secretary of the
TECHNICAL APPROACH TO CITIZENSHIP 1971 Constitutional Convention was the proper
PROBLEMS. — Our citizens no doubt constitute party to testify to such execution. The inability to
the country's greatest wealth. Citizenship is a produce the originals before the HRET was also
special privilege which one must forever cherish. testified to as aforestated by Atty. Ricafrente,
However, in order to truly revere this treasure of Atty. Nolledo, and Atty. Santos. In proving the
citizenship, we do not, on the basis of too harsh inability to produce, the law does not require the
an interpretation, have to unreasonably deny it to degree of proof to be of sufficient certainty; it is
those who qualify to share in its richness. Under enough that it be shown that after a bona fide
the overly strict jurisprudence surrounding our diligent search, the same cannot be found (see
antiquated naturalization laws only the very Government of P.I. vs. Martinez, 44 Phil. 817
affluent backed by influential patrons, who were [1918]). Since the execution of the document and
willing to suffer the indignities of a lengthy, the inability to produce were adequately
sometimes humiliating, and often corrupt process established, the contents of the questioned
of clearances by minor bureaucrats and whose documents can be proven by a copy thereof or by
lawyers knew how to overcome so many the recollection of witnesses.
technical traps of the judicial process were able
to acquire citizenship. It is time for the PADILLA, J., dissenting:
naturalization law to be revised to enable a more 1. CONSTITUTIONAL LAW; SUPREME COURT;
positive, affirmative, and meaningful examination JURISDICTION THEREOF; EXPANDED UNDER THE
of an applicant's suitability to be a Filipino. A 1987 Constitution; DECISION OF HOUSE
more humane, more indubitable and less ELECTORAL TRIBUNAL SUBJECT TO JUDICIAL
technical approach to citizenship problems is REVIEW. — I believe that, contrary to the
essential. respondents' contentions, the Court has the
jurisdiction and competence to review the
16. ID.; HOUSE OF REPRESENTATIVE; questioned decision of the House Electoral
CANDIDATES; PROPERTY OWNERSHIP; NOT A Tribunal and to decide the present controversy.
QUALIFICATION. — To require the private Article VIII, Section 1 of the 1987 Constitution
respondent to own property in order to be eligible provides that: "Judicial power includes the duty of
to run for Congress would be tantamount to a the courts of justice to settle actual controversies
property qualification. The Constitution only involving rights which are legally demandable
requires that the candidate meet the age, and enforceable, and to determine whether or not
citizenship, voting and residence requirements. there has been a grave abuse of discretion
Nowhere is it required by the Constitution that amounting to lack or excess of jurisdiction on the
the candidate should also own property in order part of any branch or instrumentality of the
to be qualified to run (see Maquera vs. Borra, 122 government." The Constitution, it is true,
Phil. 412 [1965]). constitutes the tribunal as the sole judge of all
contests relating to the election, returns, and
17. REMEDIAL LAW; BEST EVIDENCE RULE; qualifications of Members of the House of
EXCEPTION; ORIGINAL HAS BEEN LOST; Representatives. But as early as 1938, it was held
REQUIREMENTS THEREOF TO BE ADMISSIBLE; in Morrero vs. Bocar (66 Phil. 429), construing
PROPERLY LAID IN CASE AT BAR. — The Section 4, Article VI of the 1935 Constitution
petitioners' sole ground in disputing that which provided that " . . . The Electoral
respondent was a natural-born Filipino is that the Commission shall be the sole judge of all contests
documents presented to prove it were not in relating to the election, returns and qualifications
compliance with the best evidence rule. The of the Members of the National Assembly." that:
petitioners allege that the private respondent "The judgment rendered by the (electoral)
failed to present the original of the documentary commission in the exercise of such and
evidence, testimonial evidence and of the acknowledged power is beyond judicial

4|CONSTI2_Article IV_CITIZENSHIP
interference, except, in any event, 'upon a clear that stems ultimately from some legitimate area
showing of such arbitrary and improvident use of of governmental power (the Supreme Court in
the power as will constitute a denial of due Modern Role, C.B. Sevisher, 1958, p. 36)."
process of law' (Barry vs. US ex rel. Cunningham, Moreover, it is decidedly a matter of great public
279 US 597; 73 Law. ed., 867; Angara vs. interest and concern to determine whether or not
Electoral Commission, 35 Off. Gaz., 23)." And private respondent is qualified to hold so
then under the afore-quoted provisions of Article important and high a public office which is
VIII, Section 1 of the 1987 Constitution, this Court specifically reserved by the Constitution only to
is duty-bound to determine whether or not, in an natural-born Filipino citizens.
actual controversy, there has been a grave abuse
of discretion amounting to lack or excess of 4. ID.; CITIZENSHIP; NATURAL-BORN; REQUISITE;
jurisdiction on the part of any branch or NOT COMPLIED WITH IN CASE AT BAR. — The
instrumentality of the government. records show that private respondent was born
on 19 June 1948 to the spouses Jose Ong Chuan,
a Chinese citizen, and Agrifina E. Lao, a natural-
born Filipino citizen, in Laoang, Northern Samar.
2. ID.; ID.; ID.; ID.; APPLIED IN CASE AT BAR. — In other words, at birth, private respondent was a
The present controversy, involves more than Chinese citizen (not a natural-born Filipino citizen)
perceived irregularities in the conduct of a because his father was then a Chinese citizen
congressional election or a disputed appreciation (not a naturalized Filipino citizen). Under the 1935
of ballots, in which cases, it may be contended Constitution which was enforced at the time of
with great legal force and persuasion that the private respondent's birth on 19 June 1948, only
decision of the electoral tribunal should be final those whose fathers were citizens of the
and conclusive, for it is, by constitutional Philippines were considered Filipino citizens.
directive, made the sole judge of contests Those whose mothers were citizens of the
relating to such matters. The present Philippines had to elect Philippine citizenship
controversy, however, involves no less than a upon reaching the age of majority, in order to be
determination of whether the qualifications for considered Filipino citizens. Following the basic
membership in the House of Representatives, as definition in the 1987 Constitution of a natural-
prescribed by the Constitution, have been met. born citizen, in relation to the 1935 Constitution,
Indeed, this Court would be unforgivably remiss private respondent is not a natural-born Filipino
in the performance of its duties, as mandated by citizen, having been born a Chinese citizen by
the Constitution, were it to allow a person, not a virtue of the Chinese citizenship of his father at
natural-born Filipino citizen, to continue to sit as a the time of his birth, although from birth, private
Member of the House of Representatives, solely respondent had the right to elect Philippine
because the House Electoral Tribunal has citizenship, the citizenship of his mother, but only
declared him to be so. In such a case, the tribunal upon his reaching the age of majority.
would have acted with grave abuse of discretion
amounting to lack or excess of jurisdiction as to 5. ID.; ID.; ID.; SECTION 15 OF THE REVISED
require the exercise by this Court of its power of NATURALIZATION LAW (C.A. 473); DID NOT
judicial review. Besides, the citizenship and CONFER STATUS OF NATURAL-BORN IN CASE AT
residence qualifications of private respondent for BAR. — While under Section 15 of the Revised
the office of Member of the House of Naturalization Law (C.A. 473) minor children of a
Representatives, are here controverted by naturalized citizen (father), who were born in the
petitioners who, at the same time, claim that Philippines prior to the naturalization of the
they are entitled to the office illegally held by parent automatically become Filipino citizens, this
private respondent. From this additional direction, does not alter the fact that private respondent
where one asserts and earnestly perceived right was not born to a Filipino father, and the
that in turn is vigorously resisted by another, operation of Section 15 of CA 473 did not confer
there is clearly a justiciable controversy proper upon him the status of a natural-born citizen
for this Court to consider and decide. merely because he did not have to perform any
act to acquire or perfect his status as a Filipino
3. ID.; ID.; ID.; EXERCISE OF JUDICIAL REVIEW citizen.
NOT VIOLATIVE OF THE PRINCIPLE OF
SEPARATION OF POWERS. — The Court, in 6. ID.; ID.; NATURALIZATION; NATURE THEREOF;
reviewing the decision of the tribunal, does not PRIVILEGE NOT A RIGHT. — "Naturalization is not
assert supremacy over it in contravention of the a right, but a privilege of the most discriminating
time-honored principle of constitutional as well as delicate and exacting nature, affecting
separation of powers. The Court in this instance public interest of the highest order, and which
simply performs a function entrusted and may be enjoyed only under the precise conditions
assigned to it by the Constitution of interpreting, prescribed by law therefor."
in a justiciable controversy, the pertinent
provisions of the Constitution with finality. "It is 7. ID.; ID.; ID.; PETITION; GRANT THEREOF;
the role of the Judiciary to refine and, when APPEALABLE; OATH TAKEN BEFORE EXPIRATION
necessary, correct constitutional (and/or OF THE PERIOD OF APPEAL; IMPROPER. — It is
statutory) interpretation, in the context of the settled that an order granting a petition to take
interactions of the three branches of the the requisite oath of allegiance of one who has
government, almost always in situations where previously obtained a decision favorable to his
some agency of the State has engaged in action application for naturalization, is appealable. It is,

5|CONSTI2_Article IV_CITIZENSHIP
therefore, improper and illegal to authorize the for the doctrine of implied election of Philippine
taking of said oath upon the issuance of said citizenship, is not applicable to the case at bar.
order and before the expiration of the The respondent tribunal failed to consider that
reglementary period to perfect any appeal from Esteban Mallare reached the age of majority in
said order. In Cua Sun Ke vs. Republic (159 SCRA 1924, or seventeen (17) years before CA 625 was
477), this Court held that: "Administration of the approved and, more importantly, eleven (11)
oath of allegiance on the same day as issuance of years before the 1935 Constitution (which
order granting citizenship is irregular and makes granted the right of election) took effect.
the proceedings so taken null and void (Republic
vs. Guy, 115 SCRA 244 [1982]; citing the case of 11. ID.; ID.; ID.; REQUISITE PROVIDED FOR
Ong So vs. Republic of the Philippines, 121 Phil. UNDER COMMONWEALTH ACT NO. 625 NOT
1381)." COMPLIED WITH IN CASE AT BAR. — The
respondent tribunal erred in ruling that by
8. ID.; ID.; NATURAL-BORN; DEFINED AND operation of CA 473, the Revised Naturalization
INTERPRETED UNDER THE 1987 Constitution. — Law, providing for private respondent's
Article IV, Section 2 of the 1987 Constitution acquisition of Filipino citizenship by reason of the
defines natural-born (Filipino) citizens as: naturalization of his father, the law itself had
"Natural-born citizens are those who are citizens already elected Philippine citizenship for him. For,
of the Philippines from birth without having to assuming arguendo that the naturalization of
perform any act to acquire or perfect their private respondent's father was valid, and that
Philippine citizenship. Those who elect Philippine there was no further need for private respondent
citizenship in accordance with paragraph (3), to elect Philippine citizenship (as he had
Section 1 hereof shall be deemed natural-born automatically become a Filipino citizen) yet, this
citizens." Article IV, Section 1, paragraph (3) of did not mean that the operation of the Revised
the 1987 Constitution provides that: "Section 1. Naturalization Law amounted to an election by
The following are citizens of the Philippines: . . . him of Philippine citizenship as contemplated by
(3) Those born before January 17, 1973, of Filipino the Constitution. Besides, election of Philippine
mothers, who elect Philippine citizenship upon citizenship derived from one's Filipino mother, is
reaching the age of majority." It would appear made upon reaching the age of majority, not
then that the intent of the framers of the 1987 during one's minority. There is no doubt in my
Constitution in defining a natural-born Filipino mind, therefore, that private respondent did not
citizen was to equalize the position of Filipino elect Philippine citizenship upon reaching the age
fathers and Filipino mothers as to their children of majority in 1969 or within a reasonable time
becoming natural-born Filipino citizens. In other thereafter as required by CA 625. Consequently,
words, after 17 January 1973, effectivity date of he cannot be deemed a natural-born Filipino
the 1973 Constitution, all those born of Filipino citizen under Sections 2 and 1 (3), Article IV of
fathers (with alien spouse) or Filipino mothers the 1987 Constitution.
(with alien spouse) are natural-born Filipino
citizens. But those born to Filipino mothers prior
to 17 January 1973 must still elect Philippine
citizenship upon reaching the age of majority, in 12. ID.; ELECTION PROTEST; QUESTIONING
order to be deemed natural-born Filipino citizens. ELIGIBILITY OF A CANDIDATE-ELECT; IN EFFECT A
The election, which is related to the attainment of QUO WARRANTO PROCEEDING; INELIGIBILITY OF
the age of majority, may be made before or after CANDIDATE-ELECT RESULTS IN NO-CHOICE. —
17 January 1973. This interpretation appears to Neither of the petitioners may take the place of
be in consonance with the fundamental purpose private respondent in the House of
of the Constitution which is to protect and Representatives representing the second district
enhance the people's individual interests, and to of Northern Samar. The ruling of this Court in
foster equality among them. Ramon L. Labo, Jr. vs. The Commission on
Elections (COMELEC) EN BANC and Luis L.
9. ID.; ID.; ELECTION THEREOF; MUST BE MADE Lardizabal (176 SCRA 1), is controlling. There we
EXPRESSLY AS PROVIDED FOR UNDER held that Luis L. Lardizabal, who filed the quo
COMMONWEALTH ACT NO. 625. — It is settled warranto petition, could not replace Ramon L.
doctrine in this jurisdiction that election of Labo, Jr. as mayor of Baguio City for the simple
Philippine citizenship must be made in reason that as he obtained only the second
accordance with Commonwealth Act 625, highest number of votes in the election, he was
Sections 1 and 2 of the Act mandate that the obviously not the choice of the people of Baguio
option to elect Philippine citizenship must be City for mayor of that City. A petition alleging that
effected expressly, not impliedly. the candidate-elect is not qualified for the office
is, in effect, a quo warranto proceeding even if it
10. ID.; ID.; ID.; CASE OF IN RE: FLORENCIO is labelled an election protest. It is a proceeding
MALLARE (ADMINISTRATIVE CASE NO. 533, to unseat the ineligible person from office but not
SEPTEMBER 12, 1974, [59 SCRA 45]) NOT necessarily to install the protestant in his place.
APPLICABLE IN CASE AT BAR. — The respondent The general rule is that the fact that a plurality or
tribunal cites In re: Florencio Mallare which held a majority of the votes are cast for an ineligible
that Esteban Mallare's exercise of the right of candidate in an election does not entitle the
suffrage when he came of age, constituted a candidate receiving the next highest number of
positive act of election of Philippine citizenship. votes to be declared elected. In such a case, the
Mallare, cited by respondent tribunal as authority

6|CONSTI2_Article IV_CITIZENSHIP
electors have failed to make a choice and the position of Representative (Congressman) to the
election is a nullity. House of Representatives for the second district
of Northern Samar, would have to cease in office
13. ID.; ID.; PHILIPPINE BILL OF 1902; by virtue of this Court's decision, if the full
REQUIREMENTS PROVIDED THEREIN; NOT membership of the Court had participated in this
COMPLIED WITH IN CASE AT BAR. — The "test," case, with the result that the legislative district
following the premises of the 1971 Constitutional would cease to have, in the interim, a
Convention, is whether or not Ong Te, private representative in the House of Representatives.
respondent's and Emil L. Ong's grandfather was But the fundamental consideration in case of this
"an inhabitant of the Philippines who continued to nature is the Constitution and only the
reside therein and was a Spanish subject on April Constitution. It has to be assumed, therefore, that
11, 1899." If he met these requirements of the when the electorate in the second legislative
Philippine Bill of 1902, then, Ong Te was a Filipino district of Northern Samar cast the majority of
citizen; otherwise, he was not a Filipino citizen. their votes for private respondent, they seemed
Petitioners (protestants) submitted and offered in and believed that he was fully eligible and
evidence before the House Electoral Tribunal qualified for the office because he is a natural-
exhibits W, X, Y, Z, AA, BB, CC, DD and EE which born Filipino citizen. That erroneous assumption
are copies of entries in the "Registro de Chinos" and belief can not prevail over, but must yield to
from years 1896 to 1897 which show that Ong Te the majesty of the Constitution.
was not listed as an inhabitant of Samar where he
is claimed to have been a resident. Petitioners SARMIENTO, J., concurring:
(protestants) also submitted and offered in 1. CONSTITUTIONAL LAW; ELECTORAL TRIBUNAL
evidence before the House Electoral Tribunal OF THE HOUSE OF REPRESENTATIVES; AS SOLE
Exhibit V, a certification of the Chief of the JUDGE OF ALL CONTEST RELATING TO MEMBERS
Archives Division, Records and Management and THEREOF; ISSUE OF CITIZENSHIP INCLUDED;
Archives Office, stating that the name of Ong Te BEYOND JUDICIAL INTERVENTION. — The question
does not appear in the "Registro de Chinos" for of citizenship is a question of fact, and as a rule,
the province of Samar for 1895. These exhibits the Supreme Court leaves facts to the tribunal
prove or at least, as petitioners validly argue, that determined them. I am quite agreed that the
tend to prove that Ong Te was NOT a resident of Electoral Tribunal of the House of
Samar close to 11 April 1899 and, therefore, Representatives, as the "sole judge" of all
could not continue residing in Samar, Philippines contests relating to the membership in the
after 11 April 1899, contrary to private House, as follows: "Sec. 17. The Senate and the
respondent's pretense. In the face of these proofs House of Representatives shall each have an
or evidence, private respondent FAILED TO Electoral Tribunal which shall be the sole judge of
PRESENT ANY REBUTTAL OR COUNTERVAILING all contests relating to the election, returns, and
EVIDENCE. qualifications of their respective Members. Each
Electoral Tribunal shall be composed of nine
14. ID.; ID.; RES JUDICATA; NOT APPLICABLE. — Members, three of whom shall be Justices of the
The decision of the 1971 Constitutional Supreme Court to be designated by the Chief
Convention in the case of Emil L. Ong was a Justice, and the remaining six shall be Members
decision of a political body, not a court of law. of the Senate or the House of Representatives, as
And, even if we have to take such a decision as a the case may be, who shall be chosen on the
decision of a quasi-judicial body (i.e., a political basis of proportional representation from the
body exercising quasi-judicial functions), said political parties and the parties or organizations
decision in the Emil L. Ong case can not have the registered under the party-list system
category or character of res judicata in the represented therein. The senior Justice in the
present judicial controversy, because between Electoral Tribunal shall be its Chairman." is the
the two (2) cases, there is no identity of parties best judge of facts and this Court can not
(one involves Emil L. Ong, while the other substitute its judgment because it thinks it knows
involves private respondent) and, more better.
importantly, there is no, identity of causes of
action because the first involves the 1935 2. ID.; SUPREME COURT; EXPANDED JURISDICTION
Constitution while the second involves the 1987 THEREOF; REVIEW OF FACTS NOT INCLUDED. — In
Constitution. As held in Lee vs. Commissioners on the case of Aratuc vs. Commission on Elections
Immigration (G.R. No. L-23446, 20 December (88 SCRA 251), it was held that this Court can not
1971, 42 SCRA 561): " . . . Everytime the review the errors of the Commission on Elections
citizenship of a person is material or (then the "sole judge" of all election contests) —
indispensable in a judicial or administrative case, in the sense of reviewing facts and unearthing
whatever the corresponding court or mistakes — and that this Court's jurisdiction is to
administrative authority decides therein as to see simply whether or not it is guilty of a grave
such citizenship is generally not considered as res abuse of discretion. It is true that the new
judicata, hence it has to be threshed out again Constitution has conferred expanded powers on
and again as the occasion may demand." the Court, but as the Charter states, our authority
is "to determine whether or not there has been a
15. ID; SUPREMACY OF THE CONSTITUTION; MUST grave abuse of discretion amounting to lack or
BE ENFORCED. — It is regrettable that one (as excess of jurisdiction on the part of any branch or
private respondent) who unquestionably obtained instrumentality of the government." It is not to
the highest number of votes for the elective review facts.

7|CONSTI2_Article IV_CITIZENSHIP
respective members. (See Article VI, Section 17,
3. ID.; ID.; ID.; "GRAVE ABUSE OF DISCRETION" Constitution). prLL
DEFINED. — "Grave abuse of discretion" has been
defined as whimsical exercise of power The authority conferred upon the Electoral
amounting to excess of jurisdiction, or otherwise, Tribunal is full, clear and complete. The use of the
to denial of due process of law. word sole emphasizes the exclusivity of the
jurisdiction of these Tribunals.
DECISION
The Supreme Court in the case of Lazatin v. HRET
GUTIERREZ, JR., J p: (168 SCRA 391 [1988]) stated that under the
1987 Constitution, the jurisdiction of the Electoral
The petitioners come to this Court asking for the Tribunal is original and exclusive, viz:
setting aside and reversal of a decision of the
House of Representatives Electoral Tribunal "The use of the word 'sole' emphasizes the
(HRET). exclusive character of the jurisdiction conferred
(Angara v. Electoral Commission, supra at p.
The HRET declared that respondent Jose Ong, Jr. 162). The exercise of power by the Electoral
is a natural born Filipino citizen and a resident of Commission under the 1935 Constitution has
Laoang, Northern Samar for voting purposes. The been described as 'intended to be as complete
sole issue before us is whether or not, in making and unimpaired as if it had originally remained in
that determination, the HRET acted with grave the legislature.' (id., at p. 175) Earlier this grant
abuse of discretion. of power to the legislature was characterized by
Justice Malcolm as 'full, clear and complete'.
On May 11, 1987, the congressional election for (Veloso v. Board of Canvassers of Leyte and
the second district of Northern Samar was held. Samar, 39 Phil. 886 [1919]) Under the amended
1935 Constitution, the power was unqualifiedly
Among the candidates who vied for the position reposed upon the Electoral Tribunal and it
of representative in the second legislative district remained as full, clear and complete as that
of Northern Samar are the petitioners, Sixto previously granted the Legislature and the
Balinquit and Antonio Co and the private Electoral Commission, (Lachica v. Yap, 25 SCRA
respondent, Jose Ong, Jr. 140 [1968] The same may be said with regard to
the jurisdiction of the Electoral Tribunal under the
Respondent Ong was proclaimed the duly elected 1987 Constitution." (p. 401).
representative of the second district of Northern
Samar.

The petitioners filed election protests against the The Court continued further, ". . . so long as the
private respondent premised on the following Constitution grants the HRET the power to be the
grounds: sole judge of all contests relating to election,
returns and qualifications of members of the
1) Jose Ong, Jr. is not a natural born citizen of the House of Representatives, any final action taken
Philippines; and by the HRET on a matter within its jurisdiction
shall, as a rule, not be reviewed by this Court . . .
2) Jose Ong, Jr. is not a resident of the second the power granted to the Electoral Tribunal is full,
district of Northern Samar. clear and complete and excludes the exercise of
any authority on the part of this Court that would
The HRET, in its decision dated November 6, in any wise restrict it or curtail it or even affect
1989, found for the private respondent. the same." (pp. 403-404)

A motion for reconsideration was filed by the When may the Court inquire into acts of the
petitioners on November 12, 1989. This was, Electoral Tribunals under our constitutional grants
however, denied by the HRET, in its resolution of power?
dated February 22, 1989.
In the later case of Robles v. HRET (181 SCRA 780
Hence, these petitions for certiorari. [1990]) the Supreme Court stated that the
judgments of the Tribunal are beyond judicial
We treat the comments as answers and decide interference save only "in the exercise of this
the issues raised in the petitions. Court's so-called extraordinary jurisdiction, . . .
upon a determination that the Tribunal's decision
ON THE ISSUE OF JURISDICTION or resolution was rendered without or in excess of
The first question which arises refers to our its jurisdiction, or with grave abuse of discretion
jurisdiction. or paraphrasing Morrero, upon a clear showing of
such arbitrary and improvident use by the
The Constitution explicitly provides that the Tribunal of its power as constitutes a denial of
House of Representatives Electoral Tribunal due process of law, or upon a demonstration of a
(HRET) and the Senate Electoral Tribunal (SET) very clear unmitigated ERROR, manifestly
shall be the sole judges of all contests relating to constituting such GRAVE ABUSE OF DISCRETION
the election, returns, and qualifications of their that there has to be a remedy for such abuse."
(at pp. 785-786)

8|CONSTI2_Article IV_CITIZENSHIP
In the leading case of Morrero v. Bocar (66 Phil. In the case at bar, the Court finds no improvident
429 [1938]) the Court ruled that the power of the use of power, no denial of due process on the
Electoral Commission "is beyond judicial part of the HRET which will necessitate the
interference except, in any event, upon a clear exercise of the power of judicial review by the
showing of such arbitrary and improvident use of Supreme Court.
power as will constitute a denial of due process."
The Court does not venture into the perilous area ON THE ISSUE OF CITIZENSHIP
of trying to correct perceived errors of The records show that in the year 1895, the
independent branches of the Government. It private respondent's grandfather, Ong Te, arrived
comes in only when it has to vindicate a denial of in the Philippines from China. Ong Te established
due process or correct an abuse of discretion so his residence in the municipality of Laoang,
grave or glaring that no less than the Constitution Samar on land which he bought from the fruits of
calls for remedial action. LLjur hard work.

The Supreme Court under the 1987 Constitution, As a resident of Laoang, Ong Te was able to
has been given an expanded jurisdiction, so to obtain a certificate of residence from the then
speak, to review the decisions of the other Spanish colonial administration.
branches and agencies of the government to
determine whether or not they have acted within The father of the private respondent, Jose Ong
the bounds of the Constitution. (See Article VIII, Chuan was born in China in 1905. He was brought
Section 1, Constitution) by Ong Te to Samar in the year 1915.

Yet, in the exercise thereof, the Court is to merely Jose Ong Chuan spent his childhood in the
check whether or not the governmental branch or province of Samar. In Laoang, he was able to
agency has gone beyond the Constitutional limits establish an enduring relationship with his
of its jurisdiction, not that it erred or has a neighbors, resulting in his easy assimilation into
different view. In the absence of a showing that the community.
the HRET has committed grave abuse of
discretion amounting to lack of jurisdiction, there As Jose Ong Chuan grew older in the rural and
is no occasion for the Court to exercise its seaside community of Laoang, he absorbed
corrective power; it will not decide a matter which Filipino cultural values and practices. He was
by its nature is for the HRET alone to decide. (See baptized into Christianity. As the years passed,
Marcos v. Manglapus, 177 SCRA 668 [1989]) It Jose Ong Chuan met a natural born-Filipina,
has no power to look into what it thinks is Agripina Lao. The two fell in love and, thereafter,
apparent error. got married in 1932 according to Catholic faith
and practice.
As constitutional creations invested with
necessary power, the Electoral Tribunals, The couple bore eight children, one of whom is
although not powers in the tripartite scheme of the private respondent who was born in 1948.
the government, are, in the exercise of their
functions independent organs — independent of The private respondent's father never emigrated
Congress and the Supreme Court. The power from this country. He decided to put up a
granted to HRET by the Constitution is intended hardware store and shared and survived the
to be as complete and unimpaired as if it had vicissitudes of life in Samar.
remained originally in the legislature. (Angara v.
Electoral Commission, 63 Phil. 139 [1936]) The business prospered. Expansion became
inevitable. As a result, a branch was set-up in
In passing upon petitions, the Court with its Binondo, Manila. In the meantime, the father of
traditional and careful regard for the balance of the private respondent, unsure of his legal status
powers, must permit this exclusive privilege of and in an unequivocal affirmation of where he
the Tribunals to remain where the Sovereign cast his life and family, filed with the Court of
authority has place it. (See Veloso v. Boards of First Instance of Samar of application for
Canvassers of Leyte and Samar, 39 Phil. 886 naturalization on February 15, 1954. LibLex
[1919])
On April 28, 1955, the CFI of Samar, after trial,
It has been argued that under Article VI, Section declared Jose Ong Chuan a Filipino citizen.
17 of the present Constitution, the situation may
exist as it exists today where there is an On May 15, 1957, the Court of First Instance of
unhealthy one-sided political composition of the Samar issued an order declaring the decision of
two Electoral Tribunals. There is nothing in the April 28, 1955 as final and executory and that
Constitution, however, that makes the HRET Jose Ong Chuan may already take his Oath of
because of its composition any less independent Allegiance.
from the Court or its constitutional functions any
less exclusive. The degree of judicial intervention Pursuant to said order, Jose Ong Chuan took his
should not be made to depend on how many Oath of Allegiance; correspondingly, a certificate
legislative members of the HRET belong to this of naturalization was issued to him.
party or that party. The test remains the same —
manifest grave abuse of discretion.

9|CONSTI2_Article IV_CITIZENSHIP
At the time Jose Ong Chuan took his oath, the the opportunity came in 1987, he ran in the
private respondent then a minor of nine years elections for representative in the second district
was finishing his elementary education in the of Northern Samar.
province of Samar. There is nothing in the records
to differentiate him from other Filipinos insofar as Mr. Ong was overwhelmingly voted by the people
the customs and practices of the local populace of Northern Samar as their representative in
were concerned. Congress. Even if the total votes of the two
petitioners are combined, Ong would still lead the
Fortunes changed. The house of the family of the two by more than 7,000 votes.
private respondent in Laoang, Samar was burned
to the ground. The pertinent portions of the Constitution found
in Article IV read:.
Undaunted by the catastrophe, the private
respondent's family constructed another one in "SECTION 1, the following are citizens of the
place of their ruined house. Again, there is no Philippines:
showing other than that Laoang was their abode
and home. 1. Those who are citizens of the Philippines at the
time of the adoption of the Constitution;
After completing his elementary education, the
private respondent, in search for better 2. Those whose fathers or mothers are citizens of
education, went to Manila in order to acquire his the Philippines;
secondary and college education.
3. Those born before January 17, 1973, of Filipino
In the meantime, another misfortune was mothers, who elect Philippine citizenship upon
suffered by the family in 1975 when a fire gutted reaching the age of majority; and
their second house in Laoang, Samar. The
respondent's family constructed still another 4. Those who are naturalized in accordance with
house, this time a 16-door apartment building, law.
two doors of which were reserved for the family.
SECTION 2, Natural-born Citizens are those who
The private respondent graduated from college, are citizens of the Philippines from birth without
and thereafter took and passed the CPA Board having to perform any act to acquire or perfect
Examinations. their citizenship. Those who elect Philippine
citizenship in accordance with paragraph 3 hereof
Since employment opportunities were better in shall be deemed natural born citizens."
Manila, the respondent looked for work here. He
found a job in the Central Bank of the Philippines
as an examiner. Later, however, he worked in the
hardware business of his family in Manila. In The Court interprets Section 1, Paragraph 3
1971, his elder brother, Emil, was elected as a above as applying not only to those who elect
delegate to the 1971 Constitutional Convention. Philippine citizenship after February 2, 1987 but
His status as a natural born citizen was also to those who, having been born of Filipino
challenged. Parenthetically, the Convention which mothers, elected citizenship before that date.
in drafting the Constitution removed the unequal
treatment given to derived citizenship on the The provision in Paragraph 3 was intended to
basis of the mother's citizenship formally and correct an unfair position which discriminates
solemnly declared Emil Ong, respondent's full against Filipino women. There is no ambiguity in
brother, as a natural born Filipino. The the deliberations of the Constitutional
Constitutional Convention had to be aware of the Commission, viz:
meaning of natural born citizenship since it was
precisely amending the article on this subject. "Mr. Azcuna:
cdll
With respect to the provision of section 4, would
The private respondent frequently went home to this refer only to those who elect Philippine
Laoang, Samar, where he grew up and spent his citizenship after the effectivity of the 1973
childhood days. Constitution or would it also cover those who
elected it under the 1973 Constitution?
In 1984, the private respondent married a Filipina
named Desiree Lim. Fr. Bernas:

For the elections of 1984 and 1986, Jose Ong, Jr. It would apply to anybody who elected Philippine
registered himself as a voter of Laoang, Samar, citizenship by virtue of the provision of the 1935
and correspondingly, voted there during those Constitution whether the election was done
elections. before or after January l7, 1973." (Records of the
Constitutional Commission, Vol. 1, p. 228;
The private respondent after being engaged for Emphasis supplied).
several years in the management of their family
business decided to be of greater service to his xxx xxx xxx
province and ran for public office. Hence, when

10 | C O N S T I 2 _ A r t i c l e I V _ C I T I Z E N S H I P
"Mr. Trenas: elect Philippine citizenship upon reaching the age
of majority; and if they do elect, they become
The Committee on Citizenship, Bill of Rights, Filipino citizens but not natural-born Filipino
Political Rights and Obligations and Human Rights citizens." (Records of the Constitutional
has more or less decided to extend the Commission, Vol. 1, p. 356)
interpretation of who is a natural-born Citizen as
provided in section 4 of the 1973 Constitution by The foregoing significantly reveals the intent of
adding that persons who have elected Philippine the framers. To make the provision prospective
Citizenship under the 1935 Constitution shall be from February 3, 1987 is to give a narrow
natural-born? Am I right Mr. Presiding Officer? interpretation resulting in an inequitable
situation. It must also be retroactive.
Fr. Bernas:
It should be noted that in construing the law, the
yes." Courts are not always to be hedged in by the
literal meaning of its language. The spirit and
xxx xxx xxx intendment thereof, must prevail over the letter,
especially where adherence to the latter would
"Mr. Nolledo: result in absurdity and injustice. (Casela v. Court
of Appeals, 35 SCRA 279 [1970])
And I remember very well that in the Reverend
Father Bernas' well written book, he said that the A Constitutional provision should be construed so
decision was designed merely to accommodate as to give it effective operation and suppress the
former delegate Ernesto Ang and that the mischief at which it is aimed, hence, it is the spirit
definition on natural-born has no retroactive of the provision which should prevail over the
effect. Now it seems that the Reverend Father letter thereof. (Jarrolt v. Mabberly, 103 U.S. 580)
Bernas is going against this intention by
supporting the amendment? In the words of the Court in the case of J.M.
Tuason v. LTA (31 SCRA 413 [1970]:
Fr. Bernas:
"To that primordial intent, all else is subordinated.
As the Commissioner can see, there has been an Our Constitution, any constitution is not to be
evolution in my thinking. (Records of the construed narrowly or pedantically, for the
Constitutional Commission, Vol. 1, p. 189) prescriptions therein contained, to paraphrase
Justice Holmes, are not mathematical formulas
xxx xxx xxx having their essence in their form but are organic
living institutions, the significance of which is
"Mr. Rodrigo: vital not formal . . ." (p. 427)

But this provision becomes very important The provision in question was enacted to correct
because his election of Philippine citizenship the anomalous situation where one born of a
makes him not only a Filipino citizen but a Filipino father and an alien mother was
natural-born Filipino citizen entitling him to run automatically granted the status of a natural-born
for Congress . . . citizen while one born of a Filipino mother and an
alien father would still have to elect Philippine
Fr. Bernas: citizenship. If one so elected, he was not, under
earlier laws, conferred the status of a natural-
Correct. We are quite aware of that and for that born.
reason we will leave it to the body to approve
that provision of section 4. Under the 1973 Constitution, those born of
Filipino fathers and those born of Filipino mothers
Mr. Rodrigo: with an alien father were placed on equal footing.
They were both considered as natural-born
I think there is a good basis for the provision citizens.
because it strikes me as unfair that the Filipino
citizen who was born a day before January 17, Hence, the bestowment of the status of "natural-
1973 cannot be a Filipino citizen or a natural born born" cannot be made to depend on the fleeting
citizen." (Records of the Constitutional accident of time or result in two kinds of citizens
Commission, Vol. 1, p. 231) made up of essentially the same similarly
situated members.
xxx xxx xxx
It is for this reason that the amendments were
"Mr. Rodrigo: enacted, that is, in order to remedy this
accidental anomaly, and, therefore, treat equally
The purpose of that provision is to remedy an all those born before the 1973 Constitution and
inequitable situation. Between 1935 and 1973 who elected Philippine citizenship either before or
when we were under the 1935 Constitution, those after the effectivity of that Constitution. Cdpr
born of Filipino fathers but alien mothers were
natural-born Filipinos. However, those born of The Constitutional provision in question is,
Filipino mothers but alien fathers would have to therefore curative in nature. The enactment was

11 | C O N S T I 2 _ A r t i c l e I V _ C I T I Z E N S H I P
meant to correct the inequitable and absurd participated in political exercises as a Filipino and
situation which then prevailed, and thus, render has always considered himself a Filipino citizen.
those acts valid which would have been nil at the There is nothing in the records to show that he
time had it not been for the curative provisions. does not embrace Philippine customs and values,
(See Development Bank of the Philippines v. nothing to indicate any tinge of alien-ness, no
Court of Appeals, 96 SCRA 342 [1980]) acts to show that this country is not his natural
homeland. The mass of voters of Northern Samar
There is no dispute that the respondent's mother are fully aware of Mr. Ong's parentage. They
was a natural born Filipina at the time of her should know him better than any member of this
marriage. Crucial to this case is the issue of Court will ever know him. They voted by
whether or not the respondent elected or chose overwhelming numbers to have him represent
to be a Filipino citizen. them in Congress. Because of his acts since
childhood, they have considered him as a Filipino.
Election becomes material because Section 2 of
Article IV of the Constitution accords natural born The filing of sworn statement or formal
status to children born of Filipino mothers before declaration is a requirement for those who still
January 17, 1973, if they elect citizenship upon have to elect citizenship. For those already
reaching the age of majority. Filipinos when the time to elect came up, there
are acts of deliberate choice which cannot be less
To expect the respondent to have formally or in binding. Entering a profession open only to
writing elected citizenship when he came of age Filipinos, serving in public office where citizenship
is to ask for the unnatural and unnecessary. The is a qualification, voting during election time,
reason is obvious. He was already a citizen. Not running for public office, and other categorical
only was his mother a natural born citizen but his acts of similar nature are themselves formal
father had been naturalized when the respondent manifestations of choice for these persons. LLjur
was only nine (9) years old. He could not have
divined when he came of age that in 1973 and An election of Philippine citizenship presupposes
1987 the Constitution would be amended to that the person electing is an alien. Or his status
require him to have filed a sworn statement in is doubtful because he is a national of two
1969 electing citizenship inspite of his already countries. There is no doubt in this case about Mr.
having been a citizen since 1957. In 1969, Ong's being a Filipino when he turned twenty-one
election through a sworn statement would have (21).
been an unusual and unnecessary procedure for
one who had been a citizen since he was nine We repeat that any election of Philippine
years old. citizenship on the part of the private respondent
would not only have been superfluous but it
We have jurisprudence that defines "election" as would also have resulted in an absurdity. How can
both a formal and an informal process. a Filipino citizen elect Philippine citizenship?

In the case of In Re: Florencio Mallare (59 SCRA The respondent HRET has an interesting view as
45 [1974]), the Court held that the exercise of the to how Mr. Ong elected citizenship. It observed
right of suffrage and the participation in election that "when protestee was only nine years of age,
exercises constitute a positive act of election of his father, Jose Ong Chuan became a naturalized
Philippine citizenship. In the exact Filipino. Section 15 of the Revised Naturalization
pronouncement of the Court, we held: Act squarely applies its benefit to him for he was
then a minor residing in this country. Concededly,
"Esteban's exercise of the right of suffrage when it was the law itself that had already elected
he came of age, constitutes a positive act of Philippine citizenship for protestee by declaring
election of Philippine citizenship". (p. 52; him as such." (Emphasis supplied)
emphasis supplied)

The private respondent did more than merely


exercise his right of suffrage. He has established The petitioners argue that the respondent's
his life here in the Philippines. father was not, validly, a naturalized citizen
because of his premature taking of the oath of
For those in the peculiar situation of the citizenship.
respondent who cannot be expected to have
elected citizenship as they were already citizens, The Court cannot go into the collateral procedure
we apply the In Re Mallare rule. of stripping Mr. Ong's father of his citizenship
after his death and at this very late date just so
The respondent was born in an outlying rural we can go after the son.
town of Samar where there are no alien enclaves
and no racial distinctions. The respondent has The petitioners question the citizenship of the
lived the life of a Filipino since birth. His father father through a collateral approach. This can not
applied for naturalization when the child was still be done. In our jurisdiction, an attack on a
a small boy. He is a Roman Catholic. He has person's citizenship may only be done through a
worked for a sensitive government agency. His direct action for its nullity. (See Queto v. Catolico,
profession requires citizenship for taking the 31 SCRA 52 [1970]).
examinations and getting a license. He has

12 | C O N S T I 2 _ A r t i c l e I V _ C I T I Z E N S H I P
To ask the Court to declare the grant of Philippine
citizenship to Jose Ong Chuan as null and void 1. Persons born in Spanish territory.
would run against the principle of due process.
Jose Ong Chuan has already been laid to rest. 2. Children born of a Spanish father or mother,
How can he be given a fair opportunity to defend even though they were born out of Spain.
himself. A dead man cannot speak. To quote the
words of the HRET: "Ong Chuan's lips have long 3. Foreigners who may have obtained
been muted to perpetuity by his demise and naturalization papers.
obviously he could not rise beyond where his
mortal remains now lie to defend himself were 4. Those without such papers, who may have
this matter to be made a central issue in this acquired domicile in any town in the Monarchy."
case." (Emphasis supplied)

The issue before us is not the nullification of the The domicile of a natural person is the place of
grant of citizenship to Jose Ong Chuan. Our his habitual residence. This domicile, once
function is to determine whether or not the HRET established is considered to continue and will not
committed abuse of authority in the exercise of be deemed lost until a new one is established.
its powers. Moreover, the respondent traces his (Article 50, NCC; Article 40, Civil Code of Spain;
natural born citizenship through his mother, not Zuellig v. Republic, 83 Phil. 768 [1949])
through the citizenship of his father. The
citizenship of the father is relevant only to As earlier stated, Ong Te became a permanent
determine whether or not the respondent "chose" resident of Laoang, Samar around 1895.
to be a Filipino when he came of age. At that time Correspondingly, a certificate of residence was
and up to the present, both mother and father then issued to him by virtue of his being a
were Filipinos. Respondent Ong could not have resident of Laoang, Samar. (Report of the
elected any other citizenship unless he first Committee on Election Protests and Credentials
formally renounced Philippine citizenship in favor of the 1971 Constitutional Convention,
of a foreign nationality. Unlike other persons September 7,1972, p. 3)
faced with a problem of election, there was no
foreign nationality of his father which he could The domicile that Ong Te established m 1895
possibly have chosen. continued until April 11, 1899; it even went
beyond the turn of the 19th century. It is also in
There is another reason why we cannot declare this place were Ong Te set-up his business and
the HRET as having committed manifest grave acquired his real property.
abuse of discretion. The same issue of natural-
born citizenship has already been decided by the As concluded by the Constitutional Convention
Constitutional Convention of 1971 and by the Ong Te falls within the meaning of sub-paragraph
Batasang Pambansa convened by authority of the 4 of Article 17 of the Civil Code of Spain.
Constitution drafted by that Convention. Emil
Ong, full blood brother of the respondent, was Although Ong Te made brief visits to China, he,
declared and accepted as a natural born citizen nevertheless, always returned to the Philippines.
by both bodies. The fact that he died in China, during one of his
visits in said country, was of no moment. This will
Assuming that our opinion is different from that of not change the fact that he already had his
the Constitutional Convention, the Batasang domicile fixed in the Philippines and pursuant to
Pambansa, and the respondent HRET, such a the Civil Code of Spain, he had become a Spanish
difference could only be characterized as error. subject. LibLex
There would be no basis to call the HRET decision
so arbitrary and whimsical as to amount to grave If Ong Te became a Spanish subject by virtue of
abuse of discretion. having established his domicile in a town under
the Monarchy of Spain, necessarily, Ong Te was
What was the basis for the Constitutional also an inhabitant of the Philippines for an
Convention's declaring Emil Ong a natural born inhabitant has been defined as one who has
citizen? actual fixed residence in a place; one who has a
domicile in a place. (Bouvier's Law Dictionary,
Under the Philippine Bill of 1902, inhabitants of Vol. II) A priori, there can be no other logical
the Philippines who were Spanish subjects on the conclusion but to educe that Ong Te qualified as a
11th day of April 1899 and then residing in said Filipino citizen under the provisions of section 4 of
islands and their children born subsequent the Philippine Bill of 1902.
thereto were conferred the status of a Filipino
citizen. The HRET itself found this fact of absolute verity
in concluding that the private respondent was a
Was the grandfather of the private respondent a natural-born Filipino.
Spanish subject?
The petitioners' sole ground in disputing this fact
Article 17 of the Civil Code of Spain enumerates is that the documents presented to prove it were
those who were considered Spanish Subjects, viz: not in compliance with the best evidence rule.
The petitioners allege that the private respondent
"ARTICLE 17. The following are Spaniards: failed to present the original of the documentary

13 | C O N S T I 2 _ A r t i c l e I V _ C I T I Z E N S H I P
evidence, testimonial evidence and of the Justice Davide, Jr. The petitioners could have
transcript of the proceedings of the body which presented any one of the long list of delegates to
the aforesaid resolution of the 1971 refute Mr. Ong's having been declared a natural-
Constitutional Convention was predicated. born citizen. They did not do so. Nor did they
demur to the contents of the documents
On the contrary, the documents presented by the presented by the private respondent. They
private respondent fall under the exceptions to merely relied on the procedural objections
the best evidence rule. respecting the admissibility of the evidence
presented.
It was established in the proceedings before the
HRET that the originals of the Committee Report The Constitutional Convention was the sole judge
No. 12, the minutes of the plenary session of of the qualifications of Emil Ong to be a member
1971 Constitutional Convention held on of that body. The HRET, by explicit mandate of
November 28, 1972 cannot be found. the Constitution, is the sole judge of the
qualifications of Jose Ong, Jr. to be a member of
This was affirmed by Atty. Ricafrente, Assistant Congress. Both bodies deliberated at length on
Secretary of the 1971 Constitutional Convention; the controversies over which they were sole
by Atty. Nolledo, Delegate to the 1971 judges. Decisions were arrived at only after a full
Constitutional Convention; and by Atty. Antonio presentation of all relevant factors which the
Santos, Chief Librarian of the U.P. Law Center, in parties wished to present. Even assuming that we
their respective testimonies given before the disagree with their conclusions, we cannot
HRET to the effect that there is no governmental declare their acts as committed with grave abuse
agency which is the official custodian of the of discretion. We have to keep clear the line
records of the 1971 Constitutional Convention. between error and grave abuse.
(TSN, December 12, 1988, pp. 30-31; TSN,
January 17, 1989, pp. 34-35; TSN, February 1, ON THE ISSUE OF RESIDENCE
1989, p. 44; TSN, February 6, 1989, pp. 28-29) The petitioners question the residence
qualification of respondent Ong.
The execution of the originals was established by
Atty. Ricafrente, who as the Assistant Secretary of The petitioners lose sight of the meaning of
the 1971 Constitutional Convention was the "residence" under the Constitution. The term
proper party to testify to such execution. (TSN, "residence" has been understood as synonymous
December 12, 1989, pp. 11-24) with domicile not only under the previous
Constitutions but also under the 1987
The inability to produce the originals before the Constitution.
HRET was also testified to as aforestated by Atty.
Ricafrente, Atty. Nolledo, and Atty. Santos. In The deliberations of the Constitutional
proving the inability to produce, the law does not Commission reveal that the meaning of residence
require the degree of proof to be of sufficient vis-a-vis the qualifications of a candidate for
certainty; it is enough that it be shown that after Congress continues to remain the same as that of
a bona fide diligent search, the same cannot be domicile, to wit:
found. (see Government of P.I. v. Martinez, 44
Phil. 817 [1918])

Since the execution of the document and the "Mr. Nolledo:


inability to produce were adequately established,
the contents of the questioned documents can be With respect to Section 5, I remember that in the
proven by a copy thereof or by the recollection of 1971 Constitutional Convention, there was an
witnesses. attempt to require residence in the place not less
than one year immediately preceding the day of
Moreover, to erase all doubts as to the the elections. So my question is: What is the
authenticity of the documentary evidence cited in Committee's concept of residence of a candidate
the Committee Report, the former member of the for the legislature? Is it actual residence or is it
1971 Constitutional Convention, Atty. Nolledo, the concept of domicile or constructive
when he was presented as a witness in the residence?
hearing of the protest against the private
respondent, categorically stated that he saw the Mr. Davide:
disputed documents presented during the
hearing of the election protest against the brother Madame President, insofar as the regular
of the private respondent. (TSN, February 1, members of the National Assembly are
1989, pp. 8-9) concerned, the proposed section merely provides,
among others, 'and a resident thereof, that is, in
In his concurring opinion, Mr. Justice Sarmiento, a the district, for a period of not less than one year
vice-president of the Constitutional Convention, preceding the day of the election'. This was in
states that he was presiding officer of the plenary effect lifted from the 1973 Constitution, the
session which deliberated on the report on the interpretation given to it was domicile." (Records
election protest against Delegate Emil Ong. He of the 1987 Constitutional Convention, Vol. II, July
cites a long list of names of delegates present. 22, 1986, p. 87)
Among them are Mr. Chief Justice Fernan, and Mr.

14 | C O N S T I 2 _ A r t i c l e I V _ C I T I Z E N S H I P
xxx xxx xxx The properties owned by the Ong Family are in
the name of the private respondent's parents.
"Mrs. Rosario Braid: Upon the demise of his parents, necessarily, the
private respondent, pursuant to the laws of
The next question is on Section 7, page 2. I think succession, became the co-owner thereof (as a
Commissioner Nolledo has raised the same point co-heir), notwithstanding the fact that these were
that 'resident' has been interpreted at times as a still in the names of his parents.
matter of intention rather than actual residence.
Even assuming that the private respondent does
Mr. De los Reyes: not own any property in Samar, the Supreme
Court in the case of De los Reyes D. Solidum (61
Domicile. Phil. 893 [1935]) held that it is not required that a
person should have a house in order to establish
Ms. Rosario Braid: his residence and domicile. It is enough that he
should live in the municipality or in a rented
Yes, So, would the gentlemen consider at the house or in that of a friend or relative. (Emphasis
proper time to go back to actual residence rather supplied)
than mere intention to reside?
To require the private respondent to own property
Mr. De los Reyes: in order to be eligible to run for Congress would
be tantamount to a property qualification. The
But we might encounter some difficulty Constitution only requires that the candidate
especially considering that a provision in the meet the age, citizenship, voting and residence
Constitution in the Article on Suffrage says that requirements. Nowhere is it required by the
Filipinos living abroad may vote as enacted by Constitution that the candidate should also own
law. So, we have to stick to the original concept property in order to be qualified to run. (see
that it should be by domicile and not physical and Maquera v. Borra, 122 Phil. 412 [1965])
actual residence." (Records of the 1987
Constitutional Commission, Vol. II, July 22, 1986, It has also been settled that absence from
p. 110) residence to pursue studies or practice a
profession or registration as a voter other than in
The framers of the Constitution adhered to the the place where one is elected, does not
earlier definition given to the word "residence" constitute loss of residence. (Faypon v. Quirino,
which regarded it as having the same meaning as 96 Phil. 294 [1954])
domicile.
As previously stated, the private respondent
The term "domicile" denotes a fixed permanent stayed in Manila for the purpose of finishing his
residence to which when absent for business or studies and later to practice his profession. There
pleasure, one intends to return. (Ong Huan Tin v. was no intention to abandon the residence in
Republic, 19 SCRA 966 [1967]) The absence of a Laoang, Samar. On the contrary, the periodical
person from said permanent residence, no matter journeys made to his home province reveal that
how long, notwithstanding, it continues to be the he always had the animus revertendi.
domicile of that person. In other words, domicile
is characterized by animus revertendi. (Ujano v. The Philippines is made up not only of a single
Republic, 17 SCRA 147 [1966]) cdphil race; it has, rather, undergone an interracial
evolution. Throughout our history, there has been
The domicile of origin of the private respondent, a continuing influx of Malays, Chinese,
which was the domicile of his parents, is fixed at Americans, Japanese, Spaniards and other
Laoang, Samar. Contrary to the petitioners' nationalities. This racial diversity gives strength
imputation, Jose Ong, Jr. never abandoned said to our country.
domicile; it remained fixed therein even up to the
present. Many great Filipinos have not been whole-blooded
nationals, if there is such a person, for there is
The private respondent, in the proceedings before none. To mention a few, the great Jose Rizal was
the HRET, sufficiently established that after the part Chinese, the late Chief Justice Claudio
fire that gutted their house in 1961, another one Teehankee was part Chinese, and of course our
was constructed. own President, Corazon Aquino is also part
Chinese. Verily, some Filipinos of whom we are
Likewise, after the second fire which again proud were ethnically more Chinese than the
destroyed their house in 1975, a sixteen-door private respondent.
apartment was built by their family, two doors of
which were reserved as their family residence. Our citizens no doubt constitute the country's
(TSN, Jose Ong, Jr., November 18, 1988, p. 8) greatest wealth. Citizenship is a special privilege
which one must forever cherish.
The petitioners' allegation that since the private
respondent owns no property in Laoang, Samar, However, in order to truly revere this treasure of
he cannot, therefore, be a resident of said place citizenship, we do not, on the basis of too harsh
is misplaced. an interpretation, have to unreasonably deny it to
those who qualify to share in its richness.

15 | C O N S T I 2 _ A r t i c l e I V _ C I T I Z E N S H I P
Portuguese citizen, applied for a renewal of his
Under the overly strict jurisprudence surrounding Portuguese passport and represented himself as
our antiquated naturalization laws only the very such in official documents even after he had
affluent backed by influential patrons, who were become a naturalized Philippine citizen. Such
willing to suffer the indignities of a lengthy, resumption or reacquisition of Portuguese
sometimes humiliating, and often corrupt process citizenship is grossly inconsistent with his
of clearances by minor bureaucrats and whose maintenance of Philippine citizenship.
lawyers knew how to overcome so many
technical traps of the judicial process were able 2. ID.; BILL OF RIGHTS; DUE PROCESS; DENIAL
to acquire citizenship. It is time for the THEREOF OBVIATED WHEN PETITIONER WAS
naturalization law to be revised to enable a more GIVEN BY COURT THE OPPORTUNITY TO SHOW
positive, affirmative, and meaningful examination PROOF OF CONTINUED PHILIPPINE CITIZENSHIP.
of an applicant's suitability to be a Filipino. A — Denial, if any, of due process was obviated
more humane, more indubitable and less when petitioner was given by the Court the
technical approach to citizenship problems is opportunity to show proof of continued Philippine
essential. citizenship, but he has failed.

WHEREFORE, the petitions are hereby 3. ID.; CITIZENSHIP; PHILIPPINE CITIZENSHIP; NOT
DISMISSED. The questioned decision of the house A COMMODITY OR WARE TO BE DISPLAYED WHEN
of Representatives Electoral Tribunal is AFFIRMED. REQUIRED AND SUPPRESSED WHEN CONVENIENT.
Respondent Jose Ong, Jr. is declared a natural- — Philippine citizenship, it must be stressed, is
born citizen of the Philippines and a resident of not a commodity or were to be displayed when
Laoang, Northern Samar. required and suppressed when convenient.

FERNAN, C.J., dissenting:


SO ORDERED. 1. CONSTITUTIONAL LAW; BILL OF RIGHTS; DUE
PROCESS; POWERS OF REVIEW OF SUPREME
Bidin, Griño-Aquino, Medialdea and Davide, Jr., COURT CANNOT BE A SUBSTITUTE FOR DEMANDS
JJ ., concur. THEREOF SINCE SAID COURT IS NOT A TRIER OF
FACTS. — The observation of Mr. Justice Hugo E.
Fernan, C .J ., Melencio-Herrera, Cruz, Feliciano Gutierrez, Jr. in his dissenting opinion that
and Gancayco, JJ ., took no part. "(c)onsidering the serious implications of de-
Filipinization, the correct procedures according to
(In re: Yu v. Defensor-Santiago, G.R. No. law must be applied," is appropriate as it has
83882, January 24, 1989) been held that "(i)f, however, in a deportation
proceeding, the alleged alien claims citizenship
EN BANC and supports the claim by substantial evidence,
he is entitled to have his status finally determined
[G.R. No. 83882. January 24, 1989.] by a judicial, as distinguished from an executive,
tribunal" (3 Am Jur 2d 949 citing United States ex
IN RE PETITION FOR HABEAS CORPUS OF WILLIE rel. Bilokumsky v. Tod, 263 US 149, 68 L ed 221,
YU, WILLIE YU, petitioner, vs. MIRIAM DEFENSOR- 44 S Ct 54; Ng Fung Ho v. White, 259 US 276, 66
SANTIAGO, BIENVENIDO P. ALANO, JR., MAJOR Led 938, 42 S Ct 492). By this, it means a full
PABALAN, DELEO HERNANDEZ, BLODDY blown trial under the more rigid rules of evidence
HERNANDEZ, BENNY REYES AND JUN ESPIRITU prescribed in court proceedings. And certainly,
SANTO, respondents. the review powers being exercised by this Court
in this case fall short of this requirement. Said
Pelaez, Adriano and Gregorio and Bonifacio A. powers of review cannot be a substitute for the
Alentajan for petitioner. demands of due process, particularly in the light
of the well-recognized principle that this Court is
Chavez, Hechanova & Lim Law Offices not a trier of facts.
collaborating counsel for petitioner.
DECISION
Augusto Jose y. Arreza for respondents.
PADILLA, J p:
SYLLABUS
The present controversy originated with a petition
1. CONSTITUTIONAL LAW; CITIZENSHIP; EXPRESS; for habeas corpus filed with the Court on 4 July
RENUNCIATION; A RENUNCIATION MADE KNOWN 1988 seeking the release from detention of herein
DISTINCTLY AND EXPLICITLY AND NOT LEFT TO petitioner. 1 After manifestation and motion of
INFERENCE OR IMPLICATION. — In Board of the Solicitor General of his decision to refrain
Immigration Commissioners vs. Go Gallano, from filing a return of the writ on behalf of the
express renunciation was held to mean a CID, respondent Commissioner thru counsel filed
renunciation that is made known distinctly and the return. 2 Counsel for the parties were heard
explicitly and not left to inference or implication. in oral argument on 20 July 1988. The parties
Petitioner, with full knowledge, and legal were allowed to submit marked exhibits, and to
capacity, after having renounced Portuguese file memoranda. 3 An internal resolution of 7
citizenship upon naturalization as a Philippine November 1988 referred the case to the Court en
citizen resumed or reacquired his prior status as a banc. In its 10 November 1988 resolution,

16 | C O N S T I 2 _ A r t i c l e I V _ C I T I Z E N S H I P
denying the petition for habeas corpus, the Court order. Petitioner filed a reply thereto on 6 January
disposed of the pending issues of (1) jurisdiction 1989.
of the CID over a naturalized Filipino citizen and
(2) validity of warrantless arrest and detention of Petitioner's own compliance reveals that he was
the same person. originally issued a Portuguese passport in 1971,
17 valid for five (5) years and renewed for the
Petitioner filed a motion for reconsideration with same period upon presentment before the proper
prayer for restraining order dated 24 November Portuguese consular officer. Despite his
1988. 4 On 29 November 1988, the Court naturalization as a Philippine citizen on 10
resolved to deny with finality the aforesaid February 1978, on 21 July 1981, petitioner
motion for reconsideration, and further resolved applied for and was issued Portuguese Passport
to deny the urgent motion for issuance of a No. 35/81 serial N. 1517410 by the Consular
restraining order dated 28 November 1988. 5 Section of the Portuguese Embassy in Tokyo. Said
Consular Office certifies that his Portuguese
Undaunted, petitioner filed a motion for passport expired on 20 duly 1986. 18 While still a
clarification with prayer for restraining order on 5 citizen of the Philippines who had renounced,
December 1988. upon his naturalization, "absolutely and forever
all allegiance and fidelity to any foreign prince,
Acting on said motion, a temporary restraining potentate, state or sovereignty" and pledged to
order was issued by the Court on 7 December "maintain true faith and allegiance to the
1988. 6 Respondent Commissioner filed a motion Republic of the Philippines," 19 he declared his
to lift TRO on 13 December 1988, the basis of nationality as Portuguese in commercial
which is a summary judgment of deportation documents he signed, specifically, the Companies
against Yu issued by the CID Board of registry of Tai Shun Estate Ltd. 20 filed in
Commissioners on 2 December 1988. 7 Petitioner Hongkong sometime in April 1980.
also filed a motion to set case for oral argument
on 8 December 1988. To the mind of the Court, the foregoing acts
considered together constitute an express
In the meantime, an urgent motion for release renunciation of petitioner's Philippine citizenship
from arbitrary detention 8 was filed by petitioner acquired through naturalization. In Board of
on 13 December 1988. A memorandum in Immigration Commissioners vs. Go Gallano, 21
furtherance of said motion for release dated 14 express renunciation was held to mean a
December 1988 was filed on 15 December 1988 renunciation that is made known distinctly and
together with a vigorous opposition to the lifting explicitly and not left to inference or implication.
of the TRO. cdrep Petitioner, with full knowledge, and legal
capacity, after having renounced Portuguese
The lifting of the Temporary Restraining Order citizenship upon naturalization as a Philippine
issued by the Court on 7 December 1988 is citizen 22 resumed or reacquired his prior status
urgently sought by respondent Commissioner as a Portuguese citizen, applied for a renewal of
who was ordered to cease and desist from his Portuguese passport 23 and represented
immediately deporting petitioner Yu pending the himself as such in official documents even after
conclusion of hearings before the Board of Special he had become a naturalized Philippine citizen.
Inquiry, CID. To finally dispose of the case, the Such resumption or reacquisition of Portuguese
Court will likewise rule on petitioner's motion for citizenship is grossly inconsistent with his
clarification with prayer for restraining order maintenance of Philippine citizenship. LexLib
dated 5 December 1988, 9 urgent motion for
release from arbitrary detention dated 13 This Court issued the aforementioned TRO
December 1988, 10 the memorandum in pending hearings with the Board of Special
furtherance of said motion for release dated 14 Inquiry, CID. However, pleadings submitted
December 1988, 11 motion to set case for oral before this Court after the issuance of said TRO
argument dated 8 December 1988. 12 have unequivocally shown that petitioner has
expressly renounced his Philippine citizenship.
Acting on the motion to lift the temporary The material facts are not only established by the
restraining order (issued on 7 December 1988) pleadings — they are not disputed by petitioner.
dated 9 December 1988, 13 and the vigorous A rehearing on this point with the CID would be
opposition to lift restraining order dated 15 unnecessary and superfluous. Denial, if any, of
December 1988, 14 the Court resolved to give due process was obviated when petitioner was
petitioner Yu a non-extendible period of three (3) given by the Court the opportunity to show proof
days from notice within which to explain and of continued Philippine citizenship, but he has
prove why he should still be considered a citizen failed.
of the Philippines despite his acquisition and use
of a Portuguese passport. 15 While normally the question of whether or not a
person has renounced his Philippine citizenship
Petitioner filed his compliance with the resolution should be heard before a trial court of law in
of 15 December 1988 on 20 December 1988 16 adversary proceedings, this has become
followed by an earnest request for temporary unnecessary as this Court, no less, upon the
release on 22 December 1988. Respondent filed insistence of petitioner, had to look into the facts
on 2 January 1989 her comment reiterating her and satisfy itself on whether or not petitioner's
previous motion to lift temporary restraining

17 | C O N S T I 2 _ A r t i c l e I V _ C I T I Z E N S H I P
claim to continued Philippine citizenship is revolution to help in the restoration of democracy.
meritorious. He also argued that the challenge to his title
should be dismissed, being in reality a quo
Philippine citizenship, it must be stressed, is not a warranto petition that should have been filed
commodity or were to be displayed when within ten days from his proclamation, in
required and suppressed when convenient. This accordance with Section 253 of the Omhibus
then resolves adverse to the petitioner his motion Election Code. The League, moreover, was not a
for clarification and other motions mentioned in proper party because it was not a voter and so
the second paragraph, page 3 of this Decision. could not sue under the said section. prLL

WHEREFORE, premises considered, petitioner's Frivaldo moved for a preliminary hearing on his
motion for release from detention is DENIED. affirmative defenses but the respondent
Respondent's motion to lift the temporary Commission on Elections decided instead by its
restraining order is GRANTED. This Decision is Order of January 20, 1988, to set the case for
immediately executory. hearing on the merits. His motion for
reconsideration was denied in another Order
dated February 21, 1988. He then came to this
Court in a petition for certiorari and prohibition to
SO ORDERED. ask that the said orders be set aside on the
ground that they had been rendered with grave
Melencio-Herrera, Paras, Feliciano, Gancayco, abuse of discretion. Pending resolution of the
Bidin, Sarmiento, Griño-Aquino, Medialdea and petition, we issued a temporary order against the
Regalado, JJ., concur. hearing on the merits scheduled by the COMELEC
and at the same time required comments from
Narvasa, J., in the result. the respondents.

(Frivaldo v. COMELEC, G.R. No. 87193, June In their Comment, the private respondents
23, 1989) reiterated their assertion that Frivaldo was a
naturalized American citizen and had not
EN BANC reacquired Philippine citizenship on the day of the
election on January 18, 1988. He was therefore
[G.R. No. 87193. June 23, 1989.] not qualified to run for and be elected governor.
They also argued that their petition in the
JUAN GALLANOSA FRIVALDO, petitioner, vs. Commission on Elections was not really for quo
COMMISSION ON ELECTIONS AND THE LEAGUE OF warranto under Section 253 of the Omnibus
MUNICIPALITIES, SORSOGON CHAPTER, HEREIN Election Code. The ultimate purpose was to
REPRESENTED BY ITS PRESIDENT, SALVADOR NEE prevent Frivaldo from continuing as governor, his
ESTUYE, respondents. candidacy and election being null and void ab
initio because of his alienage. Even if their
J .L. Misa & Associates for petitioner. petition were to be considered as one for quo
warranto, it could not have been filed within ten
Lladoc, Huab & Associates for private respondent. days from Frivaldo's proclamation because it was
only in September 1988 that they received proof
DECISION of his naturalization. And assuming that the
League itself was not a proper party, Estuye
CRUZ, J p: himself, who was suing not only for the League
but also in his personal capacity, could
Petitioner Juan G. Frivaldo was proclaimed nevertheless institute the suit by himself alone.
governor-elect of the province of Sorsogon on
January 22, 1988, and assumed office in due Speaking for the public respondent, the Solicitor
time. On October 27, 1988, the league of General supported the contention that Frivaldo
Municipalities, Sorsogon Chapter (hereafter, was not a citizen of the Philippines and had not
League), represented by its President, Salvador repatriated himself after his naturalization as an
Estuye, who was also suing in his personal American citizen. As an alien, he was disqualified
capacity, filed with the Commission on Elections a from public office in the Philippines. His election
petition for the annulment of Frivaldo's election did not cure this defect because the electorate of
and proclamation on the ground that he was not Sorsogon could not amend the Constitution, the
a Filipino citizen, having been naturalized in the Local Government Code, and the Omnibus
United States on January 20,1983. In his answer Election Code. He also joined in the private
dated May 22, 1988, Frivaldo admitted that he respondent's argument that Section 253 of the
was naturalized in the United States as alleged Omnibus Election Code was not applicable
but pleaded the special and affirmative defenses because what the League and Estuye were
that he had sought American citizenship only to seeking was not only the annulment of the
protect himself against President Marcos. His proclamation and election of Frivaldo. He agreed
naturalization, he said, was "merely forced upon that they were also asking for the termination of
himself as a means of survival against the Frivaldo's incumbency as governor of Sorsogon
unrelenting persecution by the Martial Law on the ground that he was not a Filipino.
Dictator's agents abroad." He added that he had
returned to the Philippines after the EDSA

18 | C O N S T I 2 _ A r t i c l e I V _ C I T I Z E N S H I P
In his Reply, Frivaldo insisted that he was a the Omnibus Election Code provides that a
citizen of the Philippines because his qualified voter must be, among other
naturalization as an American citizen was not qualifications, a citizen of the Philippines, this
"impressed with voluntariness." In support he being an indispensable requirement for suffrage
cited the Nottebohm Case, [1955 I.C.J. 4; 49 under Article V, Section 1, of the Constitution.
A.J.I.L. 396 (1955)] where a German national's
naturalization in Liechtenstein was not recognized In the certificate of candidacy he filed on
because it had been obtained for reasons of November 19, 1987, Frivaldo described himself as
convenience only. He said he could not have a "natural-born" citizen of the Philippines,
repatriated himself before the 1988 elections omitting mention of any subsequent loss of such
because the Special Committee on Naturalization status. The evidence shows, however, that he
created for the purpose by LOI No. 270 had not was naturalized as a citizen of the United States
yet been organized then. His oath in his in 1983 per the following certification from the
certificate of candidacy that he was a natural- United States District Court, Northern District of
born citizen should be a sufficient act of California, as duly authenticated by Vice Consul
repatriation. Additionally, his active participation Amado P. Cortez of the Philippine Consulate
in the 1987 congressional elections had divested General in San Francisco, California, U.S.A.
him of American citizenship under the laws of the
United States, thus restoring his Philippine OFFICE OF THE CLERK
citizenship. He ended by reiterating his prayer for UNITED STATES DISTRICT COURT
the rejection of the move to disqualify him for NORTHERN DISTRICT OF CALIFORNIA
being time-barred under Section 253 of the September 23, 1988
Omnibus Election Code. LLpr
TO WHOM IT MAY CONCERN:
Considering the importance and urgency of the Our records show that JUAN GALLANOSA
question herein raised, the Court has decided to FRIVALDO, born on October 20, 1915, was
resolve it directly instead of allowing the normal naturalized in this Court on January 20, 1983, and
circuitous route that will after all eventually end issued Certificate of Naturalization No. 11690178.
with this Court, albeit only after a long delay. We
cannot permit this delay. Such delay will be Petition No. 280225.
inimical to the public interest and the vital
principles of public office to be here applied. Alien Registration No. A23 079 270.

It is true that the Commission on Elections has Very truly yours,


the primary jurisdiction over this question as the WILLIAM L. WHITTAKER
sole judge of all contests relating to the election, Clerk
returns and qualifications of the members of the by:
Congress and elective provincial and city officials.
However, the decision on Frivaldo's citizenship (Sgd.)
has already been made by the COMELEC through ARACELI V. BARENG
its counsel, the Solicitor General, who Deputy Clerk
categorically claims that Frivaldo is a foreigner. This evidence is not denied by the petitioner. In
We assume this stance was taken by him after fact, he expressly admitted it in his answer.
consultation with the public respondent and with Nevertheless, as earlier noted, he claims it was
its approval. It therefore represents the decision "forced" on him as a measure of protection from
of the COMELEC itself that we may now review. the persecution of the Marcos government
Exercising our discretion to interpret the Rules of through his agents in the United States. cdll
Court and the Constitution, we shall consider the
present petition as having been filed in The Court sees no reason not to believe that the
accordance with Article IX-A, Section 7, of the petitioner was one of the enemies of the Marcos
Constitution, to challenge the aforementioned dictatorship. Even so, it cannot agree that as a
Orders of the COMELEC. consequence thereof he was coerced into
embracing American citizenship. His feeble
The basic question we must resolve is whether or suggestion that his naturalization was not the
not Juan G. Frivaldo was a citizen of the result of his own free and voluntary choice is
Philippines at the time of his election on January totally unacceptable and must be rejected
18,1988, as provincial governor of Sorsogon. All outright.
the other issues raised in this petition are merely
secondary to this basic question. There were many other Filipinos in the United
States similarly situated as Frivaldo, and some of
The reason for this inquiry is the provision in them subject to greater risk than he, who did not
Article XI, Section 9, of the Constitution that all find it necessary — nor do they claim to have
public officials and employees owe the State and been coerced — to abandon their cherished
the Constitution "allegiance at all times" and the status as Filipinos. They did not take the oath of
specific requirement in Section 42 of the Local allegiance to the United States, unlike the
Government Code that a candidate for local petitioner who solemnly declared "on oath, that I
elective office must be inter alia a citizen of the absolutely and entirely renounce and abjure all
Philippines and a qualified voter of the allegiance and fidelity to any foreign prince,
constituency where he is running. Section 117 of potentate, state or sovereignty of whom or which

19 | C O N S T I 2 _ A r t i c l e I V _ C I T I Z E N S H I P
I have heretofore been a subject or citizen," Philippine citizenship may be reacquired by direct
meaning in his case the Republic of the act of Congress, by naturalization, or by
Philippines. The martyred Ninoy Aquino heads the repatriation.
impressive list of those Filipinos in exile who,
unlike the petitioner, held fast to their Philippine While Frivaldo does not invoke either of the first
citizenship despite the perils of their resistance to two methods, he nevertheless claims he has
the Marcos regime. reacquired Philippine citizenship by virtue of a
valid repatriation. He claims that by actively
participating in the elections in this country, he
automatically forfeited American citizenship
The Nottebohm case cited by the petitioner under the laws of the United States. Such laws do
invoked the international law principle of effective not concern us here. The alleged forfeiture is
nationality which is clearly not applicable to the between him and the United States as his
case at bar. This principle is expressed in Article 5 adopted country. It should be obvious that even if
of the Hague Convention of 1930 on the Conflict he did lose his naturalized American citizenship,
of Nationality Laws as follows: prcd such forfeiture did not and could not have the
effect of automatically restoring his citizenship in
Art. 5. Within a third State a person having more the Philippines that he had earlier renounced. At
than one nationality shall be treated as if he had best, what might have happened as a result of
only one. Without prejudice to the application of the loss of his naturalized citizenship was that he
its law in matters of personal status and of any became a stateless individual.
convention in force, a third State shall, of the
nationalities which any such person possesses, Frivaldo's contention that he could not have
recognize exclusively in its territory either the repatriated himself under LOI 270 because the
nationality of the country in which he is habitually Special Committee provided for therein had not
and principally resident or the nationality of the yet been constituted seems to suggest that the
country with which in the circumstances he lack of that body rendered his repatriation
appears to be in fact most closely connected. unnecessary. That is far-fetched if not specious.
Such a conclusion would open the floodgates, as
Nottebohm was a German by birth but a resident it were. It would allow all Filipinos who have
of Guatemala for 34 years when he applied for renounced this country to claim back their
and acquired naturalization in Liechtenstein one abandoned citizenship without formally rejecting
month before the outbreak of World War II. Many their adopted state and reaffirming their
members of his family and his business interests allegiance to the Philippines.
were in Germany. In 1943, Guatemala, which had
declared war on Germany, arrested Nottebohm It does not appear that Frivaldo has taken these
and confiscated all his properties on the ground categorical acts. He contends that by simply filing
that he was a German national. Liechtenstein his certificate of candidacy he had, without more,
thereupon filed suit on his behalf, as its citizen, already effectively recovered Philippine
against Guatemala. The International Court of citizenship. But that is hardly the formal
Justice held Nottebohm to be still a national of declaration the law envisions — surely, Philippine
Germany, with which he was more closely citizenship previously disowned is not that
connected than with Liechtenstein. cdphil cheaply recovered. If the Special Committee had
not yet been convened, what that meant simply
That case is not relevant to the petition before us was that the petitioner had to wait until this was
because it dealt with a conflict between the done, or seek naturalization by legislative or
nationality laws of two states as decided by a judicial proceedings.
third state. No third state is involved in the case
at bar; in fact, even the United States is not The argument that the petition filed with the
actively claiming Frivaldo as its national. The sole Commission on Elections should be dismissed for
question presented to us is whether or not tardiness is not well-taken. The herein private
Frivaldo is a citizen of the Philippines under our respondents are seeking to prevent Frivaldo from
own laws, regardless of other nationality laws. We continuing to discharge his office of governor
can decide this question alone as sovereign of because he is disqualified from doing so as a
our own territory, conformably to Section 1 of the foreigner. Qualifications for public office are
said Convention providing that "it is for each continuing requirements and must be possessed
State to determine under its law who are its not only at the time of appointment or election or
nationals." assumption of office but during the officer's entire
tenure. Once any of the required qualifications is
It is also worth noting that Nottebohm was lost, his title may be seasonably challenged. If,
invoking his naturalization in Liechtenstein say, a female legislator were to marry a foreigner
whereas in the present case Frivaldo is rejecting during her term and by her act or omission
his naturalization in the United States. acquires his nationality, would she have a right to
remain in office simply because the challenge to
If he really wanted to disavow his American her title may no longer be made within ten days
citizenship and reacquire Philippine citizenship, from her proclamation? It has been established,
the petitioner should have done so in accordance and not even denied, that the evidence of
with the laws of our country. Under CA No. 63 as Frivaldo's naturalization was discovered only
amended by CA No. 473 and PD No. 725,

20 | C O N S T I 2 _ A r t i c l e I V _ C I T I Z E N S H I P
eight months after his proclamation and his title
was challenged shortly thereafter. RAUL R. LEE, petitioner, vs. COMMISSION ON
ELECTIONS and JUAN G. FRIVALDO, respondents.
This Court will not permit the anomaly of a person
sitting as provincial governor in this country while [G.R. No. 105735. June 6, 1994.]
owing exclusive allegiance to another country.
The fact that he was elected by the people of RAUL R. LEE, petitioner, vs. COMMISSION ON
Sorsogon does not excuse this patent violation of ELECTIONS and JUAN G. FRIVALDO, respondents.
the salutary rule limiting public office and
employment only to the citizens of this country. The Solicitor General for petitioner in G.R. No.
The qualifications prescribed for elective office 104654.
cannot be erased by the electorate alone. The will
of the people as expressed through the ballot Yolando F. Lim counsel for private respondent.
cannot cure the vice of ineligibility, especially if
they mistakenly believed, as in this case, that the SYLLABUS
candidate was qualified. Obviously, this rule
requires strict application when the deficiency is 1. CONSTITUTIONAL LAW; REVISED
lack of citizenship. If a person seeks to serve in NATURALIZATION LAW; PROCEDURAL
the Republic of the Philippines, he must owe his REQUIREMENTS, JURISDICTIONAL; CASE AT BAR.
total loyalty to this country only, abjuring and — Private respondent, having opted to reacquire
renouncing all fealty and fidelity to any other Philippine citizenship thru naturalization under
state. LexLib the Revised Naturalization Law, is duty bound to
follow the procedure prescribed by the said law. It
It is true as the petitioner points out that the is not for an applicant to decide for himself and to
status of the natural-born citizen is favored by the select the requirement which he believes, even
Constitution and our laws, which is all the more sincerely, are applicable to his case and discard
reason why it should be treasured like a pearl of those which be believes are inconvenient or
great price. But once it is surrendered and merely of nuisance value. The law does not
renounced, the gift is gone and cannot be lightly distinguish between an applicant who was
restored. This country of ours, for all its formerly a Filipino citizen and one who was never
difficulties and limitations, is like a jealous and such a citizen. It does not provide a special
possessive mother. Once rejected, it is not quick procedure for the reacquisition of Philippine
to welcome back with eager arms its prodigal if citizenship by former Filipino citizens akin to the
repentant children. The returning renegade must repatriation of a woman who had lost her
show, by an express and unequivocal act, the Philippine citizenship by reason of her marriage to
renewal of his loyalty and love. an alien. The trial court never acquired
jurisdiction to hear the petition for naturalization
WHEREFORE, the petition is DISMISSED and of private respondent. The proceedings
petitioner JUAN G. FRIVALDO is hereby declared conducted, the decision rendered and the oath of
not a citizen of the Philippines and therefore allegiance taken therein, are null and void or
DISQUALIFIED from serving as Governor of the failure to comply with the publication and posting
Province of Sorsogon. Accordingly, he is ordered requirements under the Revised Naturalization
to vacate his office and surrender the same to the Law. Under Section 9 of the said law, both the
duly elected Vice-Governor of the said province petition for naturalization and the order setting it
once this decision becomes final and executory. for hearing must be published once a week for
The temporary restraining order dated March 9, three consecutive weeks in the Official Gazette
1989, is LIFTED. and a newspaper of general circulation.
Compliance therewith is jurisdictional (Po Yi Bo v.
SO ORDERED. Republic, 205 SCRA 400 [1992]). Moreover, the
publication and posting of the petition and the
Fernan (C .J .), Narvasa, Melencio-Herrera, Paras, order must be in its full text for the court to
Feliciano, Gancayco, Padilla, Bidin, Griño-Aquino, acquire jurisdiction (Sy v. Republic, 55 SCRA 724
Medialdea and Regalado, JJ ., concur. [1974]). The petition for naturalization lacks
Sarmiento, J ., took no part. several allegations required by Sections 2 and 6
of the Revised Naturalization Law, particularly: (1)
Cortes, J ., concurs in the result. that the petitioner is of good moral character; (2)
(Republic v. De La Rosa, G.R. No. 104654, that he resided continuously in the Philippines for
105715, 105735, June 06, 1994) at least ten years; (3) that he is able to speak and
write English and any one of the principal
EN BANC dialects; (4) that he will reside continuously in the
Philippines from the date of the filing of the
[G.R. No. 104654. June 6, 1994.] petition until his admission to Philippine
citizenship; and (5) that he has filed a declaration
REPUBLIC OF THE PHILIPPINES, petitioner, vs. of intention or if he is excused from said filing,
HON. ROSALIO G. DE LA ROSA, PRESIDING JUDGE the justification therefor. The absence of such
OF THE REGIONAL TRIAL COURT, BRANCH 28, allegations is fatal to the petition (Po Yi Bi v.
MANILA and JUAN G. FRIVALDO, respondents. Republic, 205 SCRA 400 [1992]). Likewise the
petition is not supported by the affidavit of at
[G.R. No. 105715. June 6, 1994.] least two credible persons who vouched for the

21 | C O N S T I 2 _ A r t i c l e I V _ C I T I Z E N S H I P
good moral character of private respondent as tenure; once any of the required qualification is
required by Section 7 of the Revised lost, his title may be seasonably challenged."
Naturalization Law. Private respondent also failed
to attach a copy of his certificate of arrival to the 6. ID.; ID.; ONLY FILIPINO CITIZENS CAN RUN AND
petition as required by Section 7 of the said law. BE ELECTED THERETO. — Petitioner's argument,
The proceedings of the trial court was marred by that to unseat him will frustrate the will of the
the following irregularities: (1) the hearing of the electorate, is untenable. Both the Local
petition was set ahead of the scheduled date of Government Code and the Constitution require
hearing, without a publication of the order that only Filipino citizens can run and be elected
advancing the date of hearing, and the petition to public office. We can only surmise that the
itself; (2) the petition was heard within six electorate, at the time they voted for private
months from the last publication of the petition; respondent, was of the mistaken belief that he
(3) petitioner was allowed to take his oath of had legally reacquired Filipino citizenship.
allegiance before the finality of the judgment;
and (4) petitioner took his oath of allegiance 7. ID.; ELECTIONS; WHERE THE CANDIDATE WHO
without observing the two-year waiting period. OBTAINED THE HIGHEST NUMBER OF VOTES IS
Private respondent is declared NOT a citizen of DISQUALIFIED, THE CANDIDATE WHO GARNERED
the Philippines and therefore DISQUALIFIED from THE SECOND HIGHEST NUMBER OF VOTES IS NOT
continuing to serve as GOVERNOR of the Province ENTITLED TO BE DECLARED WINNER. — Petitioner
of Sorsogon. He is ordered to VACATE his office in G.R. No. 105715, prays that the votes cast in
and to SURRENDER the same to the Vice- favor of private respondent be considered stray
Governor of the Province of Sorsogon once this and that he, being the candidate obtaining the
decision becomes final and executory. second highest number of votes, be declared
winner. In Labo, Jr. v. COMELEC , 176 SCRA 1
2. ID.; ID.; DECISION THEREON BECOMES FINAL (1989), we ruled that where the candidate who
ONLY AFTER THIRTY (30) DAYS FROM obtained the highest number of votes is later
PROMULGATION. — A decision in a petition for declared to be disqualified to hold the office to
naturalization becomes final only after 30 days which he was elected, the candidate who
from its promulgation and, insofar as the Solicitor garnered the second highest number of votes is
General is concerned, that period is counted from not entitled to be declared winner (See also
the date of his receipt of the copy of the decision Geronimo v. Ramos, 136 SCRA 435 [1985];
(Republic v. Court of First Instance of Albay, 60 Topacio v. Paredes, 23 Phil. 238 [1912]).
SCRA 195 [1974]).
DECISION
3. ID.; ID.; DECISION GRANTING NATURALIZATION
SHALL BE EXECUTORY AFTER TWO (2) FROM QUIASON, J p:
PROMULGATION; REASON. — Section 1 of R.A.
NO. 530 provides that no decision granting In Frivaldo v. Commission on Elections, 174 SCRA
citizenship in naturalization proceedings shall be 245 (1989), this Court declared private
executory until after two years from its respondent, Juan G. Frivaldo, an alien and
promulgation in order to be able to observe if: (1) therefore disqualified from serving as Governor of
the applicant has left the country; (2) the the Province of the Sorsogon.
applicant has dedicated himself continously to a
lawful calling or profession; (3) the applicant has Once more, the citizenship of private respondent
not been convicted of any offense or violation of is put in issue in these petitions docketed as G.R.
government promulgated rules; and (4) the No. 104654, G.R. No. 105715 and G.R. No.
applicant has committed any act prejudicial to 105735. The petitions were consolidated since
the interest of the country or contrary to they principally involved the same issues and
government announced policies. parties. LibLex

4. REMEDIAL LAW; SPECIAL CIVIL ACTION; QUO I


WARRANTO; PETITION NOT COVERED BY THE TEN G.R. No. 104654
(10) DAY PERIOD FOR APPEAL PRESCRIBED IN This is a petition for certiorari under Rule 45 of
SECTION 253 OF THE OMNIBUS ELECTION CODE. the Revised Rules of Court in relation to R.A. No.
— In Frivaldo v. Commission on Elections, 174 5440 and Section 25 of the Interim Rules, filed by
SCRA 245 (1989), we held that a petition for quo the Republic of the Philippines: (1) to annul the
warranto, questioning the respondent's title and Decision dated February 27, 1992 of the Regional
seeking to prevent him from holding office as Trial Court, Branch 28, Manila, in SP Proc. No. 91-
Governor for alienage, is not covered by the ten- 58645, which re-admitted private respondent as a
day period for appeal prescribed in Section 253 of Filipino citizen under the Revised Naturalization
the Omnibus Election Code. Law (C.A. No. 63 as amended by C.A. No. 473);
and (2) to nullify the oath of allegiance taken by
5. POLITICAL LAW; PUBLIC OFFICE; private respondent on February 27, 1992.
QUALIFICATIONS THEREON ARE CONTINUING
REQUIREMENTS. — We explained that On September 20, 1991, petitioner filed a petition
"qualifications for public office are continuing for naturalization captioned: "In the Matter of
requirements and must be possessed not only at Petition of Juan G. Frivaldo to be Re-admitted as a
the time of appointment or election or Citizen of the Philippines under COMMONWEALTH
assumption of office but during the officer's entire ACT NO. 63" (Rollo, pp. 17-23).

22 | C O N S T I 2 _ A r t i c l e I V _ C I T I Z E N S H I P
On the same day, private respondent was allowed
In an Order dated October 7, 1991 respondent to take his oath of allegiance before respondent
Judge set the petition for hearing on March 16, Judge (Rollo, p. 34).
1992, and directed the publication of the said
order and petition in the Official Gazette and a On March 16, a "Motion for Leave of Court to
newspaper of general circulation, for three Intervene and to Admit Motion for
consecutive weeks, the last publication of which Reconsideration" was filed by Quiterio H. Hermo.
should be at least six months before the said date He alleged that the proceedings were tainted with
of hearing. The order further required the posting jurisdictional defects, and prayed for a new trial
of a copy thereof and the petition in a to conform with the requirements of the
conspicuous place in the Office of the Clerk of Naturalization Law.
Court of the Regional Trial Court, Manila (Rollo,
pp. 24- 26). After receiving a copy of the Decision on March
18, 1992, the Solicitor General interposed a
On January 14, 1992, private respondent filed a timely appeal directly with the Supreme Court.
"Motion to Set Hearing Ahead of Schedule,"
where he manifested his intention to run for G.R. No. 105715
public office in the May 1992 elections. He This is a petition for certiorari, mandamus with
alleged that the deadline for filing the certificate injunction under Rule 65 of the Revised Rules of
of candidacy was March 15, one day before the Court in relation to Section 5(2) of Article VIII of
scheduled hearing. He asked that the hearing set the Constitution with prayer for temporary
on March 16 be cancelled and be moved to restraining order filed by Raul R. Lee against the
January 24 (Rollo, pp. 27-28). Commission on Elections (COMELEC) and private
respondent, to annul the en banc Resolution of
The Motion was granted in an Order dated the COMELEC, which dismissed his petition
January 24, 1992, wherein the hearing of the docketed as SPC Case No. 92-273. The said
petition was moved to February 21, 1992. The petition sought to annul the proclamation of
said order was not published nor a copy thereof private respondent as Governor-elect of the
posted. cdrep Province of Sorsogon.

On February 21, the hearing proceeded with Petitioner was the official candidate of the Laban
private respondent as the sole witness. He ng Demokratikong Pilipino (LDP) for the position
submitted the following documentary evidence: of governor of the Province of Sorsogon in the
(1) Affidavit of Publication of the Order dated May 1992 elections. Private respondent was the
October 7, 1991 issued by the publisher of The official candidate of the Lakas-National Union of
Philippine Star (Exh. "A"); (2) Certificate of Christian Democrats (Lakas-NUCD) for the same
Publication of the order issued by the National position.
Printing Office (Exh. "B"); (3) Notice of Hearing of
Petition (Exh. "B-1"); (4) Photocopy of a Citation Private respondent was proclaimed winner on
issued by the National Press Club with private May 22, 1992.
respondent's picture (Exhs. "C" and "C-2"); (5)
Certificate of Appreciation issued by the Rotary On June 1, petitioner filed a petition with the
Club of Davao (Exh. "D"); (6) Photocopy of a COMELEC to annul the proclamation of private
Plaque of Appreciation issued by the Republican respondent as Governor-elect of the Province of
College, Quezon City (Exh. "E"); (7) Photocopy of Sorsogon on the grounds: (1) that the
a Plaque of Appreciation issued by the Davao- proceedings and composition of the Provincial
Bicol Association (Exh. "F"); (8) Certification Board of Canvassers were not in accordance with
issued by the Records Management and Archives law; (2) that private respondent is an alien,
Office that the record of birth of private whose grant of Philippine citizenship is being
respondent was not on file (Exh. "G"); and (9) questioned by the State in G.R. No. 104654; and
Certificate of Naturalization issued by the United (3) that private respondent is not a duly
States District Court (Exh. "H"). registered voter. Petitioner further prayed that
the votes cast in favor of private respondent be
considered as stray votes, and that he, on the
basis of the remaining valid votes cast, be
Six days later, on February 27, respondent Judge proclaimed winner. llcd
rendered the assailed Decision, disposing as
follows: On June 10, the COMELEC issued the questioned
en banc resolution which dismissed the petition
"WHEREFORE, the petition is GRANTED, Petitioner for having been filed out of time, citing Section
JUAN G. FRIVALDO, is re-admitted as a citizen of 19 of R.A. No. 7166. Said section provides that
the Republic of the Philippines by naturalization, the period to appeal a ruling of the board of
thereby vesting upon him, all the rights and canvassers on questions affecting its composition
privileges of a natural born Filipino citizen" (Rollo, or proceedings was three days.
p. 33).
In this petition, petitioner argues that the
COMELEC acted with grave abuse of discretion
when it ignored the fundamental issue of private

23 | C O N S T I 2 _ A r t i c l e I V _ C I T I Z E N S H I P
respondent's disqualification in the guise of and averred: (1) that Quiterio H. Hermo, not
technicality. being a candidate for the same office for which
private respondent was aspiring, had no standing
Petitioner claims that the inclusion of private to file the petition; (2) that the decision re-
respondent's name in the list of registered voters admitting him to Philippine citizenship was
in Sta. Magdalena, Sorsogon was invalid because presumed to be valid; and (3) that no case had
at the time he registered as a voter in 1987, he been filed to exclude his name as a registered
was an American citizen. voter. LLjur

Petitioner further claims that the grant of Filipino Raul R. Lee intervened in the petition for
citizenship to private respondent is not yet cancellation of private respondent's certificate of
conclusive because the case is still on appeal candidacy (Rollo, p. 37).
before us.
On May 13, 1992, said intervenor urged the
Petitioner prays for: (1) the annulment of private COMELEC to decide the petition for cancellation,
respondent's proclamation as Governor of the citing Section 78 of the Omnibus Election Code,
Province of Sorsogon; (2) the deletion of private which provides that all petitions on matters
respondent's name from the list of candidates for involving the cancellation of a certificate of
the position of governor; (3) the proclamation of candidacy must be decided "not later than fifteen
the governor-elect based on the remaining votes, days before election," and the case of Alonto v.
after the exclusion of the votes for private Commission on Elections, 22 SCRA 878 (1968),
respondent; (4) the issuance of a temporary which ruled that all pre-proclamation
restraining order to enjoin private respondent controversies should be summarily decided
from taking his oath and assuming office; and (5) (Rollo, p. 50).
the issuance of a writ of mandamus to compel
the COMELEC to resolve the pending The COMELEC concedes that private respondent
disqualification case docketed as SPA Case No. has not yet reacquired his Filipino citizenship
92-016, against private respondent. LLphil because the decision granting him the same is
not yet final and executory (Rollo, p. 63).
G.R. No. 105735 However, it submits that the issue of
This is a petition for mandamus under Rule 65 of disqualification of a candidate is not among the
the Revised Rules of Court in relation to Section grounds allowed in a pre-proclamation
5(2) of Article VIII of the Constitution, with prayer controversy, like SPC Case No. 92-273. Moreover,
for temporary restraining order. The parties the said petition was filed out of time.
herein are identical with the parties in G.R. No.
105715. The COMELEC contends that the preparation for
the elections occupied much of its time, thus its
failure to immediately resolve SPA Case No. 92-
In substance, petitioner prays for the COMELEC's 016. It argues that under Section 5 of Rule 25 of
immediate resolution of SPA Case No. 92-016, the COMELEC Rules of Procedure, it is excused
which is a petition for the cancellation of private from deciding a disqualification case within the
respondent's certificate of candidacy filed on period provided by law for reasons beyond its
March 23, 1992 by Quiterio H. Hermo, the control. It also assumed that the same action was
intervenor in G.R. No. 104654 (Rollo, p. 18). subsequently abandoned by petitioner when he
filed before it a petition for quo warranto
The petition for cancellation alleged: (1) that docketed as EPC No. 92-35. The quo warranto
private respondent is an American citizen, and proceedings sought private respondent's
therefore ineligible to run as candidate for the disqualification because of his American
position of governor of the Province of Sorsogon; citizenship. LLjur
(2) that the trial court's decision re-admitting
private respondent as a Filipino citizen was II
fraught with legal infirmities rendering it null and G.R. No. 104654
void; (3) that assuming the decision to be valid, We shall first resolve the issue concerning private
private respondent's oath of allegiance, which respondent's citizenship.
was taken on the same day the questioned
decision was promulgated, violated Republic Act In his comment to the State's appeal of the
No. 530, which provides for a two-year waiting decision granting him Philippine citizenship in
period before the oath of allegiance can be taken G.R. No. 104654, private respondent alleges that
by the applicant; and (4) that the hearing of the the precarious political atmosphere in the country
petition on February 27, 1992, was held less than during Martial Law compelled him to seek
four months from the date of the last publication political asylum in the United States, and
of the order and petition. The petition prayed for eventually to renounce his Philippine citizenship.
the cancellation of private respondent's
certificate of candidacy and the deletion of his He claims that his petition for naturalization was
name from the list of registered voters in Sta. his only available remedy for his reacquisition of
Magdalena, Sorsogon. Philippine citizenship. He tried to reacquire his
Philippine citizenship through repatriation and
In his answer to the petition for cancellation, direct act of Congress. However, he was later
private respondent denied the allegations therein informed that repatriation proceedings were

24 | C O N S T I 2 _ A r t i c l e I V _ C I T I Z E N S H I P
limited to army deserters or Filipino women who taken therein, are null and void for failure to
had lost their citizenship by reason of their comply with the publication and posting
marriage to foreigners (Rollo, pp. 49-50). His requirements under the Revised Naturalization
request to Congress for sponsorship of a bill Law.
allowing him to reacquire his Philippine
citizenship failed to materialize, notwithstanding Under Section 9 of the said law, both the petition
the endorsement of several members of the for naturalization and the order setting it for
House of Representatives in his favor (Rollo, p. hearing must be published once a week for three
51). He attributed this to the maneuvers of his consecutive weeks in the Official Gazette and a
political rivals. newspaper of general circulation. Compliance
therewith is jurisdictional (Po Yi Bo v. Republic,
He also claims that the re-scheduling of the 205 SCRA 400 [1992]). Moreover, the publication
hearing of the petition to an earlier date, without and posting of the petition and the order must be
publication, was made without objection from the in its full text for the court to acquire jurisdiction
Office of the Solicitor General. He makes mention (Sy v. Republic, 55 SCRA 724 [1974]).
that on the date of the hearing, the court was
jampacked. LLphil The petition for naturalization lacks several
allegations required by Sections 2 and 6 of the
It is private respondent's posture that there was Revised Naturalization Law, particularly: (1) that
substantial compliance with the law and that the the petitioner is of good moral character; (2) that
public was well-informed of his petition for he resided continuously in the Philippines for at
naturalization due to the publicity given by the least ten years; (3) that he is able to speak and
media. write English and any one of the principal
dialects; (4) that he will reside continuously in the
Anent the issue of the mandatory two-year Philippines from the date of the filing of the
waiting period prior to the taking of the oath of petition until his admission to Philippine
allegiance, private respondent theorizes that the citizenship; and (5) that he has filed a declaration
rationale of the law imposing the waiting period is of intention or if he is excused from said filing,
to grant the public an opportunity to investigate the justification therefor.
the background of the applicant and to oppose
the grant of Philippine citizenship if there is basis The absence of such allegations is fatal to the
to do so. In his case, private respondent alleges petition (Po Yi Bi v. Republic, 205 SCRA 400
that such requirement may be dispensed with, [1992]).
claiming that his life, both private and public, was
well-known. Private respondent cites his Likewise the petition is not supported by the
achievements as a freedom fighter and a former affidavit of at least two credible persons who
Governor of the Province of Sorsogon for six vouched for the good moral character of private
terms. respondent as required by Section 7 of the
Revised Naturalization Law. Private respondent
The appeal of the Solicitor General in behalf of also failed to attach a copy of his certificate of
the Republic of the Philippines is meritorious. The arrival to the petition as required by Section 7 of
naturalization proceedings in SP Proc. No. 91- the said law. LLphil
58645 was full of procedural flaws, rendering the
decision an anomaly. LLphil The proceedings of the trial court was marred by
the following irregularities: (1) the hearing of the
petition was set ahead of the scheduled date of
hearing, without a publication of the order
Private respondent, having opted to reacquire advancing the date of hearing, and the petition
Philippine citizenship thru naturalization under itself; (2) the petition was heard within six
the Revised Naturalization Law, is duty bound to months from the last publication of the petition;
follow the procedure prescribed by the said law. It (3) petitioner was allowed to take his oath of
is not for an applicant to decide for himself and to allegiance before the finality of the judgment;
select the requirements which he believes, even and (4) petitioner took his oath of allegiance
sincerely, are applicable to his case and discard without observing the two-year waiting period.
those which be believes are inconvenient or
merely of nuisance value. The law does not A decision in a petition for naturalization becomes
distinguish between an applicant who was final only after 30 days from its promulgation
formerly a Filipino citizen and one who was never and, insofar as the Solicitor General is concerned,
such a citizen. It does not provide a special that period is counted from the date of his receipt
procedure for the reacquisition of Philippine of the copy of the decision (Republic v. Court of
citizenship by former Filipino citizens akin to the First Instance of Albay, 60 SCRA 195 [1974]).
repatriation of a woman who had lost her
Philippine citizenship by reason of her marriage to Section 1 of R.A. NO. 530 provides that no
an alien. decision granting citizenship in naturalization
proceedings shall be executory until after two
The trial court never acquired jurisdiction to hear years from its promulgation in order to be able to
the petition for naturalization of private observe if: (1) the applicant has left the country;
respondent. The proceedings conducted, the (2) the applicant has dedicated himself
decision rendered and the oath of allegiance continously to a lawful calling or profession; (3)

25 | C O N S T I 2 _ A r t i c l e I V _ C I T I Z E N S H I P
the applicant has not been convicted of any votes is later declared to be disqualified to hold
offense or violation of government promulgated the office to which he was elected, the candidate
rules; and (4) the applicant has committed any who garnered the second highest number of
act prejudicial to the interest of the country or votes is not entitled to be declared winner (See
contrary to government announced policies. prcd also Geronimo v. Ramos, 136 SCRA 435 [1985];
Topacio v. Paredes, 23 Phil. 238 [1912]). prLL
Even discounting the provisions of R.A. NO. 530,
the courts cannot implement any decision G.R. No. 105735
granting the petition for naturalization before its In view of the discussions of G.R. No. 104654 and
finality. G.R. No. 105715, we find the petition in G.R. No.
105735 moot and academic.
G.R. No. 105715
In view of the finding in G.R. No. 104654 that WHEREFORE, the petitions in G.R. No. 104654
private respondent is not yet a Filipino citizen, we and G.R. No. 105715 are both GRANTED while the
have to grant the petition in G.R. No. 105715 petition in G.R. No. 105735 is DISMISSED. Private
after treating it as a petition for certiorari instead respondent is declared NOT a citizen of the
of a petition for mandamus. Said petition assails Philippines and therefore DISQUALIFIED from
the en banc resolution of the COMELEC, continuing to serve as GOVERNOR of the Province
dismissing SPC Case No. 92-273, which in turn is of Sorsogon. He is ordered to VACATE his office
a petition to annul private respondent's and to SURRENDER the same to the Vice-
proclamation on three grounds: 1) that the Governor of the Province of Sorsogon once this
proceedings and composition of the Provincial decision becomes final and executory. No
Board of Canvassers were not in accordance with pronouncement as to costs. cdll
law; 2) that private respondent is an alien, whose
grant of Filipino citizenship is being questioned by SO ORDERED.
the State in G.R. No. 104654; and 3) that private Feliciano, Padilla, Bidin, Regalado, Davide, Jr.,
respondent is not a duly registered voter. The Romero, Bellosillo, Melo, Puno, Vitug and
COMELEC dismissed the petition on the grounds Kapunan, JJ., concur.
that it was filed outside the three-day period for Narvasa, C .J ., took no part. Related to a party.
questioning the proceedings and composition of Cruz, J., took no part. Related to one of the
the Provincial Board of Canvassers under Section counsel in the proceedings before the COMELEC.
19 of R.A. No. 7166. prcd

The COMELEC failed to resolve the more serious (Labo, Jr. v. COMELEC, G.R. No. 86564,
issue — the disqualification of private respondent August 01, 1989)
to be proclaimed Governor on grounds of lack of
Filipino citizenship. In this aspect, the petition is EN BANC
one for quo warranto. In Frivaldo v. Commission
on Elections, 174 SCRA 245 (1989), we held that [G.R. No. 86564. August 1, 1989.]
a petition for quo warranto, questioning the
respondent's title and seeking to prevent him RAMON L. LABO, JR., petitioner, vs. THE
from holding office as Governor for alienage, is COMMISSION ON ELECTIONS (COMELEC) EN BANC
not covered by the ten-day period for appeal AND LUIS LARDIZABAL, respondents.
prescribed in Section 253 of the Omnibus Election
Code. Furthermore, we explained that Estelito P. Mendoza for petitioner.
"qualifications for public office are continuing
requirements and must be possessed not only at Rillera and Quintana for private respondent.
the time of appointment or election or
assumption of office but during the officer's entire SYLLABUS
tenure; once any of the required qualification is
lost, his title may be seasonably challenged." 1. SPECIAL CIVIL ACTION; QUO WARRANTO;
PETITION FILED TIMELY. — The Court has
Petitioner's argument, that to unseat him will considered the arguments of the parties and
frustrate the will of the electorate, is untenable. holds that the petition for quo warranto was filed
Both the Local Government Code and the on time. We agree with the respondents that the
Constitution require that only Filipino citizens can fee was paid during the ten-day period as
run and be elected to public office. We can only extended by the pendency of the petition when it
surmise that the electorate, at the time they was treated by the COMELEC as a pre-
voted for private respondent, was of the mistaken proclamation proceeding which did not require
belief that he had legally reacquired Filipino the payment of a filing fee. At that, we reach this
citizenship. conclusion only on the assumption that the
requirement for the payment of the fees in quo
Petitioner in G.R. No. 105715, prays that the warranto proceedings was already effective.
votes cast in favor of private respondent be There is no record that Res. No. 1450 was even
considered stray and that he, being the candidate published; and as for Res. No. 1996, this took
obtaining the second highest number of votes, be effect only on March 3, 1988, seven days after its
declared winner. In Labo, Jr. v. COMELEC , 176 publication in the February 25, 1988 issues of the
SCRA 1 (1989), we ruled that where the Manila Chronicle and the Philippine Daily Inquirer,
candidate who obtained the highest number of or after the petition was filed.

26 | C O N S T I 2 _ A r t i c l e I V _ C I T I Z E N S H I P
Allegiance and/or made the Affirmation of
2. ID.; ID.; PAYMENT OF FILING FEES NECESSARY Allegiance.
FOR CONFERMENT OF JURISDICTION; COURT MAY
ALLOW PAYMENT WITHIN A REASONABLE TIME. — 6. ID.; CITIZENSHIP; MODES OF LOSING
It is true that in the Manchester Case, we PHILIPPINE CITIZENSHIP. — CA No. 63 enumerates
required the timely payment of the filing fee as a the modes by which Philippine citizenship may be
precondition for the timeliness of the filing of the lost. Among these are: (1) naturalization in a
case itself. In Sun Insurance Office, Ltd. v. foreign country; (2) express renunciation of
Asuncion, however, this Court, taking into citizenship; and (3) subscribing to an oath of
account the special circumstances of that case, allegiance to support the Constitution or laws of a
reiterated the rule that the trial court acquires foreign country, all of which are applicable to the
jurisdiction over a case only upon the payment of petitioner. It is also worth mentioning in this
the prescribed filing fee. However, this court may connection that under Article IV, Section 5, of the
allow the payment of the said fee within a present Constitution, "Dual allegiance of citizens
reasonable time. In the event of non-compliance is inimical to the national interest and shall be
therewith, the case shall be dismissed. The same dealt with by law."
idea is expressed in Rule 42, Section 18, of the
COMELEC Rules of Procedure adopted on June 20, 7. ID.; ID.; ANNULMENT OF MARRIAGE TO A
1988. FOREIGNER NOT AN AUTOMATIC RESTORATION
OF PHILIPPINE CITIZENSHIP. — Even if it be
3. CIVIL PROCEDURE; ACTION; RESOLUTION ON assumed that, as the petitioner asserts, his
THE MERITS INSTEAD OF REMANDING THE CASE naturalization in Australia was annulled after it
TO THE TRIAL COURT FOR FURTHER was found that his marriage to the Australian
PROCEEDINGS; AS DEMANDED BY THE DICTATES citizen was bigamous, that circumstance alone
OF JUSTICE. — Remand of the case to the lower did not automatically restore his Philippine
court for further reception of evidence is not citizenship. His divestiture of Australian
necessary where the court is in a position to citizenship does not concern us here. That is a
resolve the dispute based on the records before matter between him and his adopted country.
it. On many occasions, the Court, in the public What we must consider is the fact that he
interest and the expeditious administration of voluntarily and freely rejected Philippine
justice, has resolved actions on the merits instead citizenship and willingly and knowingly embraced
of remanding them to the trial court for further the citizenship of a foreign country. The possibility
proceedings, such as where the ends of justice that he may have been subsequently rejected by
would not be subserved by the remand of the Australia, as he claims, does not mean that he
case or when public interest demands an early has been automatically reinstated as a citizen of
disposition of the case or where the trial court the Philippines.
had already received all the evidence of the
parties. 8. ID.; ID.; MEANS OF REACQUIRING PHILIPPINE
CITIZENSHIP. — Under CA No. 63 as amended by
4. ID.; ID.; DOCTRINE OF RES JUDICATA NOT PD No. 725, Philippine citizenship may be
APPLICABLE TO QUESTIONS OF CITIZENSHIP; reacquired by direct act of Congress, by
DEFENSE TO BE SEASONABLY INVOKED. — There naturalization, or by repatriation. It does not
is also the claim that the decision can no longer appear in the record, nor does the petitioner
be reversed because of the doctrine of res claim, that he has reacquired Philippine
judicata, but this too must be dismissed. This citizenship by any of these methods.
doctrine does not apply to questions of
citizenship, as the Court has ruled in several 9. ID.; ID.; LACK OF PHILIPPINE CITIZENSHIP ON
cases. Moreover, it does not appear that it was THE DAY OF LOCAL ELECTIONS; GROUND FOR
properly and seasonably pleaded, in a motion to DISQUALIFICATION AS A CANDIDATE FOR MAYOR.
dismiss or in the answer, having been invoked — The petitioner is not now, nor was he on the
only when the petitioner filed his reply to the day of the local elections on January 18, 1988, a
private respondent's comment. Besides, one of citizen of the Philippines. In fact, he was not even
the requisites of res judicata, to wit, identity of a qualified voter under the Constitution itself
parties, is not present in this case. because of his alienage. He was therefore
ineligible as a candidate for mayor of Baguio City
5. POLITICAL LAW; NATURALIZATION; under Section 42 of the Local Government Code.
ACQUISITION OF CITIZENSHIP BY
NATURALIZATION. — The petitioner's contention 10. ID.; ELECTION; QUALIFICATIONS OF
that his marriage to an Australian national in CANDIDATE FOR PUBLIC OFFICE, CONTENDING
1976 did not automatically divest him of REQUIREMENTS. — The probability that many of
Philippine citizenship is irrelevant. There is no those who voted for the petitioner may have
claim or finding that he automatically ceased to done so in the belief that he was qualified only
be a Filipino because of that marriage. He strengthens the conclusion that the results of the
became a citizen of Australia because he was election cannot nullify the qualifications for the
naturalized as such through a formal and positive office now held by him. These qualifications are
process, simplified in his case because he was continuing requirements; once any of them is lost
married to an Australian citizen. As a condition for during incumbency, title to the office itself is
such naturalization, he formally took the Oath of deemed forfeited. In the case at bar, the
citizenship and voting requirements were not

27 | C O N S T I 2 _ A r t i c l e I V _ C I T I Z E N S H I P
subsequently lost but were not possessed at all in Batasang Pambansa, regional, provincial, or city
the first place on the day of the election. The officer on the ground of ineligibility or of
petitioner was disqualified from running as mayor disloyalty to the Republic of the Philippines shall
and, although elected, is not now qualified to files sworn petition for quo warranto with the
serve as such. Commission within ten days after the
proclamation of the result of the election.
11. ID.; ELECTION; CANDIDATE OBTAINING THE
SECOND HIGHEST NUMBER OF VOTES; NOT
QUALIFIED TO REPLACE THE DISQUALIFIED
CANDIDATE; SANTOS RULING REVERSED. — The petitioner adds that the payment of the filing
Finally, there is the question of whether or not the fee is required under Rule 36, Section 5, of the
private respondent, who filed the quo warranto Procedural Rules of the COMELEC providing that
petition, can replace the petitioner as mayor. He —
cannot. The simple reason is that as he obtained Sec. 5. No petition for quo warranto shall be given
only the second highest number of votes in the due course without the payment of a filing fee in
election, he was obviously not the choice of the the amount of Three Hundred Pesos (P300.00)
people of Baguio City. Re-examining Santos v. and the legal research fee as required by law.
Commission on Election, 137 SCRA 740 the Court
finds, and so holds, that it should be reversed in and stresses that there is abundant jurisprudence
favor of the earlier case of Geronimo v. Ramos, holding that the payment of the filing fee is
which represents the more logical and democratic essential to the timeliness of the filing of the
rule. There the Court held it would be extremely petition itself. He cites many rulings of the Court
repugnant to the basic concept of the to this effect, specifically Manchester v. Court of
constitutionally guaranteed right to suffrage if a Appeals. 1
candidate who has not acquired the majority or For his part, the private respondent denies that
plurality of votes is proclaimed a winner and the filing fee was paid out of time. In fact, he
imposed as the representative of a constituency, says, it was filed ahead of time. His point is that
the majority of which have positively declared when he filed his "Petition for Quo Warranto with
through their ballots that they do not choose him. Prayer for Immediate Annulment of Proclamation
and Restraining Order or Injunction" on January
DECISION 26, 1988, the COMELEC treated it as a pre-
proclamation controversy and docketed it as SPC
CRUZ, J p: Case No. 88-288. No docket fee was collected
although it was offered. It was only on February 8,
The petitioner asks this Court to restrain the 1988, that the COMELEC decided to treat his
Commission on Elections from looking into the petition as solely for quo warranto and re-
question of his citizenship as a qualification for docketed it as EPC Case No. 88-19, serving him
his office as Mayor of Baguio City. The allegation notice on February 10, 1988. He immediately paid
that he is a foreigner, he says, is not the issue. the filing fee on that date.
The issue is whether or not the public respondent The private respondent argues further that during
has jurisdiction to conduct any inquiry into this the period when the COMELEC regarded his
matter, considering that the petition for quo petition as a pre-proclamation controversy, the
warranto against him was not filed on time. cdphil time for filing an election protest or quo warranto
It is noteworthy that this argument is based on proceeding was deemed suspended under
the alleged tardiness not of the petition itself but Section 248 of the Omnibus Election Code. 2 At
of the payment of the filing fee, which the any rate, he says, Rule 36, Section 5, of the
petitioner contends was an indispensable COMELEC Rules of Procedure cited by the
requirement. The fee is, curiously enough, all of petitioner, became effective only on November
P300.00 only. This brings to mind the popular 15, 1988, seven days after publication of the said
verse that for want of a horse the kingdom was Rules in the Official Gazette pursuant to Section
lost. Still, if it is shown that the petition was 4, Rule 44 thereof. 3 These rules could not
indeed filed beyond the reglementary' period, retroact to January 26,1988, when he filed his
there is no question that this petition must be petition with the COMELEC.
granted and the challenge abated. In his Reply, the petitioner argues that even if the
The petitioner's position is simple. He was Omnibus Election Code did not require it, the
proclaimed mayor-elect of Baguio City on January payment of filing fees was still necessary under
20, 1988. The petition for quo warranto was filed Res. No. 1996 and, before that, Res. No. 1450 of
by the private respondent on January 26,1988, the respondent COMELEC, promulgated on
but no filing fee was paid on that date. This fee January 12, 1988, and February 26, 1980,
was finally paid on February 10, 1988, or twenty- respectively. To this, the private respondent
one days after his proclamation. As the petition counters that the latter resolution was intended
by itself alone was ineffectual without the filing for the local elections held on January 30, 1980,
fee, it should be deemed filed only when the fee and did not apply to the 1988 local elections,
was paid. This was done beyond the which were supposed to be governed by the first-
reglementary period provided for under Section mentioned resolution. However, Res. No. 1996
253 of the Omnibus Election Code reading as took effect only on March 3, 1988, following the
follows: lapse of seven days alter its publication as
SEC. 253. Petition for quo warranto. — Any voter required by RA No. 6646, otherwise known as the
contesting the election of a Member of the Electoral Reform Law of 1987, which became

28 | C O N S T I 2 _ A r t i c l e I V _ C I T I Z E N S H I P
effective on January 5,1988. Its Section 30 an even less important consideration than the
provides in part: reglementary period the petitioner insists upon.
Sec. 30. Effectivity of Regulations and Orders of This matter should normally end here as the sole
the Commission. — The rules and regulations issue originally raised by the petitioner is the
promulgated by the Commission shall take effect timeliness of the quo warranto proceedings
on the seventh day after their publication in the against him. However, as his citizenship is the
Official Gazette or in at least (2) daily newspapers subject of that proceeding, and considering the
of general circulation in the Philippines. necessity for an early resolution of that more
important question clearly and urgently affecting
The Court has considered the arguments of the the public interest, we shall directly address it
parties and holds that the petition for quo now in this same action.
warranto was filed on time. We agree with the The Court has similarly acted in a notable number
respondents that the fee was paid during the ten- of cases, thus:
day period as extended by the pendency of the From the foregoing brief statement of the nature
petition when it was treated by the COMELEC as a of the instant case, it would appear that our sole
pre-proclamation proceeding which did not function in this proceeding should be to resolve
require the payment of a filing fee. At that, we the single issue of whether or not the Court of
reach this conclusion only on the assumption that Appeals erred in ruling that the motion for new
the requirement for the payment of the fees in trial of the GSIS in question should indeed be
quo warranto proceedings was already effective. deemed pro forma. But going over the extended
There is no record that Res. No. 1450 was even pleadings of both parties, the Court is
published; and as for Res. No. 1996, this took immediately impressed that substantial justice
effect only on March 3, 1988, seven days after its may not be timely achieved, if we should decide
publication in the February 25, 1988 issues of the this case upon such a technical ground alone. We
Manila Chronicle and the Philippine Daily Inquirer, have carefully read all the allegations and
or after the petition was filed. cdasia arguments of the parties, very ably and
The petitioner forgets Tañada v. Tuvera 4 when he comprehensively expounded by evidently
argues that the resolutions became effective knowledgeable and unusually competent counsel,
"immediately upon approval" simply because it and we feel we can better serve the interests of
was so provided therein. We held in that case that justice by broadening the scope of our inquiry, for
publication was still necessary under the due as the record before us stands, we see that there
process clause despite such effectivity clause. is enough basis for us to end the basic
In any event, what is important is that the filing controversy between the parties here and now,
fee was paid, and whatever delay there may have dispensing, however, with procedural steps which
been is not imputable to the private respondent's would not anyway affect substantially the merits
fault or neglect. It is true that in the Manchester of their respective claims. 6
Case, we required the timely payment of the
filing fee as a precondition for the timeliness of xxx xxx xxx
the filing of the case itself. In Sun Insurance
Office, Ltd. v. Asuncion, 5 however, this Court, While it is the fault of the petitioner for appealing
taking into account the special circumstances of to the wrong court and thereby allowing the
that case, declared: period for appeal to lapse, the more correct
This Court reiterates the rule that the trial court procedure was for the respondent court to
acquires jurisdiction over a case only upon the forward the case to the proper court which was
payment of the prescribed filing fee. However, the Court of Appeals for appropriate action.
the court may allow the payment of the said fee Considering, however, the length of time that this
within a reasonable time. In the event of non- case has been pending, we apply the rule in the
compliance therewith, the case shall be case of Del Castillo v. Jaymalin, (112 SCRA 629)
dismissed. and follow the principle enunciated in Alger
Electric, Inc. v. Court of Appeals, (135 SCRA 37)
The same idea is expressed in Rule 42, Section which states:
18, of the COMELEC Rules of Procedure adopted
on June 20, 1988, thus: ". . . it is a cherished rule of procedure for this
Sec. 18. Non-payment of prescribed fees. — If the Court to always strive to settle the entire
fees above prescribed are not paid, the controversy in a single proceeding leaving no root
Commission may refuse to take action thereon or branch to bear the seeds of future litigation.
until they are paid and may dismiss the action or No useful purpose will be served if this case is
the proceeding. (Emphasis supplied.) remanded to the trial court only to have its
decision raised again to the Intermediate
The Court notes that while arguing the technical Appellate Court and from there to this Court." (p.
point that the petition for quo warranto should be 43)
dismissed for failure to pay the filing fee on time,
the petitioner would at the same time minimize Only recently in the case of Beautifont, Inc., et al.
his alleged lack of citizenship as "a futile v. Court of Appeals, et al. (G.R. No. 50141,
technicality." It is regrettable, to say the least, January 29,1988), we stated that:
that the requirement of citizenship as a
qualification for public office can be so ". . . But all those relevant facts are now before
demeaned. what is worse is that it is regarded as this Court. And those facts dictate the rendition of
a verdict in the petitioner's favor. There is

29 | C O N S T I 2 _ A r t i c l e I V _ C I T I Z E N S H I P
therefore no point in referring the case back to The first decision was penned by then COMELEC
the Court of Appeals. The facts and the legal Chairman Vicente Santiago, Jr., with
propositions involved will not change, nor should Commissioners Pabalate, Savellano and Opinion
the ultimate judgment. Considerable time has concurring in full and Commissioner Bacungan
already elapsed and, to serve the ends of justice, concurring in the dismissal of the petition
it is time that the controversy is finally laid to "without prejudice to the issue of the
rest. (See Sotto v. Samson, 5 SCRA 733; Republic respondent's citizenship being raised a new in a
v. Paredes, 108 Phil. 57; Lianga Lumber Co. v. proper case. "Commissioner Sagadraca reserved
Lianga Timber Co., Inc., 76 SCRA 197; Erico v. his vote, while Commissioner Felipe was for
Heirs of Chigas, 98 SCRA 575; Francisco v. City of deferring decision until representations shall have
Davao, 12 SCRA 628; Valencia v. Mabilangan, 105 been made with the Australian Embassy for
Phil. 162). 'Sound practice seeks to accommodate official verification of the petitioner's alleged
the theory which avoids waste of time, effort and naturalization as an Australian.
expense, both to the parties and the government, The second decision was unanimously rendered
not to speak of delay in the disposal of the case by Chairman Miriam Defensor-Santiago and
(of: Fernandez v. Garcia, 92 Phil. 592, 597). A Commissioners Alano and Geraldez of the
marked characteristic of our judicial set-up is that Commission on Immigration and Deportation.
where the dictates of justice so demand . . . the prLL
Supreme Court should act, and act with finality.' It is important to observe that in the proceeding
(Li Siu Liat v. Republic, 21 SCRA 1039, 1046, before the COMELEC, there was no direct proof
citing Samal v. CA, 99 Phil. 230 and U.S. v. that the herein petitioner had been formally
Gimenez, 34 Phil. 74). In this case, the dictates of naturalized as a citizen of Australia. This
justice do demand that this Court act, and act conjecture, which was eventually rejected, was
with finality." 7 merely inferred from the fact that he had married
an Australian citizen, obtained an Australian
xxx xxx xxx passport, and registered as en alien with the CID
upon his return to this country in 1980.
Remand of the case to the lower court for further On the other hand, the decision of the CID took
reception of evidence is not necessary where the into account the official statement of the
court is in a position to resolve the dispute based Australian Government dated August 12, 1984,
on the records before it. On many occasions, the through its Consul in the Philippines, that the
Court, in the public interest and the expeditious petitioner was still an Australian citizen as of that
administration of justice, has resolved actions on date by reason of his naturalization in 1976. That
the merits instead of remanding them to the trial statement 12 is reproduced in full as follows:
court for further proceedings, such as where the I, GRAHAM COLIN WEST, Consul of Australia in the
ends of justice would not be subserved by the Philippines, by virtue of a certificate of
remand of the case or when public interest appointment signed and sealed by the Australian
demands an early disposition of the case or Minister of State for Foreign Affairs on 19 October
where the trial court had already received all the 1983, and recognized as such by Letter of Patent
evidence of the parties. 8 signed and sealed by the Philippines Acting
Minister of Foreign Affairs on 23 November 1983,
do hereby provide the following statement in
response to the Subpoena Testificandum dated 9
This course of action becomes all the more April 1984 in regard to the Petition for
justified in the present case where, to repeat for disqualification against RAMON LABO, JR. Y
stress, it is claimed that a foreigner is holding a LOZANO (SPC No. 84-73). and do hereby certify
public office. that the statement is true and correct.
We also note in his Reply, the petitioner says:
In adopting private respondent's comment, STATEMENT
respondent COMELEC implicitly adopted as "its A) RAMON LABO, JR. Y LOZANO, date of birth 23
own" private respondent's repeated assertion December 1934, was married in the Philippines to
that petitioner is no longer a Filipino citizen. In so an Australian citizen. As the spouse of an
doing, has not respondent COMELEC effectively Australian citizen, he was not required to meet
disqualified itself, by reason of prejudgment, from normal requirements for the grant of citizenship
resolving the petition for quo warranto filed by and was granted Australian citizenship by Sydney
private respondent still pending before it? 9 on 28 July 1976.

This is still another reason why the Court has B) Any person over the age of 16 years who is
seen fit to rule directly on the merits of this case. granted Australian citizenship must take an oath
Going over the record, we find that there are two of allegiance or make an affirmation of allegiance.
administrative decisions on the question of the The wording of the oath of affirmation is: "I. . . .,
petitioner's citizenship. The first was rendered by renouncing all other allegiance . . . .," etc. This
the Commission on Elections on May 12, 1982, need not necessarily have any effect on his
and found the petitioner to be a citizen of the former nationality as this would depend on the
Philippines. 10 The second was rendered by the citizenship laws of his former country.
Commission on Immigration and Deportation on
September 13, 1988, and held that the petitioner C) The marriage was declared void in the
was not a citizen of the Philippines. 11 Australian Federal Court in Sydney on 27 June

30 | C O N S T I 2 _ A r t i c l e I V _ C I T I Z E N S H I P
1980 on the ground that the marriage had been The decision also noted the oath of allegiance
bigamous. taken by every naturalized Australian reading as
follows:
D) According to our records LABO is still an OATH OF ALLEGIANCE
Australian citizen. I, A.B., renouncing all other allegiance, swear by
Almighty God that I will be faithful and bear true
E) Should he return to Australia, LABO may face allegiance to Her Majesty Elizabeth the Second,
court action in respect of Section 50 of Australian Queen of Australia, Her heirs and successors
Citizenship Act 1948 which relates to the giving of according to law, and that I will faithfully observe
false or misleading information of a material the laws of Australia and fulfill my duties as an
nature in respect of an application for Australian Australian citizen. 14
citizenship. If such a prosecution was successful, and the Affirmation of Allegiance, which declares:
he could be deprived of Australian citizenship AFFIRMATION OF ALLEGIANCE
under Section 21 of the Act. I, A.B., renouncing all other allegiance, solemnly
and sincerely promise and declare that I will be
F) There are two further ways in which LABO faithful and bear true allegiance to Her Majesty
could divest himself of Australian citizenship: Elizabeth the Second, Queen of Australia, Her
heirs and successors according to law, and that I
(i) He could make a declaration of Renunciation of will faithfully observe the Laws of Australia and
Australian citizenship under Section 18 of the fulfill my duties as an Australian citizen. 15
Australian Citizenship Act, or
The petitioner does not question the authenticity
(ii) If he acquired another nationality, (for of the above evidence. Neither does he deny that
example, Filipino) by a formal end voluntary act he obtained Australian Passport No. 754705,
other than marriage, then he would automatically which he used in coming back to the Philippines
lose his Australian citizenship under Section 17 of in 1980, when he declared before the
the Act. immigration authorities that he was an alien and
registered as such under Alien Certificate of
IN WITNESS WHEREOF, I HAVE HEREUNTO SET Registration No. B-323985. 16 He later asked for
MAY HAND AND SEAL OF THE AUSTRALIAN the change of his status from immigrant to a
EMBASSY, MANILA, THIS 12TH DAY OF APRIL returning former Philippine citizen and was
1984. granted Immigrant Certificate of Residence No.
223809. 17 He also categorically declared that he
DONE AT MANILA IN THE PHILIPPINES. was a citizen of Australia in a number of sworn
statements voluntarily made by him and. even
(Signed) sought to avoid the jurisdiction of the barangay
court on the ground that he was a foreigner. 18
GRAHAM C. WEST The decision of the COMELEC in 1982 quaintly
dismisses all these acts as "mistakes" that did not
Consul divest the petitioner of his citizenship, although,
as earlier noted, not all the member joined in this
This was affirmed later by the letter of February finding. We reject this ruling as totally baseless.
1, 1988, addressed to the private respondent by The petitioner is not an unlettered person who
the Department of Foreign Affairs reading as was not aware of the consequences of his acts,
follows: 13 let alone the fact that he was assisted by counsel
Sir: when he performed these acts.
The private respondent questions the motives of
With reference to your letter dated 1 February the COMELEC at that time and stresses Labo's
1988, I wish to inform you that inquiry made with political affiliation with the party in power then,
the Australian Government through the Embassy but we need not go into that now.
of the Philippines in Canberra has elicited the There is also the claim that the decision can no
following information: longer be reversed because of the doctrine of res
judicata, but this too must be dismissed. This
1) That Mr. Ramon L. Labo, Jr. acquired Australian doctrine does not apply to questions of
citizenship on 28 July 1976. citizenship, as the Court has ruled in several
cases. 19 Moreover, it does not appear that it was
2) That prior to 17 July 1986, a candidate for properly and seasonably pleaded, in a motion to
Australian citizenship had to either swear an oath dismiss or in the answer, having been invoked
of allegiance or make an affirmation of allegiance only when the petitioner filed his reply 20 to the
which carries a renunciation of "all other private respondent's comment. Besides, one of
allegiance." the requisites of res judicata, to wit, identity of
parties, is not present in this case.
Very truly yours, The petitioner's contention that his marriage to
For the Secretary of Foreign Affairs: an Australian national in 1976 did not
automatically divest him of Philippine citizenship
(SGD) RODOLFO SEVERINO, JR is irrelevant. There is no claim or finding that he
automatically ceased to be a Filipino because of
Assistant Secretary that marriage. He became a citizen of Australia
because he was naturalized as such through a

31 | C O N S T I 2 _ A r t i c l e I V _ C I T I Z E N S H I P
formal and positive process, simplified in his case cancellation of his alien certificate of registration.
because he was married to an Australian citizen. And that is also the reason we must deny his
As a condition for such naturalization, he formally present claim for recognition as a citizen of the
took the Oath of Allegiance and/or made the Philippines.
Affirmation of Allegiance, both quoted above. The petitioner is not now, nor was he on the day
Renouncing all other allegiance, he swore "to be of the local elections on January 18, 1988, a
faithful and bear true allegiance to Her Majesty citizen of the Philippines. In fact, he was not even
Elizabeth the Second, Queen of Australia. . . . , a qualified voter under the Constitution itself
and to fulfill his duties as an Australian citizen." because of his alienage. 21 He was therefore
cdll ineligible as a candidate for mayor of Baguio City
The petitioner now claims that his naturalization under Section 42 of the Local Government Code
in Australia made him at worst only a dual providing in material part as follows:
national and did not divest him of his Philippine Sec. 42. Qualifications. — (1) An elective local
citizenship. Such a specious argument cannot official must be a citizen of the Philippines, at
stand against the clear provisions of CA No. 63, least twenty-three years of age on election day, a
which enumerates the modes by which Philippine qualified voter registered as such in the
citizenship may be lost. Among these are: (1) barangay, municipality, city or province where he
naturalization in a foreign country; (2) express proposes to be elected, a resident therein for at
renunciation of citizenship; and (3) subscribing to least one year at the time of the filing of his
an oath of allegiance to support the Constitution certificate of candidacy, and able to read and
or laws of a foreign country, all of which are write English, Pilipino, or any other local language
applicable to the petitioner. It is also worth or dialect.
mentioning in this connection that under Article
IV, Section 5, of the present Constitution, "Dual The petitioner argues that his alleged lack of
allegiance of citizens is inimical to the national citizenship is a "futile technicality" that should
interest and shall be dealt with by law." not frustrate the will of the electorate of Baguio
City who elected him by a "resonant and
Even if it be assumed that, as the petitioner thunderous majority." To be accurate, it was not
asserts, his naturalization in Australia was as loud as all that, for his lead over the second-
annulled after it was found that his marriage to placer was only about 2,100 votes. In any event,
the Australian citizen was bigamous, that the people of that locality could not have, even
circumstance alone did not automatically restore unanimously, changed the requirements of the
his Philippine citizenship. His divestiture of Local Government Code and the Constitution. The
Australian citizenship does not concern us here. electorate had no power to permit a foreigner
That is a matter between him and his adopted owing his total allegiance to the Queen of
country. What we must consider is the fact that Australia, or at least a stateless individual owing
he voluntarily and freely rejected Philippine no allegiance to the Republic of the Philippines, to
citizenship and willingly and knowingly embraced preside over them as mayor of their city. Only
the citizenship of a foreign country. The possibility citizens of the Philippines have that privilege over
that he may have been subsequently rejected by their countrymen.
Australia, as he claims, does not mean that he The probability that many of those who voted for
has been automatically reinstated as a citizen of the petitioner may have done so in the belief that
the Philippines. he was qualified only strengthens the conclusion
Under CA No. 63 as amended by PD No. 725, that the results of the election cannot nullify the
Philippine citizenship may be reacquired by direct qualifications for the office now held by him.
act of Congress, by naturalization, or by These qualifications are continuing requirements;
repatriation. It does not appear in the record, nor once any of them is lost during incumbency, title
does the petitioner claim, that he has reacquired to the office itself is deemed forfeited. In the case
Philippine citizenship by any of these methods. at bar, the citizenship and voting requirements
He does not point to any judicial decree of were not subsequently lost but were not
naturalization as to any statute directly conferring possessed at all in the first place on the day of
Philippine citizenship upon him. Neither has he the election. The petitioner was disqualified from
shown that he has complied with PD No. 725, running as mayor and, although elected, is not
providing that: now qualified to serve as such. LLpr
. . . (2) natural-born Filipinos who have lost their Finally, there is the question of whether or not the
Philippine citizenship may reacquire Philippine private respondent, who filed the quo warranto
citizenship through repatriation by applying with petition, can replace the petitioner as mayor. He
the Special Committee on Naturalization created cannot. The simple reason is that as he obtained
by Letter of Instruction No. 270, and, if their only the second highest number of votes in the
applications are approved, taking the necessary election, he was obviously not the choice of the
oath of allegiance to the Republic of the people of Baguio City.
Philippines, after which they shall be deemed to The latest ruling of the Court on this issue is
have reacquired Philippine citizenship. The Santos v. Commission on Elections, 22 decided in
Commission on Immigration and Deportation shall 1985. In that case, the candidate who placed
thereupon cancel their certificate of registration. second was proclaimed elected after the votes for
(Emphasis supplied.) his winning rival, who was disqualified as a
turncoat and considered a non-candidate, were
That is why the Commission on Immigration and all disregarded as stray. In effect, the second
Deportation rejected his application for the placer won by default. That decision was

32 | C O N S T I 2 _ A r t i c l e I V _ C I T I Z E N S H I P
supported by eight members of the Court then, dedication to the country he has abjured and he
23 with three dissenting 24 and another two solemnly affirms once again his total and
reserving their vote.25 One was on official leave. exclusive loyalty to the Republic of the
26 Philippines. This may not be accomplished by
Re-examining that decision, the Court finds, and election to public office.
so holds, that it should be reversed in favor of the WHEREFORE, petitioner Ramon J. Labo, Jr. is
earlier case of Geronimo v. Ramos, 27 which hereby declared NOT a citizen of the Philippines
represents the more logical and democratic rule. and therefore DISQUALIFIED from continuing to
That case, which reiterated the doctrine first serve as Mayor of Baguio City. He is ordered to
announced in 1912 in Topacio vs. Paredes, 28 was VACATE his office and surrender the same to the
supported by ten members of the Court, 29 Vice-Mayor of Baguio City once this decision
without any dissent, although one reserved his becomes final and executory. The temporary
vote, 30 another took no part, 31 and two others restraining order dated January 31, 1989, is
were an leave. 32 There the Court held: LIFTED.
". . . it would be extremely repugnant to the basic Fernan, C . J ., Narvasa, Melencio-Herrera, Paras,
concept of the constitutionally guaranteed right Feliciano, Gancayco, Padilla, Bidin, Sarmiento,
to suffrage if a candidate who has not acquired Cortes, Griño-Aquino, Medialdea and Regalado,
the majority or plurality of votes is proclaimed a JJ ., concur.
winner and imposed as the representative of a
constituency, the majority of which have (Aznar v. COMELEC, G.R. No. 83820, May 25,
positively declared through their ballots that they 1990)
do not choose him.
EN BANC
Sound policy dictates that public elective offices
are filled by those who have received the highest [G.R. No. 83820. May 25, 1990.]
number of votes cast in the election for that
office, and it is a fundamental idea in all JOSE B. AZNAR (as Provincial Chairman of PDP
republican forms of government that no one can Laban in Cebu), petitioner, vs. COMMISSION ON
be declared elected and no measure can be ELECTIONS and EMILIO MARIO RENNER OSMEÑA,
declared carried unless he or it receives a respondents.
majority or plurality of the legal votes cast in the
election. (20 Corpus Juris 2nd, S 243, p. 676.) Rufino B. Requina for petitioner.

The fact that the candidate who obtained the Angara, Abello, Concepcion, Regala & Cruz for
highest number of votes is later declared to be private respondent.
disqualified or not eligible for the office to which
he was elected does not necessarily entitle the DECISION
candidate who obtained the second highest
number of votes to be declared the winner of the PARAS, J p:
elective office. The votes cast for a dead,
disqualified, or non-eligible person may not be Before Us is a petition for certiorari assailing the
valid to vote the winner into office or maintain Resolution of the Commission on Elections
him there. However, in the absence of a statute (COMELEC) dated June 11, 1988, which dismissed
which clearly asserts a contrary political and the petition for the disqualification of private
legislative policy on the matter, if the votes were respondent Emilio "Lito" Osmeña as candidate for
cast in the sincere belief that the candidate was Provincial Governor of Cebu Province. cdll
alive, qualified, or eligible, they should not be
treated as stray, void or meaningless. The facts of the case are briefly as follows:

It remains to stress that the citizen of the On November 19, 1987, private respondent
Philippines must take pride in his status as such Emilio "Lito" Osmeña filed his certificate of
and cherish this priceless gift that, out of more candidacy with the COMELEC for the position of
than a hundred other nationalities, God has seen Provincial Governor of Cebu Province in the
fit to grant him. Having been so endowed, he January 18, 1988 local elections.
must not lightly yield this precious advantage,
rejecting it for another land that may offer him On January 22, 1988, the Cebu PDP-Laban
material and other attractions that he may not Provincial Council (Cebu-PDP Laban, for short), as
find in his own country. To be sure, he has the represented by petitioner Jose B. Aznar in his
right to renounce the Philippines if he sees fit and capacity as its incumbent Provincial Chairman,
transfer his allegiance to a state with more filed with the COMELEC a petition for the
allurements for him. 33 But having done so, he disqualification of private respondent on the
cannot expect to be welcomed back with open ground that he is allegedly not a Filipino citizen,
arms once his taste for his adopted country turns being a citizen of the United States of America.
sour or he is himself disowned by it as an
undesirable alien. On January 27, 1988, petitioner filed a Formal
Philippine citizenship is not a cheap commodity Manifestation submitting a Certificate issued by
that can be easily recovered after its the then Immigration and Deportation
renunciation. It may be restored only after the Commissioner Miriam Defensor Santiago
returning renegade makes a formal act of re- certifying that private respondent is an American

33 | C O N S T I 2 _ A r t i c l e I V _ C I T I Z E N S H I P
and is a holder of Alien Certificate of Registration "(1) Before election, pursuant to Section 78
(ACR) No. B-21448 and Immigrant Certificate of thereof which provides that:
Residence (ICR) No. 133911, issued at Manila on
March 27 and 28, 1958, respectively. (Annex "B- 'Section 78. Petition to deny due course or to
1"). cancel a certificate of candidacy. — A verified
petition seeking to deny due course or to cancel a
The petitioner also filed a Supplemental Urgent certificate of candidacy may be filed by any
Ex-Parte Motion for the Issuance of a Temporary person exclusively on the ground that any
Restraining Order to temporarily enjoin the Cebu material representation contained therein as
Provincial Board of Canvassers from required under Section 74 hereof is false. The
tabulating/canvassing the votes cast in favor of petition may be filed at any time not later than
private respondent and proclaiming him until the twenty five days from the time of the filing of the
final resolution of the main petition. certificate of candidacy and shall be decided,
after the notice and hearing, not later than fifteen
Thus, on January 28, 1988, the COMELEC en banc days before the election.
resolved to order the Board to continue
canvassing but to suspend the proclamation. and

At the hearing before the COMELEC (First "(2) After election, pursuant to Section 253
Division), the petitioner presented the following thereof, viz:
exhibits tending to show that private respondent
is an American citizen: Application for Alien 'Sec. 253. Petition for quo warranto. — Any voter
Registration Form No. 1 of the Bureau of contesting the election of any Member of the
Immigration signed by private respondent dated Batasang Pambansa, regional, provincial, or city
November 21, 1979 (Exh. "B"); Alien Certificate of officer on the ground of inelligibility or of
Registration No. 015356 in the name of private disloyalty to the Republic of the Philippines shall
respondent dated November 21, 1979 (Exh. "C"); file a sworn petition for quo warranto with the
Permit to Re-enter the Philippines dated Commission within ten days after the
November 21, 1979 (Exh. "D"); Immigration proclamation of the results of the election."
Certificate of Clearance dated January 3, 1980
(Exh. "E"). (pp. 117-118, Rollo). The records show that private respondent filed
his certificate of candidacy on November 19,
Private respondent, on the other hand, 1987 and that the petitioner filed its petition for
maintained that he is a Filipino citizen, alleging: disqualification of said private respondent on
that he is the legitimate child of Dr. Emilio D. January 22, 1988. Since the petition for
Osmeña, a Filipino and son of the late President disqualification was filed beyond the twenty five-
Sergio Osmeña, Sr.; that he is a holder of a valid day period required in Section 78 of the Omnibus
and subsisting Philippine Passport No. 0855103 Election Code, it is clear that said petition was
issued on March 25, 1987; that he has been filed out of time.
continuously residing in the Philippines since birth
and has not gone out of the country for more The petition for the disqualification of private
than six months; and that he has been a respondent cannot also be treated as a petition
registered voter in the Philippines since 1965. for quo warranto under Section 253 of the same
(pp. 107-108, Rollo) Code as it is unquestionably premature,
considering that private respondent was
On March 3, 1988, COMELEC (First Division) proclaimed Provincial Governor of Cebu only on
directed the Board of Canvassers to proclaim the March 3, 1988.
winning candidates. Having obtained the highest
number of votes, private respondent was However, We deem it is a matter of public
proclaimed the Provincial Governor of Cebu. interest to ascertain the respondent's citizenship
and qualification to hold the public office to which
Thereafter, on June 11, 1988, COMELEC (First he has been proclaimed elected. There is enough
Division) dismissed the petition for basis for us to rule directly on the merits of the
disqualification for not having been timely filed case, as the COMELEC did below.
and for lack of sufficient proof that private
respondent is not a Filipino citizen. cdrep Petitioner's contention that private respondent is
not a Filipino citizen and, therefore, disqualified
Hence, the present petition. from running for and being elected to the office of
Provincial Governor of Cebu, is not supported by
The petition is not meritorious. substantial and convincing evidence.

There are two instances where a petition In the proceedings before the COMELEC, the
questioning the qualifications of a registered petitioner failed to present direct proof that
candidate to run for the office for which his private respondent had lost his Filipino citizenship
certificate of candidacy was filed can be raised by any of the modes provided for under C.A. No.
under the Omnibus Election Code (B.P. Blg. 881), 63. Among others, these are: (1) by naturalization
to wit: in a foreign country; (2) by express renunciation
of citizenship; and (3) by subscribing to an oath
of allegiance to support the Constitution or laws

34 | C O N S T I 2 _ A r t i c l e I V _ C I T I Z E N S H I P
of a foreign country. From the evidence, it is clear statements, Labo categorically declared that he
that private respondent Osmeña did not lose his was a citizen of Australia. LexLib
Philippine citizenship by any of the three
mentioned hereinabove or by any other mode of In declaring both Frivaldo and Labo not citizens of
losing Philippine citizenship. the Philippines, therefore, disqualified from
serving as Governor of the Province of Sorsogon
In concluding that private respondent had been and Mayor of Baguio City, respectively, the Court
naturalized as a citizen of the United States of considered the fact that by their own admissions,
America, the petitioner merely relied on the fact they are indubitably aliens, no longer owing any
that private respondent was issued alien allegiance to the Republic of the Philippines since
certificate of registration and was given clearance they have sworn their total allegiance to a foreign
and permit to re-enter the Philippines by the state.
Commission on Immigration and Deportation.
Petitioner assumed that because of the foregoing, In the instant case, private respondent
the respondent is an American and "being an vehemently denies having taken the oath of
American", private respondent "must have taken allegiance of the United States (p. 81, Rollo). He
and sworn to the Oath of Allegiance required by is a holder of a valid and subsisting Philippine
the U.S. Naturalization Laws." (p. 81, Rollo) passport and has continuously participated in the
electoral process in this country since 1963 up to
Philippine courts are only allowed to determine the present, both as a voter and as a candidate
who are Filipino citizens and who are not. (pp. 107-108, Rollo). Thus, private respondent
Whether or not a person is considered an remains a Filipino and the loss of his Philippine
American under the laws of the United States citizenship cannot be presumed.
does not concern Us here.

By virtue of his being the son of a Filipino father,


the presumption that private respondent is a In the learned dissent of Mr. Justice Teodoro
Filipino remains. It was incumbent upon the Padilla, he stresses the fact that because Osmeña
petitioner to prove that private respondent had obtained Certificates of Alien Registration as an
lost his Philippine citizenship. As earlier stated, American citizen, the first in 1958 when he was
however, the petitioner failed to positively 24 years old and the second in 1979, he, Osmeña
establish this fact. should be regarded as having expressly
renounced Philippine citizenship. To Our mind,
The cases of Juan Gallanosa Frivaldo v. COMELEC this is a case of non sequitur (It does not follow).
et al., (G.R. No. 87193, June 21, 1989) and Ramon Considering the fact that admittedly Osmeña was
L. Labo v. COMELEC et al. (G.R. No. 86564, August both a Filipino and an American, the mere fact
1, 1989) are not applicable to the case at bar. that he has a Certificate stating he is an
American does not mean that he is not still a
In the Frivaldo case, evidence shows that he was Filipino. Thus, by way of analogy, if a person who
naturalized as a citizen of the United States in has two brothers named Jose and Mario states or
1983 per certification from the United States certifies that he has a brother named Jose, this
District Court, Northern District of California, as does not mean that he does not have a brother
duly authenticated by Vice Consul Amado P. named Mario; or if a person is enrolled as student
Cortez of the Philippine Consulate General in San simultaneously in two universities, namely
Francisco, California, U.S.A. University X and University Y, presents a
Certification that he is a student of University X,
Frivaldo expressly admitted in his answer that he this does not necessarily mean that he is not still
was naturalized in the United States but claimed a student of University Y. In the case of Osmeña,
that he was forced to embrace American the Certification that he is an American does not
citizenship to protect himself from the mean that he is not still a Filipino, possessed as
persecution of the Marcos government. The he is, of both nationalities or citizenships. Indeed,
Court, however, found this suggestion of there is no express renunciation here of Philippine
involuntariness unacceptable, pointing out that citizenship; truth to tell, there is even no implied
there were many other Filipinos in the United renunciation of said citizenship. When We
States similarly situated as Frivaldo who did not consider that the renunciation needed to lose
find it necessary to abandon their status as Philippine citizenship must be "express", it stands
Filipinos. to reason that there can be no such loss of
Philippine citizenship when there is no
Likewise, in the case of Labo, records show that renunciation, either "express" or "implied ".
Labo was married to an Australian citizen and
that he was naturalized as an Australian citizen in Parenthetically, the statement in the 1987
1976, per certification from the Australian Constitution that "dual allegiance of citizens is
Government through its Consul in the Philippines. inimical to the national interest and shall be dealt
This was later affirmed by the Department of with by law" (Art. IV, Sec. 5) has no retroactive
Foreign Affairs. effect. And while it is true that even before the
1987 Constitution, Our country had already
The authenticity of the above evidence was not frowned upon the concept of dual citizenship or
disputed by Labo. In fact, in a number of sworn allegiance, the fact is it actually existed. Be it
noted further that under the aforecited proviso,

35 | C O N S T I 2 _ A r t i c l e I V _ C I T I Z E N S H I P
the effect of such dual citizenship or allegiance canvassers proclaimed private respondent as vice
shall be dealt with by a future law. Said law has mayor. This petition sought the reversal of the
not yet been enacted. resolution of the COMELEC en banc and to
declare the private respondent disqualified to
WHEREFORE, the petition for certiorari is hereby hold the office of the vice mayor of Makati. cdasia
DISMISSED and the Resolution of the COMELEC is
hereby AFFIRMED. On the issue of whether the petitioner has
personality to bring this suit considering that he
SO ORDERED. was not the original party in the disqualification
case, the Supreme Court ruled that under Sec. 6
Narvasa, Bidin, Griño-Aquino, Medialdea, of R.A. No. 6646, otherwise known as the
Regalado and Feliciano, JJ., concur. Electoral Reforms Law of 1987, intervention may
be allowed in proceedings for disqualification
Fernan, C.J., took no part even after election if there has yet been no final
judgment rendered. As regards the issue of
Gutierrez, Jr., J., My stand in the cases of Willie Yu citizenship, the Court ruled that by filing a
v. Miriam Defensor Santiago, et al. (G.R. No. certificate of candidacy when he ran for his
83882, January 24, 1989) and Ramon Labo, Jr. v. present post, private respondent elected
Commission on Elections (G.R. 86564, August 2, Philippine citizenship and in effect renounced his
1989) is clear. I regret, however, that I cannot American citizenship.
participate in this case because one of the
principal counsels is my relative by affinity within SYLLABUS
the fourth civil degree.
Gancayco, J., is on official leave. 1. POLITICAL LAW; ELECTORAL REFORMS LAW OF
Cortes, J., concurs in the result. 1987 (R.A. No. 6646); INTERVENTION, ALLOWED
IN PROCEEDINGS FOR DISQUALIFICATION EVEN
(Mercado v. Manzano, G.R. No. 135083, May AFTER ELECTION IF THERE HAS BEEN NO FINAL
26, 1999) JUDGMENT RENDERED; CASE AT BAR. — Private
respondent argues that petitioner has neither
EN BANC legal interest in the matter in litigation nor an
interest to protect because he is "a defeated
[G.R. No. 135083. May 26, 1999.] candidate for the vice-mayoralty post of Makati
City [who] cannot be proclaimed as the Vice-
ERNESTO S. MERCADO, petitioner, vs. EDUARDO Mayor of Makati City even if the private
BARRIOS MANZANO and the COMMISSION ON respondent be ultimately disqualified by final and
ELECTIONS, respondents. executory judgment." The flaw in this argument is
it assumes that, at the time petitioner sought to
Balase, Tamase, Alampay Law Office for intervene in the proceedings before the
petitioner. COMELEC, there had already been a proclamation
of the results of the election for the vice
Siguion Reyna, Montecillo & Ongsiako for private mayoralty contest for Makati City, on the basis of
respondent. which petitioner came out only second to private
respondent. The fact, however, is that there had
SYNOPSIS been no proclamation at that time. Certainly,
petitioner had, and still has, an interest in ousting
Petitioner Mercado and private respondent private respondent from the race at the time he
Manzano were candidates for vice mayor of the sought to intervene. The rule in Labo vs.
City of Makati in the May 11, 1998 elections. The COMELEC, reiterated in several cases, only
proclamation of private respondent was applies to cases in which the election of the
suspended in view of a pending petition for respondent is contested, and the question is
disqualification filed by a certain Ernesto Mamaril whether one who placed second to the
who alleged that private respondent was not a disqualified candidate may be declared the
citizen of the Philippines but of the United States. winner. In the present case, at the time petitioner
The Second Division of the COMELEC granted the filed a "Motion for leave to File Intervention" on
petition of Mamaril and ordered the cancellation May 20, 1998, there had been no proclamation of
of the certificate of candidacy of private the winner, and petitioner's purpose was
respondent on the ground that he is a dual citizen precisely to have private respondent disqualified
and under Sec. 40 of the Local Government Code, "from running for [an] elective local position"
persons with dual citizenship are disqualified from under Section 40(d) of R.A. No. 7160. If Ernesto
running for any elective position. Private Mamaril (who originally instituted the
respondent filed a motion for reconsideration. The disqualification proceedings), a registered voter
motion remained pending until after the election. of Makati City, was competent to bring the action,
The board of canvassers tabulated the votes but so was petitioner since the latter was a rival
suspended the proclamation of the winner. candidate for vice mayor of Makati City. Nor is
Petitioner sought to intervene in the case for petitioner's interest in the matter in litigation any
disqualification. COMELEC en banc reversed the less because he filed a motion for intervention
decision and declared private respondent only on May 20, 1998, after private respondent
qualified to run for the position. Pursuant to the had been shown to have garnered the highest
ruling of the COMELEC en banc, the board of number of votes among the candidates for vice

36 | C O N S T I 2 _ A r t i c l e I V _ C I T I Z E N S H I P
mayor. That petitioner had a right to intervene at origin even after their naturalization. Hence, the
that stage of the proceedings for the phrase "dual citizenship" in R.A. No. 7160, Section
disqualification against private respondent is 40(d) and in R.A. No. 7854, Section 20 must be
clear from Section 6 of R.A. No. 6646, otherwise understood as referring to "dual allegiance."
known as the Electoral Reforms Law of 1987, Consequently, persons with mere dual citizenship
which provides: Any candidate who has been do not fall under this disqualification. Unlike those
declared by final judgment to be disqualified shall with dual allegiance, who must, therefore, be
not be voted for, and the votes cast for him shall subject to strict process with respect to the
not be counted. If for any reason a candidate is termination of their status, for candidates with
not declared by final judgment before an election dual citizenship, it should suffice if, upon the filing
to be disqualified and he is voted for and receives of their certificates of candidacy, they elect
the winning number of votes in such election, the Philippine citizenship to terminate their status as
Court or Commission shall continue with the trial persons with dual citizenship considering that
and hearing of the action, inquiry, or protest and, their condition is the unavoidable consequence of
upon motion of the complainant or any conflicting laws of different states. As Joaquin G.
intervenor, may during the pendency thereof Bernas, one of the most perceptive members of
order the suspension of the proclamation of such the Constitutional Commission, pointed out:
candidate whenever the evidence of guilt is "[D]ual citizenship is just a reality imposed on us
strong. Under this provision, intervention may be because we have no control of the laws on
allowed in proceedings for disqualification even citizenship of other countries. We recognize a
after election if there has yet been no final child of a Filipino mother. But whether or not she
judgment rendered. is considered a citizen of another country is
something completely beyond our control." By
2. ID.; CITIZENSHIP; DUAL CITIZENSHIP; electing Philippine citizenship, such candidates at
DISTINGUISHED FROM DUAL ALLEGIANCE. — Dual the same time forswear allegiance to the other
citizenship is different from dual allegiance. The country of which they are also citizens and
former arises when, as a result of the concurrent thereby terminate their status as dual citizens. It
application of the different laws of two or more may be that, from the point of view of the foreign
states, a person is simultaneously considered a state and of its laws, such an individual has not
national by the said states. For instance, such a effectively renounced his foreign citizenship.
situation may arise when a person whose parents
are citizens of a state which adheres to the 4. ID.; ID.; FILING OF THE CERTIFICATE OF
principle of jus sanguinis is born in a state which CANDIDACY SUFFICED TO RENOUNCE AMERICAN
follows the doctrine of jus soli. Such a person, CITIZENSHIP; CASE AT BAR. — By filing a
ipso facto and without any voluntary act on his certificate of candidacy when he ran for his
part, is concurrently considered a citizen of both present post, private respondent elected
states. Considering the citizenship clause (Art. IV) Philippine citizenship and in effect renounced his
of our Constitution, it is possible for the following American citizenship. The filing of such certificate
classes of citizens of the Philippines to posses of candidacy sufficed to renounce his American
dual citizenship: (1) Those born of Filipino fathers citizenship, effectively removing any
and/or mothers in foreign countries which follow disqualification he might have as a dual citizen.
the principle of jus soli; (2) Those born in the Thus, in Frivaldo vs. COMELEC it was held: It is
Philippines of Filipino mothers and alien fathers if not disputed that on January 20, 1983 Frivaldo
by the laws of their fathers' country such children became an American. Would the retroactivity of
are citizens of that country; (3) Those who marry his repatriation not effectively give him dual
aliens if by the laws of the latter's country the citizenship, which under Sec. 40 of the Local
former are considered citizens, unless by their act Government Code would disqualify him "from
or omission they are deemed to have renounced running for any elective local position?" We
Philippine citizenship. There may be other answer this question in the negative, as there is
situations in which a citizen of the Philippines cogent reason to hold that Frivaldo was really
may, without performing any act, be also a STATELESS at the time he took said oath of
citizen of another state; but the above cases are allegiance and even before that, when he ran for
possible given the constitutional provisions on governor in 1988. In his Comment, Frivaldo wrote
citizenship. Dual allegiance, on the other hand, that he "had long renounced and had long
refers to the situation in which a person abandoned his American citizenship — long
simultaneously owes, by some positive act, before May 8, 1995. At best, Frivaldo was
loyalty to two or more states. While dual stateless in the interim — when he abandoned
citizenship is involuntary, dual allegiance is the and renounced his US citizenship but before he
result of an individual's volition. With respect to was repatriated to his Filipino citizenship." On this
dual allegiance, Article IV, Section 5 of the point, we quote from the assailed Resolution
Constitution provides: "Dual allegiance of citizens dated December 19, 1995: "By the laws of the
is inimical to the national interest and shall be United States, petitioner Frivaldo lost his
dealt with by law." American citizenship when he took his oath of
allegiance to the Philippine Government when he
3. ID.; ID.; ID.; ID.; RATIONALE. — In including ran for Governor in 1988, in 1992, and in 1995.
Section 5 in Article IV on citizenship, the concern Every certificate of candidacy contains an oath of
of the Constitutional Commission was not with allegiance to the Philippine Government." These
dual citizens per se but with naturalized citizens factual findings that Frivaldo has lost his foreign
who maintain their allegiance to their countries of nationality long before the elections of 1995 have

37 | C O N S T I 2 _ A r t i c l e I V _ C I T I Z E N S H I P
not been effectively rebutted by Lee. In its resolution, dated May 7, 1998, 2 the Second
Furthermore, it is basic that such findings of the Division of the COMELEC granted the petition of
Commission are conclusive upon this Court, Mamaril and ordered the cancellation of the
absent any showing of capriciousness or certificate of candidacy of private respondent on
arbitrariness or abuse. Until the filing of his the ground that he is a dual citizen and, under
certificate of candidacy on March 21, 1998, §40(d) of the Local Government Code, persons
private respondent had dual citizenship. The acts with dual citizenship are disqualified from running
attributed to him can be considered simply as the for any elective position. The COMELEC's Second
assertion of his American nationality before the Division said:
termination of his American citizenship. What this What is presented before the Commission is a
Court said in Aznar vs. COMELEC applies mutatis petition for disqualification of Eduardo Barrios
mutandis to private respondent in the case at Manzano as candidate for the office of Vice-Mayor
bar: . . . Considering the fact that admittedly of Makati City in the May 11, 1998 elections. The
Osmeña was both a Filipino and an American, the petition is based on the ground that the
mere fact that he has a Certificate stating he is respondent is an American citizen based on the
an American does not mean that he is not still a record of the Bureau of Immigration and
Filipino. . . [T]he Certification that he is an misrepresented himself as a natural-born Filipino
American does not mean that he is not still a citizen.
Filipino, possessed as he is, of both nationalities
or citizenships. Indeed, there is no express In his answer to the petition filed on April 27,
renunciation here of Philippine citizenship; truth 1998, the respondent admitted that he is
to tell, there is even no implied renunciation of registered as a foreigner with the Bureau of
said citizenship. When We consider that the Immigration under Alien Certificate of
renunciation needed to lose Philippine citizenship Registration No. B-31632 and alleged that he is a
must be "express," it stands to reason that there Filipino citizen because he was born in 1955 of a
can be no such loss of Philippine citizenship when Filipino father and a Filipino mother. He was born
there is no renunciation, either "express" or in the United States, San Francisco, California, on
"implied." To recapitulate, by declaring in his September 14, 1955, and is considered an
certificate of candidacy that he is a Filipino American citizen under US Laws. But
citizen; that he is not a permanent resident or notwithstanding his registration as an American
immigrant of another country; that he will defend citizen, he did not lose his Filipino citizenship.
and support the Constitution of the Philippines
and bear true faith and allegiance thereto and Judging from the foregoing facts, it would appear
that he does so without mental reservation, that respondent Manzano is both a Filipino and a
private respondent has, as far as the laws of this US citizen. In other words, he holds dual
country are concerned, effectively repudiated his citizenship.
American citizenship and anything which he may
have said before as a dual citizen. On the other The question presented is whether under our
hand, private respondent's oath of allegiance to laws, he is disqualified from the position for which
the Philippines, when considered with the fact he filed his certificate of candidacy. Is he eligible
that he has spent his youth and adulthood, for the office he seeks to be elected?
received his education, practiced his profession
as an artist, and taken part in past elections in Under Section 40(d) of the Local Government
this country, leaves no doubt of his election of Code, those holding dual citizenship are
Philippine citizenship. acCITS disqualified from running for any elective local
position.

WHEREFORE, the Commission hereby declares


DECISION the respondent Eduardo Barrios Manzano
DISQUALIFIED as candidate for Vice-Mayor of
MENDOZA, J p: Makati City.

Petitioner Ernesto S. Mercado and private On May 8, 1998, private respondent filed a
respondent Eduardo B. Manzano were candidates motion for reconsideration. 3 The motion
for vice mayor of the City of Makati in the May 11, remained pending even until after the election
1998 elections. The other one was Gabriel V. held on May 11, 1998.
Daza III. The results of the election were as
follows: Accordingly, pursuant to Omnibus Resolution No.
Eduardo B. Manzano 103,853 3044, dated May 10, 1998, of the COMELEC, the
board of canvassers tabulated the votes cast for
Ernesto S. Mercado 100,894 vice mayor of Makati City but suspended the
proclamation of the winner.
Gabriel V. Daza III 54,275 1
The proclamation of private respondent was On May 19, 1998, petitioner sought to intervene
suspended in view of a pending petition for in the case for disqualification. 4 Petitioner's
disqualification filed by a certain Ernesto Mamaril motion was opposed by private respondent.
who alleged that private respondent was not a
citizen of the Philippines but of the United States. The motion was not resolved. Instead, on August
31, 1998, the COMELEC en banc rendered its

38 | C O N S T I 2 _ A r t i c l e I V _ C I T I Z E N S H I P
resolution. Voting 4 to 1, with one commissioner respondent Eduardo Luis Barrios Manzano as the
abstaining, the COMELEC en banc reversed the winning candidate for vice-mayor of Makati City.
ruling of its Second Division and declared private
respondent qualified to run for vice mayor of the Pursuant to the resolution of the COMELEC en
City of Makati in the May 11, 1998 elections. 5 banc, the board of canvassers, on the evening of
The pertinent portions of the resolution of the August 31, 1998, proclaimed private respondent
COMELEC en banc read: as vice mayor of the City of Makati. cdasia
This is a petition for certiorari seeking to set aside
As aforesaid, respondent Eduardo Barrios the aforesaid resolution of the COMELEC en banc
Manzano was born in San Francisco, California, and to declare private respondent disqualified to
U.S.A. He acquired US citizenship by operation of hold the office of vice mayor of Makati City.
the United States Constitution and laws under the Petitioner contends that —
principle of jus soli. [T]he COMELEC en banc ERRED in holding that:

He was also a natural born Filipino citizen by A. Under Philippine law, Manzano was no longer a
operation of the 1935 Philippine Constitution, as U.S. citizen when he:
his father and mother were Filipinos at the time of
his birth. At the age of six (6), his parents brought 1. He renounced his U.S. citizenship when he
him to the Philippines using an American passport attained the age of majority when he was already
as travel document. His parents also registered 37 years old; and,
him as an alien with the Philippine Bureau of
Immigration. He was issued an alien certificate of 2. He renounced his U.S. citizenship when he
registration. This, however, did not result in the (merely) registered himself as a voter and voted
loss of his Philippine citizenship, as he did not in the elections of 1992, 1995 and 1998.
renounce Philippine citizenship and did not take
an oath of allegiance to the United States. B. Manzano is qualified to run for and or hold the
elective office of Vice-Mayor of the City of Makati;
It is an undisputed fact that when respondent
attained the age of majority, he registered C. At the time of the May 11, 1998 elections, the
himself as a voter, and voted in the elections of resolution of the Second Division adopted on 7
1992, 1995 and 1998, which effectively May 1998 was not yet final so that, effectively,
renounced his citizenship under American law. petitioner may not be declared the winner even
Under Philippine law, he no longer had U.S. assuming that Manzano is disqualified to run for
citizenship. and hold the elective office of Vice-Mayor of the
City of Makati.
At the time of the May 11, 1998 elections, the
resolution of the Second Division, adopted on May We first consider the threshold procedural issue
7, 1998, was not yet final. Respondent Manzano raised by private respondent Manzano — whether
obtained the highest number of votes among the petitioner Mercado has personality to bring this
candidates for vice-mayor of Makati City, suit considering that he was not an original party
garnering one hundred three thousand eight in the case for disqualification filed by Ernesto
hundred fifty-three (103,853) votes over his Mamaril nor was petitioner's motion for leave to
closest rival, Ernesto S. Mercado, who obtained intervene granted.
one hundred thousand eight hundred ninety-four I. PETITIONER'S RIGHT TO BRING THIS SUIT
(100,894) votes, or a margin of two thousand Private respondent cites the following provisions
nine hundred fifty-nine (2,959) votes. Gabriel of Rule 8 of the Rules of Procedure of the
Daza III obtained third place with fifty four COMELEC in support of his claim that petitioner
thousand two hundred seventy-five (54,275) has no right to intervene and, therefore, cannot
votes. In applying election laws, it would be far bring this suit to set aside the ruling denying his
better to err in favor of the popular choice than motion for intervention:
be embroiled in complex legal issues involving SECTION 1. When proper and when may be
private international law which may well be permitted to intervene. — Any person allowed to
settled before the highest court (Cf. Frivaldo vs. initiate an action or proceeding may, before or
Commission on Elections, 257 SCRA 727). during the trial of an action or proceeding, be
permitted by the Commission, in its discretion to
WHEREFORE, the Commission en banc hereby intervene in such action or proceeding, if he has
REVERSES the resolution of the Second Division, legal interest in the matter in litigation, or in the
adopted on May 7, 1998, ordering the success of either of the parties, or an interest
cancellation of the respondent's certificate of against both, or when he is so situated as to be
candidacy. adversely affected by such action or proceeding.

We declare respondent Eduardo Luis Barrios xxx xxx xxx


Manzano to be QUALIFIED as a candidate for the
position of vice-mayor of Makati City in the May SECTION 3. Discretion of Commission. — In
11, 1998, elections. allowing or disallowing a motion for intervention,
the Commission or the Division, in the exercise of
ACCORDINGLY, the Commission directs the Makati its discretion, shall consider whether or not the
City Board of Canvassers, upon proper notice to intervention will unduly delay or prejudice the
the parties, to reconvene and proclaim the adjudication of the rights of the original parties

39 | C O N S T I 2 _ A r t i c l e I V _ C I T I Z E N S H I P
and whether or not the intervenor's rights may be election if there has yet been no final judgment
fully protected in a separate action or proceeding. rendered.
The failure of the COMELEC en banc to resolve
Private respondent argues that petitioner has petitioner's motion for intervention was
neither legal interest in the matter in litigation tantamount to a denial of the motion, justifying
nor an interest to protect because he is "a petitioner in filing the instant petition for
defeated candidate for the vice-mayoralty post of certiorari. As the COMELEC en banc instead
Makati City [who] cannot be proclaimed as the decided the merits of the case, the present
Vice-Mayor of Makati City even if the private petition properly deals not only with the denial of
respondent be ultimately disqualified by final and petitioner's motion for intervention but also with
executory judgment." the substantive issues respecting private
The flaw in this argument is it assumes that, at respondent's alleged disqualification on the
the time petitioner sought to intervene in the ground of dual citizenship.
proceedings before the COMELEC, there had This brings us to the next question, namely,
already been a proclamation of the results of the whether private respondent Manzano possesses
election for the vice mayoralty contest for Makati dual citizenship and, if so, whether he is
City, on the basis of which petitioner came out disqualified from being a candidate for vice
only second to private respondent. The fact, mayor of Makati City.
however, is that there had been no proclamation II. DUAL CITIZENSHIP AS A GROUND FOR
at that time. Certainly, petitioner had, and still DISQUALIFICATION
has, an interest in ousting private respondent The disqualification of private respondent
from the race at the time he sought to intervene. Manzano is being sought under §40 of The Local
The rule in Labo v. COMELEC, 6 reiterated in Government Code of 1991 (R.A. No. 7160), which
several cases, 7 only applies to cases in which declares as "disqualified from running for any
the election of the respondent is contested, and elective local position: . . . (d) Those with dual
the question is whether one who placed second citizenship." This provision is incorporated in the
to the disqualified candidate may be declared the Charter of the City of Makati. 8
winner. In the present case, at the time petitioner Invoking the maxim dura lex sed lex, petitioner,
filed a "Motion for Leave to File Intervention" on as well as the Solicitor General, who sides with
May 20, 1998, there had been no proclamation of him in this case, contends that through §40(d) of
the winner, and petitioner's purpose was the Local Government Code, Congress has
precisely to have private respondent disqualified "command[ed] in explicit terms the ineligibility of
"from running for [an] elective local position" persons possessing dual allegiance to hold local
under §40(d) of R.A. No. 7160. If Ernesto Mamaril elective office."
(who originally instituted the disqualification To begin with, dual citizenship is different from
proceedings), a registered voter of Makati City, dual allegiance. The former arises when, as a
was competent to bring the action, so was result of the concurrent application of the
petitioner since the latter was a rival candidate different laws of two or more states, a person is
for vice mayor of Makati City. simultaneously considered a national by the said
states. 9 For instance, such a situation may arise
Nor is petitioner's interest in the matter in when a person whose parents are citizens of a
litigation any less because he filed a motion for state which adheres to the principle of jus
intervention only on May 20, 1998, after private sanguinis is born in a state which follows the
respondent had been shown to have garnered the doctrine of jus soli. Such a person, ipso facto and
highest number of votes among the candidates without any voluntary act on his part, is
for vice mayor. That petitioner had a right to concurrently considered a citizen of both states.
intervene at that stage of the proceedings for the Considering the citizenship clause (Art. IV) of our
disqualification against private respondent is Constitution, it is possible for the following
clear from §6 of R.A. No. 6646, otherwise known classes of citizens of the Philippines to possess
as the Electoral Reforms Law of 1987, which dual citizenship:
provides: (1) Those born of Filipino fathers and/or mothers
Any candidate who has been declared by final in foreign countries which follow the principle of
judgment to be disqualified shall not be voted for, jus soli;
and the votes cast for him shall not be counted. If
for any reason a candidate is not declared by final (2) Those born in the Philippines of Filipino
judgment before an election to be disqualified mothers and alien fathers if by the laws of their
and he is voted for and receives the winning fathers' country such children are citizens of that
number of votes in such election, the Court or country;
Commission shall continue with the trial and
hearing of the action, inquiry, or protest and, (3) Those who marry aliens if by the laws of the
upon motion of the complainant or any latter's country the former are considered
intervenor, may during the pendency thereof citizens, unless by their act or omission they are
order the suspension of the proclamation of such deemed to have renounced Philippine citizenship.
candidate whenever the evidence of guilt is
strong. There may be other situations in which a citizen
of the Philippines may, without performing any
Under this provision, intervention may be allowed act, be also a citizen of another state; but the
in proceedings for disqualification even after above cases are clearly possible given the
constitutional provisions on citizenship.

40 | C O N S T I 2 _ A r t i c l e I V _ C I T I Z E N S H I P
Dual allegiance, on the other hand, refers to the And so, this is exactly what we ask — that the
situation in which a person simultaneously owes, Committee kindly consider incorporating a new
by some positive act, loyalty to two or more section, probably Section 5, in the article on
states. While dual citizenship is involuntary, dual Citizenship which will read as follows: DUAL
allegiance is the result of an individual's volition. ALLEGIANCE IS INIMICAL TO CITIZENSHIP AND
With respect to dual allegiance, Article IV, §5 of SHALL BE DEALT WITH ACCORDING TO LAW.
the Constitution provides: "Dual allegiance of
citizens is inimical to the national interest and In another session of the Commission, Ople spoke
shall be dealt with by law." This provision was on the problem of these citizens with dual
included in the 1987 Constitution at the instance allegiance, thus: 11
of Commissioner Blas F. Ople who explained its . . . A significant number of Commissioners
necessity as follows: 10 expressed their concern about dual citizenship in
. . . I want to draw attention to the fact that dual the sense that it implies a double allegiance
allegiance is not dual citizenship. I have under a double sovereignty which some of us who
circulated a memorandum to the Bernas spoke then in a freewheeling debate thought
Committee according to which a dual allegiance would be repugnant to the sovereignty which
— and I reiterate a dual allegiance — is larger and pervades the Constitution and to citizenship itself
more threatening than that of mere double which implies a uniqueness and which elsewhere
citizenship which is seldom intentional and, in the Constitution is defined in terms of rights
perhaps, never insidious. That is often a function and obligations exclusive to that citizenship
of the accident of mixed marriages or of birth on including, of course, the obligation to rise to the
foreign soil. And so, I do not question double defense of the State when it is threatened, and
citizenship at all. back of this, Commissioner Bernas, is, of course,
the concern for national security. In the course of
What we would like the Committee to consider is those debates, I think some noted the fact that as
to take constitutional cognizance of the problem a result of the wave of naturalizations since the
of dual allegiance. For example, we all know what decision to establish diplomatic relations with the
happens in the triennial elections of the People's Republic of China was made in 1975, a
Federation of Filipino-Chinese Chambers of good number of these naturalized Filipinos still
Commerce which consists of about 600 chapters routinely go to Taipei every October 10; and it is
all over the country. There is a Peking ticket, as asserted that some of them do renew their oath
well as a Taipei ticket. Not widely known is the of allegiance to a foreign government maybe just
fact that the Filipino-Chinese community is to enter into the spirit of the occasion when the
represented in the Legislative Yuan of the anniversary of the Sun Yat-Sen Republic is
Republic of China in Taiwan. And until recently, commemorated. And so, I have detected a
the sponsor might recall, in Mainland China in the genuine and deep concern about double
People's Republic of China, they have the citizenship, with its attendant risk of double
Associated Legislative Council for overseas allegiance which is repugnant to our sovereignty
Chinese wherein all of Southeast Asia including and national security. I appreciate what the
some European and Latin countries were Committee said that this could be left to the
represented, which was dissolved after several determination of a future legislature. But
years because of diplomatic friction. At that time, considering the scale of the problem, the real
the Filipino-Chinese were also represented in that impact on the security of this country, arising
Overseas Council. from, let us say, potentially great numbers of
double citizens professing double allegiance, will
When I speak of double allegiance, therefore, I the Committee entertain a proposed amendment
speak of this unsettled kind of allegiance of at the proper time that will prohibit, in effect, or
Filipinos, of citizens who are already Filipinos but regulate double citizenship?
who, by their acts, may be said to be bound by a
second allegiance, either to Peking or Taiwan. I Clearly, in including §5 in Article IV on citizenship,
also took close note of the concern expressed by the concern of the Constitutional Commission was
some Commissioners yesterday, including not with dual citizens per se but with naturalized
Commissioner Villacorta, who were concerned citizens who maintain their allegiance to their
about the lack of guarantees of thorough countries of origin even after their naturalization.
assimilation, and especially Commissioner Hence, the phrase "dual citizenship" in R.A. No.
Concepcion who has always been worried about 7160, §40(d) and in R.A. No. 7854, §20 must be
minority claims on our natural resources. understood as referring to "dual allegiance."
Consequently, persons with mere dual citizenship
Dual allegiance can actually siphon scarce do not fall under this disqualification. Unlike those
national capital to Taiwan, Singapore, China or with dual allegiance, who must, therefore, be
Malaysia, and this is already happening. Some of subject to strict process with respect to the
the great commercial places in downtown Taipei termination of their status, for candidates with
are Filipino-owned, owned by Filipino-Chinese — it dual citizenship, it should suffice if, upon the filing
is of common knowledge in Manila. It can mean a of their certificates of candidacy, they elect
tragic capital outflow when we have to endure a Philippine citizenship to terminate their status as
capital famine which also means economic persons with dual citizenship considering that
stagnation, worsening unemployment and social their condition is the unavoidable consequence of
unrest. conflicting laws of different states. As Joaquin G.
Bernas, one of the most perceptive members of

41 | C O N S T I 2 _ A r t i c l e I V _ C I T I Z E N S H I P
the Constitutional Commission, pointed out: SENATOR ENRILE. But we are talking from the
"[D]ual citizenship is just a reality imposed on us viewpoint of Philippine law, Mr. President. He will
because we have no control of the laws on always have one citizenship, and that is the
citizenship of other countries. We recognize a citizenship invested upon him or her in the
child of a Filipino mother. But whether or not she Constitution of the Republic.
is considered a citizen of another country is
something completely beyond our control." 12 SENATOR PIMENTEL. That is true, Mr. President.
But if he exercises acts that will prove that he
By electing Philippine citizenship, such also acknowledges other citizenships, then he will
candidates at the same time forswear allegiance probably fall under this disqualification.
to the other country of which they are also
citizens and thereby terminate their status as This is similar to the requirement that an
dual citizens. It may be that, from the point of applicant for naturalization must renounce "all
view of the foreign state and of its laws, such an allegiance and fidelity to any foreign prince,
individual has not effectively renounced his potentate, state, or sovereignty" 14 of which at
foreign citizenship. That is of no moment as the the time he is a subject or citizen before he can
following discussion on §40(d) between Senators be issued a certificate of naturalization as a
Enrile and Pimentel clearly shows: 13 citizen of the Philippines. In Parado v. Republic, 15
SENATOR ENRILE. Mr. President, I would like to it was held:
ask clarification of line 41, page 17: "Any person [W]hen a person applying for citizenship by
with dual citizenship" is disqualified to run for any naturalization takes an oath that he renounces
elective local position. Under the present his loyalty to any other country or government
Constitution, Mr. President, someone whose and solemnly declares that he owes his
mother is a citizen of the Philippines but his allegiance to the Republic of the Philippines, the
father is a foreigner is a natural-born citizen of condition imposed by law is satisfied and
the Republic. There is no requirement that such a complied with. The determination whether such
natural born citizen, upon reaching the age of renunciation is valid or fully complies with the
majority, must elect or give up Philippine provisions of our Naturalization Law lies within
citizenship. the province and is an exclusive prerogative of
our courts. The latter should apply the law duly
On the assumption that this person would carry enacted by the legislative department of the
two passports, one belonging to the country of Republic. No foreign law may or should interfere
his or her father and one belonging to the with its operation and application. If the
Republic of the Philippines, may such a situation requirement of the Chinese Law of Nationality
disqualify the person to run for a local were to be read into our Naturalization Law, we
government position? would be applying not what our legislative
department has deemed it wise to require, but
SENATOR PIMENTEL. To my mind, Mr. President, it what a foreign government has thought or
only means that at the moment when he would intended to exact. That, of course, is absurd. It
want to run for public office, he has to repudiate must be resisted by all means and at all cost. It
one of his citizenships. would be a brazen encroachment upon the
sovereign will and power of the people of this
SENATOR ENRILE. Suppose he carries only a Republic.
Philippine passport but the country of origin or
the country of the father claims that person, III. PETITIONER'S ELECTION OF PHILIPPINE
nevertheless, as a citizen? No one can renounce. CITIZENSHIP
There are such countries in the world. The record shows that private respondent was
born in San Francisco, California on September 4,
SENATOR PIMENTEL. Well, the very fact that he is 1955, of Filipino parents. Since the Philippines
running for public office would, in effect, be an adheres to the principle of jus sanguinis, while
election for him of his desire to be considered as the United States follows the doctrine of jus soli,
a Filipino citizen. the parties agree that, at birth at least, he was a
national both of the Philippines and of the United
SENATOR ENRILE. But, precisely, Mr. President, States. However, the COMELEC en banc held that,
the Constitution does not require an election. by participating in Philippine elections in 1992,
Under the Constitution, a person whose mother is 1995, and 1998, private respondent "effectively
a citizen of the Philippines is, at birth, a citizen renounced his U.S. citizenship under American
without any overt act to claim the citizenship. law," so that now he is solely a Philippine
national.
SENATOR PIMENTEL. Yes. What we are saying, Mr. Petitioner challenges this ruling. He argues that
President, is: Under the Gentleman's example, if merely taking part in Philippine elections is not
he does not renounce his other citizenship, then sufficient evidence of renunciation and that, in
he is opening himself to question. So, if he is any event, as the alleged renunciation was made
really interested to run, the first thing he should when private respondent was already 37 years
do is to say in the Certificate of Candidacy that: "I old, it was ineffective as it should have been
am a Filipino citizen, and I have only one made when he reached the age of majority.
citizenship." In holding that by voting in Philippine elections
private respondent renounced his American
citizenship, the COMELEC must have in mind

42 | C O N S T I 2 _ A r t i c l e I V _ C I T I Z E N S H I P
§349 of the Immigration and Nationality Act of the On this point, we quote from the assailed
United States, which provided that "A person who Resolution dated December 19, 1995:
is a national of the United States, whether by
birth or naturalization, shall lose his nationality "By the laws of the United States, petitioner
by: . . . (e) Voting in a political election in a Frivaldo lost his American citizenship when he
foreign state or participating in an election or took his oath of allegiance to the Philippine
plebiscite to determine the sovereignty over Government when he ran for Governor in 1988, in
foreign territory." To be sure this provision was 1992, and in 1995. Every certificate of candidacy
declared unconstitutional by the U.S. Supreme contains an oath of allegiance to the Philippine
Court in Afroyim v. Rusk 16 as beyond the power Government."
given to the U.S. Congress to regulate foreign
relations. However, by filing a certificate of These factual findings that Frivaldo has lost his
candidacy when he ran for his present post, foreign nationality long before the elections of
private respondent elected Philippine citizenship 1995 have not been effectively rebutted by Lee.
and in effect renounced his American citizenship. Furthermore, it is basic that such findings of the
Private respondent's certificate of candidacy, filed Commission are conclusive upon this Court,
on March 27, 1998, contained the following absent any showing of capriciousness or
statements made under oath: arbitrariness or abuse.
6. I AM A FILIPINO CITIZEN (STATE IF "NATURAL-
BORN" OR "NATURALIZED") NATURAL-BORN There is, therefore, no merit in petitioner's
contention that the oath of allegiance contained
xxx xxx xxx in private respondent's certificate of candidacy is
insufficient to constitute renunciation of his
10. I AM A REGISTERED VOTER OF PRECINCT NO. American citizenship. Equally without merit is
747-A, BARANGAY SAN LORENZO, petitioner's contention that, to be effective, such
CITY/MUNICIPALITY OF MAKATI, PROVINCE OF renunciation should have been made upon
NCR. private respondent reaching the age of majority
since no law requires the election of Philippine
11. I AM NOT A PERMANENT RESIDENT OF, OR citizenship to be made upon majority age.
IMMIGRANT TO, A FOREIGN COUNTRY. Finally, much is made of the fact that private
respondent admitted that he is registered as an
12. I AM ELIGIBLE FOR THE OFFICE I SEEK TO BE American citizen in the Bureau of Immigration
ELECTED. I WILL SUPPORT AND DEFEND THE and Deportation and that he holds an American
CONSTITUTION OF THE PHILIPPINES AND WILL passport which he used in his last travel to the
MAINTAIN TRUE FAITH AND ALLEGIANCE United States on April 22, 1997. There is no merit
THERETO; THAT I WILL OBEY THE LAWS, LEGAL in this. Until the filing of his certificate of
ORDERS AND DECREES PROMULGATED BY THE candidacy on March 21, 1998, he had dual
DULY CONSTITUTED AUTHORITIES OF THE citizenship. The acts attributed to him can be
REPUBLIC OF THE PHILIPPINES; AND THAT I considered simply as the assertion of his
IMPOSE THIS OBLIGATION UPON MYSELF American nationality before the termination of his
VOLUNTARILY, WITHOUT MENTAL RESERVATION American citizenship. What this Court said in
OR PURPOSE OF EVASION. I HEREBY CERTIFY Aznar vs. COMELEC 18 applies mutatis mutandis
THAT THE FACTS STATED HEREIN ARE TRUE AND to private respondent in the case at bar:
CORRECT OF MY OWN PERSONAL KNOWLEDGE. . . . Considering the fact that admittedly Osmeña
was both a Filipino and an American, the mere
fact that he has a Certificate stating he is an
The filing of such certificate of candidacy sufficed American does not mean that he is not still a
to renounce his American citizenship, effectively Filipino. . . . [T]he Certification that he is an
removing any disqualification he might have as a American does not mean that he is not still a
dual citizen. Thus, in Frivaldo v. COMELEC it was Filipino, possessed as he is, of both nationalities
held: 17 or citizenships. Indeed, there is no express
It is not disputed that on January 20, 1983 renunciation here of Philippine citizenship; truth
Frivaldo became an American. Would the to tell, there is even no implied renunciation of
retroactivity of his repatriation not effectively said citizenship. When We consider that the
give him dual citizenship, which under Sec. 40 of renunciation needed to lose Philippine citizenship
the Local Government Code would disqualify him must be "express," it stands to reason that there
"from running for any elective local position?" We can be no such loss of Philippine citizenship when
answer this question in the negative, as there is there is no renunciation, either "express" or
cogent reason to hold that Frivaldo was really "implied."
STATELESS at the time he took said oath of
allegiance and even before that, when he ran for
governor in 1988. In his Comment, Frivaldo wrote
that he "had long renounced and had long To recapitulate, by declaring in his certificate of
abandoned his American citizenship-long before candidacy that he is a Filipino citizen; that he is
May 8, 1995. At best, Frivaldo was stateless in the not a permanent resident or immigrant of another
interim-when he abandoned and renounced his country; that he will defend and support the
US citizenship but before he was repatriated to Constitution of the Philippines and bear true faith
his Filipino citizenship." and allegiance thereto and that he does so
without mental reservation, private respondent

43 | C O N S T I 2 _ A r t i c l e I V _ C I T I Z E N S H I P
has, as far as the laws of this country are proof of his citizenship. In compliance therewith,
concerned, effectively repudiated his American on 27 July 1999, Ching filed a Manifestation with
citizenship and anything which he may have said attached Affidavit of Election of Philippine
before as a dual citizen. Citizenship and Oath of Allegiance dated 15 July
On the other hand, private respondent's oath of 1999. DSAEIT
allegiance to the Philippines, when considered
with the fact that he has spent his youth and The Court held that Ching failed to validly elect
adulthood, received his education, practiced his Philippine citizenship. The span of fourteen (14)
profession as an artist, and taken part in past years that lapsed from the time he reached the
elections in this country, leaves no doubt of his age of majority until he finally expressed his
election of Philippine citizenship. intention to elect Philippine citizenship was
His declarations will be taken upon the faith that clearly way beyond the contemplation of the
he will fulfill his undertaking made under oath. requirement of electing "upon reaching the age of
Should he betray that trust, there are enough majority." Moreover, Ching had offered no reason
sanctions for declaring the loss of his Philippine why he delayed his election of Philippine
citizenship through expatriation in appropriate citizenship. The prescribed procedure in electing
proceedings. In Yu v. Defensor-Santiago, 19 we Philippine citizenship is certainly not a tedious
sustained the denial of entry into the country of and painstaking process. All that is required of
petitioner on the ground that, after taking his the elector is to execute an affidavit of election of
oath as a naturalized citizen, he applied for the Philippine citizenship and, thereafter, file the
renewal of his Portuguese passport and declared same with the nearest civil registry. Ching's
in commercial documents executed abroad that unreasonable and unexplained delay in making
he was a Portuguese national. A similar sanction his election cannot be simply glossed over.
can be taken against any one who, in electing
Philippine citizenship, renounces his foreign SYLLABUS
nationality, but subsequently does some act
constituting renunciation of his Philippine 1. POLITICAL LAW; CONSTITUTIONAL LAW;
citizenship. cdasia CITIZENSHIP; CHILDREN WITH ALIEN FATHER AND
WHEREFORE, the petition for certiorari is FILIPINO MOTHER BORN BEFORE JANUARY 17,
DISMISSED for lack of merit. 1973 MUST ELECT THEIR CITIZENSHIP PURSUANT
SO ORDERED. TO 1935 Constitution. — When Ching was born in
Davide, Jr., C.J., Romero, Bellosillo, Melo, Puno, 1964, the governing charter was the 1935
Vitug, Kapunan, Quisumbing, Buena, Gonzaga- Constitution. Under Article IV, Section 1(3) of the
Reyes, and Ynares-Santiago, JJ., concur. 1935 Constitution, the citizenship of a legitimate
Panganiban and Purisima, JJ., are on leave. child born of a Filipino mother and an alien father
Pardo, J., took no part. followed the citizenship of the father, unless,
upon reaching the age of majority, the child
(Re: Vicente D. Ching, B.M. No. 914 elected Philippine citizenship. This right to elect
(Resolution), October 01, 1999) Philippine citizenship was recognized in the 1973
Constitution when it provided that "(t)hose who
EN BANC elect Philippine citizenship pursuant to the
[B.M. No. 914 . October 1, 1999.] provisions of the Constitution of nineteen
RE: APPLICATION FOR ADMISSION TO THE hundred and thirty-five" are citizens of the
PHILIPPINE BAR Philippines. Likewise, this recognition by the 1973
VICENTE D. CHING, applicant. Constitution was carried over to the 1987
The Solicitor General for applicant. Constitution which states that "(t)hose born
before January 17, 1973 of Filipino mothers, who
SYNOPSIS elect Philippine citizenship upon reaching the age
Vicente D. Ching is the legitimate son of spouses of majority" are Philippine citizens.
Tat Ching, a Chinese citizen and Prescila A. Dulay,
a Filipino, Ching was born in Francia West, Tubao, 2. ID.; ID.; ID.; 1973 AND 1987 ConstitutionAL
La Union on 11 April 1964. Since birth, he resided PROVISIONS ON ELECTION OF PHILIPPINE
in the Philippines. He is also a Certified Public CITIZENSHIP HAVE NO CURATIVE EFFECT. — It
Accountant and a registered voter of Tubao, La should be noted, however, that the 1973 and
Union. In fact, he was elected as member of the 1987 Constitutional provisions on the election of
Sangguniang Bayan of Tubao, La Union during the Philippine citizenship should not be understood as
12 May 1992 synchronized elections. On 17 July having a curative effect on any irregularity in the
1998, having completed a Bachelor of Laws acquisition of citizenship for those covered by the
course at the St. Louis University, Baguio City, he 1935 Constitution. If the citizenship of a person
filed an application to take the 1998 Bar was subject to challenge under the old charter, it
Examinations. He was conditionally admitted to remains subject to challenge under the new
take the Bar Examinations, subject to the charter even if the judicial challenge had not
condition that he must submit to the Court proof been commenced before the effectivity of the
of his Philippine citizenship. On 5 April 1999, the new Constitution.
1998 Bar Examinations were released and Ching
was one of the successful examinees. However, 3. ID.; ID.; ID.; COMMONWEALTH ACT NO. 625;
because of the questionable status of his PRESCRIBES PROCEDURE FOR ELECTION OF
citizenship, he was not allowed to take his oath CITIZENSHIP. — C.A. No. 625 which was enacted
and instead, he was required to submit further pursuant to Section 1(3), Article IV of the 1935

44 | C O N S T I 2 _ A r t i c l e I V _ C I T I Z E N S H I P
Constitution, prescribes the procedure that 1999, or over fourteen (14) years after he had
should be followed in order to make a valid reached the age of majority. Based on the
election of Philippine citizenship. Under Section 1 interpretation of the phrase "upon reaching the
thereof, legitimate children born of Filipino age of majority," Ching's election was clearly
mothers may elect Philippine citizenship by beyond, by any reasonable yardstick, the
expressing such intention "in a statement to be allowable period within which to exercise the
signed and sworn to by the party concerned privilege. It should be stated, in this connection,
before any officer authorized to administer oaths, that the special circumstances invoked by Ching,
and shall be filed with the nearest civil registry. i.e., his continuous and uninterrupted stay in the
The said party shall accompany the aforesaid Philippines and his being a certified public
statement with the oath of allegiance to the accountant, a registered voter and a former
Constitution and the Government of the elected public official, cannot vest in him
Philippines." Philippine citizenship as the law specifically lays
down the requirements for acquisition of
4. ID.; ID.; ID.; ID.; ID.; ELECTION SHOULD BE Philippine citizenship by election.
MADE WITHIN REASONABLE TIME AFTER
ATTAINING AGE OF MAJORITY. — However, the 7. ID.; ID.; ID.; ID.; APPLICANT FAILED TO VALIDLY
1935 Constitution and C.A. No. 625 did not ELECT PHILIPPINE CITIZENSHIP; CASE AT BAR. —
prescribe a time period within which the election Consequently, we hold that Ching failed to validly
of Philippine citizenship should be made. The elect Philippine citizenship. The span of fourteen
1935 Charter only provides that the election (14) years that lapsed from the time he reached
should be made "upon reaching the age of the age of majority until he finally expressed his
majority." The age of majority then commenced intention to elect Philippine citizenship is clearly
upon reaching twenty-one (21) years. In the way beyond the contemplation of the
opinions of the Secretary of Justice on cases requirement of electing "upon reaching the age of
involving the validity of election of Philippine majority." Moreover, Ching has offered no reason
citizenship, this dilemma was resolved by basing why he delayed his election of Philippine
the time period on the decisions of this Court citizenship. The prescribed procedure in electing
prior to the effectivity of the 1935 Constitution. In Philippine citizenship is certainly not a tedious
these decisions, the proper period for electing and painstaking process. All that is required of
Philippine citizenship was, in turn, based on the the elector is to execute an affidavit of election of
pronouncements of the Department of State of Philippine citizenship and, thereafter, file the
the United States Government to the effect that same with the nearest civil registry. Ching's
the election should be made within a "reasonable unreasonable and unexplained delay in making
time" after attaining the age of majority. his election cannot be simply glossed over.
HaAISC
5. ID.; ID.; ID.; ID.; ID.; ID.; "REASONABLE TIME";
CONSTRUED. — The phrase "reasonable time" 8. ID.; ID.; ID.; PERSON PRIVILEGED TO ELECT
has been interpreted to mean that the election PHILIPPINE CITIZENSHIP HAS ONLY AN INCHOATE
should be made within three (3) years from RIGHT TO SUCH CITIZENSHIP. — Philippine
reaching the age of majority. However, we held in citizenship can never be treated like a commodity
Cuenco vs. Secretary of Justice, that the three (3) that can be claimed when needed and
year period is not an inflexible rule. We said: It is suppressed when convenient. One who is
true that this clause has been construed to mean privileged to elect Philippine citizenship has only
a reasonable period after reaching the age of an inchoate right to such citizenship. As such, he
majority, and that the Secretary of Justice has should avail of the right with fervor, enthusiasm
ruled that three (3) years is the reasonable time and promptitude.
to elect Philippine citizenship under the
constitutional provision adverted to above, which RESOLUTION
period may be extended under certain
circumstances, as when the person concerned KAPUNAN, J p:
has always considered himself a Filipino.
However, we cautioned in Cuenco that the Can a legitimate child born under the 1935
extension of the option to elect Philippine Constitution of a Filipino mother and an alien
citizenship is not indefinite: Regardless of the father validly elect Philippine citizenship fourteen
foregoing, petitioner was born on February 16, (14) years after he has reached the age of
1923. He became of age on February 16, 1944. majority? This is the question sought to be
His election of citizenship was made on May 15, resolved in the present case involving the
1951, when he was over twenty-eight (28) years application for admission to the Philippine Bar of
of age, or over seven (7) years after he had Vicente D. Ching. cdlex
reached the age of majority. It is clear that said
election has not been made "upon reaching the The facts of this case are as follows:
age of majority."
Vicente D. Ching, the legitimate son of the
6. ID.; ID.; ID.; ID.; ID.; ID.; NOT COMPLIED IN spouses Tat Ching, a Chinese citizen, and Prescila
CASE AT BAR. — In the present case, Ching, A. Dulay, a Filipino, was born in Francia West,
having been born 11 April 1964, was already Tubao, La Union on 11 April 1964. Since his birth,
thirty-five (35) years old when he complied with Ching has resided in the Philippines.
the requirements of C.A. No. 625 on 15 June

45 | C O N S T I 2 _ A r t i c l e I V _ C I T I Z E N S H I P
On 17 July 1998, Ching, after having completed a then explains the meaning of the phrase "upon
Bachelor of Laws course at the St. Louis reaching the age of majority:"
University in Baguio City, filed an application to
take the 1998 Bar Examinations. In a Resolution The clause "upon reaching the age of majority"
of this Court, dated 1 September 1998, he was has been construed to mean a reasonable time
allowed to take the Bar Examinations, subject to after reaching the age of majority which had been
the condition that he must submit to the Court interpreted by the Secretary of Justice to be three
proof of his Philippine citizenship. (3) years (VELAYO, supra at p. 51 citing Op., Sec.
of Justice No. 70, s. 1940, Feb. 27, 1940). Said
period may be extended under certain
circumstances, as when a (sic) person concerned
In compliance with the above resolution, Ching has always considered himself a Filipino (ibid.,
submitted on 18 November 1998, the following citing Op. Nos. 355 and 422, s. 1955; 3, 12, 46,
documents: 86 and 97, s. 1953). But in Cuenco, it was held
that an election done after over seven (7) years
1. Certification, dated 9 June 1986, issued by the was not made within a reasonable time.
Board of Accountancy of the Professional
Regulations Commission showing that Ching is a In conclusion, the OSG points out that Ching has
certified public accountant; not formally elected Philippine citizenship and, if
ever he does, it would already be beyond the
2. Voter Certification, dated 14 June 1997, issued "reasonable time" allowed by present
by Elizabeth B. Cerezo, Election Officer of the jurisprudence. However, due to the peculiar
Commission on Elections (COMELEC) in Tubao, La circumstances surrounding Ching's case, the OSG
Union showing that Ching is a registered voter of recommends the relaxation of the standing rule
the said place; and on the construction of the phrase "reasonable
period" and the allowance of Ching to elect
3. Certification, dated 12 October 1998, also Philippine citizenship in accordance with C.A. No.
issued by Elizabeth B. Cerezo, showing that Ching 625 prior to taking his oath as a member of the
was elected as a member of the Sangguniang Philippine Bar. llcd
Bayan of Tubao, La Union during the 12 May 1992
synchronized elections. cdphil On 27 July 1999, Ching filed a Manifestation,
attaching therewith his Affidavit of Election of
On 5 April 1999, the results of the 1998 Bar Philippine Citizenship and his Oath of Allegiance,
Examinations were released and Ching was one both dated 15 July 1999. In his Manifestation,
of the successful Bar examinees. The oath-taking Ching states:
of the successful Bar examinees was scheduled
on 5 May 1999. However, because of the 1. I have always considered myself as a Filipino;
questionable status of Ching's citizenship, he was
not allowed to take his oath. Pursuant to the 2. I was registered as a Filipino and consistently
resolution of this Court, dated 20 April 1999, he declared myself as one in my school records and
was required to submit further proof of his other official documents;
citizenship. In the same resolution, the Office of
the Solicitor General (OSG) was required to file a 3. I am practicing a profession (Certified Public
comment on Ching's petition for admission to the Accountant) reserved for Filipino citizens;
bar and on the documents evidencing his
Philippine citizenship. 4. I participated in electoral process[es] since the
time I was eligible to vote;
The OSG filed its comment on 8 July 1999, stating
that Ching, being the "legitimate child of a 5. I had served the people of Tubao, La Union as a
Chinese father and a Filipino mother born under member of the Sangguniang Bayan from 1992 to
the 1935 Constitution was a Chinese citizen and 1995;
continued to be so, unless upon reaching the age
of majority he elected Philippine citizenship" 1 in 6. I elected Philippine citizenship on July 15, 1999
strict compliance with the provisions of in accordance with Commonwealth Act No. 625;
Commonwealth Act No. 625 entitled "An Act
Providing for the Manner in which the Option to 7. My election was expressed in a statement
Elect Philippine Citizenship shall be Declared by a signed and sworn to by me before a notary
Person Whose Mother is a Filipino Citizen." The public;
OSG adds that "(w)hat he acquired at best was
only an inchoate Philippine citizenship which he 8. I accompanied my election of Philippine
could perfect by election upon reaching the age citizenship with the oath of allegiance to the
of majority." 2 In this regard, the OSG clarifies Constitution and the Government of the
that "two (2) conditions must concur in order that Philippines; LexLib
the election of Philippine citizenship may be
effective, namely: (a) the mother of the person 9. I filed my election of Philippine citizenship and
making the election must be a citizen of the my oath of allegiance to (sic) the Civil Registrar of
Philippines; and (b) said election must be made Tubao La Union, and
'upon reaching the age of majority.'" 3 The OSG

46 | C O N S T I 2 _ A r t i c l e I V _ C I T I Z E N S H I P
10. I paid the amount of TEN PESOS (Ps. 10.00) as Philippine citizenship was, in turn, based on the
filing fees. pronouncements of the Department of State of
the United States Government to the effect that
Since Ching has already elected Philippine the election should be made within a "reasonable
citizenship on 15 July 1999, the question raised is time" after attaining the age of majority. 10 The
whether he has elected Philippine citizenship phrase "reasonable time" has been interpreted to
within a "reasonable time." In the affirmative, mean that the election should be made within
whether his citizenship by election retroacted to three (3) years from reaching the age of majority.
the time he took the bar examination. 11 However, we held in Cuenco vs. Secretary of
Justice, 12 that the three (3) year period is not an
When Ching was born in 1964, the governing inflexible rule. We said:
charter was the 1935 Constitution. Under Article
IV, Section 1(3) of the 1935 Constitution, the It is true that this clause has been construed to
citizenship of a legitimate child born of a Filipino mean a reasonable period after reaching the age
mother and an alien father followed the of majority, and that the Secretary of Justice has
citizenship of the father, unless, upon reaching ruled that three (3) years is the reasonable time
the age of majority, the child elected Philippine to elect Philippine citizenship under the
citizenship. 4 This right to elect Philippine constitutional provision adverted to above, which
citizenship was recognized in the 1973 period may be extended under certain
Constitution when it provided that "(t)hose who circumstances, as when the person concerned
elect Philippine citizenship pursuant to the has always considered himself a Filipino. 13
provisions of the Constitution of nineteen
hundred and thirty-five" are citizens of the However, we cautioned in Cuenco that the
Philippines. 5 Likewise, this recognition by the extension of the option to elect Philippine
1973 Constitution was carried over to the 1987 citizenship is not indefinite:
Constitution which states that "(t)hose born
before January 17, 1973 of Filipino mothers, who Regardless of the foregoing, petitioner was born
elect Philippine citizenship upon reaching the age on February 16, 1923. He became of age on
of majority" are Philippine citizens. 6 It should be February 16, 1944. His election of citizenship was
noted, however, that the 1973 and 1987 made on May 15, 1951, when he was over
Constitutional provisions on the election of twenty-eight (28) years of age, or over seven (7)
Philippine citizenship should not be understood as years after he had reached the age of majority. It
having a curative effect on any irregularity in the is clear that said election has not been made
acquisition of citizenship for those covered by the "upon reaching the age of majority." 14
1935 Constitution. 7 If the citizenship of a person
was subject to challenge under the old charter, it In the present case, Ching, having been born on
remains subject to challenge under the new 11 April 1964, was already thirty-five (35) years
charter even if the judicial challenge had not old when he complied with the requirements of
been commenced before the effectivity of the C.A. No. 625 on 15 June 1999, or over fourteen
new Constitution. 8 (14) years after he had reached the age of
majority. Based on the interpretation of the
C.A. No. 625 which was enacted pursuant to phrase "upon reaching the age of majority,"
Section 1(3), Article IV of the 1935 Constitution, Ching's election was clearly beyond, by any
prescribes the procedure that should be followed reasonable yardstick, the allowable period within
in order to make a valid election of Philippine which to exercise the privilege. It should be
citizenship. Under Section 1 thereof, legitimate stated, in this connection, that the special
children born of Filipino mothers may elect circumstances invoked by Ching, i.e., his
Philippine citizenship by expressing such continuous and uninterrupted stay in the
intention "in a statement to be signed and sworn Philippines and his being a certified public
to by the party concerned before any officer accountant, a registered voter and a former
authorized to administer oaths, and shall be filed elected public official, cannot vest in him
with the nearest civil registry. The said party shall Philippine citizenship as the law specifically lays
accompany the aforesaid statement with the oath down the requirements for acquisition of
of allegiance to the Constitution and the Philippine citizenship by election. cda
Government of the Philippines." cdasia

However, the 1935 Constitution and C.A. No. 625


did not prescribe a time period within which the Definitely, the so-called special circumstances
election of Philippine citizenship should be made. cannot constitute what Ching erroneously labels
The 1935 Charter only provides that the election as informal election of citizenship. Ching cannot
should be made "upon reaching the age of find a refuge in the case of In re: Florencio
majority." The age of majority then commenced Mallare, 15 the pertinent portion of which reads:
upon reaching twenty-one (21) years. 9 In the
opinions of the Secretary of Justice on cases And even assuming arguendo that Ana Mallare
involving the validity of election of Philippine were (sic) legally married to an alien, Esteban's
citizenship, this dilemma was resolved by basing exercise of the right of suffrage when he came of
the time period on the decisions of this Court age, constitutes a positive act of election of
prior to the effectivity of the 1935 Constitution. In Philippine citizenship. It has been established that
these decisions, the proper period for electing Esteban Mallare was a registered voter as of April

47 | C O N S T I 2 _ A r t i c l e I V _ C I T I Z E N S H I P
14, 1928, and that as early as 1925 (when he was Filipinos when the time to elect came up, there
about 22 years old), Esteban was already are acts of deliberate choice which cannot be less
participating in the elections and campaigning for binding. Entering a profession open only to
certain candidate[s]. These acts are sufficient to Filipinos, serving in public office where citizenship
show his preference for Philippine citizenship. 16 is a qualification, voting during election time,
running for public office, and other categorical
Ching's reliance on Mallare is misplaced. The acts of similar nature are themselves formal
facts and circumstances obtaining therein are manifestations for these persons.
very different from those in the present case,
thus, negating its applicability. First, Esteban An election of Philippine citizenship presupposes
Mallare was born before the effectivity of the that the person electing is an alien. Or his status
1935 Constitution and the enactment of C.A. No. is doubtful because he is a national of two
625. Hence, the requirements and procedures countries. There is no doubt in this case about Mr.
prescribed under the 1935 Constitution and C.A. Ong's being a Filipino when he turned twenty-one
No. 625 for electing Philippine citizenship would (21). cdpr
not be applicable to him. Second, the ruling in
Mallare was an obiter since, as correctly pointed We repeat that any election of Philippine
out by the OSG, it was not necessary for Esteban citizenship on the part of the private respondent
Mallare to elect Philippine citizenship because he would not only have been superfluous but it
was already a Filipino, he being a natural child of would also have resulted in an absurdity. How can
a Filipino mother. In this regard, the Court stated: a Filipino citizen elect Philippine citizenship? 19

Esteban Mallare, natural child of Ana Mallare, a The Court, like the OSG, is sympathetic with the
Filipina, is therefore himself a Filipino, and no plight of Ching. However, even if we consider the
other act would be necessary to confer on him all special circumstances in the life of Ching like his
the rights and privileges attached to Philippine having lived in the Philippines all his life and his
citizenship (U.S. vs. Ong Tianse, 29 Phil. 332; consistent belief that he is a Filipino, controlling
Santos Co vs. Government of the Philippine statutes and jurisprudence constrain us to
Islands, 42 Phil. 543, Serra vs. Republic, L-4223, disagree with the recommendation of the OSG.
May 12, 1952, Sy Quimsuan vs. Republic, L-4693, Consequently, we hold that Ching failed to validly
Feb. 16, 1953; Pitallano vs. Republic, L-5111, June elect Philippine citizenship. The span of fourteen
28, 1954). Neither could any act be taken on the (14) years that lapsed from the time he reached
erroneous belief that he is a non-Filipino divest the age of majority until he finally expressed his
him of the citizenship privileges to which he is intention to elect Philippine citizenship is clearly
rightfully entitled. 17 way beyond the contemplation of the
requirement of electing "upon reaching the age of
The ruling in Mallare was reiterated and further majority." Moreover, Ching has offered no reason
elaborated in Co vs. Electoral Tribunal of the why he delayed his election of Philippine
House of Representatives, 18 where we held: citizenship. The prescribed procedure in electing
Philippine citizenship is certainly not a tedious
We have jurisprudence that defines 'election' as and painstaking process. All that is required of
both a formal and an informal process. the elector is to execute an affidavit of election of
Philippine citizenship and, thereafter, file the
In the case of In re: Florencio Mallare (59 SCRA 45 same with the nearest civil registry. Ching's
[1974]), the Court held that the exercise of the unreasonable and unexplained delay in making
right of suffrage and the participation in election his election cannot be simply glossed over.
exercises constitute a positive act of election of
Philippine citizenship. In the exact Philippine citizenship can never be treated like a
pronouncement of the Court, we held: commodity that can be claimed when needed and
suppressed when convenient. 20 One who is
"Esteban's exercise of the right of suffrage when privileged to elect Philippine citizenship has only
he came of age, constitutes a positive act of an inchoate right to such citizenship. As such, he
Philippine citizenship". (p. 52; emphasis should avail of the right with fervor, enthusiasm
supplied)" and promptitude. Sadly, in this case, Ching slept
on his opportunity to elect Philippine citizenship
The private respondent did more than merely and, as a result, this golden privilege slipped
exercise his right of suffrage. He has established away from his grasp. LLjur
his life here in the Philippines.
IN VIEW OF THE FOREGOING, the Court Resolves
For those in the peculiar situation of the to DENY Vicente D. Ching's application for
respondent who cannot be expected to have admission to the Philippine Bar.
elected Philippine citizenship as they were
already citizens, we apply the In Re Mallare rule. SO ORDERED.

xxx xxx xxx Davide, Jr., C.J., Bellosillo, Melo, Puno, Vitug,
Mendoza, Panganiban, Quisumbing, Purisima,
The filing of sworn statement or formal Pardo, Buena, Gonzaga-Reyes and Ynares-
declaration is a requirement for those who still Santiago, JJ., concur.
have to elect citizenship. For those already

48 | C O N S T I 2 _ A r t i c l e I V _ C I T I Z E N S H I P
(Bengson III v. HRET, G.R. No. 142840, May commissioned in, the armed forces of a foreign
07, 2001) country under any of the circumstances
mentioned in paragraph (a) or (b), shall not be
EN BANC permitted to participate nor vote in any election
of the Republic of the Philippines during the
[G.R. No. 142840. May 7, 2001.] period of his service to, or commission in, the
armed forces of said country. Upon his discharge
ANTONIO BENGSON III, petitioner, vs. HOUSE OF from the service of the said foreign country, he
REPRESENTATIVES ELECTORAL TRIBUNAL and shall be automatically entitled to the full
TEODORO C. CRUZ, respondents. enjoyment of his civil and political rights as a
Filipino citizen . . . .
DECISION
Whatever doubt that remained regarding his loss
KAPUNAN, J p: of Philippine citizenship was erased by his
naturalization as a U.S. citizen on June 5, 1990, in
The citizenship of respondent Teodoro C. Cruz is connection with his service in the U.S. Marine
at issue in this case, in view of the constitutional Corps.
requirement that "no person shall be a Member of
the House of Representatives unless he is a On March 17, 1994, respondent Cruz reacquired
natural-born citizen." 1 his Philippine citizenship through repatriation
under Republic Act No. 2630. 3 He ran for and
Respondent Cruz was a natural-born citizen of the was elected as the Representative of the Second
Philippines. He was born in San Clemente, Tarlac, District of Pangasinan in the May 11, 1998
on April 27, 1960, of Filipino parents. The elections. He won by a convincing margin of
fundamental law then applicable was the 1935 26,671 votes over petitioner Antonio Bengson III,
Constitution. 2 who was then running for reelection.

On November 5, 1985, however, respondent Cruz Subsequently, petitioner filed a case for Quo
enlisted in the United States Marine Corps and, Warranto Ad Cautelam with respondent House of
without the consent of the Republic of the Representatives Electoral Tribunal (HRET)
Philippines, took an oath of allegiance to the claiming that respondent Cruz was not qualified
United States. As a consequence, he lost his to become a member of the House of
Filipino citizenship for under COMMONWEALTH Representatives since he is not a natural-born
ACT NO. 63, Section 1(4), a Filipino citizen may citizen as required under Article VI, Section 6 of
lose his citizenship by, among others, "rendering the Constitution. 4
service to or accepting commission in the armed
forces of a foreign country." Said provision of law On March 2, 2000, the HRET rendered its decision
reads: 5 dismissing the petition, for quo warranto and
declaring respondent Cruz the duly elected
SECTION 1. How citizenship may be lost. — A Representative of the Second District of
Filipino citizen may lose his citizenship in any of Pangasinan in the May 1998 elections. The HRET
the following ways and/or events: likewise denied petitioner's motion for
reconsideration of the decision in its resolution
xxx xxx xxx dated April 27, 2000. 6

(4) By rendering services to, or accepting Petitioner thus filed the present petition for
commission in, the armed forces of a foreign certiorari assailing the HRET's decision on the
country: Provided, That the rendering of service following grounds:
to, or the acceptance of such commission in, the
armed forces of a foreign country, and the taking 1. The HRET committed serious errors and grave
of an oath of allegiance incident thereto, with the abuse of discretion, amounting to excess of
consent of the Republic of the Philippines, shall jurisdiction, when it ruled that private respondent
not divest a Filipino of his Philippine citizenship if is a natural-born citizen of the Philippines despite
either of the following circumstances is present: the fact that he had ceased being such in view of
the loss and renunciation of such citizenship on
(a) The Republic of the Philippines has a his part. SDEHCc
defensive and/or offensive pact of alliance with
said foreign country; or 2. The HRET committed serious errors and grave
abuse of discretion, amounting to excess of
(b) The said foreign country maintains armed jurisdiction, when it considered private
forces on Philippine territory with the consent of respondent as a citizen of the Philippines despite
the Republic of the Philippines: Provided, That the the fact that he did not validly acquire his
Filipino citizen concerned, at the time of Philippine citizenship.
rendering said service, or acceptance of said
commission, and taking the oath of allegiance 3. Assuming that private respondent's acquisition
incident thereto, states that he does so only in of Philippine citizenship was invalid, the HRET
connection with his service to said foreign committed serious errors and grave abuse of
country; And provided, finally, That any Filipino discretion, amounting to excess of jurisdiction,
citizen who is rendering service to, or is when it dismissed the petition despite the fact

49 | C O N S T I 2 _ A r t i c l e I V _ C I T I Z E N S H I P
that such reacquisition could not legally and becomes executory only after two (2) years from
constitutionally restore his natural-born status. 7 its promulgation when the court is satisfied that
during the intervening period, the applicant has
The issue now before us is whether respondent (1) not left the Philippines; (2) has dedicated
Cruz, a natural-born Filipino who became an himself to a lawful calling or profession; (3) has
American citizen, can still be considered a not been convicted of any offense or violation of
natural-born Filipino upon his reacquisition of Government promulgated rules; or (4) committed
Philippine citizenship. any act prejudicial to the interest of the nation or
contrary to any Government announced policies.
Petitioner asserts that respondent Cruz may no 14
longer be considered a natural-born Filipino since
he lost his Philippine citizenship when he swore Filipino citizens who have lost their citizenship
allegiance to the United States in 1995, and had may however reacquire the same in the manner
to reacquire the same by repatriation. He insists provided by law. COMMONWEALTH ACT NO. 63
that Article IV, Section 2 of the Constitution (CA No. 63), enumerates the three modes by
expressly states that natural-born citizens are which Philippine citizenship may be reacquired by
those who are citizens from birth without having a former citizen: (1) by naturalization, (2) by
to perform any act to acquire or perfect such repatriation, and (3) by direct act of Congress. 15
citizenship.
Naturalization is a mode for both acquisition and
Respondent on the other hand contends that he reacquisition of Philippine citizenship. As a mode
reacquired his status as a natural-born citizen of initially acquiring Philippine citizenship,
when he was repatriated since the phrase "from naturalization is governed by Commonwealth Act
birth" in Article IV, Section 2 refers to the innate, No. 473, as amended. On the other hand,
inherent and inborn characteristic of being a naturalization as a mode for reacquiring
natural-born citizen. Philippine citizenship is governed by
COMMONWEALTH ACT NO. 63. 16 Under this law,
The petition is without merit. a former Filipino citizen who wishes to reacquire
Philippine citizenship must possess certain
The 1987 Constitution enumerates who are qualifications 17 and none of the disqualifications
Filipino citizens as follows: mentioned in Section 4 of C.A. 473. 18

(1) Those who are citizens of the Philippines at Repatriation, on the other hand, may be had
the time of the adoption of this Constitution; under various statutes by those who lost their
citizenship due to: (1) desertion of the armed
(2) Those whose fathers or mothers are citizens forces; 19 (2) service in the armed forces of the
of the Philippines; allied forces in World War II; 20 (3) service in the
Armed Forces of the United States at any other
(3) Those born before January 17, 1973 of Filipino time; 21 (4) marriage of a Filipino woman to an
mothers, who elect Philippine citizenship upon alien; 22 and (5) political and economic necessity.
reaching the age of majority, and 23

(4) Those who are naturalized in accordance with As distinguished from the lengthy process of
law. 8 naturalization, repatriation simply consists of the
taking of an oath of allegiance to the Republic of
There are two ways of acquiring citizenship: (1) the Philippines and registering said oath in the
by birth, and (2) by naturalization. These ways of Local Civil Registry of the place where the person
acquiring citizenship correspond to the two kinds concerned resides or last resided.
of citizens: the natural-born citizen, and the
naturalized citizen. A person who at the time of In Angara v. Republic, 24 we held:
his birth is a citizen of a particular country, is a
natural-born citizen thereof. 9 . . . . Parenthetically, under these statutes
[referring to RA Nos. 965 and 2630], the person
As defined in the same Constitution, natural-born desiring to reacquire Philippine citizenship would
citizens "are those citizens of the Philippines from not even be required to file a petition in court,
birth without having to perform any act to acquire and all that he had to do was to take an oath of
or perfect his Philippine citizenship." 10 allegiance to the Republic of the Philippines and
to register that fact with the civil registry in the
On the other hand, naturalized citizens are those place of his residence or where he had last
who have become Filipino citizens through resided in the Philippines. [Emphasis in the
naturalization, generally under Commonwealth original.] 25
Act No. 473, otherwise known as the Revised
Naturalization Law, which repealed the former
Naturalization Law (Act No. 2927), and by
Republic Act No. 530. 11 To be naturalized, an Moreover, repatriation results in the recovery of
applicant has to prove that he possesses all the the original nationality. 26 This means that a
qualifications 12 and none of the disqualifications naturalized Filipino who lost his citizenship will be
13 provided by law to become a Filipino citizen. restored to his prior status as a naturalized
The decision granting Philippine citizenship Filipino citizen. On the other hand, if he was

50 | C O N S T I 2 _ A r t i c l e I V _ C I T I Z E N S H I P
originally a natural-born citizen before he lost his
Philippine citizenship, he will be restored to his The present Constitution, however, now considers
former status as a natural-born Filipino. those born of Filipino mothers before the
effectivity of the 1973 Constitution and who
In respondent Cruz's case, he lost his Filipino elected Philippine citizenship upon reaching the
citizenship when he rendered service in the majority age as natural-born. After defining who
Armed Forces of the United States. However, he are natural-born citizens, Section 2 of Article IV
subsequently reacquired Philippine citizenship adds a sentence: "Those who elect Philippine
under R.A. No. 2630, which provides: citizenship in accordance with paragraph (3),
Section 1 hereof shall be deemed natural-born
SECTION 1. Any person who had lost his citizens." Consequently, only naturalized Filipinos
Philippine citizenship by rendering service to, or are considered not natural-born citizens. It is
accepting commission in, the Armed Forces of the apparent from the enumeration of who are
United States, or after separation from the Armed citizens under the present Constitution that there
Forces of the United States, acquired United are only two classes of citizens: (1) those who are
States citizenship, may reacquire Philippine natural-born and (2) those who are naturalized in
citizenship by taking an oath of allegiance to the accordance with law. A citizen who is not a
Republic of the Philippines and registering the naturalized Filipino, i.e., did not have to undergo
same with Local Civil Registry in the place where the process of naturalization to obtain Philippine
he resides or last resided in the Philippines. The citizenship, necessarily is a natural-born Filipino.
said oath of allegiance shall contain a Noteworthy is the absence in said enumeration of
renunciation of any other citizenship. a separate category for persons who, after losing
Philippine citizenship, subsequently reacquire it.
Having thus taken the required oath of allegiance The reason therefor is clear: as to such persons,
to the Republic and having registered the same in they would either be natural-born or naturalized
the Civil Registry of Magantarem, Pangasinan in depending on the reasons for the loss of their
accordance with the aforecited provision, citizenship and the mode prescribed by the
respondent Cruz is deemed to have recovered his applicable law for the reacquisition thereof. As
original status as a natural-born citizen, a status respondent Cruz was not required by law to go
which he acquired at birth as the son of a Filipino through naturalization proceedings in order to
father. 27 It bears stressing that the act of reacquire his citizenship, he is perforce a natural-
repatriation allows him to recover, or return to, born Filipino. As such, he possessed all the
his original status before he lost his Philippine necessary qualifications to be elected as member
citizenship. of the House of Representatives.

Petitioner's contention that respondent Cruz is no A final point. The HRET has been empowered by
longer a natural-born citizen since he had to the Constitution to be the "sole judge" of all
perform an act to regain his citizenship is contests relating to the election, returns, and
untenable. As correctly explained by the HRET in qualifications of the members of the House. 29
its decision, the term "natural-born citizen" was The Court's jurisdiction over the HRET is merely
first defined in Article III, Section 4 of the 1973 to check "whether or not there has been a grave
Constitution as follows: abuse of discretion amounting to lack or excess
of jurisdiction" on the part of the latter. 30 In the
SECTION 4. A natural-born citizen is one who is a absence thereof, there is no occasion for the
citizen of the Philippines from birth without Court to exercise its corrective power and annul
having to perform any act to acquire or perfect the decision of the HRET nor to substitute the
his Philippine citizenship. Court's judgment for that of the latter for the
simple reason that it is not the office of a petition
Two requisites must concur for a person to be for certiorari to inquire into the correctness of the
considered as such: (1) a person must be a assailed decision. 31 There is no such showing of
Filipino citizen from birth and (2) he does not grave abuse of discretion in this case.
have to perform any act to obtain or perfect his
Philippine citizenship. WHEREFORE, the petition is hereby DISMISSED.
Davide, Jr., C.J., Bellosillo and Puno, JJ., concur.
Under the 1973 Constitution definition, there Melo, J., took no part. Chairman of the HRET
were two categories of Filipino citizens which which rendered the decision under review.
were not considered natural-born: (1) those who Vitug, J.,took no part. A member of the HRET
were naturalized and (2) those born before which rendered the appealed judgment.
January 17, 1973, 28 of Filipino mothers who, Mendoza, J., took no part, being ponente of
upon reaching the age of majority, elected decision under review.
Philippine citizenship. Those "naturalized citizens" Panganiban, J., please see concurring opinion.
were not considered natural-born obviously Quisumbing, Buena and De Leon, Jr., JJ., are on
because they were not Filipinos at birth and had leave.
to perform an act to acquire Philippine Pardo, J., concurs on this and the concurring
citizenship. Those born of Filipino mothers before opinion of J. Panganiban.
the effectivity of the 1973 Constitution were Gonzaga-Reyes, J., also joins concurring opinion
likewise not considered natural-born because of J. Panganiban.
they also had to perform an act to perfect their
Philippine citizenship.

51 | C O N S T I 2 _ A r t i c l e I V _ C I T I Z E N S H I P
Ynares-Santiago, J., hereby certifies that J. prejudice public interest, or construction resulting
Santiago joins with the majority opinion of J. in unreasonableness, as well as a construction
Kapunan. which will result in absurd consequences.
Gutierrez, J., Please see dissenting opinion.
4. ID.; CONSTRUCTION AVOIDED IF INCONSISTENT
(Moy Ya Lim Yao v. Commissioner of WITH LEGISLATIVE INTENT. — So a construction
Immigration, G.R. No. L-21289, October 04, should, if possible, be avoided if the result would
1971) be an apparent inconsistency in legislative intent,
as has been determined by the judicial decisions,
EN BANC or which would result in futility, redundancy, or a
conclusion not contemplated by the legislature;
[G.R. No. L-21289. October 4, 1971.] and the court should adopt that construction
which will be the least likely to produce mischief.
MOY YA LIM YAO alias EDILBERTO AGUINALDO LIM Unless plainly shown to have been the intention
and LAU YUEN YEUNG, petitioners-appellants, vs. of the legislature an interpretation which would
THE COMMISSIONER OF IMMIGRATION, render the requirements of the statute uncertain
respondent-appellee. and vague is to be avoided, and the court will not
ascribe to the legislature an intent to confer an
Aruego, Mamaril & Associates for petitioners- illusory right.
appellants.
5. POLITICAL LAW; CITIZENSHIP;
Solicitor General Arturo A. Alafriz, Asst. Sol. Gen. NATURALIZATION; POLICY OF SELECTIVE
Frine C . Zaballero and Solicitor Sumilang V . ADMISSION, EXPLAINED. — The avowed policy of
Bernardo for respondent-appellee. "selective admission" more particularly refers to a
case where a citizenship is sought to be acquired
SYLLABUS in a judicial proceeding for naturalization. In such
a case, the courts should no doubt apply the
1. POLITICAL LAW; CITIZENSHIP; IMMIGRATION national policy of selecting only those who are
ACT; SECTION 9 (G) THEREOF, NOT APPLICABLE worthy to be come citizens. There is here a choice
TO ALIEN WHO LEGITIMATELY BECOMES FILIPINO. between accepting or rejecting the application for
— Section 9 (g) of the Immigration Act does not citizenship. But this policy finds no application is
apply to aliens who after coming into the cases where citizenship is conferred by operation
Philippines as temporary visitors, legitimately of law. In such cases, the courts have no choice to
become Filipino citizens or acquire Filipino accept or reject. If the individual claiming
citizenship. Such change of nationality naturally citizenship by operations of law proves in legal
bestows upon them the right to stay in the proceedings that he satisfies the statutory
Philippines permanently or not, as they may requirements, the cannot do otherwise than to
choose, and if they elect to reside here, the declare that he is a citizens of the Philippines.
immigration authorities may neither deport them
nor confiscate their bonds. 6. ID.; ID.; ID.; ALIEN WOMAN MARRYING FILIPINO
IPSO FACTO BECOME CITIZEN PROVIDED NOT
2. ID.; ID.; NATURALIZATION; EFFECTS. — The DISQUALIFIED BY LAW. — We now hold, all
naturalization of an alien visitor as a Philippine previous decisions of this Court indicating
citizen logically produces the effect of conferring otherwise notwithstanding, that under Section 15
upon him ipso facto all the rights of citizenship of Commonwealth Act 473, an alien woman
including that of being entitled to permanently marrying a Filipino, native-born or naturalized,
stay in the Philippines outside the orbit of becomes ipso facto a Filipina provided she is not
authority of the Commissioner of Immigration vis- disqualified to be a citizen of the Philippines
avis aliens, if only because by its very nature and under Section 4 of the same law. Likewise, an
express provisions, the Immigration Law is a law alien woman married an alien who is
only for aliens and is inapplicable to citizens of subsequently naturalized here follows the
the Philippines. Philippine citizenship of her husband the moment
he takes his oath as Filipino citizens, provided
3. STATUTORY CONSTRUCTION; WHERE that she does not suffer from any of the
LANGUAGE OF STATUTE IS SUSCEPTIBLE OF TWO disqualifications under said Section 4.
CONSTRUCTIONS, THAT WHICH CARRIES OUT
OBJECT PREVAILS. — A statute is to be construed 7. ID.; ID.; ID.; ID.; NATURALIZATION
with reference to its manifest object, and if the PROCEEDING, NOT REQUIRED. — Section 16 is a
language is susceptible of two constructions, one parallel provision to Section 15. If the widow of an
which will carry out and the other defeat such applicant for naturalization a Filipino, who dies
manifest object, it should receive the former during the proceedings, is not required to go
construction. A construction will cause through a naturalization proceeding, in order to
objectionable results should be avoided and the be considered as a Filipino citizen hereof, it
court will, if possible, place on the statute a should not follow that the wife of a living Filipino
construction which will not result in injustice, and cannot be denied that same privilege. This is
in accordance with the decisions construing plain common sense and there is absolutely no
statutes, a construction will not result in evidence that the Legislature intended to treat
oppression, hardship, or inconveniences will also them differently.
be avoided, as will a construction which will

52 | C O N S T I 2 _ A r t i c l e I V _ C I T I Z E N S H I P
8. ID.; ID.; ID.; MODES OF. — The Constitution be threshed out again and again as the occasion
itself recognizes as Philippine citizens "Those who may demand.
are naturalized in accordance with law" (Section 1
[5], Article IV, Philippine Constitution). Citizens by 12. ID.; ID.; NATURALIZATION; PROCEDURES FOR
naturalization, under this provision, include not ALIEN WIFE TO ACQUIRE PHILIPPINE CITIZENSHIP.
only those who are naturalized in accordance — Regarding the steps that should be taken by an
with legal proceedings for the acquisition of alien woman married to a Filipino citizen in order
citizenship, but also those who acquire citizenship to acquire Philippine citizenship, the procedure
by "derivative naturalization" or by operation of followed in the Bureau of Immigration is as
law, as, for example, the "naturalization" of an follows: The alien woman must file a petition for
alien wife through the naturalization of her the cancellation of her alien certificate of
husband, or by marriage of an alien woman to a registration alleging, among other things, that
citizen. she is married to a Filipino citizen and that she is
not disqualified from acquiring her husband's
9. ID.; ID.; ID.; SECTION 15 OF REVISED citizenship pursuant to Section 4 of
NATURALIZATION LAW; PURPOSE. — The leading Commonwealth Act No. 473, as amended. Upon
idea or purpose of Section 15 was to confer the filing of the said petition, which should be
Philippine citizenship by operation of law upon accompanied or supported by the joint affidavit of
certain classes of aliens as a legal consequence the petitioner and her Filipino husband to the
of their relationship, by blood or by affinity, to effect that the petitioner does not belong to any
persons who are already citizens of the of the groups disqualified by the cited Section
Philippines. Whenever the fact of relationship of from becoming naturalized Filipino citizen, the
the persons enumerated in the provisions concurs Bureau of Immigration conducts an investigation
with the fact of citizenship of the person to who and thereafter promulgates its order or decision
they are related, the effect is for said person to granting or denying the petition.
become ipso facto citizens of the Philippines.
"Ipso facto" as here used does not mean that all REYES, J.B.L., J., dissenting:
alien wives and all minor children of the
Philippine citizens, from the mere fact of POLITICAL LAW; CITIZENSHIP; NATURALIZATION;
relationship, necessarily become such citizens ALIEN WOMAN MARRIED TO FILIPINO MUST
also. Those who do not meet the statutory PROVE QUALIFICATIONS UNDER SECTION 3. —
requirements do not ipso facto become citizens; Our naturalization law separates qualifications
they must apply for naturalization in order to from disqualifications; the positive qualifications
acquire such status. What it does mean, however, under Section 3 thereof express a policy of
is that in respect of those persons enumerated in restriction as to candidates for naturalization as
Section 15, the relationship to a citizen of the much as the disqualifications under Section 4.
Philippines is the operative fact which establishes And it has been shown in our decision in the
the acquisition of Philippine citizenship by them. second Ly Giok Ha case (Ly Giok Ha vs. Galang, L-
Necessarily, it also determines the point of time 21332 March 18, 1966, 16 SCRA 416) that those
at which such citizenship commences. not disqualified under Section 4 would not
necessarily qualify under Section 3, even if the
10. ID.; ID.; ID.; ID.; ALIEN WIFE DEEMED A residence qualification were disregarded. In other
CITIZEN IF SHE MIGHT HERSELF BE NATURALIZED. words, by giving to Section 15 of our
— The legislature could not have intended that an Naturalization Law the effect of excluding only
alien wife should not be deemed a Philippine those women suffering from disqualification
citizen unless and until she proves that she might under Section 3 could result in admitting to
herself be lawfully naturalized. Far from it, the law citizenship woman that Section 2 intends to
states in plain terms that she shall be deemed a exclude. In these circumstances, I do not see why
citizen of the Philippines if she is one "who might American interpretation of the words who might
herself be lawfully naturalized." The proviso that herself be lawfully naturalized should be
she must be one "who might herself be lawfully considered hinding in this jurisdiction.
naturalized" is not a condition precedent to the
vesting or acquisition of citizenship; it is only a
condition or a state of fact necessary to establish
her citizenship as a factum probandum, i.e., as a DECISION
fact established and proved in evidence. The
word "might," as used in that phrase, precisely BARREDO, J p:
implies that at the time of her marriage to
Philippine citizen, the alien woman "had (the) Appeal from the following decision of the Court of
power" to become such a citizen herself under First Instance of Manila in its Civil Case No. 49705
the laws then in force. entitled Moy Ya Lim Yao, etc., et al. vs. The
Commissioner of Immigration which, brief as it is,
11. ID.; ID.; RES JUDICATA NOT APPLICABLE TO sufficiently depicts the factual setting of and the
RULINGS THEREON. — Everytime the citizenship fundamental issues involved in this case thus:
of a person is material or indispensable in a
judicial or administrative case, whatever the "In the instant case, petitioners seek the issuance
corresponding court or administrative authority of a writ of injunction against the Commissioner
decides therein as to such citizenship is generally of Immigration, 'restraining the latter and/or his
not considered as res adjudicata, hence it has to authorized representative from ordering plaintiff

53 | C O N S T I 2 _ A r t i c l e I V _ C I T I Z E N S H I P
Lau Yuen Yeung to leave the Philippines and might herself be lawfully naturalized shall be
causing her arrest and deportation and the deemed a citizen of the Philippines."
confiscation of her bond, upon her failure to do
so.' The above-quoted provision is clear and its
import unequivocal and hence it should be held
"The prayer for preliminary injunction embodied to mean what it plainly and explicitly expresses in
in the complaint, having been denied, the case unmistakable terms. The clause 'who might
was heard on the merits and the parties herself be lawfully naturalized' incontestably
submitted their respective evidence. implies that an alien woman may be deemed a
citizen of the Philippines by virtue of her marriage
"The facts of the case, as substantially and to a Filipino citizen only if she possesses all the
correctly stated by the Solicitor General are qualifications and none of the disqualifications
these: specified in the law, because these are the
explicit requisites provided by law for an alien to
'On February 8, 1961, Lau Yuen Yeung applied for be naturalized. (Lee Suan Ay, Alberto Tan and Lee
a passport visa to enter the Philippines as a non- Chiao vs. Emilio Galang, etc., G. R. No. L-11855).
immigrant. In the interrogation made in However, from the allegation of paragraph 3 of
connection with her application for a temporary the complaint, to wit:
visitor's visa to enter the Philippines, she stated
that she was a Chinese residing at Kowloon, "'3. That plaintiff Lau Yuen Yeung, Chinese by
Hongkong, and that she desired to take a birth, who might herself be lawfully naturalized as
pleasure trip to the Philippines to visit her great a Filipino citizen (not being disqualified to become
(grand) uncle Lau Ching Ping for a period of one such by naturalization), is a Filipino citizen by
month (Exhibits '1,' '1-a,' and '2'). She was virtue of her marriage on January 25, 1962 to
permitted to come into the Philippines on March plaintiff MOY YA LIM YAO alias EDILBERTO
13, 1961, and was permitted to stay for a period AGUINALDO LIM, under the Naturalization Laws of
of one month which would expire on April 13, the Philippines."
1961. On the date of her arrival, Asher Y, Cheng
filed a bond in the amount of P1,000.00 to it can be deduced beyond debate that petitioner
undertake, among others, that said Lau Yuen Lau Yuen Yeung while claiming not to be
Yeung would actually depart from the Philippines disqualified, does not and cannot allege that she
on or before the expiration of her authorized possesses all the qualifications to be naturalized,
period of stay in this country or within the period naturally because, having been admitted as a
as in his discretion the Commissioner of temporary visitor only on March 13, 1961, it is
Immigration or his authorized representative obvious at once that she lacks at least, the
might properly allow. After repeated extensions, requisite length of residence in the Philippines
petitioner Lau Yuen Yeung was allowed to stay in (Revised Naturalization Law, Sec. 2, Case No. 2,
the Philippines up to February 13, 1962 (Exhibit Sec. 3, Case No. 3).
'4'). On January 25, 1962, she contracted
marriage with Moy Ya Lim Yao alias Edilberto 'Were if the intention of the law that the alien
Aguinaldo Lim an alleged Filipino citizen. Because woman, to be deemed a citizen of the Philippines
of the contemplated action of respondent to by virtue of marriage to a Filipino citizen, need
confiscate her bond and order her arrest and only be not disqualified under the Naturalization
immediate deportation, after the expiration of her Law, it would have been worded "and who herself
authorized stay, she brought this action for is not disqualified to become a citizen of the
injunction with preliminary injunction. At the Philippines."
hearing which took place one and a half years
after her arrival, it was admitted that petitioner 'Second, Lau Yuen Yeung, a temporary Chinese
Lau Yuen Yeung could not write either English or woman visitor, whose authorized stay in the
Tagalog. Except for a few words, she could not Philippines, after repeated extensions thereof,
speak either English or Tagalog. She could not was to expire last February 28, 1962, having
name any Filipino neighbor, with a Filipino name married her co-plaintiff only on January 25, 1962,
except one, Rosa. She did not know the names of or just a little over one month before the expiry
her brothers-in-law, or sisters-in-law.' date of her stay, it is evident that said marriage
was effected merely for convenience to defeat or
"Under the facts unfolded above, the Court is of avoid her then impending compulsory departure,
the considered opinion, and so holds, that the not to say deportation. This cannot be permitted.
instant petition for injunction cannot be sustained
for the same reasons set forth in the Order of this 'Third, as the Solicitor General has well stated:
Court, dated March 19, 1962, the pertinent
portions of which read: "'5. That petitioner Lau Yuen Yeung, having been
admitted as a temporary alien visitor on the
'First, Section 15 of the Revised Naturalization strength of a deliberate and voluntary
Law provides: representation that she will enter and stay only
for a period of one month and thereby secured a
"'Effect of the naturalization on wife and children. visa, cannot go back on her representation to
— Any woman who is now or may hereafter be stay permanently without first departing from the
married to a citizen of the Philippines, and who Philippines as she had promised." (Chung Tiao
Bing, et al. vs. Commissioner of Immigration, G.

54 | C O N S T I 2 _ A r t i c l e I V _ C I T I Z E N S H I P
R. No. L-9966, September 29, 1956; Ong Se Lun CITIZEN OF THE PHILIPPINES BY VIRTUE OF HER
vs. Board of Commissioners, G. R. No. L-6017, MARRIAGE TO A FILIPINO CITIZEN, ONLY IF SHE
September 16, 1954; Sec. 9, last par., Phil. POSSESSES ALL THE QUALIFICATIONS AND NONE
Immigration Law). OF THE DISQUALIFICATIONS SPECIFIED IN THE
LAW.
The aforequoted argument of the Solicitor
General is well buttressed, not only by the II
decided cases of the Supreme Court on the point
mentioned above, but also on the very provisions THE LOWER COURT ERRED IN HOLDING THAT A
of Section 9, sub-paragraph (g) of the Philippine WOMAN FOREIGNER WHO DOES NOT POSSESS
Immigration Act of 1940 which reads: ANY OF THE DISQUALIFICATIONS FOR
CITIZENSHIP AND WHO MARRIED A FILIPINO
" 'An alien who is admitted as a non-immigrant CITIZEN IS STILL CONSIDERED AN ALIEN EVEN
cannot remain in the Philippines permanently. To AFTER SUCH MARRIAGE AS TO FALL WITHIN THE
obtain permanent admission, a non-immigrant REQUIREMENT OF SECTION 9, SUB-PARAGRAPH
alien must depart voluntarily to some foreign (9) OF THE PHILIPPINE IMMIGRATION ACT OF
country and procure from the appropriate 1940.
Philippine Consul the proper visa and thereafter
undergo examination by the Officers of the III
Bureau of Immigration at a Philippine port of
entry for determination of his admissibility in THE COURT ERRED IN CONCLUDING THAT LAU
accordance with the requirements of this Act. YUEN YEUNG'S MARRIAGE TO A FILIPINO CITIZEN
(This paragraph is added by Republic Act 503).'" WAS ONLY FOR CONVENIENCE, MERELY BECAUSE
(Sec. 9, subparagraph (g) of the Philippine THE SAME WAS CELEBRATED JUST OVER A
Immigration Act of 1940). MONTH BEFORE THE EXPIRY DATE OF HER
AUTHORIZED STAY.
'And fourth, respondent Commissioner of
Immigration is charged with the administration of IV
all laws relating to immigration (Sec. 3, Com. Act
No. 613) and in the performance of his duties in THE LOWER COURT ERRED IN FAILING TO FIND
relation to alien immigrants, the law gives the THAT THE COMMISSIONER OF IMMIGRATION
Commissioner of Immigration a wide discretion, a ACTED WITH ABUSE OF DISCRETION OR IN
quasi-judicial function in determining cases EXCESS OF HIS JURISDICTION WHEN SAID
presented to him (Pedro Uy So vs. Commissioner OFFICER THREATENED TO SEND OUT OF THE
of Immigration CA-G. R. No. 23336-R, Dec 15, COUNTRY PLAINTIFF LAU YUEN YEUNG WITH
1960), so that his decision thereon may not be WARNING THAT HER FAILURE TO DO SO WOULD
disturbed unless he acted with abuse of MEAN CONFISCATION OF HER BOND, ARREST
discretion or in excess of his jurisdiction.' AND IMMEDIATE DEPORTATION, IN SPITE OF THE
FACT THAT LAU YUEN YEUNG IS NOW A FILIPINO
"It may also be not amiss to state that wife Lau CITIZEN.
Yuen Yeung, while she barely and insufficiently
talk in broken Tagalog and English, she admitted V
that she cannot write either language."
THE LOWER COURT ERRED IN DISMISSING
The only matter of fact not clearly passed upon PLAINTIFFS-APPELLANTS' COMPLAINT AND IN
by His Honor which could have some bearing in REFUSING TO PERMANENTLY ENJOIN THE
the resolution of this appeal is the allegation in COMMISSIONER FROM ORDERING PLAINTIFF LAU
the brief of petitioners-appellants, not denied in YUEN YEUNG TO LEAVE THE PHILIPPINES AS A
the government's brief, that "in the hearing . . . , TEMPORARY VISITOR WHICH SHE IS NOT.
it was shown thru the testimony of the plaintiff
Lau Yuen Yeung that she does not possess any of
the disqualifications for naturalization." Of course,
as an additional somehow relevant factual VI
matter, it is also emphasized by said appellants
that during the hearing in the lower court, held THE LOWER COURT ERRED IN REFUSING TO
almost ten months after the alleged marriage of GRANT PLAINTIFFS-APPELLANTS' MOTION FOR
petitioners, "Lau Yuen Yeung was already carrying PRELIMINARY INJUNCTION EMBODIED IN THEIR
in her womb for seven months a child by her COMPLAINT, IN AN ORDER DATED MARCH 19,
husband." 1962. (PAGES 36-41, RECORD ON APPEAL).
Appellants have assigned six errors allegedly
committed by the court a quo, thus: We need not discuss these assigned errors
separately. In effect, the above decision upheld
I the two main grounds of objection of the Solicitor
General to the petition in the court below, viz:
THE LOWER COURT ERRED IN HOLDING THAT THE
CLAUSE 'WHO MIGHT HERSELF BE LAWFULLY "That petitioner Lau Yuen Yeung, having been
NATURALIZED' (OF SECTION 15, REVISED admitted as a temporary alien visitor on the
NATURALIZATION LAW) INCONTESTABLY IMPLIES strength of a deliberate and voluntary
THAT AN ALIEN WOMAN MAY BE DEEMED A representation that she will enter and stay only

55 | C O N S T I 2 _ A r t i c l e I V _ C I T I Z E N S H I P
for a period of one month and thereby secured a does not apply to aliens who after coming into
visa, cannot go back on her representation to the Philippines as temporary visitors, legitimately
stay permanently without first departing from the become Filipino citizens or acquire Filipino
Philippines as she had promised (Chung Tiao citizenship. Such change of nationality naturally
Bing, et al. vs. Commissioner of Immigration, G.R. bestows upon them the right to stay in the
No. L-9966, September 29, 1956; Ong Se Lun vs. Philippines permanently or not, as they may
Board of Commissioners, G.R. No. L-6017, Sept. choose, and if they elect to reside here, the
16, 1954, Sec. 9, last par. Phil. Immigration Law); immigration authorities may neither deport them
nor confiscate their bonds. True it is that this
"That the mere marriage of a Filipino citizen to an Court has vehement]y expressed disapproval of
alien does not automatically confer on the latter convenient ruses employed by aliens to convert
Philippine citizenship. The alien wife must their status from temporary visitors to permanent
possess all the qualifications required by law to residents in circumvention of the procedure
become a Filipino citizen by naturalization and prescribed by the legal provision already
none of the disqualifications. (Lee Suan Ay, mentioned, such as in Chiong Tiao Bing vs.
Alberto Tan and Lee Chiao vs. Galang, etc., G. R. Commissioner of Immigration, 99 Phil. 1020,
No. L-11855, Dec. 25, 1959)" wherein, thru Mr. Justice J.B.L. Reyes, the Court,
reiterating the ruling in Ong Se Lun vs. Board of
It is obvious from the nature of these objections Immigration Commissioners, 95 Phil. 785, said:
that their proper resolution would necessarily ". . . It is clear that if an alien gains admission to
cover all the points raised in appellants' the Islands on the strength of a deliberate and
assignments of error, hence, We will base our voluntary representation that he will enter only
discussions, more or less, on said objections. for a limited time, and thereby secures the
I. benefit of a temporary visa, the law will not allow
The first objection of the Solicitor General which him subsequently to go back on his
covers the matters dealt with in appellants' representation and stay permanently, without
second and fourth assignments of error does not first departing from the Philippines as he had
require any lengthy discussion. As a matter of promised. No officer can relieve him of the
fact, it seems evident that the Solicitor General's departure requirements of section 9 of the
pose that an alien who has been admitted into Immigration Act, under the guise of 'change' or
the Philippines as a non-immigrant cannot remain 'correction', for the law makes no distinctions,
here permanently unless he voluntarily leaves the and no officer is above the law. Any other ruling
country first and goes to a foreign country to would, as stated in our previous decision,
secure thereat from the appropriate Philippine encourage aliens to enter the Islands on false
consul the proper visa and thereafter undergo pretences; every alien so permitted to enter for a
examination by officers of the Bureau of limited time, might then claim a right to
Immigration at a Philippine port of entry for permanent admission, however flimsy such claim
determination of his admissibility in accordance should be, and thereby compel our government
with the requirements of the Philippine to spend time, money and effort to examining
Immigration Act of 1940, as amended by Republic and verifying whether or not every such alien
Act 503, is premised on the assumption that really has a right to take up permanent residence
petitioner Lau Yuen Yeung is not a Filipino citizen. here. In the meanwhile, the alien would be able
We note the same line of reasoning in the to prolong his stay and evade his return to the
appealed decision of the court a quo. Accordingly, port whence he came, contrary to what he
it is but safe to assume that were the Solicitor promised to do when he entered. The damages
General and His Honor of the view that said inherent in such ruling are self-evident."
petitioner had become ipso facto a Filipina by
virtue of her marriage to her Filipino husband, On the other hand, however, We cannot see any
they would have held her as entitled to assume reason why an alien who has been here as a
the status of a permanent resident without temporary visitor but who has in the meanwhile
having to depart as required of aliens by Section become a Filipino should be required to still leave
9(g) of the law. the Philippines for a foreign country, only to apply
thereat for a re-entry here and undergo the
In any event, to set this point at rest, We hereby process of showing that he is entitled to come
hold that portion of Section 9(g) of the back, when after all, such right has become
Immigration Act providing: incontestible as a necessary concomitant of his
assumption of our nationality by whatever legal
"An alien who is admitted as a non-immigrant means this hag been conferred upon him.
cannot remain in the Philippines permanently. To Consider, for example, precisely the case of the
obtain permanent admission, a non-immigrant minor children of an alien who is naturalized. It is
alien must depart voluntarily to some foreign indubitable that they become ipso facto citizens
country and procure from the appropriate of the Philippines. Could it be the law that before
Philippine consul the proper visa and thereafter they can be allowed permanent residence, they
undergo examination by the officers of the still have to be taken abroad so that they may be
Bureau of Immigration at a Philippine port of processed to determine whether or not they have
entry for determination of his admissibility in a right to have permanent residence here? The
accordance with the requirements of this Act." difficulties and hardships which such a
requirement entails and its seeming
unreasonableness argue against such a rather

56 | C O N S T I 2 _ A r t i c l e I V _ C I T I Z E N S H I P
absurd construction. Indeed, as early as 1957, in prescribed by the Revised Naturalization Law and
Ly Giok Ha vs. Galang, 101 Phil. 459, Mr. Justice prove in said naturalization proceeding not only
Concepcion, our present Chief Justice, already that she has all the qualifications and none of the
ruled thus: disqualifications provided in the law but also that
she has complied with all the formalities required
". . . (P)etitioners allege that, upon her marriage thereby like any other applicant for
to a Filipino, Ly Giok Ha became also a citizen of naturalization, 2 albeit said decision is not yet
the Philippines. Indeed, if this conclusion were part of our jurisprudence inasmuch as the motion
correct, it would follow that, in consequence of for its reconsideration is still pending resolution.
her marriage, she had been naturalized as such Appellants are in effect urging Us, however, in
citizen, and, hence the decision appealed from their first and second assignments of error, not
would have to be affirmed, for section 40(c) of only to reconsider Burca but to even reexamine
Commonwealth Act 613 provides that 'in the Lee Suan Ay which, as a matter of fact, is the
event of the naturalization as a Philippine citizen . prevailing rule, having been reiterated in all
. . of the alien on whose behalf the bond deposit subsequent decisions up to Go Im Ty. 3
is given, the bond shall be cancelled or the be
deposited shall be returned to the depositor or his
legal representative.'" (At. pp. 462-463) In other
words, the applicable statute itself more than Actually, the first case in which Section 15 of the
implies that the naturalization of an alien visitor Naturalization Law, Commonwealth Act 473,
as a Philippine citizen logically produces the underwent judicial construction was in the first Ly
effect of conferring upon him ipso facto all the Giok Ha case, 4 one almost identical to the one at
rights of citizenship including that of being bar. Ly Giok Ha, a woman of Chinese nationality,
entitled to permanently stay in the Philippines was a temporary visitor here whose authority to
outside the orbit of authority of the Commissioner stay was to expire on March 14, 1956. She filed a
of Immigration vis-a-vis aliens, if only because by bond to guaranty her timely departure. On March
its very nature and express provisions, the 8, 1956, eight days before the expiration of her
Immigration Law is a law only for aliens and is authority to stay, she married a Filipino by the
inapplicable to citizens of the Philippines. In the name of Restituto Lacasta. On March 9, 1956, her
sense thus discussed, therefore, appellants' husband notified the Commissioner of
second and fourth assignments of error are well Immigration of said marriage and, contending
taken. that his wife had become a Filipina by reason of
said marriage, demanded for the cancellation of
II. her bond, but instead of acceding to such
Precisely, the second objection of the Solicitor request, the Commissioner required her to leave,
General sustained by the trial judge is that and upon her failure to do so, on March 16, 1956,
appellant Lau Yuen Yeung's marriage to appellant the Commissioner confiscated her bond; a suit
Moya Lim Yao alias Edilberto Aguinaldo whose was filed for the recovery of the bond; the lower
Filipino citizenship is not denied did not have the court sustained her contention that she had no
effect of making her a Filipino, since it has not obligation to leave because she had become
been shown that she "might herself be lawfully Filipina by marriage, hence her bond should be
naturalized," it appearing clearly in the record returned. The Commissioner appealed to this
that she does not possess all the qualifications Court. In the said appeal, Mr. Justice Roberto
required of applicants for naturalization by the Concepcion, our present Chief Justice, spoke for
Revised Naturalization Law, Commonwealth Act the Court, thus:
473, even if she has proven that she does not
suffer from any of the disqualifications "The next and most important question for
thereunder. In other words, the Solicitor General determination is whether her marriage to a
implicitly concedes that had it been established Filipino justified or, at least, excused the aforesaid
in the proceedings below that appellant Lau Yuen failure of Ly Giok Ha to depart from the
Yeung possesses all the qualifications required by Philippines on or before March 14, 1956. In
the law of applicants for naturalization, she would maintaining the affirmative view, petitioners
have been recognized by the respondent as a alleged that, upon her marriage to a Filipino, Ly
Filipino citizen in the instant case, without Giok Ha became, also, a citizen of the Philippines.
requiring her to submit to the usual proceedings Indeed, if this conclusion were correct, it would
for naturalization. follow that, in consequence of her marriage, she
had been naturalized as such citizen, and, hence,
To be sure, this position of the Solicitor General is the decision appealed from would have to be
in accord with what used to be the view of this affirmed, for section 40(c) of Commonwealth Act
Court since Lee Suan Ay, et al. v. Emilio Galang, No. 613 provides that 'in the event of the
etc., et al., G.R. No. L-11855, promulgated naturalization as a Philippine citizen . . . of the
December 23, 1959, 106 Phil., 706, 713, 1 for it alien on whose behalf the bond deposit is given,
was only in Zita Ngo Burca vs. Republic, G.R. No. the bond shall be cancelled or the sum deposited
L-24252 which was promulgated on January 30, shall be returned to the depositor or his legal
1967 (19 SCRA 186), that over the pen of Mr. representative." Thus the issue boils down to
Justice Conrado Sanchez, this Court held that for whether an alien female who marries a male
an alien woman who marries a Filipino to be citizen of the Philippines follows ipso facto his
deemed a Filipina, she has to apply for political status.
naturalization in accordance with the procedure

57 | C O N S T I 2 _ A r t i c l e I V _ C I T I Z E N S H I P
"The pertinent part of section 15 of court, had the parties seemingly felt that there
Commonwealth Act No. 473, upon which was an issue on whether Ly Giok Ha may 'be
petitioners rely, reads: lawfully naturalized,' and this being a case of first
impression in our courts, we are of the opinion
'Any woman who is now or may hereafter be that, in the interest of equity and justice, the
married to a citizen of the Philippines, and who parties herein should be given an opportunity to
might herself be lawfully naturalized shall be introduce evidence, if they have any, on said
deemed a citizen of the Philippines.' issue." (At pp. 462-464.).

"Pursuant thereto, marriage to a male Filipino As may be seen, although not specifically in so
does not vest Philippine citizenship to his foreign many words, no doubt was left in the above
wife, unless she 'herself may be lawfully decision as regards the following propositions:
naturalized.' As correctly held in an opinion of the
Secretary of Justice (O.p. No. 52, series of 1950), 1. That under Section 15 of Commonwealth Act
* this limitation of section 15 excludes, from the 473, the Revised Naturalization Law, the marriage
benefits of naturalization by marriage, those of an alien woman to a Filipino makes her a
disqualified from being naturalized as citizens of Filipina, if she "herself might be lawfully
the Philippines under section 4 of said naturalized";
Commonwealth Act No. 473, namely:
2. That this Court declared as correct the opinion
'(a) Persons opposed to organized government or of the Secretary of Justice that the limitation of
affiliated with any association or group of persons Section 15 of the Naturalization Law excludes
who uphold and teach doctrines opposing all from the benefits of naturalization by marriage,
organized governments; only those disqualified from being naturalized
under Section 4 of the law quoted in the decision;
'(b) Persons defending or teaching the necessity
or propriety of violence, personal assault, or 3. That evidence to the effect that she is not
assassination for the success and predominance disqualified may be presented in the action to
of their ideas; recover her bond confiscated by the
Commissioner of Immigration;
'(c) Polygamists or believers in the practice of
polygamy; 4. That upon proof of such fact, she may be
recognized as Filipina; and
'(d) Persons convicted of crimes involving moral
turpitude; 5. That in referring to the disqualifications
enumerated in the law, the Court somehow left
'(e) Persons suffering from mental alienation or the impression that no inquiry need be made as
incurable contagious diseases; to qualifications, 5 specially considering that the
decision cited and footnoted several opinions of
'(f) Persons who, during the period of their the Secretary of Justice, the immediate superior
residence in the Philippines, have not mingled of the Commissioner of Immigration, the most
socially with the Filipinos, or who have not important of which are the following:
evinced a sincere desire to learn and embrace the
customs, traditions, and ideals of the Filipinos; "Paragraph (a), section 13 of Act No. 2927, as
amended, (now section 15, Commonwealth Act
'(g) Citizens or subjects of nations with whom No. 473), provided that 'any woman who is now
the . . . Philippines are at war, during the period or may hereafter be married to a citizen of the
of such war; Philippines, and who might herself be lawfully
naturalized shall be deemed a citizen of the
'(h) Citizens or subjects of a foreign country other Philippines.' A similar provision in the
than the United States, whose laws does not naturalization law of the United States has been
grant Filipinos the right to become naturalized construed as not requiring the woman to have
citizens or subjects thereof.' the qualifications of residence, good character,
etc., as in the case of naturalization by judicial
"In the case at bar, there is neither proof nor proceedings, but merely that she is of the race of
allegation in the pleadings that Ly Giok Ha does persons who may be naturalized. (Kelly v. Owen
not fall under any of the classes disqualified by [Dist. Col. 1868] 7 Wall 496, 5F, 11, 12; ex parte
law. Moreover, as the parties who claim that, Tryason [D. C. Wash. 1914] 215 F. 449, 27 Op.
despite her failure to depart from the Philippines Atty. Gen. 507). (Op. No. 168, s. 1940 of Justice
within the period specified in the bond in Sec. Jose Abad Santos.)
question, there has been no breach thereof,
petitioners have the burden of proving her "In a previous opinion rendered for your Office, I
alleged change of political status, from alien to stated that the clause 'who might herself be
citizen. Strictly speaking, petitioners have not lawfully naturalized', should be construed as not
made out, therefore a case against the requiring the woman to have the qualifications of
respondents-appellants. residence, good character, etc., as in cases of
naturalization by judicial proceedings, but merely
"Considering, however, that neither in the that she is of the race of persons who may be
administrative proceedings, nor in the lower naturalized. (Op. No. 79, s. 1940)

58 | C O N S T I 2 _ A r t i c l e I V _ C I T I Z E N S H I P
deportation proceedings and so, the husband
"Inasmuch as the race qualification has been filed prohibition and mandamus proceedings. The
removed by the Revised Naturalization Law, it lower court denied the petition. Although this
results that any woman who married a citizen of Court affirmed said decision, it held, on the other
the Philippines prior to or after June 17, 1939, and hand, that:
the marriage not having been dissolved, and on
the assumption that she possesses none of the
disqualifications mentioned in Section 4 of
Commonwealth Act No. 473, follows the "Granting the validity of marriage, this Court has
citizenship of her husband." (Op. No. 176, v. 1940 ruled in the recent case of Ly Giok Ha v. Galang,
of Justice Sec. Jose Abad Santos.) supra, p. 459, that the bare fact of a valid
marriage to a citizen does not suffice to confer his
"From the foregoing narration of facts, it would citizenship upon the wife. Section 15 of the
seem that the only material point of inquiry is as Naturalization Law requires that the alien woman
to the citizenship of Arce Machura. If he shall be who marries a Filipino must show, in addition,
found to be a citizen of the Philippines, his wife, that she 'might herself be lawfully naturalized' as
Mrs. Lily James Machura, shall likewise be a Filipino citizen. As construed in the decision
deemed a citizen of the Philippines pursuant to cited, this last condition requires proof that the
the provision of Section 15, Commonwealth Act woman who married a Filipino is herself not
No. 473, which reads in part as follows: disqualified under section 4 of the Naturalization
Law.
'Any woman who is now or may hereafter be
married to a citizen of the Philippines, and who "No such evidence appearing on record, the claim
might herself be lawfully naturalized shall be of assumption of Filipino citizenship by Tjioe Wu
deemed a citizen of the Philippines.' Suan, upon her marriage to petitioner, is
untenable. The lower court, therefore, committed
"The phrase 'who might herself be lawfully no error in refusing to interfere with the
naturalized', as contained in the above provision, deportation proceedings, where she can anyway
means that the woman who is married to a establish the requisites indispensable for her
Filipino citizen must not belong to any of the acquisition of Filipino citizenship, as well as the
disqualified classes enumerated in Section 4 of alleged validity of her Indonesian passport."
the Naturalization Law (Ops., Sec. of Jus., No. 28, (Ricardo Cua v. The Board of Immigration
s. 1950; No. 43, s. 1948, No. 95, s. 1941: Nos. 79 Commissioners, G. R. No. L-9997, May 22, 1957,
and 168, s. 1940). Under the facts stated in the 101 Phil. 521, 523.) [Emphasis supplied]
within papers, Mrs. Machura does not appear to
be among the disqualified classes mentioned in For emphasis, it is reiterated that in the above
the law. two cases, this Court expressly gave the parties
concerned opportunity to prove the fact that they
"It having been shown that Arce Machura or were not suffering from any of the
Arsenio Guevara was born as an illegitimate of a disqualifications of the law without the need of
Filipino mother, he should be considered as a undergoing any judicial naturalization proceeding.
citizen of the Philippines in consonance with the It may be stated, therefore, that according to the
well-settled rule that an illegitimate child follows above decisions, the law in this country, on the
the citizenship of his only legally recognized matter of the effect of marriage of an alien
parent, the mother (Op., Sec. of Jus., Nos. 58, 98 woman to a Filipino is that she thereby becomes
& 281, s. 1948; No. 96, s. 1949). Her husband a Filipina, if it can be proven that at the time of
being a Filipino, Mrs. Machura must necessarily such marriage, she does not possess any of the
be deemed as a citizen of the Philippines by disqualifications enumerated in Section 4 of the
marriage (Sec. 15, Com. Act No. 473.) (Op. No. Naturalization Law, without the need of
52, s. 1950 of Justice Sec. Ricardo Nepomuceno.) submitting to any naturalization proceedings
under said law.
The logic and authority of these opinions,
compelling as they are, must have so appealed to It is to be admitted that both of the above
this Court that five days later, on May 22, 1957, decisions made no reference to qualifications,
in Ricardo Cua v. The Board of Commissioners, that is, as to whether or not they need also to be
101 Phil. 521, Mr. Justice J.B.L. Reyes, reiterated proved, but, in any event, it is a fact that the
the same ruling on the basis of the following Secretary of Justice understood them to mean
facts: that such qualifications need not be possessed
nor proven. Then Secretary of Justice Jesus
Tjioe Wu Suan, an Indonesian, arrived in Manila Barrera, who later became a distinguished
on November 1, 1952, but it turned out that her member of this Court, 6 so ruled in opinions
passport was forged. On December 10, 1953, a rendered by him subsequent to Ly Giok Ha, the
warrant was issued for her arrest for purposes of most illustrative of which held:
deportation. Later, on December 20, 1953, she
married Ricardo Cua, a Filipino, and because of "At the outset it is important to note that an alien
said marriage, the Board of Special Inquiry woman married to a Filipino citizen needs only to
considered her a Filipina. Upon a review of the show that she 'might herself be lawfully
case, however, the Board of Immigration naturalized' in order to acquire Philippine
Commissioners insisted on continuing with the citizenship. Compliance with other conditions of

59 | C O N S T I 2 _ A r t i c l e I V _ C I T I Z E N S H I P
the statute, such as those relating to the "This view finds support in the case of Ly Giok Ha
qualifications of an applicant for naturalization et al., v. Galang et al. (G.R. No. L-10760,
through judicial proceedings, is not necessary promulgated May 17, 1957), where the Supreme
(See: Leonard v. Grant, 5 Fed. 11; 27 Ops. Atty. Court, construing the above-quoted section in the
Gen [U.S.] 507; Ops Sec. of Justice, No. 776, s. Revised Naturalization Law, held that 'marriage to
1940, and No. 111, s. 1953. a male Filipino does not vest Philippine citizenship
to his foreign wife, unless she 'herself may be
"This view finds support in the case of Ly Giok Ha lawfully naturalized,' and that 'this limitation of
et al. v. Galang et al., G.R. No. L-10760, Section 15 excludes from the benefits of
promulgated May 17, 1957, where the Supreme naturalization by marriage those disqualified from
Court, construing the abovequoted section of the being naturalized as citizens of the Philippines
Naturalization Law, held that 'marriage to a male under Section 4 of said Commonwealth Act No.
Filipino does not vest Philippine citizenship to his 473.' In other words, disqualification for any of
foreign wife, unless she 'herself may he lawfully the causes enumerated in section 4 of the Act is
naturalized,' and that 'this limitation of Section 15 the decisive factor that defeats the right of an
excludes, from the benefits of naturalization by alien woman married to a Filipino citizen to
marriage, those disqualified from being acquire Philippine citizenship." (Op. 57, s. 1958 of
naturalized as citizens of the Philippines under Justice Sec. Jesus G. Barrera.)
Section 4 of said Commonwealth Act No. 473.' In
other words, disqualification for any of the causes "The contention is untenable. The doctrine
enumerated in Section 4 of the Act is the decisive enunciated in the Ly Giok Ha case is not a new
factor that defeats the right of the foreign wife of one. In that case, the Supreme Court held that
a Philippine citizen to acquire Philippine under paragraph 1 of Section 15 of
citizenship. Commonwealth Act No. 473, 'marriage to a male
Filipino does not vest Philippine citizenship to his
xxx xxx xxx foreign wife unless she "herself may be lawfully
"Does petitioner, Lim King Bian, belong to any of naturalized"', and, quoting several earlier
these groups ? The Commissioner of Immigration opinions of the Secretary of Justice, namely: No.
does not say so but merely predicates his 52, s. 1950; No. 168, s. 1940; No. 95, s. 1941; No.
negative action on the ground that a warrant of 63, s. 1948; No. 28, s. 1950, 'this limitation of
deportation for 'overstaying' is pending against section 15 excludes from the benefits of
the petitioner. naturalization by marriage, those disqualified
from being naturalized as citizens of the
"We do not believe the position is well taken. Philippines under section 4 of said
Since the grounds for disqualification for Commonwealth Act No. 473." (Op. 134, B. 1962
naturalization are expressly enumerated in the of Justice Undersec. Magno S. Gatmaitan.)
law, a warrant of deportation not based on a
finding of unfitness to become naturalized for any It was not until more than two years later that, in
of those specified causes may not be invoked to one respect, the above construction of the law
negate acquisition of Philippine citizenship by a was importantly modified by this Court in Lee
foreign wife of a Philippine citizen under Section Suan Ay, supra, in which the facts were as
15 of the Naturalization Law. (Inclusio unius est follows:
exclusio alterius)" (Op. No. 12, s. 1958 of Justice
Undersec. Jesus G. Barrera.) "Upon expiration of the appellant Lee Suan Ay's
authorized period of temporary stay in the
"Regarding the steps that should be taken by an Philippines (25 March 1955), on 26 March 1955
alien woman married to a Filipino citizen in order the Commissioner of Immigration asked the
to acquire Philippine citizenship, the procedure bondsman to present her to the Bureau of
followed in the Bureau of Immigration is as Immigration within 24 hours from receipt of
follows: The alien woman must file a petition for notice, otherwise the bond will be confiscated
the cancellation of her alien certificate of (Annex 1). For failure of the bondsman to comply
registration alleging, among other things, that with the foregoing order, on 1 April 1955 the
she is married to a Filipino citizen and that she is Commissioner of Immigration ordered the cash
not disqualified from acquiring her husband's bond confiscated (Annex E). Therefore, there was
citizenship pursuant to section 4 of an order issued by the Commissioner of
Commonwealth Act No. 473, as amended. Upon Immigration confiscating or forfeiting the cash
the filing of said petition, which should be bond. Unlike in forfeiture of bail bonds in criminal
accompanied or supported by the joint affidavit of proceedings, where the Court must enter an
the petitioner and her Filipino husband to the order forfeiting the bail bond and the bondsman
effect that the petitioner does not belong to any must be given an opportunity to present his
of the groups disqualified by the cited section principal or give a satisfactory reason for his
from becoming naturalized Filipino citizen (please inability to do so, before final judgment may be
see attached CEB Form 1), the Bureau of entered against the bondsman, (section 15, Rule
Immigration conducts an investigation and 110; U.S. v. Bonoan, 22 Phil. 1.) in forfeiture of
thereafter promulgates its order or decision bonds posted for the temporary stay of an alien
granting or denying the petition." (Op. No. 38, B. in the Philippines, no court proceeding is
1958 of Justice Sec. Jesus G. Barrera.) necessary. Once a breach of the terms and
conditions of the undertaking in the bond is
committed, the Commissioner of Immigration

60 | C O N S T I 2 _ A r t i c l e I V _ C I T I Z E N S H I P
may, under the terms and conditions thereof, squarely raised therein similarly as in Lee Suan
declare it forfeited in favor of the Government." Ay, hence, anything said on the said matter would
(In the meanwhile, on April 1, 1955, Lee Suan Ay at best be no more than obiter dictum, Justice
and Alberto Tan, a Filipino, were joined in Reyes limited himself to holding that "Under
marriage by the Justice of the Peace of Las Piñas, Section 15 of the Naturalization Act, the wife is
Rizal.) deemed a citizen of the Philippines only if she
'might herself be lawfully naturalized,' so that the
Mr. Justice Sabino Padilla speaking for a fact of marriage to a citizen, by itself alone, does
unanimous court which included Justices not suffice to confer citizenship, as this Court has
Concepcion and Reyes who had penned Ly Giok previously ruled in Ly Giok Ha v. Galang, 54 O.G.
Ha and Ricardo Cua, ruled thus: 356, and in Cua v. Board of Immigration
"The fact that Lee Suan Ay (a Chinese) was Commissioners, 53 O.G. 8567; and there is here
married to a Filipino citizen does not relieve the no evidence of record as to the qualifications or
bondsman from his liability on the bond. The absence of disqualifications of appellee Kua Suy",
marriage took place on 1 April 1955, and the without explaining the apparent departure
violation of the terms and conditions of; the already pointed out from Ly Giok Ha and Ricardo
undertaking in the bond — failure to depart from Cua. Even Justice Makalintal, who wrote a
the Philippines upon expiration of her authorized separate concurring and dissenting opinion
period of temporary stay in the Philippines (25 merely lumped together Ly Giok Ha, Ricardo Cua
March 1955) and failure to report to the and Lee Suan Ay and opined that both
Commissioner of Immigration within 24 hours qualifications and non-disqualifications have to be
from receipt of notice — were committed before shown without elucidating on what seemed to be
the marriage. Moreover, the marriage of a Filipino departure from the said first two decisions.
citizen to an alien does not automatically confer
Philippine citizenship upon the latter. She must It was only on November 30, 1963 that to Mr.
possesses the qualifications required by law to Justice Roberto Regala fell the task of
become a Filipino citizen by naturalization. ** rationalizing the Court's position. In La San Tuang
There is no showing that the appellant Lee Suan v. Galang, G.R. No. L-18775, November 30, 1963,
Ay possesses all the qualifications and none of 9 SCRA 638, the facts were simply these: 10 San
the disqualifications provided for by law to Tuang, a Chinese woman, arrived in the
become a Filipino citizen by naturalization." Philippines on July 1, 1960 as a temporary visitor
with authority to stay up to June 30, 1961. She
Pertinently to be noted at once in this ruling, married a Filipino on January 7, 1961, almost six
which, to be sure, is the one relied upon in the months before the expiry date at her permit, and
appealed decision now before Us, is the fact that when she was refused to leave after her authority
the footnote of the statement therein that the to stay had expired, she refused to do so,
alien wife "must possess the qualifications claiming she had become a Filipina by marriage,
required by law to become a Filipino citizen by and to bolster her position, she submitted an
naturalization" makes reference to Section 15, affidavit stating explicitly that she does not
Commonwealth Act 473 and precisely, also to Ly possess any of the disqualifications enumerated
Giok Ha v. Galang, supra. As will be recalled, on in the Naturalization Law, Commonwealth Act
the other hand, in the opinions of the Secretary of 473. When the case reached the court, the trial
Justice explicitly adopted by the Court in Ly Giok judge held for the government that in addition to
Ha, among them, Opinion No. 176, Series of not having any of the disqualifications referred to,
1940, above-quoted, it was clearly held that "(I)n there was need that Lo San Tuang should have
a previous opinion rendered for your Office, I also possessed all the qualifications of residence,
stated that the clause 'who might herself be moral character, knowledge of a native principal
lawfully naturalized', should be construed as not dialect, etc., provided by the law. Recognizing
requiring the woman to have the qualifications of that the issue squarely to be passed upon was
residence, good character, etc., as in cases of whether or not the possession of all the
naturalization by judicial proceedings, but merely qualifications were indeed needed to be shown
that she is of the race by persons who may be apart from non-disqualification, Justice Regala
naturalized." (Op. Na. 79, s. 1940) held affirmatively for the Court, reasoning out
thus:

"It is to be noted that the petitioner has anchored


Since Justice Padilla gave no reason at all for the her claim for citizenship on the basis of the
obviously significant modification of the decision laid down in the case of Leonard v.
construction of the law, it could be said that there Grant, 5 Swy. 603, 5 F 11, where the Circuit Court
was need for clarification of the seemingly new of Oregon held that it was only necessary that the
posture of the Court. The occasion for such woman 'should be a person of the class or race
clarification should have been in Kua Suy, etc., et permitted to be naturalized by existing laws, and
al. vs. The Commissioner of Immigration, G.R. No. that in respect of the qualifications arising out of
L-13790, October 31, 1963, penned by Mr. Justice her conduct or opinions, being the wife of a
J.B.L. Reyes, who had rendered the opinion in citizen, she is to be regarded as qualified for
Ricardo Cua, supra, which followed that in Ly Giok citizenship, and therefore considered a citizen.'
Ha, supra, but apparently seeing no immediate (In explanation of its conclusion, the Court said:
relevancy in the case on hand then of the 'If, whenever during the life of the woman or
particular point in issue now, since it was not afterwards, the question of her citizenship arises

61 | C O N S T I 2 _ A r t i c l e I V _ C I T I Z E N S H I P
in a legal proceeding, the party asserting her 'First. Residence in the Philippine Islands for a
citizenship by reason of her marriage with a continuous period of not less than five years,
citizen must not only prove such marriage, but except as provided in the next following section;
also that the woman then possessed all the
further qualifications necessary to her becoming 'Second. To have conducted themselves in a
naturalized under existing laws, the statute will proper and irreproachable manner during the
be practically nugatory, if not a delusion and a entire Period of their residence in the Philippine
snare. The proof of the facts may have existed at Islands, in their relation with the constituted
the time of the marriage, but years after, when a government as well as with the community in
controversy arises upon the subject, it may be which they are living;
lost or difficult to find.')
'Third. To hold in the Philippine Islands real estate
"In other words, all that she was required to prove worth not less than one thousand pesos,
was that she was a free white woman or a woman Philippine currency, or have some known trade or
of African descent or nativity, in order to be profession; and
deemed an American citizen, because, with
respect to the rest of the qualifications on 'Fourth. To speak and write English, Spanish, or
residence, moral character, etc., she was some native tongue.
presumed to be qualified.
'In case the petitioner is a foreign subject, he
"Like the law in the United States, our former shall, besides, declare in writing and under oath
Naturalization Law (Act No. 2927, as amended by his intention of renouncing absolutely and
Act No. 3448) specified the classes of persons perpetually all faith and allegiance to the foreign
who alone might become citizens of the authority, state or sovereignty of which he was a
Philippines, even as it provided who were native, citizen or subject.'
disqualified. Thus, the pertinent provisions of that
law provided: "Applying the interpretation given by Leonard v.
Grant, supra, to our law as it then stood, alien
'Section 1. Who may become Philippine citizens. women married to citizens of the Philippines
— Philippine citizenship may be acquired by (a) must, in order to be deemed citizens of the
natives of the Philippines who are not citizens Philippines, be either (1) natives of the Philippines
thereof under the Jones Law; (b) natives of the who were not citizens thereof under the Jones
Insular possessions of the United States; (c) Law, or (2) natives of other Insular possessions of
citizens of the United States, or foreigners who the United States, or (3) citizens of the United
under the laws of the United States may become States or foreigners who under the laws of the
citizens of said country if residing therein. United States might become citizens of that
country if residing therein. With respect to the
'Section 2. Who are disqualified. — The following qualifications set forth in Section 3 of the former
cannot be naturalized as Philippine citizens: (a) law, they were deemed to have the same for all
Persons opposed to organized government or intents and purposes.
affiliated with any association or group of persons
who uphold and teach doctrines opposing all "But, with the approval of the Revised
organized government; (b) persons defending or Naturalization Law (Commonwealth Act No. 473)
teaching the necessity or propriety of violence, on June 17, 1939, Congress has since discarded
personal assault or assassination for the success class or racial consideration from the
and predominance of their ideas; (c) polygamists qualifications of applicants for naturalization
or believers in the practice of polygamy; (d) (according to its proponent, the purpose in
persons convicted of crimes involving moral eliminating this consideration was, first, to
turpitude; (e) persons suffering from mental remove the features of the existing naturalization
alienation or incurable contagious diseases; (f) act which discriminated in favor of the
citizens or subjects of nations with whom the Caucasian} and against Asiatics who are our
United States and the Philippines are at war, neighbors, and are related to us by racial affinity
during the period of such war. and, second, to foster amity with all nations
[Sinco, Phil. Political Law 502 — 11 ed.]), even as
'Section 3. Qualifications. — The persons it retained in Section 15 the phrase in question.
comprised in subsection (a) of section one of this The result is that the phrase 'who might herself
Act, in order to be able to acquire Philippine be lawfully naturalized' must be understood in
citizenship, must be not less than twenty-one the context in which it is now found, in a setting
years of age on the day of the hearing of their so different from that in which it was found by the
petition. Court in Leonard v. Grant.

'The persons comprised in subsections (b) and (c) "The only logical deduction from the elimination
of said section one shall, in addition to being not of class or racial consideration is that, as the
less than twenty-one years of age on the day of Solicitor General points out, the phrase 'who
the hearing of the petition, have all and each of might herself be lawfully naturalized' must now
the following qualifications: be understood as referring to those who under
Section 2 of the law are qualified to become
citizens of the Philippines.

62 | C O N S T I 2 _ A r t i c l e I V _ C I T I Z E N S H I P
"There is simply no support for the view that the naturalization petition of her husband is no
phrase 'who might herself be lawfully naturalized' assurance that he (the husband) would become a
must now be understood as requiring merely that citizen, as to make a basis for the extension of
the alien woman must not belong to the class of her temporary stay."
disqualified persons under Section 4 of the
Revised Naturalization Law. Such a proposition On the same day, in Tong Siok Sy v. Vivo, G.R. No.
misreads the ruling laid down in Leonard v. Grant. L-21136, December 27, 1963, 9 SCRA 876, Justice
A person who is not disqualified is not necessarily Barrera reiterated the same ruling and citing
qualified to become a citizen of the Philippines, particularly Lo San Tuang and Kua Suy, held that
because the law treats 'qualifications' and the marriage of Tong Siok Sy to a Filipino on
'disqualifications' in separate sections. And then November 12, 1960 at Taichung, Taiwan and her
it must not be lost sight of that even under the taking oath of Filipino citizenship before the
interpretation given to the former law, it was to Philippine Vice Consul at Taipeh, Taiwan on
be understood that the alien woman was not January 6, 1961 did not make her a Filipino
disqualified under Section 2 of that law. Leonard citizen, since she came here only in 1961 and
v. Grant did not rule that it was enough if the obviously, she had not had the necessary ten-
alien woman does not belong to the class of year residence in the Philippines required by the
disqualified persons in order that she may be law.
deemed to follow the citizenship of her husband: Such then was the status of the jurisprudential
What that case held was that the phrase 'who law on the matter under discussion when Justice
might herself be lawfully naturalized, merely Makalintal sought a reexamination thereof in
means that she belongs to the class or race of Choy King Tee v. Galang, G.R. No. L-18351, March
persons qualified to become citizens by 26, 1965, 13 SCRA 402. Choy King Tee's husband
naturalization — the assumption being always was granted Philippine citizenship on January 13,
that she is not otherwise disqualified. 1959 and took the oath on January 31 of the
same year, Choy King Tee first came to the
Philippines in 1955 and kept commuting between
Manila and Hongkong since then, her last visa
"We therefore hold that under the first paragraph before the case being due to expire on February
of Section 15 of the Naturalization Law, an alien 14, 1961. On January 27, 1961, her husband
woman, who is married to a citizen of the asked the Commissioner of Immigration to cancel
Philippines, acquires the citizenship of her her alien certificate of registration, as well as
husband only if she has all the qualifications and their child's, for the reason that they were
none of the disqualifications provided by law. Filipinos, and when the request was denied as to
Since there is no proof in this case that petitioner the wife, a mandamus was sought, which the trial
has all the qualifications and is not in any way court granted. Discussing anew the issue of the
disqualified, her marriage to a Filipino citizen need for qualifications, Justice Makalintal not on]y
does not automatically make her a Filipino citizen. reiterated the arguments of Justice Regala in Lo
Her affidavit to the effect that she is not in any San Tuang but added further that the ruling is
way disqualified to become a citizen of this believed to be in line with the national policy of
country was correctly disregarded by the trial selective admission to Philippine citizenship. 7
court, the same being self-serving."
No wonder, upon this authority, in Austria v.
Naturally, almost a month later in Sun Peck Yong Conchu, G.R. No. L-20716, June 22, 1965, 14
V. Commissioner of Immigration, G.R. No L-20784, SCRA 336, Justice J.P. Bengzon readily reversed
December 27, 1963, 9 SCRA 875, wherein the the decision of the lower court granting the writs
Secretary of Foreign Affairs reversed a previous of mandamus and prohibition against the
resolution of the preceding administration to Commissioner of Immigration, considering that
allow Sun Peck Yong and her minor son to await Austria's wife, while admitting she did not
the taking of the oath of Filipino citizenship of her possess all the qualifications for naturalization,
husband two years after the decision granting had submitted only an affidavit that she had none
him nationalization and required her to leave and of the disqualifications therefor. So also did
this order was contested in court, Justice Barrera Justice Dizon similarly hold eight days later in
held: Brito v. Commissioner, G.R. No. L-16829, June 30,
1965, 14 SCRA 539.
"In the case of Lo San Tuang v. Commissioner of
Immigration (G.R. No. L-18775, promulgated Then came the second Ly Giok Ha case 8 wherein
November 30, 1963; Kua Suy vs. Commissioner of Justice J. B. L. Reyes took occasion to expand on
Immigration, L-13790, promulgated October 31, the reasoning of Choy King Tee by illustrating with
1963), we held that the fact that the husband examples "the danger of relying exclusively on
became a naturalized citizen does not the absence of disqualifications, without taking
automatically make the wife a citizen of the into account the other affirmative requirements of
Philippines. It must also be shown that she herself the law." 9
possesses all the qualifications, and none of the
disqualifications, to become a citizen. In this Lastly, in Go Im Ty v. Republic, G.R. No. L-17919,
case, there is no allegation, much less showing, decided on July 30, 1966, 10 Justice Zaldivar held
that petitioner-wife is qualified to become a for the Court that an alien woman who is
Filipino citizen herself. Furthermore, the fact that widowed during the pendency of the
a decision was favorably made on the naturalization proceedings of her husband, in

63 | C O N S T I 2 _ A r t i c l e I V _ C I T I Z E N S H I P
order that she may be allowed to take the oath as change in the membership of the Court since Go
Filipino, must, aside from proving compliance with Im Ty, and of those who were in the Court already
the requirements of Republic Act 530, show that when Burca was decided, two members, Justice
she possesses all the qualifications and does not Makalintal and Castro concurred only in the
suffer from any of the disqualifications under the result, precisely, according to them, because they
Naturalization Law, citing in the process the wanted to leave the point now under discussion
decision to such effect discussed above, 1 1 even open in so far as they are concerned. 12 Truth to
as he impliedly reversed pro tanto the ruling in tell, the views and arguments discussed at length
Tan Lin v. Republic, G.R. No. L-13786, May 31, with copious relevant authorities, in the motion
1961, 2 SCRA 383. for reconsideration as well as in the
memorandum of the amici curiae 13 in the Burca
Accordingly, in Burca, Justice Sanchez premised case cannot just be taken lightly and summarily
his opinion on the assumption that the point now ignored, since they project in the most forceful
under discussion is settled law. manner, not only the legal and logical angles of
the issue, but also the imperative practical
In the case now at bar, the Court is again called aspects thereof in the light of the actual situation
upon to rule on the same issue. Under Section 15 of the thousands of alien wives of Filipinos who
of the Naturalization Law, Commonwealth Act have so long, even decades, considered
473, providing that: themselves as Filipinas and have always lived and
acted as such, officially or otherwise, relying on
"SEC. 15. Effect of the naturalization on wife and the long standing continuous recognition of their
children. — Any woman, who is now or may status as such by the administrative authorities in
hereafter be married to a citizen of the charge of the matter, as well as by the courts.
Philippines, and who might herself be lawfully Under these circumstances, and if only to afford
naturalized shall be deemed a citizen of the the Court an opportunity to consider the views of
Philippines. the five justices who took no part in Ga Im Ty
(including the writer of this opinion), the Court
"Minor children of persons naturalized under this decided to further reexamine the matter. After all,
law who have been born in the Philippines shall the ruling first laid in Lee Suan Ay, and later in Lo
be considered citizens thereof. San Tuang, Choy King Tee and the second (1966)
Ly Giok Ha, did not categorically repudiate the
"A foreign-born minor child, if dwelling in the opinions of the Secretary of Justice relied upon by
Philippines at the time of the naturalization of the the first (1959) Ly Giok Ha. Besides, some points
parent, shall automatically become a Philippine brought to light during the deliberations in this
citizen, and a foreign-born child, who is not in the case would seem to indicate that the premises of
Philippines at the time the parent is naturalized, the later cases can still bear further
shall be deemed a Philippine citizen only during consideration.
his minority, unless he begins to reside Whether We like it or not, it is undeniably factual
permanently in the Philippines when still a minor, that the legal provision We are construing,
in which case, he will continue to be a Philippine Section 15, aforequoted, of the Naturalization
citizen even after becoming of age. Law has been taken directly, copied and adopted
from its American counterpart. To be more
"A child born outside of the Philippines after the accurate, said provision is nothing less than a
naturalization of his parent, shall be considered a reenactment of the American provision. A brief
Philippine citizen, unless within one year after review of its history proves this beyond per
reaching the age of majority he fails to register adventure of doubt.
himself as a Philippine citizen at the American
Consulate of the country where he resides, and to The first Naturalization Law of the Philippines
take the necessary oath of allegiance. approved by the Philippine Legislature under
American sovereignty was that of March 26,
is it necessary, in order that an alien woman who 1920, Act No. 2927. Before then, as a
marries a Filipino or who is married to a man who consequence of the Treaty of Paris, our citizenship
subsequently becomes a Filipino, may become a laws were found only in the Organic Laws, the
Filipino citizen herself, that, aside from not Philippine Bill of 1902, the Act of the United
suffering from any of the disqualifications States Congress of March 23, 1912 and later the
enumerated in the law, she must also possess all Jones Law of 1916. In fact, Act No. 2927 was
the qualifications required by said law? If nothing enacted pursuant to express authority granted by
but the unbroken line from Lee Suan Ay to Go Im the Jones Law. For obvious reasons, the
Ty, as recounted above, were to be considered, it Philippines gained autonomy on the subjects of
is obvious that an affirmative answer to the citizenship and immigration only after the
question would be inevitable, specially, if it is effectivity of the Philippine Independence Act.
noted that the present case was actually This made it practically impossible for our laws on
submitted for decision on January 21, 1964 yet, said subject to have any perspective or
shortly after Lo San Tuang, Tong Siok Sy and Sun orientation of our own; everything was American.
Peck Yong, all supra, and even before Choy King
Tee, supra, were decided. There are other
circumstances, however, which make it desirable,
if not necessary, that the Court take up the The Philippine Bill of 1902 provided pertinently:
matter anew. There has been a substantial

64 | C O N S T I 2 _ A r t i c l e I V _ C I T I Z E N S H I P
"SECTION 4. That all inhabitants of the Philippine Filipino before the marriage, although Section 13
Islands continuing to reside therein who were thereof provided thus:
Spanish subjects on the eleventh day of April,
eighteen-hundred and ninety-nine, and then "SEC. 13. Right of widow and children of
resided in said Islands, and their children born petitioners who have died. — In case a petitioner
subsequent thereto, shall be deemed and held to should die before the final decision has been
be citizens of the Philippine Islands and as such rendered, his widow and minor children may
entitled to the protection of the United States, continue the proceedings. The decision rendered
except such as shall have elected to preserve in the case shall, so far as the widow and minor
their allegiance to the Crown of Spain in children are concerned, produce the same legal
accordance with the provisions of the treaty of effect as if it had been rendered during the life of
peace between the United States and Spain the petitioner."
signed at Paris December tenth, eighteen
hundred and ninety-eight." It was not until November 30, 1928, upon the
approval of Act 3448, amending Act 2977, that
This Section 4 of the Philippine Bill of 1902 was the following provisions were added to the above
amended by Act of Congress of March 23, 1912, Section 13:
by adding a provision as follows:
"Provided, That the Philippine Legislature is "SECTION 1. The following new sections are
hereby authorized to provide by law for the hereby inserted between sections thirteen and
acquisition of Philippine citizenship by those fourteen of Act Numbered Twenty-nine hundred
natives of the Philippine Islands who do not come and Twenty-seven:
within the foregoing provisions, the natives of
other insular possessions of the United States, 'SEC. 13 (a). Any woman who is now or may
and such other persons residing in the Philippine hereafter be married to a citizen of the Philippine
Islands who would become citizens of the United Islands and who might herself be lawfully
States, under the laws of the United States, if naturalized, shall be deemed a citizen of the
residing therein." Philippine Islands.

The Jones Law reenacted these provisions 'SEC. 13 (b). Children of persons who have been
substantially: duly naturalized under this law, being under the
age of twenty-one years at the time of the
"SECTION 2. That all inhabitants of the Philippine naturalization of their parents, shall, if dwelling in
Islands who were Spanish subjects on the the Philippine Islands, be considered citizens
eleventh day of April, eighteen hundred and thereof.
ninety-nine, and then resided in said islands, and
their children born subsequent thereto, shall be 'SEC. 13 (c). Children of persons naturalized
deemed and held to be citizens of the Philippine under this law who have been born in the
Islands, except such as shall have elected to Philippine Islands after the naturalization of their
preserve their allegiance to the Crown of Spain in parents shall be considered citizens thereof.' "
accordance with the provisions of the treaty of
peace between the United States and Spain, When Commonwealth Act 473, the current
signed at Paris December tenth, eighteen naturalization law, was enacted on June 17, 1939,
hundred and ninety-eight and except such others the above Section 13 became its Section 15
as have since become citizens of some other which has already been quoted earlier in this
country: Provided, That the Philippine Legislature, decision. As can be seen, Section 13(a)
herein provided for, is hereby authorized to abovequoted was re-enacted practically word for
provide by law for the acquisition of Philippine word in the first paragraph of this Section 15
citizenship by those natives of the Philippine except for the change of Philippine Islands to
Islands who do not come within the foregoing Philippines. And it could not have been on any
provisions, the natives of the insular possessions other basis than this legislative history of our
of the United States, and such other persons naturalization law that each and everyone of the
residing in the Philippine Islands who are citizens decisions of this Court from the first Ly Giok Ha to
of the United States under the laws of the United Go Im Ty, discussed above, were rendered.
States if residing therein."
As stated earlier, in the opinion of Chief Justice
For aught that appears, there was nothing in any Concepcion in the first Ly Giok Ha, it was quite
of the said organic laws regarding the effect of clear that for an alien woman who marries a
marriage to a Filipino upon the nationality of an Filipino to become herself a Filipino citizen, there
alien woman, albeit under the Spanish Civil Code is no need for any naturalization proceeding
provisions on citizenship, Articles 17 to 27, which because she becomes a Filipina ipso facto from
were, however, abrogated upon the change of the time of such marriage, provided she does not
sovereignty, it was unquestionable that the suffer any of the disqualifications enumerated in
citizenship of the wife always followed that of the Section 4 of Commonwealth Act 473, with no
husband. Not even Act 2927 contained any mention being made of whether or not the
provision regarding the effect of naturalization of qualifications enumerated in Section 2 thereof
an alien upon the citizenship of his alien wife, nor need be shown. It was only in Lee Suan Ay in
of the marriage of such alien woman with a 1959 that the possession of qualifications were
native born Filipino or one who had become a specifically required, but it was not until 1963, in

65 | C O N S T I 2 _ A r t i c l e I V _ C I T I Z E N S H I P
Lo San Tuang, that Justice Regala reasoned out there was at the time a similarity between the
why the possession of the qualifications provided naturalization laws of the two countries, and
by the law should also be shown to be possessed hence there was reason to accord here
by the alien wife of a Filipino, for her to become a persuasive force to the interpretation given in the
Filipina by marriage. United States to the statutory provision
concerning the citizenship of alien women
As may be recalled, the basic argument advanced marrying American citizens.
by Justice Regala was briefly as follows: That "like
the law in the United States, our Naturalization "This Court, however, believes that such reason
Law specified the classes of persons who alone has ceased to exist since the enactment of the
might become citizens, even as it provided who Revised Naturalization Law (Commonwealth Act
were disqualified," and inasmuch as No. 473) on June 17, 1939. The racial restrictions
Commonwealth Act 473, our Naturalization Law have been eliminated in this Act, but the
since 1939 did not reenact the section providing provision found in Act No. 3448 has been
who might become citizens, allegedly in order to maintained. It is logical to presume that when
remove racial discrimination in favor of Congress chose to retain the said provision —
Caucasians and against Asiatics, "the only logical that to be deemed a Philippine citizen upon
deduction . . . is that the phrase 'who might marriage the alien wife must be one 'who might
herself be lawfully naturalized' must now be herself be lawfully naturalized,' the reference is
understood as referring to those who under no longer to the class or race to which the woman
Section 2 of the law are qualified to become belongs, for class or race has become immaterial,
citizens of the Philippines" and "there is simply no but to the qualifications and disqualifications for
support for the view that the phrase 'who might naturalization as enumerated in Sections 2 and 4
herself be lawfully naturalized' must now be of the statute. Otherwise the requirement that
understood as requiring merely that the alien the woman 'might herself be lawfully naturalized'
woman must not belong to the class of would be meaningless surplusage, contrary to
disqualified persons under Section 4 of the settled norms of statutory construction.
Revised Naturalization Law." 14
"The rule laid down by this Court in this and in
A similar line of reasoning was followed in Choy other cases heretofore decided is believed to be
King Tee, which for ready reference may be in line with the national policy of selective
quoted: admission to Philippine citizenship, which after all
is a privilege granted only to those who are found
"The question has been settled by the uniform worthy thereof, and not indiscriminately to
ruling of this Court in a number of cases. The anybody at all on the basis alone of marriage to a
alien wife of a Filipino citizen must first prove that man who is a citizen of the Philippines,
she has all the qualifications required by Section irrespective of moral character, ideological
2 and none of the disqualifications enumerated in beliefs, and identification with Filipino ideals,
Section 4 of the Naturalization Law before she customs and traditions.
may he deemed a Philippine citizen (Lao Chay v.
Galang, L-19977, Oct. 30, 1964, citing Lo San
Tuang v. Galang, L-18775, Nov. 30, 1963; Sun
Peck Yong v. Commissioner of Immigration, L- "Appellee here having failed to prove that she has
20784, December 27, 1963; Tong Siok Sy v. Vivo, all the qualifications for naturalization, even,
L-21136, December 27, 1963). The writer of this indeed, that she has none of the disqualifications,
opinion has submitted the question anew to the she is not entitled to recognition as a Philippine
court for a possible reexamination of the said citizen."
ruling in the light of the interpretation of a similar
law in the United States after which Section 15 of In the second Ly Giok Ha, the Court further
our Naturalization Law was patterned. That law fortified the arguments in favor of the same
was section 2 of the Act of February 10, 1855 conclusion thus:
(Section 1994 of the Revised Statutes of the
U.S.). The local law, Act No. 3448, was passed on "On cross-examination, she (Ly Giok Ha) failed to
November 30, 1928 as an amendment to the establish that: (1) she has been residing in the
former Philippine Naturalization Law, Act No. Philippines for a continuous period of at least (10)
2927, which was approved on March 26, 1920. years (p. 27, t.s.n., id.); (2) she has a lucrative
Under this Naturalization Law, acquisition of trade, profession, or lawful occupation (p. 13.
Philippine citizenship was limited to three classes t.s.n., id.); and (3) she can speak and write
of persons, (a) Natives of the Philippines who English, or any of the principal Philippine
were not citizens thereof; (b) natives of the other languages (pp. 12, 13, t.s.n., id.)
insular possessions of the United States; and (c)
citizens of the United States, or foreigners who, "While the appellant Immigration Commissioner
under the laws of the United States, may become contends that the words emphasized indicate
citizens of the latter country if residing therein. that the present Naturalization Law requires that
The reference in subdivision (c) to foreigners who an alien woman who marries a Filipino husband
may become American Citizens is restrictive in must possess the qualifications prescribed by
character, for only persons of certain specified section 2 in addition to not being disqualified
races were qualified thereunder. In other words, under any of the eight ('a' to 'h') subheadings of
in so far as racial restrictions were concerned section 4 of Commonwealth Act No. 473, in order

66 | C O N S T I 2 _ A r t i c l e I V _ C I T I Z E N S H I P
to claim our citizenship by marriage, both the disqualified under section 4, as long as she is not
appellee and the court below (in its second 'opposed to organized government,' nor affiliated
decision) sustain the view that all that the law to groups 'upholding or teaching doctrines
demands is that the woman be not disqualified opposing all organized governments', nor
under section 4. 'defending or teaching the necessity or propriety
of violence, personal assault or assassination for
"At the time the present case was remanded to the success or predominance of their ideas.' Et
the court of origin (1960) the question at issue sic de caeteris.
could be regarded as not conclusively settled,
there being only the concise pronouncement in "The foregoing instances should suffice to
Lee Suan Ay, et al. v. Galang, G. R. No. L-11855, illustrate the danger of relying exclusively on the
Dec. 23, 1959, to the effect that: absence of disqualifications, without taking into
account the other affirmative requirements of the
'The marriage of a Filipino citizen to an alien does law, which, in the case at bar, the appellee Ly
not automatically confer Philippine citizenship Giok Ha admittedly does not possess.
upon the latter. She must possess the
qualifications required by law to become a Filipino "As to the argument that the phrase 'might
citizen by naturalization.' herself be lawfully naturalized' was derived from
the U.S. Revised Statutes (section 1994) and
"Since that time, however, a long line of decisions should be given the same territorial and racial
of this Court has firmly established the rule that significance given to it by American courts, this
the requirement of section 15 of Commonwealth Court has rejected the same in Lon San Tuang v.
Act 473 (the Naturalization Act), that an alien Galang, L-18775, November 30, 1963; and in
woman married to a citizen should be one who Choy King Tee v. Galang, L-18351, March 26,
'might herself be lawfully naturalized," means not 1965."
only woman free from the disqualifications
enumerated in section 4 of the Act but also one It is difficult to minimize the persuasive force of
who possesses the qualifications prescribed by the foregoing rationalizations, but a closer study
section 2 of Commonwealth Act 473 (San Tuan v. thereof cannot but reveal certain relevant
Galang, L-18775, Nov. 30, 1963; Sun Peck Yong v. considerations which adversely affect the
Com. of Immigration, L-20784, Dec. 27, 1963; premises on which they are predicated, thus
Tong Siok Sy v. Vivo, L-21136, Dec. 27, 1963; rendering the conclusions arrived thereby not
Austria v. Conchu, L-20716, June 22, 1965; Choy entirely unassailable.
King Tee v. Galang, L-18351, March 26, 1965;
Brito v. Com. of Immigration, L-16829, June 30, 1. The main proposition, for instance, that in
1965). eliminating Section 1 of Act 2927 providing who
are eligible for Philippine citizenship, the purpose
"Reflection will reveal why this must be so. The of Commonwealth Act 473, the Revised
qualifications prescribed under section 2 of the Naturalization Law, was to remove the racial
Naturalization Act, and the disqualifications requirements for naturalization, thereby opening
enumerated in its section 4 are not mutually the door of Filipino nationality to Asiatics instead
exclusive; and if all that were to be required is of allowing the admission thereto of Caucasians
that the wife of a Filipino be not disqualified only, suffers from lack of exact accuracy. It is
under section 4, the result might well be that important to note, to start with, that
citizenship would be conferred upon persons in Commonwealth Act 473 did away with the whole
violation of the policy of the statute. For example, Section 1 of Act 2927 which reads thus:
section 4 disqualifies only —
"SECTION 1. Who may become Philippines
'(c) Polygamists or believers in the practice of citizens. — Philippine citizenship may be acquired
polygamy; and by: (a) natives of the Philippines who are not
citizens thereof under the Jones Law; (b) natives
'(d) Persons convicted of crimes involving moral of the other Insular possessions of the United
turpitude,' States; (c) citizens of the United States, or
foreigners who under the laws of the United
so that a blackmailer, or a maintainer of gambling States may become citizens of said country if
or bawdy houses, not previously convicted by a residing therein."
competent court would not be thereby
disqualified; still, it is certain that the law did not and not only subdivision (c) thereof. Nowhere in
intend such person to be admitted as a citizen in this whole provision was there any mention of
view of the requirement of section 2 that an race or color of the persons who were then
applicant for citizenship 'must be of good moral eligible for Philippine citizenship. What is more
character.' evident from said provision is that it reflected the
inevitable subordination of our legislation during
"Similarly, the citizen's wife might be a convinced the pre-Commonwealth American regime to the
believer in racial supremacy, in government by understandable limitations flowing from our
certain selected classes, in the right to vote status as a territory of the United States by virtue
exclusively by certain 'herrenvolk', and thus of the Treaty of Paris. In fact, Section 1 of Act
disbelieve in the principles underlying the 2927 was precisely approved pursuant to express
Philippine Constitution; yet she would not be authority, without which it could not have been

67 | C O N S T I 2 _ A r t i c l e I V _ C I T I Z E N S H I P
done, granted by an amendment to Section 4 of 6, 1882 not being among those expressly
the Philippine Bill of 1902 introduced by the Act repealed by this law, hence it is clear that when
of the United States Congress of March 23, 1912 Act 2927 was enacted, subdivision (c) of its
and which was reenacted as part of the Jones Law Section 1 could not have had any connotation of
of 1916, the pertinent provisions of which have racial exclusion necessarily, even if it were traced
already been quoted earlier. In truth, therefore, it back to its origin in the Act of the United States
was because of the establishment of the Congress of 1912 already mentioned above. 16
Philippine Commonwealth and in the exercise of Thus, it would seem that the nationalization in
our legislative autonomy on citizenship matters the quoted decisions predicated on the theory
under the Philippine Independence Act that that the elimination of Section 1 of Act 2927 by
Section 1 of Act 2927 was eliminated, 15 and not Commonwealth Act 473 was purposely for no
purposely to eliminate any racial discrimination other end than the abolition of racial
contained in our Naturalization Law. The discrimination in our naturalization law has no
Philippine Legislature naturally wished to free our clear factual basis. 17
Naturalization Law from the impositions of
American legislation. In other words, the fact that 3. In view of these considerations, there appears
such discrimination was removed was one of the to be no cogent reason, why the construction
effects rather than the intended purpose of the adopted in the opinions of the Secretary of Justice
amendment. referred to in the first Ly Giok Ha decision of the
2. Again, the statement in Choy King Tee to the Chief Justice should not prevail. It is beyond
effect that "the reference in subdivision (c) (of dispute that the first paragraph of Section 15 of
Section 1 of Act 2927) to foreigners who may Commonwealth Act 473 is a reenactment of
become American citizens is restrictive in Section 13(a) of Act 2927, as amended by Act
character, for only persons of certain specified 3448, and that the latter is nothing but an exact
races were qualified thereunder" fails to consider copy, deliberately made, of Section 1994 of the
the exact import of the said subdivision. Revised Statutes of the United States as it stood
Explicitly, the thrust of the said subdivision was before it repeal in 1922. 18 Before such repeal,
to confine the grant under it of Philippine the phrase "who might herself be lawfully
citizenship only to the three classes of persons naturalized" found in said Section 15 had a
therein mentioned, the third of which were definite unmistakable construction uniformly
citizens of the United States and, corollarily, followed in all courts of the United States that
persons who could be American citizens under had occasion to apply the same and which,
her laws. The words used in the provision do not therefore, must be considered as if it were written
convey any idea of favoring aliens of any in the statute itself. It is almost trite to say that
particular race or color and of excluding others, when our legislators enacted said section, they
but more accurately, they refer to all the knew of its unvarying construction in the United
disqualifications of foreigners for American States and that, therefore, in adopting verbatim
citizenship under the laws of the United States. the American statute, they have in effect
The fact is that even as of 1906, or long before incorporated into the provision, as thus enacted,
1920, when our Act 2927 became a law, the the construction given to it by the American
naturalization laws of the United States already courts as well as the Attorney General of the
provided for the following disqualifications in the United States and all administrative authorities
Act of the Congress of June 29, 1906: charged with the implementation of the
naturalization and immigration laws of that
"SEC. 7. That no person who disbelieves in or who country. (Lo Cham v. Ocampo, 77 Phil., 635
is opposed to organized government, or who is a [1946]; Laxamana v. Baltazar, 92 Phil., 32 [1952];
member of or affiliated with any organization Hartley v. Commissioner, 295 U.S. 216, 79 L. ed.
entertaining and teaching such disbelief in or 1399, 55 S Ct. 756 [1935]; Helvering v. Windmill,
opposition to organized government, or who 305 U.S. 79, 83 L ed. 52, 59 S Ct. 45 [1938];
advocates or teaches the duty, necessity, or Helvering v. R. J. Reynolds Tobacco Co., 306 U.S.
propriety of the unlawful assaulting or killing of 110, 83 L ed. 536, 59 S Ct. 423 [1939]. [p. 32,
any officer or officers, either of specific Memo of Amicus Curiae]).
individuals or of officers generally, of the
Government of the United States, or of any other A fairly comprehensive summary of the said
organized government, because of his or their construction by the American courts and
official character, or who is a polygamist, shall be administrative authorities is contained in United
naturalized or be made a citizen of the United Stats of America ex rel. Dora Sejnensky v. Robert
States." E. Tod, Commissioner of Immigration, Appt., 285
Fed. 523, decided November 14, 1922, 26 A. L. R.
and all these disqualified persons were, therefore, 1316 as follows:
ineligible for Philippine citizenship under Section
1 of Act 2927 even if they happened to be "Section 1994 of the Revised Statutes (Comp.
Caucasians. More importantly, as a matter of fact, Stat. § 3948, 2 Fed. Sta. Anno. 2d ed. p. 117)
said American law, which was the first "Act to provides as follows: 'Any woman who is now or
Establish a Bureau of Immigration and may hereafter be married to a citizen of the
Naturalization and to Provide for a Uniform Rule United States, and who might herself be lawfully
for Naturalization of Aliens throughout the United naturalized, shall be deemed a citizen.'
States" contained no racial disqualification
requirement, except as to Chinese, the Act of May

68 | C O N S T I 2 _ A r t i c l e I V _ C I T I Z E N S H I P
"Section 1944 of the Revised Stat. is said to construed as in effect declaring that an alien
originate in the Act of Congress of February 10, woman, who is of the class or race that may be
1855 (10 Stat. at L. 604, chap. 71), which in its lawfully naturalized under the existing laws, and
second section provided 'that any woman, who who marries a citizen of the United States, is such
might lawfully be naturalized under the existing a citizen also, and it was not necessary that it
laws, married, or who shall be married to a citizen should appear affirmatively that she possessed
of the United States, shall be deemed and taken the other qualifications at the time of her
to be a citizen.' marriage to entitle her to naturalization.

"And the American Statute of 1855 is "In 1882, the Act of 1855 came before Mr. Justice
substantially a copy of the earlier British Statute Harlan, sitting in the circuit court, in United States
7 & 8 Vict. chap. 66, . . . 16, 1844, which provided v. Kellar, 13 Fed. 82. An alien woman, a subject of
that 'any woman married, or who shall be Prussia came to the United States and married
married, to a natural-born subject or person here a naturalized citizen. Mr. Justice Harlan, with
naturalized, shall be deemed and taken to be the concurrence of Judge Treat, held that upon
herself naturalized, and have all the rights and her marriage she became ipso facto a citizen of
privileges of a natural born subject.' the United States as fully as if she had complied
with all of the provisions of the statutes upon the
"The Act of Congress of September 22, 1922 (42 subject of naturalization. He added: 'There can be
Stat. at L. 1021, chap. 411, Comp. Stat. § 4358b, no doubt of this, in view of the decision of the
Fed. Stat. Anno. Supp. 1922, p. 255), being 'An Supreme Court of the United States in Kelly v.
Act Relative to the Naturalization and Citizenship Owen, 7 Wall. 496, 19 L. ed. 283.' The alien
of Married Women,' in § 2, provides 'that any 'belonged to the class of persons' who might be
woman who marries a citizen of the United States lawfully naturalized.
after the passage of this Act, . . . shall not
become a citizen of the United States by reason "In 1904, in Hopkins v. Fachant, 65 C. C. A. 1, 130
of such marriage . . .' Fed. 839, an alien woman came to the United
States from France and entered the country
"Section 6 of the act also provides 'that . . . 1994 contrary to the immigration laws. The
of the Revised Statutes . . . are repealed.' immigration authorities took her into custody at
the port of New York, with the view of deporting
"Section 6 also provides that 'such repeal shall her. She applied for her release under a writ of
not terminate citizenship acquired or retained habeas corpus, and pending the disposition of the
under either of such sections, . . .' meaning § § 2 matter she married a naturalized American
and 6. So that this Act of September 22, 1922, citizen. The circuit court of appeals for the ninth
has no application to the facts of the present circuit held, affirming the court below, that she
case, as the marriage of the relator took place was entitled to be discharged from custody. The
prior to its passage. This case, therefore, depends court declared: 'The rule is well settled that her
upon the meaning to be attached to § 1994 of the marriage to a naturalized citizen of the United
Revised Statutes. States entitled her to be discharged. The status of
the wife follows that of her husband, . . . and by
"In 1868 the Supreme Court, in Kelly v. Owen, 7 virtue of her marriage her husband's domicil
Wall. 496, 498, 19 L. ed. 283, 284, construed this became her domicil.'
provision as found in the Act of 1855 as follows:
'The term, "who might lawfully be naturalized "In 1908, the circuit court for the district of Rhode
under the existing laws," only limits the Island in Re Rustigian, 165 Fed. 980, had before it
application of the law to free white women. The the application of a husband for his final decree
previous Naturalization Act, existing at the time, of naturalization. It appeared that at that time his
only required that the person applying for its wife was held by the immigration authorities at
benefits should be "a free white person," and not New York on the ground that she was afflicted
an alien enemy.' with a dangerous and contagious disease.
Counsel on both sides agreed that the effect of
"This construction limited the effect of the statute the husband's naturalization would be to confer
to those aliens who belonged to the class or race citizenship upon the wife. In view of that
which might be lawfully naturalized, and did not contingency District Judge Brown declined to pass
refer to any of the other provisions of the upon the husband's application for naturalization,
naturalization laws as to residence or moral and thought it best to wait until it was
character, or to any of the provisions of the determined whether the wife's disease was
immigration laws relating to the exclusion or curable. He placed his failure to act on the
deportation of aliens. express ground that the effect of naturalizing the
husband might naturalize her. At the same time
"In 1880, in Leonard v. Grant (C. C.) 5 Fed. 11, he expressed his opinion that the husband's
District Judge Deady also construed the Act of naturalization would not effect her naturalization,
1855, declaring that 'any woman who is now or as she was not one who could become lawfully
may hereafter be married to a citizen of the naturalized. 'Her own capacity (to become
United States, and might herself be lawfully naturalized),' the court stated, 'is a prerequisite
naturalized, shall be deemed a citizen.' He held to her attaining citizenship. If herself lacking in
that 'upon the authorities, and the reason, if not that capacity, the married status cannot confer it
the necessity, of the case,' the statute must be upon her.' Nothing, however, was actually

69 | C O N S T I 2 _ A r t i c l e I V _ C I T I Z E N S H I P
decided in that case, and the views expressed immigration authorities in the following
therein are really nothing more than mere dicta. September, and in October a warrant for her
But, if they can be regarded as something more deportation was issued. Pending hearings as to
than that, we find ourselves, with all due respect the validity of that order, she was paroled in the
for the learned judge, unable to accept them. custody of her counsel. The ground alleged for
her deportation was that she was afflicted with a
"In 1909, in United States ex rel. Nicola v. dangerous and contagious disease at the time of
Williams, 173 Fed, 626, District Judge Learned her entry. One of the reasons assigned to defeat
Hand held that an alien woman, a subject of the deportation was that the woman had married a
Turkish Empire, who married an American citizen citizen of the United States pending the
while visiting Turkey, and then came to the United proceedings for her deportation. Judge Dodge
States, could not be excluded, although she had, declared himself unable to believe that a
at the time of her entry, a disease which under marriage under such circumstances 'is capable of
the immigration laws would have been sufficient having the effect claimed, in view of the facts
ground for her exclusion, if she had not had the shown.' He held that it was no part of the
status of a citizen. The case was brought into this intended policy of § 1994 to annul or override the
court on appeal, and in 1911 was affirmed, in 106 immigration laws, so as to authorize the
C. C. A. 464, 184 Fed. 322. In that case, however admission into the country of the wife of a
at the time the relators married, they might have naturalized alien not otherwise entitled to enter,
been lawfully naturalized, and we said: 'Even if and that an alien woman, who is of a class of
we assume the contention of the district attorney persons excluded by law from admission to the
to be correct that marriage will not make a citizen United States does not come within the provisions
of a woman who would be excluded under our of that section. The court relied wholly upon the
immigration laws, it does not affect these dicta contained in the Rustigian Case. No other
relators.' authorities were cited.

"We held that, being citizens, they could not be "In 1914, District Judge Neterer, in Ex parte
excluded as aliens; and it was also said to be Grayson, 215 Fed. 449, construed § 1994 and
inconsistent with the policy of our law that the held that where, pending proceedings to deport
husband should be a citizen and the wife an alien. an alien native of France as an alien prostitute,
The distinction between that case and the one she was married to a citizen of the United States,
now before the court is that, in the former case, she thereby became a citizen, and was not
the marriage took place before any order of subject to deportation until her citizenship was
exclusion had been made, while in this the revoked by due process of law. It was his opinion
marriage was celebrated after such an order was that if, as was contended, her marriage was
made. But such an order is a mere administrative conceived in fraud, and was entered into for the
provision, and has not the force of a judgment of purpose of evading the immigration laws and
a court, and works no estoppel. The preventing her deportation, such fact should be
administrative order is based on the established in a court of competent jurisdiction in
circumstances that existed at the time the order an action commenced for the purpose. The case
of exclusion was made. If the circumstances was appealed and the appeal was dismissed. 134
change prior to the order being carried into C. C. A. 666, 219 Fed. 1022.
effect, it cannot be executed. For example, if an
order of exclusion should be based on the ground "It is interesting also to observe the construction
that the alien was at the time afflicted with a placed upon the language of the statute by the
contagious disease, and it should be made Department of Justice. In 1874, Attorney General
satisfactorily to appear, prior to actual Williams, 14 Ops. Atty. Gen. 402, passing upon
deportation, that the alien had entirely recovered the Act of February 10, 1855, held that residence
from the disease, we think it plain that the order within the United States for the period required
could not be carried into effect. So, in this case, by the naturalization laws was not necessary in
if, after the making of the order of exclusion and order to constitute an alien woman a citizen, she
while she is permitted temporarily to remain, she having married a citizen of the United States
in good faith marries an American citizen, we abroad, although she never resided in the United
cannot doubt the validity of her marriage, and States, she and her husband having continued to
that she thereby acquired, under international reside abroad after the marriage.
law and under § 1994 of the Revised Statutes,
American citizenship, and ceased to be an alien. "In 1909, a similar construction was given to the
There upon, the immigration authorities lost their Immigration Act of May 5, 1907, in an opinion
jurisdiction over her, as that jurisdiction applies rendered by Attorney General Wickersham. It
only to aliens, and not to citizens. appeared an unmarried woman, twenty-eight
years of age and a native of Belgium, arrived in
New York and went at once to a town in
Nebraska, where she continued to reside. About
"In 1910, District Judge Dodge, in Ex parte fifteen months after her arrival she was taken
Kaprielian, 188 Fed. 694, sustained the right of before a United States commissioner by way of
the officials to deport a woman under the instituting proceedings under the Immigration Act
following circumstances: She entered this country (34 Stat. at L. 898, chap. 1134, Comp. Stat. §
in July, 1910, being an alien and having been 4242, 3 Fed. Stat. Anno. 2d ed. p. 637) for her
born in Turkey. She was taken into custody by the deportation, on the ground that she had entered

70 | C O N S T I 2 _ A r t i c l e I V _ C I T I Z E N S H I P
this country for the purpose of prostitution, and persons for whose naturalization the previous
had been found an inmate of a house of Acts of Congress provide. The terms 'married' or
prostitution and practicing the same within three 'who shall be married,' do not refer, in our
years after landing. It appeared, however, that judgment, to the time when the ceremony of
after she was taken before the United States marriage is celebrated, but to a state of marriage.
commissioner, but prior to her arrest under a They mean that, whenever a woman, who under
warrant by the Department of Justice, she was previous Acts might be naturalized, is in a state of
lawfully married to a native-born citizen of the marriage to a citizen, whether his citizenship
United States. The woman professed at the time existed at the passage of the Act or
of her marriage an intention to abandon her subsequently, or before or after the marriage, she
previous mode of life and to remove with her becomes, by that fact, a citizen also. His
husband to his home in Pennsylvania. He knew citizenship, whenever it exists, confers, under the
what her mode of life had been, but professed to Act, citizenship upon her. The construction which
believe in her good intentions. The question was would restrict the Act to women whose husbands,
raised as to the right to deport her, the claim at the time of marriage, are citizens, would
being advance that by her marriage she had exclude far the greater number, for whose
become an American citizen and therefore could benefit, as we think, the Act was intended. Its
not be deported. The Attorney General ruled object, in our opinion, was to allow her citizenship
against the right to deport her as she had to follow that of her husband, without the
become an American citizen. He held that the necessity of any application for naturalization on
words, 'who might herself be lawfully naturalized,' her part; and, if this was the object, there is no
refer to a class or race who might be lawfully reason for the restriction suggested.
naturalized, and that compliance with the other
conditions of the naturalization laws was not "The terms, 'who might lawfully be naturalized
required. 27 Ops. Atty. Gen. 507. under the existing laws,' only limit the application
of the law to free white women. The previous
"Before concluding this opinion, we may add that Naturalization Act, existing at the time only
it has not escaped our observation that Congress, required that the person applying for its benefits
in enacting the Immigration Act of 1917, co as to should be 'a free white person,' and not an alien
provide, in § 19, 'that the marriage to an enemy. Act of April 14th, 1802, 2 Stat. at L. 153.
American citizen of a female of the sexually
immoral classes . . . shall not invest such female "A similar construction was given to the Act by
with United States citizenship if the marriage of the Court of Appeals of New York, in Burton v.
such alien female shall be solemnized after her Burton, 40 N. Y. 373; and is the one which gives
arrest or after the commission of acts which make the widest extension to its provisions"
her liable to deportation under this act.'
Note that while the court did say that "the terms,
"Two conclusions seem irresistibly to follow from 'who might lawfully be naturalized under existing
the above change in the law: laws' only limit the application to free white
women" 20 it hastened to add that "the previous
"(1) Congress deemed legislation essential to Naturalization Act, existing at the time, . . .
prevent women of the immoral class avoiding required that the person applying for its benefits
deportation through the device of marrying an should be (not only) a 'free white person' (but
American citizen. also) . . . not an alien enemy." This is simply
because under the Naturalization Law of the
"(2) If Congress intended that the marriage of an United States at the time the case was decided,
American citizen with an alien woman of any the disqualification of enemy aliens had already
other of the excluded classes, either before or been removed by the Act of July 30, 1813, as may
after her detention should not confer upon her be seen in the corresponding footnote hereof
American citizenship, thereby entitling her to anon. In other words, if in the case of Kelly v.
enter the country, its intention would have been Owen only the race requirement was mentioned,
expressed, and § 19 would not have been the reason was that there was no other non-racial
confined solely to women of the immoral class." requirement or no more alien-enemy
disqualification at the time; and this is
Indeed, We have examined all the leading demonstrated by the fact that the court took care
American decisions on the, subject and We have to make it clear that under the previous
found no warrant for the proposition that the naturalization law, there was also such
phrase "who might herself be lawfully requirement in addition to race. This is important,
naturalized" in Section 1994 of the Revised Status since as stated in re Rustigian, 165 Fed. Rep. 980,
was meant solely as a racial bar, even if loose "The expression used by Mr. Justice Field, (in Kelly
statements in some decisions and other treaties v. Owen) the terms 'who might lawfully be
and other writings on the subject would seem to naturalized under existing laws' only limit the
give such impression. The case of Kelly v. Owen, application of the law to free white women, must
supra, which appears to be the most cited among be interpreted in the application to the special
the first of these decisions 19 simply held: facts and to the incapacities under the then
existing laws," (at p. 982) meaning that whether
"As we construe this Act, it confers the privileges or not an alien wife marrying a citizen would be a
of citizenship upon women married to citizens of citizen was dependent, not only on her race and
the United States, if they are of the class of nothing more necessarily, but on whether or not

71 | C O N S T I 2 _ A r t i c l e I V _ C I T I Z E N S H I P
there were other disqualifications under the law such person is on that account practically any the
in force at the time of her marriage or the less a citizen. The word 'deemed' is the
naturalization of her husband. equivalent of 'considered' or 'judged'; and,
therefore, whatever an act of Congress requires
4. As already stated, in Lo San Tuang, Choy King to be 'deemed' or 'taken' as true of any person or
Tee and the second Ly Giok Ha, the Court drew thing, must, in law, he considered as having been
the inference that because Section 1 of Act 2927 duly adjudged or established concerning such
was eliminated by Commonwealth Act 473, it. person or thing, and have force and effect
follows that in place of the said eliminated accordingly. When, therefore, Congress declares
section, particularly its subdivision (c), being the that an alien woman shall, under certain
criterion of whether or not an alien wife "may be circumstances, be 'deemed' an American citizen,
lawfully naturalized," what should be required is the effect when the contingency occurs, is
not only that she must not be disqualified under equivalent to her being naturalized directly by an
Section 4 but that she must also possess the act of Congress, or in the usual mode thereby
qualifications enumerated in Section 2, such as prescribed."
those of age, residence, good moral character,
adherence to the underlying principles of the Unless We disregard now the long settled familiar
Philippine Constitution, irreproachable conduct, rule of statutory construction that in a situation
lucrative employment or ownership of real estate, like this wherein our legislature has copied an
capacity to speak and write English or Spanish American statute word for word, it is understood
and one of the principal local languages, that the construction already given to such
education of children in certain schools, etc., statute before its being copied constitute part of
thereby implying that, in effect, said Section 2 our own law, there seems to be no reason how
has been purposely intended to take the place of We can give a different connotation or meaning
Section 1 of Act 2927. Upon further consideration to the provision in question. At least, We have
of the proper premises, We have come to the already seen that the views sustaining the
conclusion that such inference is not sufficiently contrary conclusion appear to be based on
justified. inaccurate factual premises related to the real
legislative background of the framing of our
naturalization law in its present form.
Thirdly, the idea of equating the qualifications
To begin with, nothing extant in the legislative enumerated in Section 2 of Commonwealth Act
history, which We have already examined above 473 with the eligibility requirements of Section 1
of the mentioned provisions has been shown or of Act 2927 cannot bear close scrutiny from any
can be shown to indicate that such was the clear point of view. There is no question that Section 2
intent of the legislature. Rather, what is definite is of Commonwealth Act 473 is more or less
that Section 15 an exact copy of Section 1994 of substantially the same as Section 3 of Act 2927.
the Revised Statutes of the United States, which, In other words, Section 1 of Act 2927 co-existed
at the time of the approval of Commonwealth Act already with practically the same provision as
473 had already a settled construction by Section 2 of Commonwealth Act 473. If it were
American courts and administrative authorities. true that the phrase "who may be lawfully
naturalized" in Section 13(a) of Act 2927, as
Secondly, as may be gleaned from the summary amended by Act 3448, referred to the so called
of pertinent American decisions quoted above, racial requirement in Section 1 of the same Act,
there can be no doubt that in the construction of without regard to the provisions of Section 3
the identically worded provision in the Revised thereof, how could the elimination of Section 1
Statutes of the United States, (Section 1994, have the effect of shifting the reference to
which was taken from the Act of February 10, Section 3, when precisely, according to the
1855) all authorities in the United States are American jurisprudence, which was prevailing at
unanimously agreed that the qualifications of the time Commonwealth Act 473 was approved,
residence, good moral character, adherence to such qualifications as were embodied in said
the Constitution, etc. are not supposed to be Section 3, which had their counterpart in the
considered, and that the only eligibility to be corresponding American statutes, are not
taken into account is that of the race or class to supposed to be taken into account and that what
which the subject belongs, the conceptual scope should be considered only are the requirements
of which, We have just discussed. 21 In the very similar to those provided for in said Section 1
case of Leonard v. Grant, supra, discussed by together with the disqualifications enumerated in
Justice Regala in Lo San Tuang, the explanation Section 4?
for such posture of the American authorities was
made thus: Fourthly, it is difficult to conceive that the phrase
"who might be lawfully naturalized" in Section 15
"The phrase, 'shall be deemed a citizen,' in could have been intended to convey a meaning
section 1994 Rev. St., or as it was in the Act of different than that given to it by the American
1855, supra, 'shall be deemed and taken to be a courts and administrative authorities. As already
citizen,' while it may imply that the person to stated, Act 3448 which contained said phrase and
whom it relates has not actually become a citizen from which it was taken by Commonwealth Act
by ordinary means or in the usual way, as by the 473, was enacted in 1928. By that time, Section
judgment of a competent court, upon a proper 1994 of the Revised Statutes of the United States
application and proof, yet it does not follow that was no longer in force because it had been

72 | C O N S T I 2 _ A r t i c l e I V _ C I T I Z E N S H I P
repealed expressly the Act of September 22, Act 473. More accurately, they have always been
1922 which did away with the automatic considered as disqualifications, in the sense that
naturalization of alien wives of American citizens those who did not possess them were the ones
and required, instead, that they submit to regular who could not "be lawfully naturalized," just as if
naturalization proceedings, albeit under more they were suffering from any of the
liberal terms than those of other applicants. In disqualifications under Section 2 of Act 2927 and
other words, when our legislature adopted the later those under Section 4 of Commonwealth Act
phrase in question, which, as already 473, which, incidentally, are practically identical
demonstrated, had a definite construction in to those in the former law, except those in
American law, the Americans had already paragraphs (f) and (h) of the latter. 22 Indeed,
abandoned said phraseology in favor of a such is the clear impression anyone will surely
categorical compulsion for alien wives to be get after going over all the American decisions
naturalized judicially. Simple logic would seem to and opinions quoted and/or cited in the latest
dictate that, since our lawmakers, at the time of USCA (1970), Title 8, section 1430, pp. 598-602,
the approval of Act 3448, had two choices, one to and the first decisions of this Court on the matter,
adopt the phraseology of Section 1994 with its Ly Giok Ha (1959) and Ricardo Cua, citing with
settled construction and the other to follow the approval the opinions of the Secretary of Justice.
new posture of the Americans of requiring judicial 23 Such being the case, that is, that the so-called
naturalization, and it appears that they have racial requirements were always treated as
opted for the first, We have no alternative but to disqualifications in the same light as the other
conclude that our law still follows the old or disqualifications under the law, why should their
previous American law on the subject. Indeed, elimination not be viewed or understood as a
when Commonwealth Act 473 was approved in subtraction from or a lessening of the
1939, the Philippine Legislature, already disqualifications? Why should such elimination
autonomous then from the American Congress, have instead the meaning that what were
had a clearer chance to disregard the old previously considered as irrelevant qualifications
American law and make one of our own, or, at have become disqualifications, as seems to be
least, follow the trend of the Act of the U.S. the import of the holding in Choy King Tee to the
Congress of 1922, but still, our legislators chose effect that the retention in Section 15 of
to maintain the language of the old law. What Commonwealth Act 473 of the same language of
then is significantly important is not that the what used to be Section 13 (a) of Act 2927 (as
legislature maintained said phraseology after amended by Act 3448), notwithstanding the
Section 1 of Act 2927 was eliminated, but that it elimination of Section 1 of the latter, necessarily
continued insisting on using it even after the indicates that the legislature had in mind making
Americans had amended their law in order to the phrase in question "who may be lawfully
provide for what is now contended to be the naturalized" refer no longer to any racial
construction that should be given to the phrase in disqualification but to the qualification under
question. Stated differently, had our legislature Section 2 of Commonwealth Act 473? Otherwise
adopted a phrase from an American statute stated, under Act 2927, there were two groups of
before the American courts had given it a persons that could not be naturalized, namely,
construction which was acquiesced to by those those falling under Section 1 and those falling
given upon to apply the same, it would be under Section 2, and surely, the elimination of
possible for Us to adopt a construction here one group, i.e. those belonging to Section 1,
different from that of the Americans, but as could not have had, by any process of reasoning,
things stand, the fact is that our legislature the effect of increasing, rather than decreasing
borrowed the phrase when there was already a the disqualifications that used to be before such
settled construction thereof, and what is more, it elimination. We cannot see by what alchemy of
appears that our legislators even ignored the logic such elimination could have converted
modification of the American law and persisted in qualifications into disqualifications, specially in
maintaining the old phraseology. Under these the light of the fact that, after all, these are
circumstances, it would be in defiance of reason disqualifications clearly set out as such in the law
and the principles of Statutory construction to say distinctly and separately from qualifications and,
that Section 15 has a nationalistic and selective as already demonstrated, in American
orientation and that it should be construed jurisprudence, qualifications had never been
independently of the previous American posture considered to be of any relevance in determining
because of the difference of circumstances here "who might be lawfully naturalized," as such
and in the United States. It is always safe to say phrase is used in the statute governing the status
that in the construction of a statute, We cannot of alien wives of American citizens, and our law
fall on possible judicial fiat or perspective when on the matter was merely copied verbatim from
the demonstrated legislative point of view seems the American statutes.
to indicate otherwise.

5. Viewing the matter from another angle, there


is need to emphasize that in reality and in effect, 6. In addition to these arguments based on the
the so-called racial requirements, whether under applicable legal provisions and judicial opinions,
the American laws or the Philippine laws, have whether here or in the United States, there are
hardly been considered as qualifications in the practical considerations that militate towards the
same sense as those enumerated in Section 3 of same conclusions. As aptly stated in the motion
Act 2927 and later in Section 2 of Commonwealth for reconsideration of counsel for petitioner-

73 | C O N S T I 2 _ A r t i c l e I V _ C I T I Z E N S H I P
appellee dated February 23, 1967, filed in the can she hope to acquire a lucrative income of her
case of Zita Ngo Burca v. Republic, supra: own to qualify her for citizenship?

"Unreasonableness of requiring alien wife to "3. Under Section 2 of the law, the applicant for
prove 'qualifications — naturalization 'must have enrolled his minor
children of school age, in any of the public
"There is one practical consideration that strongly schools or private schools recognized by the
militates against a construction that Section 15 of Office of the Private Education of the Philippines,
the law requires that an alien wife of a Filipino where Philippine history, government and civics
must affirmatively prove that she possesses the are taught or prescribed as part of the school
qualifications prescribed under Section 2, before curriculum during the entire period of residence
she may be deemed a citizen. Such condition, if in the Philippines required of him prior to the
imposed upon an alien wife, becomes hearing of his petition for naturalization as
unreasonably onerous and compliance therewith Philippine citizen.' If an alien woman has minor
manifestly difficult. The unreasonableness of such children by a previous marriage to another alien
requirement is shown by the following: before she marries a Filipino, and such minor
children had not been enrolled in Philippine
"1. One of the qualifications required of an schools during her period of residence in the
applicant for naturalization under Section 2 of the country, she cannot qualify for naturalization
law is that the applicant 'must have resided in the under the interpretation of this Court. The reason
Philippines for a continuous period of not less behind the requirement that children should be
than ten years.' If this requirement is applied to enrolled in recognized educational institutions is
an alien wife married to a Filipino citizen, this that they follow the citizenship of their father.
means that for a period of ten years at least, she (Chan Ho Lay v. Republic, L-5666, March 30,
cannot hope to acquire the citizenship of her 1954; Tan Hi v. Republic, 88 Phil. 117 [1951]; Hao
husband. If the wife happens to be a citizen of a Lian Chu v. Republic, 87 Phil. 668 [950]; Yap Chin
country whose law declares that upon her v. Republic, L-4177, May 29, 1953; Lim Lian Hong
marriage to a foreigner she automatically loses v. Republic, L-3575, Dec. 26, 1950). Considering
her citizenship and acquires the citizenship of her that said minor children by her first husband
husband, this could mean that for a period of ten generally follow the citizenship of their alien
years at least, she would be stateless. And even father, the basis for such requirement as applied
after having acquired continuous residence in the to her does not exist. Cessante ratione legis
Philippines for ten years, there is no guarantee cessat ipsa lex.
that her petition for naturalization will be granted,
in which case she would remain stateless for an "4. Under Section 3 of the law, the 10-year
indefinite period of time. continuous residence prescribed by Section 2
'shall be understood as reduced to five years for
"2. Section 2 of the law likewise requires of the any petitioner (who is) married to a Filipino
applicant for naturalization that he 'must own real woman.' It is absurd that an alien male married to
estate in the Philippines worth not less than five a Filipino wife should be required to reside only
thousand pesos, Philippine currency, or must for five years in the Philippines to qualify for
have some known lucrative trade, profession, or citizenship, whereas an alien woman married to a
lawful occupation.' Considering the constitutional Filipino husband must reside for ten years.
prohibition against acquisition by an alien of real
estate except in cases of hereditary succession "Thus under the interpretation given by this
(Art. XIII, Sec. 5, Constitution), an alien wife Court, it is more difficult for an alien wife related
desiring to acquire the citizenship of her husband by marriage to a Filipino citizen to become such
must have to prove that she has a lucrative citizen, than for a foreigner who is not so related.
income derived from a lawful trade, profession or And yet, it seems more than clear that the
occupation. The income requirement has been general purpose of the first paragraph of Section
interpreted to mean that the petitioner herself 15 was obviously to accord to an alien woman, by
must be the one to possess the said income. (Uy reason of her marriage to a Filipino, a privilege
v. Republic, L-19578, Oct. 27, 1964; Tanpa Ong not similarly granted to other aliens. It will be
vs. Republic, L-20605, June 30, 1965; Li Tong Pek recalled that prior to the enactment of Act No.
v. Republic, L-20912, November 29, 1965). In 3448 in 1928, amending Act No. 2927 (the old
other words, the wife must prove that she has a Naturalization Law), there was no law granting
lucrative income derived from sources other than any special privilege to alien wives of Filipinos.
her husband's trade, profession or calling. It is of They were treated as any other foreigner. It was
common knowledge, and judicial notice may be precisely to remedy this situation that the
taken of the fact that most wives in the Philippine legislature enacted Act No. 3448. On
Philippines do not have gainful occupations of this point, the observation made by the Secretary
their own. Indeed, Philippine law, recognizing the of Justice in 1941 is enlightening:
dependence of the wife upon the husband,
imposes upon the latter the duty of supporting 'It is true that under Article 22 of the (Spanish)
the former. (Art. 291, Civil Code). It should be Civil Code, the wife follows the nationality of the
borne in mind that universally, it is an accepted husband; but the Department of State of the
concept that when a woman marries, her primary United States on October 31, 1921, ruled that the
duty is to be a wife, mother and housekeeper. If alien wife of a Filipino citizen is not a Filipino
an alien wife is not to be remiss in this duty, how citizen, pointing out that our Supreme Court in

74 | C O N S T I 2 _ A r t i c l e I V _ C I T I Z E N S H I P
the leading case of Roa v. Collector of Customs construction of Section 15 with "the national
(23 Phil. 315) held that Articles 17 to 27 of the policy of selective admission to Philippine
Civil Code being political have been abrogated citizenship." But the question may be asked, is it
upon the cession of the Philippine Islands to the reasonable to suppose that in the pursuit of such
United States. Accordingly, the stand taken by policy, the legislature contemplated to make it
the Attorney-General prior to the enactment of more difficult, if not practically impossible in
Act No. 3448, was that marriage of alien women some instances, for an alien woman marrying a
to Philippine citizens did not make the former Filipino to become a Filipina than any ordinary
citizens of this country.' (Op. Atty. Gen., March 16, applicant for naturalization, as has just been
1928). demonstrated above? It seems but natural and
logical to assume that Section 15 was intended to
'To remedy this anomalous condition, Act No. extend special treatment to alien women who by
5448 was enacted in 1928 adding section 13(a) marrying a Filipino irrevocably deliver
to Act No. 2997 which provides that "any woman themselves, their possessions, their fate and
who is now or may hereafter be married to a fortunes and all that marriage implies to a citizen
citizen of the Philippine Islands, and who might of this country, "for better or for worse." Perhaps
herself be lawfully naturalized, shall be deemed a there can and will be cases wherein the personal
citizen of the Philippine Islands.' (Op. No. 22, s. conveniences and benefits arising from Philippine
1941; emphasis ours) citizenship may motivate such marriage, but
must the minority, as such cases are bound to be,
"If Section 15 of the Revised Naturalization Law serve as the criterion for the construction of law?
were to be interpreted, as this Court did, in such Moreover, it is not farfetched to believe that in
a way as to require that the alien wife must prove joining a Filipino family, the alien woman is
the qualifications prescribed in Section 2, the somehow disposed to assimilate the customs,
privilege granted to alien wives would become beliefs and ideals of Filipinos among whom, after
illusory. It is submitted that such a construction, all, she has to live and associate, but surely, no
being contrary to the manifested object of the one should expect her to do so even before
statute, must be rejected. marriage. Besides, it may be considered that in
reality the extension of citizenship to her is made
'A statute is to be construed with reference to its by the law not so much for her sake as for the
manifest object, and if the language is husband. Indeed, We find the following
susceptible of two constructions, one which will observations anent the national policy
carry out and the other defeat such manifest rationalization in Choy King Tee and Ly Giok Ha
object, it should receive the former construction.' (the second) to be quite persuasive:
(In re National Guard, 71 Vt. 493, 45 A. 1051;
Singer v. United States, 323 U.S. 333, 89 L. ed.
285. See also, U.S. v. Navarro, 19 Phil. 134
[1911]; U. S. v. Toribio, 15 Phil. 85 [1910]). "We respectfully suggest that this articulation of
the national policy begs the question. The
'. . . A construction which will cause objectionable avowed policy of 'selective admission' more
results should be avoided and the court will, if particularly refers to a case where citizenship is
possible, place on the statute a construction sought to be acquired in a judicial proceeding for
which will not result in injustice, and in naturalization. In such a case, the courts should
accordance with the decisions construing no doubt apply the national policy of selecting
statutes, a construction which will result in only those who are worthy to become citizens.
oppression, hardship, or inconveniences will also There is here a choice between accepting or
be avoided, as will a construction which will rejecting the application for citizenship. But this
prejudice public interest, or construction resulting policy finds no application in cases where
in unreasonableness, as well as a construction citizenship is conferred by operation of law. In
which will result in absurd consequences.' such cases, the courts have no choice to accept
or reject. If the individual claiming citizenship by
'So a construction should, if possible, be avoided operation of law proves in legal proceedings that
if the result would be an apparent inconsistency he satisfies the statutory requirements, the
in legislative intent, as has been determined by courts cannot do otherwise than to declare that
the judicial decisions, or which would result in he is a citizen of the Philippines. Thus, an
futility, redundancy, or a conclusion not individual who is able to prove that his father is a
contemplated by the legislature; and the court Philippine citizen, is a citizen of the Philippines,
should adopt that construction which will be the 'irrespective of his moral character, ideological
least likely to produce mischief. Unless plainly beliefs, and identification with Filipino ideals,
shown to have been the intention of the customs, and traditions.' A minor child of a
legislature, an interpretation which would render person naturalized under the law, who is able to
the requirements of the statute uncertain and prove the fact of his birth in the Philippines, is
vague is to be avoided, and the court will not likewise a citizen, regardless of whether he has
ascribe to the legislature an intent to confer an lucrative income, or he adheres to the principles
illusory right . . .' (82 C.J.S., Statutes, sec. 326, of the Constitution. So it is with an alien wife of a
pp. 623-632)." Philippine citizen. She is required to prove only
that she may herself be lawfully naturalized, i.e.,
7. In Choy King Tee and the second Ly Giok Ha, that she is not one of the disqualified persons
emphasis was laid on the need for aligning the

75 | C O N S T I 2 _ A r t i c l e I V _ C I T I Z E N S H I P
enumerated in Section 4 of the law, in order to concerned that Section 15 of the Naturalization
establish her citizenship status as a fact. Law be given effect in the same way as it was
understood and construed when the phrase "who
"A paramount policy consideration of graver may be lawfully naturalized," found in the
import should not be overlooked in this regard, American statute from which it was borrowed and
for it explains and justifies the obviously copied verbatim, was applied by the American
deliberate choice of words. It is universally courts and administrative authorities. There is
accepted that a State, in extending the privilege merit, of course, in the view that Philippine
of citizenship to an alien wife of one of its citizens statutes should be construed in the light of
could have had no other objective than to Philippine circumstances, and with particular
maintain a unity of allegiance among the reference to our naturalization laws. We should
members of the family. (Nelson v. Nelson, 113 realize the disparity in the circumstances
Neb. 453, 203 N. W. 640 [1925]; see also between the United States, as the so-called
'Convention on the Nationality of Married Women: "melting pot" of peoples from all over the world,
Historical Background and Commentary.' UNITED and the Philippines as a developing country
NATIONS, Department of Economic and Social whose Constitution is nationalistic almost in the
Affairs E/CN, 6/399, pp. 8 et seq.). Such objective extreme. Certainly, the writer of this opinion
can only be satisfactorily achieved by allowing cannot be the last in rather passionately insisting
the wife to acquire citizenship derivatively that our jurisprudence should speak our own
through the husband. This is particularly true in concepts and resort to American authorities, to
the Philippines where tradition and law has be sure, entitled to admiration and respect,
placed the husband as head of the family, whose should not be regarded as source of pride and
personal status and decisions govern the life of indisputable authority. Still, We cannot close our
the family group. Corollary to this, our laws look eyes to the undeniable fact that the provision of
with favor on the unity and solidarity of the family law now under scrutiny has no local origin and
(Art. 220, Civil Code), in whose preservation of orientation; it is purely American, factually taken
State as a vital and enduring interest. (See Art. bodily from American law when the Philippines
216, Civil Code). Thus, it has been said that by was under the dominating influence of statutes of
tradition in our country, there is a theoretic the United States Congress. It is indeed a sad
identity of person and interest between husband commentary on the work of our own legislature of
and wife, and from the nature of the relation, the the late 1920's and 1930's that given the
home of one is that of the other. (See De la Viña opportunity to break away from the old American
v. Villareal, 41 Phil. 13). It should likewise be said pattern, it took no step in that direction. Indeed,
that because of the theoretic identity of husband even after America made it patently clear in the
and wife, and the primacy of the husband, the Act of Congress of September 22, 1922 that alien
nationality of husband should be the nationality women marrying Americans cannot be citizens of
of the wife, and the laws upon one should be the the United States without undergoing
law upon the other. For as the court, in Hopkins v. naturalization proceedings, our legislators still
Fachant (9th Cir., 1904) 65 C.C.A., 1, 130 Fed. chose to adopt the previous American law of
839, held: 'The status of the wife follows that of August 10, 1855 as embodied later in Section
the husband, . . . and by virtue of her marriage 1994 of the Revised Statutes of 1874, which, it is
her husband's domicile became her domicile.' worth reiterating, was consistently and uniformly
And the presumption under Philippine law being understood as conferring American citizenship to
that the property relations of husband and wife alien women marrying Americans ipso facto,
are under the regime of conjugal partnership (Art. without having to submit to any naturalization
119, Civil Code), the income of one is also that of proceeding and without having to prove that they
the other. possess the special qualifications of residence,
moral character, adherence to American ideals
"It is, therefore, not congruent with our cherished and American constitution, provided they show
traditions of family unity and identity that a they did not suffer from any of the
husband should be a citizen and the wife an alien, disqualifications enumerated in the American
and that the national treatment of one should be Naturalization Law. Accordingly, We now hold, all
different from that of the other. Thus, it cannot be previous decisions of this Court indicating
that the husband's interests in property and otherwise notwithstanding, that under Section 15
business activities reserved by law to citizens of Commonwealth Act 473, an alien woman
should not form part of the conjugal partnership marrying a Filipino, native born or naturalized,
and be denied to the wife, nor that she herself becomes ipso facto a Filipina provided she is not
cannot, through her own efforts but for the disqualified to be a citizen of the Philippines
benefit of the partnership, acquire such interests. under Section 4 of the same law. Likewise, an
Only in rare instances should the identity of alien woman married to an alien who is
husband and wife be refused recognition, and we subsequently naturalized here follows the
submit that in respect of our citizenship laws, it Philippine citizenship of her husband the moment
should only be in the instances where the wife he takes his oath as Filipino citizen, provided that
suffers from the disqualifications stated in Section she does not suffer from any of the
4 of the Revised Naturalization Law." (Motion for disqualifications under said Section 4.
Reconsideration, Burca vs. Republic, supra.)
As under any other law rich in benefits for those
With all these considerations in mind, We are coming under it, doubtless there will be instances
persuaded that it is in the best interest of all where unscrupulous persons will attempt to take

76 | C O N S T I 2 _ A r t i c l e I V _ C I T I Z E N S H I P
advantage of this provision of law by entering for citizenship'. This is as it should be. Because a
into fake and fictitious marriages or mala fide reading of the petition will reveal at once that
matrimonies. We cannot as a matter of law hold efforts were made to act forth therein. and to
that just because of these possibilities, the prove afterwards, compliance with Sections 2 and
construction of the provision should be otherwise 4 of the Revised Naturalization law. The trial court
than as dictated inexorably by more ponderous itself apparently considered the petition as one
relevant considerations, legal, juridical and for naturalization, and, in fact, declared petitioner
practical. There can always be means of 'a citizen of the Philippines.'"
discovering such undesirable practices and every
case can be dealt with accordingly as it arises. In other words, under this holding, in order for an
alien woman marrying a Filipino to be vested with
III. Filipino citizenship, it is not enough that she
The third aspect of this case requires necessarily possesses the qualifications prescribed by
a re-examination of the ruling of this Court in Section 2 of the law and none of the
Burca, supra, regarding the need of judicial disqualifications enumerated in its Section 4.
naturalization proceedings before the alien wife Over and above all these, she has to pass thru
of a Filipino may herself be considered or deemed the whole process of judicial naturalization,
a Filipino. If this case which, as already noted, apparently from declaration of intention to oath-
was submitted for decision in 1964 yet, had only taking, before she can become a Filipina. In plain
been decided earlier, before Go Im Ty, the words, her marriage to a Filipino is absolutely of
foregoing discussions would have been sufficient no consequence to her nationality vis-a-vis that of
to dispose of it. The Court could have held that her Filipino husband; she remains to be the
despite her apparent lack of qualifications, her national of the country to which she owed
marriage to her co-petitioner made her a Filipina, allegiance before her marriage, and if she desires
without her undergoing any naturalization to be of one nationality with her husband, she has
proceedings, provided she could sustain her claim to wait for the same time that any other applicant
that she is not disqualified under Section 4 of the for naturalization needs to complete, the required
law. But as things stand now, with the Burca period of ten year residence, gain the knowledge
ruling, the question We have still to decide is, of English or Spanish and one of the principal
may she be deemed a Filipina without submitting local languages, make her children study in
to a naturalization proceeding? Filipino schools, acquire real property or engage
in some lawful occupation of her own
Naturally, if Burca is to be followed, it is clear that independently of her husband, file her declaration
the answer to this question must necessarily be of intention and after one year her application for
in the affirmative. As already stated, however, naturalization, with the affidavits of two credible
the decision in Burca has not yet become final witnesses of her good moral character and other
because there is still pending with Us a motion for qualifications, etc., etc., until a decision is
its reconsideration which vigorously submits rendered in her favor, after which, she has to
grounds worthy of serious consideration by this undergo the two years of probation, and only
Court. On this account, and for the reasons then, but not before she takes her oath as citizen,
expounded earlier in this opinion, this case is as will she begin to be considered and deemed to be
good an occasion as any other to re-examine the a citizen of the Philippines. Briefly; she can
issue. become a Filipino citizen only by judicial
declaration.
In the said decision, Justice Sanchez held for the
Court: Such being the import of, the Court's ruling, and
it being quite obvious, on the other hand, upon a
"We accordingly rule that: (1) An alien woman cursory reading of the provision, in question, that
married to a Filipino who desires to be a citizen of the law intends by it to spell out what is the
this country must apply therefore by filing a "effect of naturalization on (the) wife and
petition for citizenship reciting that she possesses children" of an alien, as plainly indicated by its
all the qualifications set forth in Section 2 and title, and inasmuch as the language of the
none of the disqualifications under Section 4, provision itself clearly conveys the thought that
both of the Revised Naturalization Law; (2) Said some effect beneficial to the wife is intended by
petition must be filed in the Court of First it, rather than that she is not in any manner to be
Instance where petitioner has resided at least one benefited thereby, it behooves Us to take a
year immediately preceding the filing of the second hard look at the ruling, if only to see
petition; and (3) Any action by any other office, whether or not the Court might have overlooked
agency, board or official, administrative or any relevant consideration warranting a
otherwise — other than the judgment of a conclusion different from that contained therein.
competent court of justice — certifying or It is undeniable that the issue before Us is of
declaring that an alien wife of the Filipino citizen grave importance, considering its consequences
is also a Filipino citizen, is hereby declared null upon tens of thousands of persons affected by
and void. the ruling therein made by the Court, and surely,
it is for Us to avoid, whenever possible, that Our
decision in any case should produce any adverse
effect upon them not contemplated either by the
"3. We treat the present petition as one for law or by the national policy it seeks to enforce.
naturalization. Or, in the words of law, a 'petition

77 | C O N S T I 2 _ A r t i c l e I V _ C I T I Z E N S H I P
AMICI CURIAE in the Burca case, respectable and "Accordingly, and in view of the implications of
impressive by their number and standing in the the issue under consideration, the Solicitor
Bar and well known for their reputation for General was required, not only, to comment
intellectual integrity, legal acumen and incisive thereon, but, also, to state 'how many cases
and comprehensive resourcefulness in research, there are, like the one at bar, in which certificates
truly evident in the quality of the memorandum of naturalization have been issued after notice of
they have submitted in said case, invite Our the filing of the petition for naturalization had
attention to the impact of the decision therein been published in the Official Gazette only once,
thus: within the periods (a) from January 28, 1950'
(when the decision in Delgado v. Republic was
"The doctrine announced by this Honorable Court promulgated) 'to May 29, 1957' (when the Ong
for the first time in the present case — that an Son Cui was decided) 'and (b) from May 29, 1957
alien woman who marries a Philippine citizen not to November 29, 1965' (when the decision in the
only does not ipso facto herself become a citizen present case was rendered).
but can acquire such citizenship only through
ordinary naturalization proceedings under the "After mature deliberation, and in the light of the
Revised Naturalization Law, and that all reasons adduced in appellant's motion for
administrative actions 'certifying or declaring' reconsideration and in the reply thereto of the
such woman to be a Philippine citizen are 'null Government, as well as of the data contained in
and void' — has consequences that reach far the latter, the Court holds that the doctrine laid
beyond the confines of the present case. down in the Ong Son Cui case shall apply and
Considerably more people are affected, and affect the validity of certificates of naturalization
affected deeply, than simply Mrs. Zita N. Burca. issued after, not on or before May 29, 1957."
The newspapers report that as many as 15
thousand women married to Philippine citizens Here We are met again by the same problem. In
are affected by this decision of the Court. These Gan Tsitung, the Court had to expressly enjoin the
are women of many and diverse nationalities, prospective application of its construction of the
including Chinese, Spanish, British, American, law made in a previous decision 24 which had
Columbian, Finnish, Japanese, Chilean, and so on. already become final, to serve the ends of justice
These members of the community, some of and equity. In the case at bar, We do not have to
whom have been married to citizens for two or go that far. As already observed, the decision in
three decades, have all exercised rights and Burca is still under reconsideration, while the
privileges reserved by law to Philippine citizens. ruling in Lee Suan Ay, Lo San Tuang, Choy King
They will have acquired, separately or in conjugal Tee and other that followed them have at the
partnership with their citizen husbands, real most become the law of the case only for the
property, and they will have sold and transferred parties thereto. If there are good grounds
such property. Many of these women may be in therefor, all We have to do now is to re-examine
professions membership in which is limited to the said rulings and clarify them.
citizens. Others are doubtless stockholders or
officers or employees in companies engaged in For ready reference, We requote Section 15:
business activities for which a certain percentage
of Filipino equity content is prescribed by law. All "Sec. 15. Effect of the naturalization on wife and
these married women are now faced with children .— Any woman who is now or may
possible divestment of personal status and of hereafter be married to a citizen of the
rights acquired and privileges exercised in Philippines, and who might herself be lawfully
reliance, in complete good faith, upon a reading naturalized shall be deemed a citizen of the
of the law that has been accepted as correct for Philippines.
more than two decades by the very agencies of
government charged with the administration of "Minor children of persons naturalized under this
that law. We must respectfully suggest that law who have been born in the Philippines shall
judicial doctrines which would visit such be consider citizens thereof.
comprehensive and far-reaching injury upon the
wives and mothers of Philippine citizens deserve "A foreign-born minor child, if dwelling in the
intensive scrutiny and re-examination." Philippines at the time of naturalization of the
parents, shall automatically become a Philippine
To be sure, this appeal can be no less than what citizen, and a foreign-born minor child, who is not
this Court attended to in Gan Tsitung vs. Republic, in the Philippines at the time the parent is
G.R. No L-20819, Feb. 21, 1967, 19 SCRA 401— naturalized, shall be deemed a Philippines citizen
when Chief Justice Concepcion observed: only during his minority, unless he begins to
"The Court realizes, however, that the rulings in reside permanently in the Philippines when still a
the Barretto and Delgado cases — although minor, in which case, he will continue to be a
referring to situations the equities of which are Philippine citizen even after becoming of age.
not identical to those obtaining in the case at bar
— may have contributed materially to the "A child born outside of the Philippines after the
irregularities committed therein and in other naturalization of his parent, shall be considered a
analogous cases, and induced the parties Philippine citizen, unless within one year after
concerned to believe, although erroneously, that reaching the age of minority, he fails to register
the procedure followed was valid under the law. himself as a Philippine citizen at the American

78 | C O N S T I 2 _ A r t i c l e I V _ C I T I Z E N S H I P
Consulate of the country where he resides, and to Of course, it goes without saying that it is
take the necessary oath of allegiance." perfectly within the constitutional authority of the
Congress of the Philippines to confer or vest
It is obvious that the main subject-matter and citizenship status by legislative fiat. (U.S. v. Wong
purpose of the statute, the Revised Naturalization Kim Ark, 169 U.S. 649, 42 L ed. 890 [1898]; See,
Law or Commonwealth Act 473, as a whole is to 1 Tañada & Carreon, Political Law of the
establish a complete procedure for the judicial Philippines 152 [1961 ed.] ) In fact, it has done so
conferment of the of the status of citizenship for particular individuals, like two foreign religious
upon qualified aliens. After having out such a prelates, 27 hence there is no reason it cannot do
procedure, remarkable for its elaborate and it for classes or groups of persons under general
careful inclusion of all safeguards against the conditions applicable to all of the members of
possibility of any undesirable persons becoming a such class or group, like women who marry
part of our citizenry, it carefully but categorically Filipinos, whether native-born or naturalized. The
states the consequence of the naturalization of issue before Us in this case is whether or not the
an alien undergoing such procedure it prescribes legislature has done so in the disputed provisions
upon members of his immediate family, his wife of Section 15 of the Naturalization Law. And Dr.
and children, 25 and, to that end, in no uncertain Vicente G. Sinco, one of the most respected
terms it ordains that: (a) all his minor children authorities on political law in the Philippines 28
who have been born in the Philippines shall be observes in this connection thus: " A special form
"considered citizens" also; (b) all such minor of naturalization is often observed by some states
children, if born outside the Philippines but with respect to women. Thus in the Philippines a
dwelling here at the time of such naturalization foreign woman married to a Filipino citizen
"shall automatically become" Filipinos also, but becomes ipso facto naturalized, if she belongs to
those not born in the Philippines and not in the any of the classes who may apply for
Philippines at the time of such naturalization, are naturalization under the Philippine Laws" (Sinco,
also "deemed citizens" of this country provided Phil. Political Law 498-499 [10th ed. 1954];
that they shall lose said status if they transfer emphasis ours; this comment is substantially
their permanent residence to a foreign country reiterated in the 1962 edition, citing Ly Giok Ha
before becoming of age; (c) all such minor and Ricardo Cua , supra.)
children, if born outside of the Philippines after
such naturalization, shall also be "considered" More importantly, it may be stated at this
Filipino citizens, unless they expatriate juncture, that in construing the provision of the
themselves by failing to register as Filipinos at United States statutes from which our law has
the Philippine (American) Consulate of the been copies, 28a the American citizenship by
country where they reside and take the necessary choice but by operation of law. "In the Revised
oath of allegiance; and (d) as to the wife, she Statutes the words 'and taken' are omitted. The
"shall be deemed a citizen of the Philippines" if effect of this statute is that every alien woman
she is one "who might herself be lawfully who marries a citizen of the United States
naturalized". 26 becomes perforce a citizen herself, without the
formality of naturalization, and regardless of her
wish in that respect." (USCA 8, p. 601 [1970 ed.],
citing Mackenzie v. Hare, 1913, 134 P. 713, 165
No doubt whatever is entertained, so Burca holds Cal. 766, affirmed 36 S. Ct. 106, 239 U.S. 299, 60
very correctly, as to the point that the minor L ed. 297.)
children, failing within the conditions of place and
time of birth and residence prescribed in the We need not recount here again how this
provision, are vested with Philippines citizenship provision in question was first enacted as
directly by legislative fiat or by force of the law paragraph (a) of Section 13, by way of an
itself and without the need for any judicial insertion into Act 2927 by Act 3448 of November
proceeding or declaration. (At p. 192 SCRA). 30, 1928, and that , in turn, and paragraph was
Indeed, the language of the provision is not copied verbatim from Section 1994 of the Revised
susceptible of any other interpretation. But it is Statutes of the United States, which by that time
claimed that the same expression "shall be already had a long accepted construction among
deemed a citizen of the Philippines" in reference the courts and administrative authorities in that
to the wife, does not necessarily connote the country holding that under such provision an
vesting of citizenship status upon her by alien woman who married a citizen became, upon
legislative fiat because the antecedent phrase such marriage, likewise a citizen by force of law
requiring that she must be one "who might and as a consequence of the marriage itself
herself be lawfully naturalized" implies that such without having to undergo any naturalization
status is intended to attach only after she has proceedings, provided that it could be shown that
undergone the whole process of judicial at the time of such marriage, she was not
naturalization required of any person desiring to disqualified to be naturalized under the laws then
become a Filipino. Stated otherwise, the ruling in in force. To repeat the discussion We already
Burca is that while Section 15 envisages and made of these undeniable facts would
intends legislative naturalization as to the minor unnecessarily make this decision doubly
children, the same section deliberately treats the extensive. The only point which might be
wife differently and leaves her out for ordinary reiterated for emphasis at this juncture is that
judicial naturalization. whereas in the United States, the American
Congress, recognizing the uniform construction of

79 | C O N S T I 2 _ A r t i c l e I V _ C I T I Z E N S H I P
Section 1994 of the Revised Statutes to be as final decision is rendered, but before the
stated above, and finding it desirable to avoid the judgment becomes executory.
effects of such construction, approved the Act of
September 22, 1922 explicitly requiring all such "There is force in the first and second arguments.
alien wives to submit to judicial naturalization, Even the second sentence of said Section 16
albeit under more liberal terms than those for contemplates the fact that the qualifications of
other applicants for citizenship, on the other the original petitioner remains the subject of
hand, the Philippines Legislature, instead of inquiry, for the simple reason that it states that
following suit and adopting a requirement, "The decision rendered in the case shall, so far as
enacted Act 3448 on November 30, 1928 which the widow and minor children are concerned,
copied verbatim the aforementioned Section produce the same legal effect as if it had been
1994 of the Revised Statutes, thereby indicating rendered during the life of the petitioner.' This
its preferences to adopts the latter law and its phraseology emphasizes the intent of the law to
settled constitution rather than the reform continue the proceedings with the deceased as
introduced by the Act of 1992. the theoretical petitioner, for if it were otherwise,
it would have been unnecessary to consider the
Obviously, these considerations leave Us no decision rendered, as far as it effected the widow
choice. Much as this Court may feel that as the and the minor children.
United States herself has evidently found it to be
an improvement of her national policy vis-a-vis xxx xxx xxx
the alien wives of her citizens to discontinue their "The Chua Chian case (supra), cited by the
automatic incorporation into the body of her appellee, declared that a dead person can not be
citizenry without passing through the judicial bound to do things stipulated in the oath of
scrutiny of a naturalization proceeding, as it used allegiance, because an oath is a personal matter.
to be before 1922, it seems but proper, without Therein, the widow prayed that she be allowed to
evidencing any bit of colonial mentality, that as a take the oath of allegiance for the deceased. IN
developing country, the Philippines adopt a the case at bar, petitioner Tan Lin merely asked
similar policy, unfortunately, the manner in which that she be allowed to take the oath of allegiance
our own legislature has enacted our laws on the and the proper certificate of naturalization, once
subject, as recounted above, provides no basis the naturalization proceedings of her deceased
for Us to construe said law along the line of the husband, shall have been completed, not on
1922 modification of the American Law. For Us to behalf of the deceased, but on her own behalf
do so would be to indulge in judicial legislation and of her children, as recipients of the benefits
which it is not constitutionally permissible for this of his naturalization. In other words, the herein
Court to do. Worse, this Court would be going petitioner proposed to take the oath of allegiance,
precisely against the grain of the implicit as a citizen of the Philippines, by virtue of the
Legislative intent. legal provision that 'any woman who is now or
may hereafter be married to a citizen of the
There is at least one decision of this Court before Philippines and who might be lawfully naturalized
Burca wherein it seems it is quite clearly implied shall be deemed a citizen of the Philippines. Minor
that this Court is of the view that under Section children of persons naturalized under this law
16 of the Naturalization Law, the widow and who have been born in the Philippines shall be
children of an applicant for naturalization who considered citizens thereof.' (Section 15,
dies during the proceeding do not have to submit Commonwealth Act No. 473). The decision
themselves to another naturalization proceeding granting citizenship to Lee Pa and the record of
in order to avail of the benefits of the proceedings the case at bar, do not show that the petitioning
involving the husband. Section 16 provides: widow could not have been lawfully naturalized,
at the time Lee Pa filed his petition, apart from
"SEC. 16. Right of widow and children of the fact that his 9 minor children were all born in
petitioners who have died. — In case a petitioner the Philippines. (Decision, In the Matter of the
should die before the final decision has been P)etition of Lee Pa to be admitted a citizen of the
rendered, his widow and minor children may Philippines, Civil Case No. 16287, CFI, Manila,
continue the proceedings. The decision rendered Annex A; Record on Appeal, pp. 8-11). The
in the case shall, so far same legal effect as if it reference for Chua case is, therefore, premature."
had been rendered during the life of the
petitioner." Section 16, as may be seen, is a parallel provision
to Section 15. If the widow of an applicant for
In Tan Lin v. Republic, G.R. No. L-13706, May 31, naturalization as Filipino, who dies during the
1961, 2 SCRA 383 this Court held: proceedings, is not required to go through a
naturalization proceedings, in order to be
"Invoking the above provisions in their favor, considered as a Filipino citizen hereof, it should
petitioners-appellants argue (1) that under said follow that the wife of a living Filipino cannot be
Sec. 16, the widow and minor children are denied the same privilege. This is plain common
allowed to continue the same proceedings and sense and there is absolutely no evidence that
are not substituted for the original petitioner; (2) the Legislature intended to treat them differently.
that the qualifications of the original petitioner
remain to be in issue and not those of the widow
and minor children, and (3) that said Section 16
applies whether the petitioner dies before or after

80 | C O N S T I 2 _ A r t i c l e I V _ C I T I Z E N S H I P
Additionally, We have carefully considered the shall, under certain circumstances, be "deemed"
arguments advanced in the motion for an American to her being naturalized directly by
reconsideration in Burca, and We see no reason an Act of Congress or in the usual mode thereby
to disagree with the following views of counsel: prescribed.' (Van Dyne, Citizenship of the United
States 239, cited in Velayo, Philippine Citizenship
"It is obvious that the provision itself is a and Naturalization 146-147 [1965 ed.] ; italics
legislative declaration of who may be considered ours).
citizens of the Philippines. It is a proposition too
plain to be disputed that Congress has the power "That this was likewise the intent of the Philippine
not only to prescribe the mode or manner under legislature when it enacted the first paragraph of
which foreigners may acquire citizenship, but also Section 15 of the Revised Naturalization
the very power of conferring citizenship by provision. In its entirely, Section 15 reads:
legislative fiat. (U.S. v. Wong Kim Ark, 169 U.S.
649, 42 L. Ed. 890 [1898]; see 1 Tañada and (See supra)
Carreon, Political Law of the Philippine citizens
ed.]). The constitutional itself recognizes as The phrases 'shall be deemed,' shall be
Philippines citizens 'Those who are naturalized in considered,' and 'shall automatically become,' as
accordance with law' (Section 1[5], Article IV, used in the above provision , are undoubtedly
Philippine Constitution). Citizens by synonymous. The leading idea or purpose of the
naturalization, under this provision, include not provision was to confer Philippine citizenship by
only those who are naturalized in accordance operation of law upon certain classes of aliens as
with legal proceedings for the acquisition of a legal consequence of their relationship, by
citizenship, but also those who acquire citizenship blood affinity, to persons who are already citizens
by 'derivative naturalization' or by operation of of the Philippines. Whenever the fact of
law, as. for example, the 'naturalization' of an relationship of the persons enumerated in the
alien wife through the naturalization of her provision concurs related, the effect is for said
husband, or by marriage of an alien woman to a persons to become ipso facto citizens of the
citizen. (See Tañada & Carreon, op. cit supra, at Philippines. 'Ipso facto' as here used does not
152 172; Velayo, Philippine Citizenship and mean that all alien wives and all minor children of
Naturalization 2 [1965 ed.]: 1 Paras, Civil code Philippine citizens, from the mere fact of
186 [1967 ed.]; see also 3 Hackworth, Digest of relationship, necessarily become such citizens
International Law 3). also. Those who do not meet the statutory
requirements do not ipso facto become citizens;
"The phrase 'shall be deemed a citizen of the they must apply for naturalization in order to
Philippines found in Section 14 of the Revised acquire such status. What it does mean, however,
Naturalization Law clearly manifests an intent to is that in respect of those persons numerated in
confer citizenship. Construing a similar phrase Section 15, the relationship to a citizen of the
found in the old U.S. naturalization law (Revised Philippines is the operative fact which establishes
Statutes, 1994) , American courts have uniformly the acquisition of Philippine citizenship by them.
taken it to mean that upon her marriage, the Necessarily, it also determines the points of time
alien woman becomes by operation of law a at which such citizenship commences. Thus,
citizen of the United States as fully as if she had under the second paragraph of Section 15, a
complied with all the provisions of the statutes minor child of a Filipino naturalized under the law,
upon the subject of naturalization. (U.S. v. Keller, who was born in the Philippines, becomes ipso
13 F. 82; U.S. Opinions of the US Attorney General facto a citizen of the Philippines from the time the
dated June 4, 1874 [14 Op. 402], July 20, 1909 fact of relationship concurs with the fact of a
[27 Op. 507], December 1, 1910 [28 Op. 508], citizenship of his parent, and the time when child
Jan. 15, 1920 [32 Op. 209] and Jan. 12, 1923 [23 became a citizen does not depend upon the time
398] ). that he is able to prove that he was born in the
Philippines. The child may prove some 25 years
'The phrase "shall be deemed a citizen, " in after the naturalization of his father that he was
Section 1994 Revised Statute (U.S. Comp. Stat. born in the Philippines and should, therefore, be
1091 1268) or as it was in the Act of 1855 910 'considered' a citizen thereof. It does not mean
Stat. at L. 604, Chapt. 71, Sec. 2), "shall be that he became a Philippine citizen only at that
deemed and taken to be a citizen", while it may later time. Similarly, an alien woman who married
imply that the person to whom it relates has not a Philippine citizen may be able to prove only
actually become a citizen by the ordinary means some 25 years after her marriage (perhaps,
or in the usual way, as by the judgment of a because it was only 25 years after the marriage
competent court, upon a proper application and that her citizenship status became in question),
proof, yet it does not follow that such person is on that she is one who 'might herself be lawfully
that account practically any the less a citizen. naturalized.' It is not reasonable to conclude that
The word "deemed" is the equivalent of she acquired Philippine citizenship only after she
"considered" or "judged," and therefore, whatever had proven that she 'might herself be lawfully
an Act of Congress requires to be "deemed" or naturalized.'
"taken" as true of any person or thing must, in
law, be considered as having been duly adjudged "The point that bears emphasis in this regard is
or established concerning such person or thing, that in adopting the very phraseology of the law ,
and have force and effect accordingly. When, the legislature could not have intended that an
therefore, Congress declares that an alien woman alien wife should not be deemed a Philippine

81 | C O N S T I 2 _ A r t i c l e I V _ C I T I Z E N S H I P
citizen unless and until she proves that she might claim to citizenship. *** His citizenship status
herself be lawfully naturalized' is not a condition commences from the time of birth, although his
precedent to the vesting or acquisition of claim thereto is established as a fact only at a
citizenship; it is only a condition or a state of fact subsequent time. Likewise, an alien woman who
necessary to establish her citizenship as a factum might herself be a lawfully naturalized becomes a
probandum i.e., as a fact established and proved Philippine citizen at the time of her marriage to a
in evidence. The word 'might,' as used in that Filipino husband, not at the time she is able to
phrase, precisely implies that at the time of her establish that status as a proven fact by showing
marriage to a Philippine citizen, the alien woman that she might herself be lawfully naturalized.
'had (the) power' to become such a citizen herself Indeed, there is no difference between a statutory
under the laws then in force. (Owen v. Kelly, 6 DC declaration that a person is deemed a citizen of
191 [1867], aff'd Kelly v. Owen, power long after the Philippines provided his father is such citizen
her marriage does not alter the fact that at her from a declaration that an alien woman married
marriage, she became a citizen. to a Filipino citizen of the Philippines provided his
father is such citizen from a declaration that an
"(This Court has held) that 'an alien wife of a alien woman married to a Filipino citizen of the
Filipino citizen may not acquire the status of a Philippines provided she might herself be lawfully
citizen of the Philippines unless there is proof that naturalized. Both become citizens by operation of
she herself may be lawfully naturalized' law; the former becomes a citizen ipso facto upon
(Decision, pp. 3-4). Under this view, the birth; the later ipso facto upon marriage.
acquisition' of citizenship by the alien wife
depends on her having proven her qualifications
for citizenship, that is, she is not a lawfully
naturalized. It is clear from the words of the law "It is true that unless and until the alien wife
that the proviso does not mean that she must proves that she might herself be lawfully
first prove that deemed (by Congress, not by the naturalized, it cannot be said that she has
courts) a citizen. Even the 'uniform' decisions established her status as a proven fact. But
cited by this Court (at fn. 2) to support its holding neither can it be said that on account, she did not
did not rule that the alien wife becomes a citizen become a citizen of the Philippines. If her
only after she has proven her qualifications for citizenship status is not questioned in any legal
citizenship. What those decisions ruled was that proceeding, she obviously has no obligation to
the alien wives in those cases failed to prove their establish her status as a fact. In such a case, the
qualifications and therefore they failed to presumption of law should be what she claims to
establish their claim to citizenship. Thus in Ly be. (U.S. v. Roxas, 5 Phil. 375 [1905] : Hilado v.
Giok Ha v. Galang, 101 Phil. 459 [1957], the case Assad, 51 O.G. 4527 [1955] ). There is a
was remanded to the lower court for presumption that a representation shown to have
determination of whether petitioner, whose claim been made is true. (Aetna Indemnity Co. v.
to citizenship by marriage to a Filipino was George A. Fuller, Co., 73 A. 738 A. 369, 111 ME.
disputed by the Government, 'might herself be 321)."
lawfully naturalized,' for the purpose of 'proving
her alleged change of political status from alien The question that keeps bouncing back as a
to citizen' (at 464). In Cua v. Board, 101 Phil. 521 consequence of the foregoing views is, what
[1957], the alien wife who was being deported, substitute is there for naturalization proceeding
claimed she was a Philippine citizen by marriage to enable the alien wife of a Philippine citizen to
to a Filipino. This Court finding that there was no have the matter of her own citizenship settled
proof that she was not disqualified under Section and established so that she may not have to be
4 of the Revised Naturalization Law, ruled that: called upon to prove it everytime she has to
'No such evidence appearing on record, the claim perform an act or enter into a transaction or
of assumption of Philippine citizenship by Tijoe business or exercise a right reserved only to
Wu Suan, upon her marriage to petitioner, is Filipinos? The ready answer to such question is
untenable.' (at 523) It will be observed that in that as the laws of our country, both substantive
these decisions cited by this Court, the lack of and procedural, stand today, there is no such
proof that the alien wives 'might (themselves) be procedure, but such is no proof that the
lawfully naturalized' did not necessarily imply citizenship under discussion is not vested as of
that they did not become, in truth and in fact, the date of marriage or the husband's acquisition
citizens upon their marriage to Filipinos. What the of citizenship, as the case may be, for the truth is
decisions merely held was that these wives failed that the situation obtains even as to native-born
to establish their claim to that status as a proven Filipinos. Everytime the citizenship of a person is
fact. material or indispensible in a judicial or
administrative case, Whatever the corresponding
"In all instances where citizenship is conferred by court or administrative authority decides therein
operation of law, the time when citizenship is as to such citizenship is generally not considered
conferred should not be confused with the time as res adjudicata, hence it has to be threshed out
when citizenship status is established as a proven again and again as the occasion may demand.
fact. Thus, even a natural-born citizen of the This, as We view it, is the sense in which Justice
Philippines, whose citizenship status is put in Dizon referred to "appropriate proceeding" in
issue in any proceeding would be required to Brito v. Commissioner, supra. Indeed, only the
prove, for instance, that his father is a citizen of good sense and judgment of those subsequently
the Philippines in order to factually establish his inquiring into the matter may make the effort

82 | C O N S T I 2 _ A r t i c l e I V _ C I T I Z E N S H I P
easier or simpler for the persons concerned by Fernando, J ., concurs except as the interpretation
relying somehow on the antecedent official accorded some American decisions as to which
findings, even if these are not really binding. he is not fully persuaded.

It may not be amiss to suggest, however, that in APPENDIX


order to have a good starting point and so that The following review of all naturalization statutes
the most immediate relevant public records may of the United States from 1790 to 1970 ravel: (1)
be kept in order the following observations in that aside from race, various other
Opinion No. 38, series of 1958, of then Acting disqualifications have also been provided for in
Secretary of Justice Jesus G. Barrera, may be the said statutes from time to time, although it
considered as the most appropriate initial step by was only in 1906 that the familiar and usual
the interested parties. grounds of disqualification, like not being
anarchists, polygamists, etc. were incorporated
"Regarding the steps that should be taken by an therein, and (2) that qualifications of applicants
alien woman married to a Filipino citizen in order for naturalization also varied from time to time.
to acquire Philippine citizenship, the procedure
followed in the Bureau of Immigration is as A — DISQUALIFICATIONS
follows: The alien woman must file a petition for 1. In the first naturalization statute of March 26,
the cancellation of her alien certificate of 1790, only a "free white person" could be
registration alleging, among other things, that naturalized, provided he was not "proscribed" by
she is married to a Filipino citizen and that she is any state, unless it be with the consent of such
not disqualified from acquiring her husband's state. (Chap. V. 1 Stat. 103)
citizenship pursuant to section 4 of
Commonwealth Act No. 473, as amended. Upon 2. In the Act of January 29, 1795, to the same
the filing of said petition, which should be provisions was added the disqualification of those
accompanied or supported by the joint affidavit of "legally convicted of having joined the army of
the petitioner and her Filipino husband to the Great Britain, during the late war." (Chap. XX, 1
effect that the petitioner does not belong to any Stat. 414).
of the groups disqualified by the cited section
from becoming naturalized Filipino citizen (please 3. In the Act of June 18, 1798, Section 1 thereto
see attached CEB Form 1), the Bureau of provided:
Immigration conducts an investigation and
thereafter promulgates its order or decision "SECTION 1. Be it enacted by the Senate and
granting or denying the petition." House of Representatives of the United States of
America in Congress assembled, That no alien
Once the Commissioner of Immigration cancels shall be admitted to become a citizen of the
the subject's registration as an alien, there will United States, or of any state, unless in the
probably be less difficulty in establishing her manner prescribed by the act, entitled 'An Act to
Filipino citizenship in any other proceeding, establish an uniform rule of naturalization; and to
depending naturally on the substance and vigor repeal the act heretofore passed on that subject,
of the opposition. 'he shall have declared his intention to become a
citizen of the United States, five years, at least,
Before closing, it is perhaps best to clarify that before his admission, and shall, at the time of his
this third issue We have passed upon was not application to be admitted, declare and prove, to
touched by the trial court, but as the point is the satisfaction of the court having jurisdiction in
decisive in this case, the Court prefers that the the case, that he has resided within the United
matter be settled once and for all now. States fourteen years, at least, and within the
state or territory where, or for which such court is
IN VIEW OF ALL THE FOREGOING, the judgment of at the time held five years, at least, besides
the Court a quo dismissing appellants' petition for conforming to the other declarations,
injunction is hereby reversed and the renunciations and proofs, by the said act
Commissioner of Immigration and/or his required, any thing therein to the contrary hereof
authorized representative is permanently notwithstanding: Provided, that any alien, who
enjoined from causing the arrest and deportation was residing within the limelights, and under the
and the confiscation of the bond of appellant Lau jurisdiction of the United States, before the
Yuen Yeung, who is hereby declared to have twenty-ninth day of January, one thousand seven
become a Filipino citizen from and by virtue of hundred and ninety-five, may, within one year
her marriage to her co-appellant Moy Ya Lim Yao after the passing of this act—and any alien who
al as Edilberto Aguinaldo Lim, a Filipino citizen of shall have made the declaration of his intention
January 25, 1962. No costs. to become a citizen of the United States, in
conformity to the provisions of the act, entitled
Dizon, Castro, Teehankee and Villamor, JJ ., 'An act to establish an uniform rule of
concur. naturalization, and to repeal the act heretofore
passed on that subject,' may, within four years
Makalintal J ., reserves his separate concurring after having made the declaration aforesaid, be
opinion. admitted to become a citizen, in the manner
prescribed by the said act, upon his making proof
that he has resided five years, at least, within the
limits, and under the jurisdiction of the United

83 | C O N S T I 2 _ A r t i c l e I V _ C I T I Z E N S H I P
States: And provided also, that no alien, who shall enemies at the times and in the manner
be a native, citizen, denizen or subject of any prescribed by the laws heretofore passed on that
nation or state with whom the United States shall subject: Provided, That nothing herein contained
be at war, at the time of his application, shall be shall be taken or construed to interfere with or
then admitted to become a citizen of the United prevent the apprehension and removal, agreeably
States." to law, of any alien enemy at any time previous
to the actual naturalization of such alien." (Chap.
There is here no mention of "white persons." XXXVI, 3 Stat. 53)
(Chap. LIV, 1 Stat. 566).
4. In the Act of April 14, 1802, mentioned in Kelly
v. Owen, supra, reference was made again to
"free white persons," and the same enemy alien 7. Neither the Act of March 22, 1816 nor those of
and "state-proscribed" disqualifications in the May 26, 1824 and May 24, 1828 made any
former statutes were carried over. (Chap. XXVIII, change in the above requirements. (Chap. XXXII,
2 Stat. 153.) 3 Stat. 258; Chap. CLXX-XVI, 4 Stat. 69; and
Chap. CXVI, 4 Stat. 310).
5. The Act of March 26, 1804 provided in its
Section 1 thus: 8. Then the Act of February 10, 1855, important
because it gave alien wives of citizens ,the status
"Be it enacted by the Senate and House of of citizens, was enacted providing:
Representatives of the United States of America
in Congress assembled, That any alien, being a "CHAP. LXXI. — An Act to secure the Right of
free white person, who was residing within the Citizenship to Children of Citizens of the United
limits and under the jurisdiction of the United States born out of the Limits thereof.
States, at any time between the eighteenth day
of June, one thousand seven hundred and ninety- "Be it enacted by the Senate and House of
eight, and the fourteenth day of April one Representatives of the United States of America
thousand eight hundred and two, and who has in Congress assembled, That persons heretofore
continued to reside within the same, may be born, or hereafter to be born, out of the limits and
admitted to become a citizen of the United jurisdiction of the United States, whose fathers
States, without a compliance with the first were or shall be at the time of their birth citizens
condition specified in the first section of the act, of the United States, shall be deemed and
entitled 'An act to establish an uniform rule of considered and are hereby declared to be citizens
naturalization, and to repeal the acts heretofore of the United States: Provided, however, That the
passed on that subject.' " rights of citizenship shall not descend to persons
whose fathers never resided in the United States.
In its Section 2, this Act already provided that:
"SEC. 2. And be it further enacted, That any
"SEC. 2. And be it further enacted, That when any woman who might lawfully be naturalized under
alien who shall have complied with the first the existing laws, married, or who shall be
condition specified in ,the first section of the said married to a citizen of the United States, shall
original act, and who shall have pursued the loyal be deemed and taken to be a citizen."
directions prescribed in the second section of the (Chap. LXXI, 10 Stat. 604.)
said act, may die, before he is actually
naturalized, the widow and the children of such
alien shall be considered as citizens of the United 9. The Act of July 14, 1870 mainly provided only
States, and shall be entitled to rights and for penalties for certain acts related to
privileges as such, upon taking the oaths naturalization, as punished thereby, but added in
prescribed by law." (CHAP. XLVII, 2 Stat. 292) its Section 7 "that the naturalization laws are
hereby extended to aliens of African nativity and
6. In the Act of July 30, 1813, the disqualification to African descent." (Chap. CCLIV, 16 Stat. 254.)
of enemy aliens was removed as follows:
10. The Act of February 1, 1876 contained no
"CHAP. XXXVI. — An Act supplementary to the relevant amendment. (Chap. 5, 19 Stat. 2.)
acts heretofore passed on the subject of an
uniform rule of naturalization. (a) 11. When the statutes of the United States were
"Be it enacted by the Senate and House of revised on June 22, 1874, the naturalization law
Representatives of the United States of America of the country was embodied in Sections 2165-
in Congress assembled, That persons resident 2174 of saddle Revised Statutes. This contained
within the United States, or the territories thereof, no racial disqualification. In fact, it reenacted
on the eighteenth day of June, in the year one ;Section 2 of the Act of February 10, 1855 as its
thousand eight hundred and twelve, who had Section 1994 thereof, thus:
before that day made declaration according to
law, of their intention to become citizens of the "SEC. 1994. Any person who is now or may
United States, or who by the existing laws of the hereafter be married to a citizen of the United
United States, were on that day entitled to States, and who might herself be lawfully
become citizens, without making such naturalized, shall be deemed a citizen." (18 Stat.
declaration, may be admitted to become citizens 351.)
thereof, notwithstanding they shall be alien

84 | C O N S T I 2 _ A r t i c l e I V _ C I T I Z E N S H I P
12. The Act of May 6, 1882 provided expressly any officer or officers, either of specific
that no State court or court of the United State individuals or of officers generally of the
shall admit Chinese to citizenship. (Chap. 126, Government of the United States, or of any other
Sec. 14, 22 Stat. , 61.) organized government, because of his or their
official character, or who is a polygamist, shall be
13. The Act of August 9, 1888 extended the naturalized or be made a citizen of the United
benefits of American citizenship to Indian woman States." (36 Stat. 598)
married to Americans thus:
Incidentally, the 6th paragraph of its Section 4
"CHAP. 818. — An Act in relation to marriage provided:
between white men and Indian women.
"Sixth. When any alien who has declared his
"Be it enacted, That no white man, not otherwise intention to become a citizen of the United States
a member of any tribe of Indians, who may dies before he is actually naturalized the widow
hereafter marry, an Indian woman, member of and minor children of such alien may, by
any Indian tribe in the United States, or any of its complying with the other provisions of this Act, be
Territories except the five civilized tribes in the naturalized without making any declaration of
Indian Territory, shall by such marriage hereafter intention." (36 Stat. 598)
acquire any right to any tribal property, privilege,
or interest whatever to which any member of 16. By the Act of March 2, 1907, alien women
such tribe is entitled. who acquired American citizenship by marriage
retained said citizenship, if she continued to
"SEC. 2. That every Indian woman, member of reside in the United States and did not renounce
any such tribe of Indians, who may hereafter be it, or, if she resided outside of the United States
married to any citizen of the United States, is by registering with the U.S. Consul of her place of
hereby declared to become by such marriage a residence. (CHAP. 2534, Sec. 4, 34 Stat. 1229.)
citizen of the United States, with all the right,
privileges, and immunities of any such citizen, 17. Since United States legislation treats
being a married woman: naturalization and citizenship per se separately,
Section 1994 of the Revised Statutes remained
"Provided, That nothing in this act contained shall untouched. In the Act of February 24, 1911 it was
impair or in any way affect the right or title of provided:
such married woman to any tribal property or any
interest therein. "Be it enacted by the Senate and House of
Representatives of the United States of America
"SEC. 2. That whenever the marriage of any white in Congress assembled, That when any alien, who
man with any Indian woman, a member of any has declared his intention to become a citizen of
such tribe of Indians, is required or offered to be the United States, becomes insane before he is
proved in any judicial ,proceeding, evidence of actually naturalized, and his wife shall thereafter
the admission of such fact by the party against make a homestead entry under the land laws of
whom the proceeding is had, or evidence of the United States, she and their minor children
general repute, or of cohabitation as married may, by complying with the other provisions of
persons, or any other circumstantial or the naturalization laws be naturalized without
presumptive evidence from which the fact may making any declaration of intention." (36 Stat.
be inferred, shall be competent. (Aug. 9, 1888) " 929.)
[25 Stat. 392, Suppl. 1.]
18. The Act of August 11, 1916 merely validated
14. The Act of April 19, 1900 extended American entries filed in certain countries. (CHAP. 316, 39
citizenship to all citizens of the Republic of Hawaii Stat. 926.)
on August 12, 1898 as well as the laws of the
United States to said Republic, including, of 19. In the Act of May 9, 1918, the U.S. Congress
course, those on naturalization. (Chap. 339, Sec. amended the naturalization laws to make
4, 31 Stat. 141.) possible the admission of Filipino navy
servicemen, and understandably, because of the
15. On June 29, 1906. "An Act to establish a war then, it provided:
Bureau of Immigration and Naturalization, and to
provide a uniform rule for the naturalization of "Seventh. Any native-born Filipino of the age of
aliens throughout the United States" was twenty-one years and upward who has declared
approved. No reference was made therein to "free his intention to become a citizen of the United
white persons''; it merely provided in its Section 7 States and who has enlisted or may hereafter
that: enlist in the United States Navy or Marine Corps
or the Naval Auxiliary Service, and who, after
"SEC. 7. That no person who disbelieve in or who service of not less than three years, may be
is opposed to organized government, or who is a honorably discharged therefrom, or who may
member of or affiliated with any organization receive an ordinary discharge with
entertaining and teaching such disbelief in or recommendation for reenlistment; or any alien, or
opposition to organized government, or who any Porto Rican not a citizen of the United States,
advocates or teaches the duty, necessity, or . . . ." (40 Stat. 542.)
propriety of the unlawful assaulting or killing of

85 | C O N S T I 2 _ A r t i c l e I V _ C I T I Z E N S H I P
20. On September 22, 1922, "An Act Relative to status she; shall have resided within the United
the Naturalization and citizenship of married States. After her naturalization she shall have the
women" was appareled repeating Section 1994 of same citizenship status as if her marriage had
the Revised Statutes and otherwise adopting a taken place after the passage of this Act.
different attitude as regards the citizenship and
naturalization of married women thus:

"Be it enacted by the Senate cleaned House of "Sec. 5. That no woman whose husband is not
Representatives of the United States of America eligible to citizenship shall be naturalized during
in Congress assembled, That the right of any the continuance of the marital status.
woman to become a naturalized citizen of the
United States shall not be denied or abridged "Sec. 6. That section 1994 of the Revised Statutes
because of her sex or because she is a married and section 4 of the Expatriation Act of 1907 are
woman. repealed. Such repeal shall not terminate
citizenship acquired or retained under either of
"Sec. 2. That any woman who marries a citizen of such sections nor restore citizenship lost under
the United States after the passage of this Act, or section 4 of the Expatriation Act of 1907.
any woman whose husband is naturalized after
the passage of this Act, shall not become a "Sec. 7. That section 3 of the Expatriation Act of
citizen of the United States by reason of such 1901 is repealed. Such repeal shall not restore
marriage or naturalization; but, if eligible to citizenship lost under such section nor terminate
citizenship, she may be naturalized upon full and citizenship resumed under such section. A woman
complete compliance with all the requirements of who has resumed under such section citizenship
the naturalization laws, with the following lost by marriage shall, upon the passage of this
exceptions: Act, have for all purposes the same citizenship
status as immediately preceding her marriage."
(a) No declaration of intention shall be required; (Chap. 411, 42 Stat. 10211022.)

(b) In lieu of the five-year period of residence 21. When "The Code of the Laws of to United
within the United States and the one-year period States of America of a General and Permanent
of residence within the State or Territory where Character in Force on December 7, 1925" was
the naturalization court is held, she shall have approved, the provisions, corresponding to the
resided continuously in the United States Hawaii, disqualifications for naturalization and the
Alaska, or Porto Rico for at least one year citizenship and naturalization of women
immediately preceding the filing of the petition. embodied therein were:

"Sec. 3. That a woman citizen of the United "367. Naturalization of woman; sex or marriage
States shall not cease to be a citizen of the not a bar. — The right of any woman to become a
United States by reason of her marriage after the naturalized citizen of the United States shall not
passage of this Act, unless she makes a formal be denied or abridged because of her sex or
renunciation of her citizenship before a court because she is a married woman. ( Sept. 22,
having jurisdiction over naturalization of aliens; 1922, c.411, 1, 42 Stat. 1021.)
Provided, That any woman citizen who marries an
alien ineligible to citizenship shall cease to be a "368. Same; women marrying citizens or persons
citizen of the United States. If at the termination becoming naturalized; procedure. — Any woman
of the marital status she is a citizen of the United who marries a citizen of the United States after
States she shall retain her citizenship regardless September 22, 1922, or any woman whose
of her residence. If during the continuance of the husband is naturalized after that date, shall not
marital status she resides continuously for two become a citizen of the United States by reason
years in a foreign State of which her husband is a of such marriage or naturalization; but, if eligible
citizen or subject, or for five years continuously to citizenship, she may be naturalized upon full
outside the United States, she shall thereafter be and complete compliance with the following
subject to the same presumption as is a exceptions:
naturalized citizen of the United States under the
second paragraph of section 2 of the Act entitled (a) No declaration of intention shall be required;
"An Act in reference to the expatriation of citizens
and their protection abroad," approved March 2 (b) In lieu of the five-year period of residence
1907. Nothing herein shall be construed to repeal within the United States and the one-year period
or amend the provisions of Revised Statutes 1999 of residence within the State or Territory where
or of section 2 of the Expatriation Act of 1907 the naturalization court is held, she shall have
with reference to expatriation. resided continuously in the United States, Hawaii,
Alaska, or Porto Rico for at least one year
"Sec. 4. That a woman who, before the passage immediately preceding the filing of the petition.
of this Act, has lost her United States citizenship (Sept. 22, 1922, c. 411, § 2, 42 Stat. 1022.)
by reason of her marriage to an alien eligible for
citizenship, may be naturalized as provided by "369. Same; women who have lost citizenship by
section 2 of this Act: Provided, That no certificate Marrying aliens eligible to citizenship; procedure.
of arrival shall be required to be filed with her — A woman, who, before September 22, 1922,
petition if during the continuance of the marital has lost her United States citizenship by reason

86 | C O N S T I 2 _ A r t i c l e I V _ C I T I Z E N S H I P
for her marriage to an alien eligible for
citizenship, may be naturalized as provided in the (b) He shall not be required to pay any fee under
preceding section. No certificate of arrival shall section 2 or section 7 of the Immigration Act of
be required to be filed with her petition if during 1924;
the continuance of the marital status she shall
have resided within the United Stators. After her (c) If otherwise admissible, he shall not be
naturalization she shall have the same citizenship excluded under section 3 of the Immigration Act
status as if her marriage had taken place after of 1917, unless excluded under the provisions of
September 22, 1922. (Swept. 22, 1922, c. 411, § that section relating to —
4, 42 Stat. 1022.)
(1) Persons afflicted with a loathsome or
"370. Same; Women married to persons ineligible dangerous contagious disease, except
to citizenship. — No woman whose husband is not tuberculosis in any form;
eligible to citizenship shall be naturalized during
the continuance of the marital status. (Swept. 22, (2) Polygamy;
1922, c. 411, 5, 42 Stat. 1022.)
(3) Prostitutes, procurers, or other like immoral
"371. Same, wife of alien declaring becoming persons;
insane before naturalization; minor children. —
When any alien, who has declared his intention to (4) Contract laborers;
become a citizen of the United States, becomes
insane before he is actually naturalized, and his (5) Persons previously deported;
wife shall thereafter make a homestead entry
under the land laws of the United States, she and (6) Persons convicted of crime.
their minor children may, by complying with the
other provisions of the naturalization laws be "Sec. 3. The unmarried child under eighteen
naturalized without making any declaration of years of age, the wife, or the husband, of an alien
intention. (Feb. 24, 1911, c. 151, 36 Stat. 929.)" veteran shall, for the purposes of the Immigration
(Chap. 9, 44 Stat. 156, 158.) Act of 1924, be considered as a non quota
immigrant when accompanying or following
which, of course, must be read together with the within six months to join him, but shall be subject
provisions on inadmissibility of Chinese, to all other provisions of that Act and of the
anarchists, polygamists, non-English speaking immigration laws.
persons, etc. in Sections 363-365 of the same
Code. 'Sec. 4. The foregoing provisions of this Act shall
22. The Act of May 26, 1926 extended not apply to any alien unless the immigration visa
naturalization privileges to alien veterans of is issued to him before the expiration of one year
World War I, thus: after the enactment of this Act." (Chap. 398, 44
Stat. 654-655.)
"Be it enacted by the Senate anal House of
Representatives of the United States of America 23. The Act of June 21, 1930 authorized
in Congress assembled, That (a) as used in this repatriation of certain veterans of World War I.
Act, the term "alien veteran" means an individual, (Chap. 559, 46 Stat. 791.)
a member of the military or naval forces of the
United States at any time after April 5, 1917, and 24. On March 3, 1931, the Act of September 22,
before November 12, 1918, who is now an alien 1922 as amended as follows:
not ineligible to citizenship; but does not include
(1) any individual at anytime during such period "Sec. 4.(a) Section 3 of the Act entitled "An Act
or thereafter separated from such forces under relative to the naturalization and citizenship of
other than honorable conditions, (2) any married women," approved September 22, 1922,
conscientious objector who performed no military as amended, is amended to read as follows:
duty whatever or refused to wear the uniform, or
(3) any alien at any time during such period or 'Sec. 3.(a) A woman citizen of the United States
thereafter discharged from the military or naval shall not cease to be a citizen of the United
forces on account of his alienage. States by reason of her marriage after this
section, as amended, takes effect, unless she
(b) Terms defined in the Immigration Act of 1924 makes a formal renunciation of her citizenship
shall, when used in this Act, have the meaning before a court having jurisdiction over
assigned to such terms in that Act. naturalization of aliens.

"Sec. 2. An alien veteran shall for the purposes of '(b) Any woman who before this section, as
the Immigration Act of 1924 be considered as a amended takes effect, has lost her United States
non-quota immigrant, but shall be subject to all citizenship by residence abroad after marriage to
the other provisions of that Act and of the an alien or by marriage to an alien ineligible to
immigration laws, except that — citizenship may, if she has not acquired any other
nationality by affirmative act, be naturalized in
(a) He shall not be subject to the head tax the manner prescribed in section 4 of this Act, as
imposed by section 2 of the Immigration Act of amended. Any woman who was a citizen of the
1917; United States at birth shall not be denied

87 | C O N S T I 2 _ A r t i c l e I V _ C I T I Z E N S H I P
naturalization under section 4 on account of her "(b) This section shall not be applied in the case
race. of any individual whose petition for naturalization
has been filed before the enactment of this Act.
'(c) No woman shall be entitled to naturalization
under section 4 of this Act, as amended, if her "Sec. 3. The last proviso in the first paragraph of
United States citizenship originated solely by a the seventh subdivision of section 4 of such Act of
reason of her marriage to a citizen of the United June 29, 1906, as amended, is amended by
States or by reason of the acquisition of United striking out the period at the end thereof and
States citizenship by her husband.' inserting in lieu thereof a semicolon and the
following: 'except that this proviso shall not apply
"(b) Section 5 of such Act of September 22, 1922, in the case of service on American-owned vessels
is repealed." (Chap. 442, 46 Stat. 1511-1512.) by an alien who has been lawfully admitted to the
United States for permanent residence.'
25. The Act of May 25, 1932 contained the
following somewhat pertinent provisions: "Sec. 4. Section 32 of such Act of June 29, 1906,
as amended, is amended by adding at the end
"Be it enacted by the Senate and House of thereof the following new subdivisions:
Representatives of the United States of America
in Congress assembled, That '(c) If the name of any naturalized citizen has,
subsequent to naturalization, been changed by
(a) an alien veteran, as defined in section 1 of the order of a court of competent jurisdiction, or by
Act of May 26, 1926 (ch. 398, 44 Stat. 654; title 8, marriage, the citizen may, upon the payment to
sec. 241, U.S. C. Supp. 1), if residing in the United the commissioner of a fee of $10, make
States, be entitled at any time within two years application (accompanied by two photographs of
after the enactment of this Act to naturalization the applicant) for a new certificate of citizenship
upon the same terms, conditions, and exemptions in the new name of such citizen. If the
which would have been accorded to such alien if commissioner finds the name of the applicant to
he had petitioned before the armistice of the have been changed as claimed he shall issue to
World War, except that (1) such alien shall be the applicant a new certificate with one of such
required to prove that immediately preceding the photographs of the applicant affixed thereto.
date of this petition he has resided continuously
within the United States for at least two years, in '(d) The Commissioner of Naturalization is
pursuance of a legal admission for permanent authorized to make and issue, without fee,
residence, and that during all such period he has certifications of any part of the naturalization
behaved as a person of good moral character; (2) records of any court, or of any certificate of
if such admission was subsequent to March 3, citizenship, for use in complying with any statute,
1924, such alien shall file with his petition a State or Federal, or in any judicial proceeding.
certificate of arrival issued by the Commissioner Any such certification shall be admitted in
of Naturalization; (3) final action shall not be had evidence equally with the original from which
upon the petition until at least ninety days have such certification was made in any case in which
elapsed after filing of such petition; and (4) such the original thereof might be admissible as
alien shall be required to appear and file his evidence. No such certification shall be made by
petition in person, and to take the prescribed any clerk of court except upon order of the court.'
oath of allegiance in open court. Such residence
and good moral character shall be proved either "Sec. 5. So much of subdivision (a) of section 33
by the affidavits of two credible witnesses who of such Act of June 29, 1906, as amended, as
are citizens of the United States, or by read 'Upon obtaining a certificate from the
depositions by two such witnesses made before a Secretary of Labor showing the date, place, and
naturalization examiner, for each place of manner of arrival in the United States,' is hereby
residence. repealed.

"(b) All petitions for citizenship made outside the "Sec. 6. Section 4 of the Act entitled 'An Act to
United States in accordance with the seventh supplement the naturalization laws, and for other
subdivision of section 4 of the Naturalization Act purposes,' approved March 2, 1929, is amended
of June 29, 1906, as amended, upon which by striking out the period at the end thereof and
naturalization has not been heretofore granted, inserting in lieu thereof a semicolon and the
are hereby declared to be invalid for all purposes. following: 'except that no such certificate shall be
required if the entry was on or before June 29,
"Sec. 2. (a) The seventh subdivision of section 4 1906.'
of the Naturalization Act of June 29, 1906, as
amended, is amended by striking out 'the "Sec. 7. Despite the provisions of subdivision (a)
National Guard or Naval Militia of any State, of section 1 of the Act entitled 'An Act making it a
Territory, or the District of Columbia, or the State felony with penalty for certain aliens to enter the
Militia in Federal Service.' United States of America under certain conditions
in violation of law,' approved March 4, 1929, as
amended, an alien, if otherwise admissible, shall
not be excluded from admission to the United
States under the provisions of such subdivision
after the expiration of one year after the date of

88 | C O N S T I 2 _ A r t i c l e I V _ C I T I Z E N S H I P
deportation if, prior to his reembarkation at a capacity to read and write English, or of the
place outside of the United States, or prior to his principles of the constitution and form of
application in foreign contiguous territory for government of the United States, being opposed
admission to the United States, the Secretary of to organized government of law, favoring
Labor, in his discretion, shall have granted such totalitarian forms of government, deserters from
alien permission to reapply for admission. the armed forces, etc. (Secs. 1422 to 1426, USCA
8-9, 1953; See also Secs. 1421 et seq., USCA 8,
"Sec. 8. The compilation of the statistics to show 1970.)
races nationalities, and other information,
authorized and directed to be prepared by the B — QUALIFICATIONS
Commissioner of Naturalization, shall be Apart from the above disqualifications, the
completed and published at the same time, as statutes referred tea contained express
near as practicable, as the Publication of the requirements as to qualifications as follows:
statistics of the 1930 census except that reports
covering the census of 1910 shall be completed (1) The Act of 1790 required residence, good
and submitted not later than January 31, 1933, moral character and adherence to the principles
and reports covering the census of 1920 not later of the United States Constitution.
than December 31, 1938. Such statistics shall
show the records of registry made under the (2) That of 1795 required a declaration of
provisions of the Act entitled 'An Act to intention. residence, adherence to the U.S.
supplement the naturalization laws, and for other Constitution, good moral character and no title of
purposes,' approved March 2, 1929. Payment for nobility.
the equipment used in preparing such
compilation shall be made from appropriations for (3) That of 1798 referred only declaration of
miscellaneous expenses of the Bureau of intention and residence.
Naturalization.
(4) That of 1802 required residence, renunciation
"Sec. 9. The Secretary of the Treasury, upon the of allegiance to former government, adherence to
recommendation of the Secretary of Labor, is U.S. Constitution, good moral character and
authorized to provide quarters without payment declaration of intention.
of rent, in the building occupied by the
Naturalization Service in New York City, for a (5) That of 1804 was practically I the same as
photographic studio operated by welfare that of 1802.
organizations without profit and solely for the
benefit of aliens seeking naturalization. Such (6) So also were those of 1813, 1816 and 1824.
studio shall be under the supervision of the
Commissioner of Naturalization. (7) That of 1828 mentioned only residence and
declaration of intention.
"Sec. 10. The tenth subdivision of section 4 of the
Act of June 29, 1906 (ch. 3592, 34 Stat. 598), as (8) Those of 1855, 1870 and 1888 amended the
amended by the Act of May 9, 1918 (ch. 69, 40, law in other respects.
40 Stat. 545; U.S.C., title 8 sec. 377), is hereby
amended to read as follows: (9) That of 1906 contained the following
provisions:
'Tenth. That any person not an alien enemy, who
resided uninterruptedly within the United States "SEC. 4. That an alien may be admitted to
during the period of five years next preceding July become a citizen of the United States in the
1, 1920, and was on that date otherwise qualified following manner and not otherwise:
to become a citizen of the United States, except
that he had not made a declaration of intention "First. He A hall declare on oath before the clerk
required by law and who during or prior to that of any court authorized by this Act to naturalize
time, because of misinformation regarding his aliens, or his authorized deputy, in the district in
citizenship status erroneously exercised the rights which such alien resides, two years at least prior
and performed the duties of a citizen of the to his admission, and after he has reached the
United States in good faith, may file the petition age of eighteen years, that it is bona fide his
for naturalization prescribed by law without intention to become a citizen of the united States,
making the preliminary declaration of intention and to renounce forever all allegiance and fidelity
required of other aliens, and upon satisfactory to any foreign prince, potentate, state, or
proof to the court that he has so acted may be sovereignty, and particularly, by name, to the
admitted as a citizen of the United States upon prince, potentate, state, or sovereignty of which
complying in all respects with the other the alien may be at the time a citizen or subject.
requirements of the naturalization law.' (Chap. And such declaration shall set forth, the name,
203, 47 Stat. 165-167.) age, occupation, personal description, place of
birth, last foreign residence and allegiance, the
26. By June 27, 1952, the right of a person to be date of arrival, the, name of the vessel, if any, in
naturalized could no longer be denied by reason which he came to the United states, and the
of race or sex or because such person was present place of residence in the United States of
married, although various disqualifications were said alien: Provided, however, That no alien who,
still maintained, such as lack understanding, in conformity with the law in force at the date of

89 | C O N S T I 2 _ A r t i c l e I V _ C I T I Z E N S H I P
his declaration, has declared his intention to
become a citizen of the United States shall be "At the time of filing of his petition there shall be
required to renew such declaration. filed with the clerk of the court a certificate from
the Department of Commerce and Labor, if the
"Second. Not less than two years nor more than petitioner arrives in the United States after the
seven years after he has made such declaration passage of this Act, stating the date, place and
of intention he shall make and file, in duplicate, a manner of his arrival in the United States, and the
petition in writing, signed by the applicant in his declaration of intention of such petitioner, which
own handwriting and duly verified, in which certificate and declaration shall be attached to
petition such applicant shall state his full name, and made a part of said petition.
his place of residence (by street and number, if
possible), his occupation, and, if possible, the "Third. He shall, before he is admitted to
date and place of his birth; the place from which citizenship, declare on oath in open court that he
he emigrated, and the date and place of his will support the Constitution of the United States,
arrival in the United States, and, if he entered and he absolutely and entirely renounces and
through a port, the name of the vessel on which abjures all allegiance and fidelity to any foreign
he arrived; the time when and the place and prince potentate, state, or sovereignty, and
name of the court where he declared his intention particularly by name to the prince, potentate,
to become a citizen of the United States; if he is state, or sovereignty of which he was before a
married he shall state the name of his wife and, if citizen or subject; that he will support and defend
possible, the country of her nativity and her place the Constitution and laws of the United States
of residence at the time of filing his petition; and against all enemies, foreign and domestic, and
if he has children, the name, date, and place of bear true faith and allegiance to the same.
birth and place of residence of each child living at
the time of his petition: Provided, That if he has "Fourth. It shall be made to appear to the
filed his declaration before the passage of this Act satisfaction of the court admitting any alien to
he shall not be required to sign the petition in his citizenship that immediately preceding the date
own handwriting. of his application he has resided continuously
within the United States five years at least, and
"The petition shall set forth that he is not a within the State or Territory where such court is at
disbeliever in or opposed to organized the time held one year at least, and that during
government, or a member of or affiliated with any that time he has behaved as a man of good moral
organization or body of persons teaching disbelief character, attached to the principles of the
in or opposed to organized government, a Constitution of the United States, and well
polygamist or believer in the practice of disposed to the good order and happiness of the
polygamy, and that it is his intention to become a same. In addition to the oath of the applicant, the
citizen of the United States and to renounce testimony of at least two witnesses, citizens of
absolutely and forever all allegiance and fidelity the United States, as to the facts of residence,
to any foreign prince, potentate, state, or moral character, and attachment to the principles
sovereignty, and particularly by name to the of the Constitution shall be required, and the
prince, potentate, state, or sovereignty of which name, ,place of residence, and occupation of
he at the time of filing of his petition may be a each witness shall be set forth in the record.
citizen or subject, and that it is his intention to "Fifth. In case the alien applying to be admitted to
reside permanently within the United States, and citizenship has borne any hereditary title, or has
whether or not he has been denied admission a, a been of any of the orders of nobility in the
citizen of the United States, and, if denied, the kingdom or state from which he came, he shall, in
ground or grounds of such denial, the court or addition to the above requisites make an express
courts in which such decision was rendered, and renunciation of his title or order of nobility in the
that the cause for such denial has since been court to which his application is made, and his
cured or removed, and every fact material to his renunciation shall be recorded in the court.
naturalization ailed required to be proved upon "Sixth. When any alien who has declared his
the final hearing of his application. intention to become a citizen of the United States
dies before he is actually naturalized the widow
and minor children of such alien may, by
complying with the other provisions of this Act, be
"The petition shall also be verified by the naturalized without making any declaration of
affidavits of at least two credible witnesses, who intention." (34 stat. 596-98.)
are citizens of the United States, and who shall 10. Those of 1911 and 1916 contained
state in their affidavits that they have personally amendments as to other matters.
known the applicant to be a resident of the 11. That of 1918 provided for different
United States for a period of at least five years qualifications for Filipinos, Porto Ricans, etc. for
continuously, and of the State, Territory, or naturalization in addition to service in the U.S.
district in which the application is made for a Navy or Philippine Constabulary.
period of at least one year immediately preceding 12. Those of years after 1922 when Section 1994
the date of the filing of his petition, and that they was repealed would have no material bearing in
each have personal knowledge that the petitioner this case.
is a person of good moral character, and that he
is in every way qualified, in their opinion, to be Amen.
admitted as a citizen of the United States.

90 | C O N S T I 2 _ A r t i c l e I V _ C I T I Z E N S H I P
(Tecson v. COMELEC, G.R. No. 161434, Commission on Elections ("COMELEC") to
161634, 161824, March 03, 2004) disqualify FPJ and to deny due course or to cancel
his certificate of candidacy upon the thesis that
EN BANC FPJ made a material misrepresentation in his
[G.R. No. 161434. March 3, 2004.] certificate of candidacy by claiming to be a
MARIA JEANETTE C. TECSON and FELIX B. natural- born Filipino citizen when in truth,
DESIDERIO, JR., petitioners, vs. The COMMISSION according to Fornier, his parents were foreigners;
ON ELECTIONS, RONALD ALLAN KELLY POE (a.k.a. his mother, Bessie Kelley Poe, was an American,
FERNANDO POE, JR.) and VICTORINO X. FORNIER, and his father, Allan Poe, was a Spanish national,
respondents. being the son of Lorenzo Pou, a Spanish subject.
Granting, petitioner asseverated, that Allan F. Poe
[G.R. No. 161634. March 3, 2004.] was a Filipino citizen, he could not have
ZOILO ANTONIO VELEZ, petitioner, vs. RONALD transmitted his Filipino citizenship to FPJ, the
ALLAN KELLEY POE, a.k.a. FERNANDO POE, JR., latter being an illegitimate child of an alien
respondent. mother. Petitioner based the allegation of the
illegitimate birth of respondent on two assertions
[G.R. No. 161824. March 3, 2004.] — first, Allan F. Poe contracted a prior marriage to
VICTORINO X. FORNIER, petitioner, vs. HON. a certain Paulita Gomez before his marriage to
COMMISSION ON ELECTIONS and RONALD ALLAN Bessie Kelley and, second, even if no such prior
KELLEY POE, ALSO KNOWN AS FERNANDO POE marriage had existed, Allan F. Poe, married Bessie
JR., respondents. Kelly only a year after the birth of respondent.

DECISION In the hearing before the Third Division of the


VITUG, J p: COMELEC on 19 January 2004, petitioner, in
Citizenship is a treasured right conferred on those support of his claim, presented several
whom the state believes are deserving of the documentary exhibits — 1) a copy of the
privilege. It is a "precious heritage, as well as an certificate of birth of FPJ, 2) a certified photocopy
inestimable acquisition," 1 that cannot be taken of an affidavit executed in Spanish by Paulita Poe
lightly by anyone — either by those who enjoy it y Gomez attesting to her having filed a case for
or by those who dispute it. bigamy and concubinage against the father of
respondent, Allan F. Poe, after discovering his
Before the Court are three consolidated cases, all bigamous relationship with Bessie Kelley, 3) an
of which raise a single question of profound English translation of the affidavit aforesaid, 4) a
importance to the nation. The issue of citizenship certified photocopy of the certificate of birth of
is brought up to challenge the qualifications of a Allan F. Poe, 5) a certification issued by the
presidential candidate to hold the highest office Director of the Records Management and
of the land. Our people are waiting for the Archives Office, attesting to the fact that there
judgment of the Court with bated breath. Is was no record in the National Archives that a
Fernando Poe, Jr., the hero of silver screen, and Lorenzo Poe or Lorenzo Pou resided or entered
now one of the main contenders for the the Philippines before 1907, and 6) a certification
presidency, a natural-born Filipino or is he not? from the Officer-In-Charge of the Archives
Division of the National Archives to the effect that
The moment of introspection takes us face to no available information could be found in the
face with Spanish and American colonial roots files of the National Archives regarding the birth
and reminds us of the rich heritage of civil law of Allan F. Poe.
and common law traditions, the fusion resulting
in a hybrid of laws and jurisprudence that could On his part, respondent, presented twenty-two
be no less than distinctly Filipino. documentary pieces of evidence, the more
significant ones being — a) a certification issued
Antecedent Case Settings by Estrella M. Domingo of the Archives Division of
On 31 December 2003, respondent Ronald Allan the National Archives that there appeared to be
Kelly Poe, also known as Fernando Poe, Jr. no available information regarding the birth of
(hereinafter "FPJ"), filed his certificate of Allan F. Poe in the registry of births for San Carlos,
candidacy for the position of President of the Pangasinan, b) a certification issued by the
Republic of the Philippines under the Koalisyon ng Officer-In-Charge of the Archives Division of the
Nagkakaisang Pilipino (KNP) Party, in the National Archives that no available information
forthcoming national elections. In his certificate about the marriage of Allan F. Poe and Paulita
of candidacy, FPJ, representing himself to be a Gomez could be found, c) a certificate of birth of
natural-born citizen of the Philippines, stated his Ronald Allan Poe, d) Original Certificate of Title
name to be "Fernando Jr.," or "Ronald Allan" Poe, No. P-2247 of the Registry of Deeds for the
his date of birth to be 20 August 1939 and his Province of Pangasinan, in the name of Lorenzo
place of birth to be Manila. Pou, e) copies of Tax Declaration No. 20844, No.
20643, No. 23477 and No. 23478 in the name of
Victorino X. Fornier, petitioner in G.R. No. 161824, Lorenzo Pou, f) a copy of the certificate of death
entitled "Victorino X. Fornier, Petitioner, versus of Lorenzo Pou, g) a copy of the purported
Hon. Commission on Elections and Ronald Allan marriage contract between Fernando Pou and
Kelley Poe, also known as Fernando Poe, Jr., Bessie Kelley, and h) a certification issued by the
Respondents," initiated, on 09 January 2004, a City Civil Registrar of San Carlos City, Pangasinan,
petition docketed SPA No. 04-003 before the stating that the records of birth in the said office

91 | C O N S T I 2 _ A r t i c l e I V _ C I T I Z E N S H I P
during the period of from 1900 until May 1946 Decisions of the COMELEC on disqualification
were totally destroyed during World War II. cases may be reviewed by the Supreme Court per
Rule 64 2 in an action for certiorari under Rule 65
On 23 January 2004, the COMELEC dismissed SPA 3 of the Revised Rules of Civil Procedure. Section
No. 04-003 for lack of merit. Three days later, or 7, Article IX, of the 1987 Constitution also reads
on 26 January 2004, Fornier filed his motion for —
reconsideration. The motion was denied on 06
February 2004 by the COMELEC en banc. On 10 "Each Commission shall decide by a majority vote
February 2004, petitioner assailed the decision of of all its Members any case or matter brought
the COMELEC before this Court conformably with before it within sixty days from the date of its
Rule 64, in relation to Rule 65, of the Revised submission for decision or resolution. A case or
Rules of Civil Procedure. The petition, docketed matter is deemed submitted for decision or
G.R. No. 161824, likewise prayed for a temporary resolution upon the filing of the last pleading,
restraining order, a writ of preliminary injunction brief, or memorandum, required by the rules of
or any other resolution that would stay the the Commission or by the Commission itself.
finality and/or execution of the COMELEC Unless otherwise provided by this Constitution or
resolutions. by law, any decision, order, or ruling of each
Commission may be brought to the Supreme
The other petitions, later consolidated with G.R. Court on certiorari by the aggrieved party within
No. 161824, would include G.R. No. 161434, thirty days from receipt of a copy thereof."
entitled "Maria Jeanette C. Tecson, and Felix B.
Desiderio, Jr., vs. The Commission on Elections, Additionally, Section 1, Article VIII, of the same
Ronald Allan Kelley Poe (a.k.a. 'Fernando Poe, Jr.'), Constitution provides that judicial power is vested
and Victorino X. Fornier," and the other, docketed in one Supreme Court and in such lower courts as
G.R. No. 161634, entitled "Zoilo Antonio G. Velez, may be established by law which power "includes
vs. Ronald Allan Kelley Poe, a.k.a. Fernando Poe, the duty of the courts of justice to settle actual
Jr.," both challenging the jurisdiction of the controversies involving rights which are legally
COMELEC and asserting that, under Article VII, demandable and enforceable, and to determine
Section 4, paragraph 7, of the 1987 Constitution, whether or not there has been a grave abuse of
only the Supreme Court had original and discretion amounting to lack or excess of
exclusive jurisdiction to resolve the basic issue on jurisdiction on the part of any branch or
the case. instrumentality of the Government."

Jurisdiction of the Court


In G.R. No. 161824 It is sufficiently clear that the petition brought up
in G.R. No. 161824 was aptly elevated to, and
In seeking the disqualification of the candidacy of could well be taken cognizance of, by this Court.
FPJ and to have the COMELEC deny due course or A contrary view could be a gross denial to our
to cancel FPJ's certificate of candidacy for alleged people of their fundamental right to be fully
misrepresentation of a material fact (i.e., that FPJ informed, and to make a proper choice, on who
was a natural-born citizen) before the COMELEC, could or should be elected to occupy the highest
petitioner Fornier invoked Section 78 of the government post in the land.
Omnibus Election Code —

"Section 78.Petition to deny due course or to


cancel a certificate of candidacy. — A verified In G.R. No. 161434 and G.R. No. 161634
petition seeking to deny due course or to cancel a
certificate of candidacy may be filed by any Petitioners Tecson, et al., in G.R. No. 161434, and
person exclusively on the ground that any Velez, in G.R. No. 161634, invoke the provisions
material representation contained therein as of Article VII, Section 4, paragraph 7, of the 1987
required under Section 74 hereof is false" — Constitution in assailing the jurisdiction of the
COMELEC when it took cognizance of SPA No. 04-
in consonance with the general powers of 003 and in urging the Supreme Court to instead
COMELEC expressed in Section 52 of the Omnibus take on the petitions they directly instituted
Election Code — before it. The Constitutional provision cited reads:
"Section 52.Powers and functions of the
Commission on Elections. In addition to the "The Supreme Court, sitting en banc, shall be the
powers and functions conferred upon it by the sole judge of all contests relating to the election,
Constitution, the Commission shall have exclusive returns, and qualifications of the President or
charge of the enforcement and administration of Vice-President, and may promulgate its rules for
all laws relative to the conduct of elections for the the purpose."
purpose of ensuring free, orderly and honest
elections" — The provision is an innovation of the 1987
Constitution. The omission in the 1935 and the
and in relation to Article 69 of the Omnibus 1973 Constitution to designate any tribunal to be
Election Code which would authorize "any the sole judge of presidential and vice-
interested party" to file a verified petition to deny presidential contests, has constrained this Court
or cancel the certificate of candidacy of any to declare, in Lopez vs. Roxas, 4 as "not (being)
nuisance candidate. justiciable" controversies or disputes involving

92 | C O N S T I 2 _ A r t i c l e I V _ C I T I Z E N S H I P
contests on the elections, returns and It is fair to conclude that the jurisdiction of the
qualifications of the President or Vice-President. Supreme Court, defined by Section 4, paragraph
The constitutional lapse prompted Congress, on 7, of the 1987 Constitution, would not include
21 June 1957, to enact Republic Act No. 1793, "An cases directly brought before it, questioning the
Act Constituting an Independent Presidential qualifications of a candidate for the presidency or
Electoral Tribunal to Try, Hear and Decide Protests vice-presidency before the elections are held.
Contesting the Election of the President-Elect and
the Vice-President-Elect of the Philippines and Accordingly, G.R. No. 161434, entitled "Maria
Providing for the Manner of Hearing the Same." Jeanette C. Tecson, et al., vs. Commission on
Republic Act 1793 designated the Chief Justice Elections, et al.," and G.R. No. 161634, entitled
and the Associate Justices of the Supreme Court "Zoilo Antonio Velez vs. Ronald Allan Kelley Poe
to be the members of the tribunal. Although the a.k.a. Fernando Poe, Jr." would have to be
subsequent adoption of the parliamentary form of dismissed for want of jurisdiction.
government under the 1973 Constitution might
have implicitly affected Republic Act No. 1793, The Citizenship Issue
the statutory set-up, nonetheless, would now be Now, to the basic issue; it should be helpful to
deemed revived under the present Section 4, first give a brief historical background on the
paragraph 7, of the 1987 Constitution. concept of citizenship.

Ordinary usage would characterize a "contest" in Perhaps, the earliest understanding of citizenship
reference to a post-election scenario. Election was that given by Aristotle, who, sometime in 384
contests consist of either an election protest or a to 322 B.C., described the "citizen" to refer to a
quo warranto which, although two distinct man who shared in the administration of justice
remedies, would have one objective in view, i.e., and in the holding of an office. 6 Aristotle saw its
to dislodge the winning candidate from office. A significance if only to determine the constituency
perusal of the phraseology in Rule 12, Rule 13, of the "State," which he described as being
and Rule 14 of the "Rules of the Presidential composed of such persons who would be
Electoral Tribunal," promulgated by the Supreme adequate in number to achieve a self-sufficient
Court en banc on 18 April 1992, would support existence. 7 The concept grew to include one who
this premise — would both govern and be governed, for which
qualifications like autonomy, judgment and
"Rule 12.Jurisdiction. — The Tribunal shall be the loyalty could be expected. Citizenship was seen
sole judge of all contests relating to the election, to deal with rights and entitlements, on the one
returns, and qualifications of the President or hand, and with concomitant obligations, on the
Vice-President of the Philippines. other. 8 In its ideal setting, a citizen was active in
public life and fundamentally willing to submit his
"Rule 13.How Initiated. — An election contest is private interests to the general interest of society.
initiated by the filing of an election protest or a
petition for quo warranto against the President or The concept of citizenship had undergone
Vice-President. An election protest shall not changes over the centuries. In the 18th century,
include a petition for quo warranto. A petition for the concept was limited, by and large, to civil
quo warranto shall not include an election citizenship, which established the rights
protest. necessary for individual freedom, such as rights
to property, personal liberty and justice. 9 Its
"Rule 14.Election Protest. — Only the registered meaning expanded during the 19th century to
candidate for President or for Vice-President of include political citizenship, which encompassed
the Philippines who received the second or third the right to participate in the exercise of political
highest number of votes may contest the election power. 10 The 20th century saw the next stage of
of the President or the Vice-President, as the case the development of social citizenship, which laid
may be, by filing a verified petition with the Clerk emphasis on the right of the citizen to economic
of the Presidential Electoral Tribunal within thirty well-being and social security. 11 The idea of
(30) days after the proclamation of the winner." citizenship has gained expression in the modern
welfare state as it so developed in Western
The rules categorically speak of the jurisdiction of Europe. An ongoing and final stage of
the tribunal over contests relating to the election, development, in keeping with the rapidly
returns and qualifications of the "President" or shrinking global village, might well be the
"Vice-President", of the Philippines, and not of internationalization of citizenship. 12
"candidates" for President or Vice-President. A
quo warranto proceeding is generally defined as The Local Setting — from Spanish Time to the
being an action against a person who usurps, Present
intrudes into, or unlawfully holds or exercises a There was no such term as "Philippine citizens"
public office. 5 In such context, the election during the Spanish regime but "subjects of Spain"
contest can only contemplate a post-election or "Spanish subjects." 13 In church records, the
scenario. In Rule 14, only a registered candidate natives were called 'indios', denoting a low regard
who would have received either the second or for the inhabitants of the archipelago. Spanish
third highest number of votes could file an laws on citizenship became highly codified during
election protest. This rule again presupposes a the 19th century but their sheer number made it
post-election scenario. difficult to point to one comprehensive law. Not
all of these citizenship laws of Spain however,

93 | C O N S T I 2 _ A r t i c l e I V _ C I T I Z E N S H I P
were made to apply to the Philippine Islands subject in respect thereof to such laws as are
except for those explicitly extended by Royal applicable to foreigners. In case they remain in
Decrees. 14 the territory they may preserve their allegiance
to the Crown of Spain by making, before a court
Spanish laws on citizenship were traced back to of record, within a year from the date of the
the Novisima Recopilacion, promulgated in Spain exchange of ratifications of this treaty, a
on 16 July 1805 but as to whether the law was declaration of their decision to preserve such
extended to the Philippines remained to be the allegiance; in default of which declaration they
subject of differing views among experts; 15 shall be held to have renounced it and to have
however, three royal decrees were undisputably adopted the nationality of the territory in which
made applicable to Spaniards in the Philippines — they reside.
the Order de la Regencia of 14 August 1841, 16
the Royal Decree of 23 August 1868 specifically
defining the political status of children born in the
Philippine Islands, 17 and finally, the Ley Thus —
Extranjera de Ultramar of 04 July 1870, which was
expressly made applicable to the Philippines by "The civil rights and political status of the native
the Royal Decree of 13 July 1870. 18 inhabitants of the territories hereby ceded to the
United States shall be determined by the
The Spanish Constitution of 1876 was never Congress." 22
extended to the Philippine Islands because of the
express mandate of its Article 89, according to Upon the ratification of the treaty, and pending
which the provisions of the Ultramar among legislation by the United States Congress on the
which this country was included, would be subject, the native inhabitants of the Philippines
governed by special laws. 19 ceased to be Spanish subjects. Although they did
not become American citizens, they, however,
It was only the Civil Code of Spain, made effective also ceased to be "aliens" under American laws
in this jurisdiction on 18 December 1889, which and were thus issued passports describing them
came out with the first categorical enumeration to be citizens of the Philippines entitled to the
of who were Spanish citizens. — protection of the United States. LibLex
The term "citizens of the Philippine Islands"
"(a)Persons born in Spanish territory, appeared for the first time in the Philippine Bill of
1902, also commonly referred to as the Philippine
"(b)Children of a Spanish father or mother, even if Organic Act of 1902, the first comprehensive
they were born outside of Spain, legislation of the Congress of the United States
on the Philippines —
"(c)Foreigners who have obtained naturalization
papers, ". . . that all inhabitants of the Philippine Islands
continuing to reside therein, who were Spanish
"(d)Those who, without such papers, may have subjects on the 11th day of April, 1891, and then
become domiciled inhabitants of any town of the resided in said Islands, and their children born
Monarchy." 20 subsequent thereto, shall be deemed and held to
be citizens of the Philippine Islands and as such
The year 1898 was another turning point in entitled to the protection of the United States,
Philippine history. Already in the state of decline except such as shall have elected to preserve
as a superpower, Spain was forced to so cede her their allegiance to the Crown of Spain in
sole colony in the East to an upcoming world accordance with the provisions of the treaty of
power, the United States. An accepted principle peace between the United States and Spain,
of international law dictated that a change in signed at Paris, December tenth eighteen
sovereignty, while resulting in an abrogation of all hundred and ninety eight." 23
political laws then in force, would have no effect
on civil laws, which would remain virtually intact. Under the organic act, a "citizen of the
Philippines" was one who was an inhabitant of the
The Treaty of Paris was entered into on 10 Philippines, and a Spanish subject on the 11th
December 1898 between Spain and the United day of April 1899. The term "inhabitant" was
States. 21 Under Article IX of the treaty, the civil taken to include 1) a native-born inhabitant, 2) an
rights and political status of the native inhabitant who was a native of Peninsular Spain,
inhabitants of the territories ceded to the United and 3) an inhabitant who obtained Spanish
States would be determined by its Congress — papers on or before 11 April 1899. 24
Controversy arose on to the status of children
"Spanish subjects, natives of the Peninsula, born in the Philippines from 11 April 1899 to 01
residing in the territory over which Spain by the July 1902, during which period no citizenship law
present treaty relinquishes or cedes her was extant in the Philippines. Weight was given to
sovereignty may remain in such territory or may the view, articulated in jurisprudential writing at
remove therefrom, retaining in either event all the time, that the common law principle of jus
their rights of property, including the right to sell soli, otherwise also known as the principle of
or dispose of such property or of its proceeds; territoriality, operative in the United States and
and they shall also have the right to carry on England, governed those born in the Philippine
their industry, commerce, and professions, being

94 | C O N S T I 2 _ A r t i c l e I V _ C I T I Z E N S H I P
Archipelago within that period. 25 More about this "Section 1, Article III, 1935 Constitution. The
later. following are citizens of the Philippines —

In 23 March 1912, the Congress of the United "(1)Those who are citizens of the Philippine
States made the following amendment to the Islands at the time of the adoption of this
Philippine Bill of 1902 — Constitution

"Provided, That the Philippine Legislature is "(2)Those born in the Philippines Islands of
hereby authorized to provide by law for the foreign parents who, before the adoption of this
acquisition of Philippine citizenship by those Constitution, had been elected to public office in
natives of the Philippine Islands who do not come the Philippine Islands.
within the foregoing provisions, the natives of
other insular possession of the United States, and "(3)Those whose fathers or mothers are citizens
such other persons residing in the Philippine of the Philippines.
Islands who would become citizens of the United
States, under the laws of the United States, if "(4)Those whose mothers are citizens of the
residing therein." 26 Philippines and upon reaching the age of
majority, elect Philippine citizenship.
With the adoption of the Philippine Bill of 1902,
the concept of "Philippine citizens" had for the "(5)Those who are naturalized in accordance with
first time crystallized. The word "Filipino" was law."
used by William H. Taft, the first Civil Governor
General in the Philippines when he initially made Subsection (4), Article III, of the 1935
mention of it in his slogan, "The Philippines for Constitution, taken together with existing civil law
the Filipinos." In 1916, the Philippine Autonomy provisions at the time, which provided that
Act, also known as the Jones Law restated women would automatically lose their Filipino
virtually the provisions of the Philippine Bill of citizenship and acquire that of their foreign
1902, as so amended by the Act of Congress in husbands, resulted in discriminatory situations
1912 — that effectively incapacitated the women from
"That all inhabitants of the Philippine Islands who transmitting their Filipino citizenship to their
were Spanish subjects on the eleventh day of legitimate children and required illegitimate
April, eighteen hundred and ninety-nine, and then children of Filipino mothers to still elect Filipino
resided in said Islands, and their children born citizenship upon reaching the age of majority.
subsequently thereto, shall be deemed and held Seeking to correct this anomaly, as well as fully
to be citizens of the Philippine Islands, except cognizant of the newly found status of Filipino
such as shall have elected to preserve their women as equals to men, the framers of the 1973
allegiance to the Crown of Spain in accordance Constitution crafted the provisions of the new
with the provisions of the treaty of peace Constitution on citizenship to reflect such
between the United States and Spain, signed at concerns —
Paris December tenth, eighteen hundred and
ninety-eight and except such others as have "Section 1, Article III, 1973 Constitution — The
since become citizens of some other country; following are citizens of the Philippines:
Provided, That the Philippine Legislature, herein
provided for, is hereby authorized to provide for "(1)Those who are citizens of the Philippines at
the acquisition of Philippine citizenship by those the time of the adoption of this Constitution.
natives of the Philippine Islands who do not come
within the foregoing provisions, the natives of the "(2)Those whose fathers or mothers are citizens
insular possessions of the United States, and such of the Philippines.
other persons residing in the Philippine Islands
who are citizens of the United States, or who "(3)Those who elect Philippine citizenship
could become citizens of the United States under pursuant to the provisions of the Constitution of
the laws of the United States, if residing therein." nineteen hundred and thirty-five.

Under the Jones Law, a native-born inhabitant of "(4)Those who are naturalized in accordance with
the Philippines was deemed to be a citizen of the law."
Philippines as of 11 April 1899 if he was 1) a
subject of Spain on 11 April 1899, 2) residing in For good measure, Section 2 of the same article
the Philippines on said date, and, 3) since that also further provided that —
date, not a citizen of some other country. "A female citizen of the Philippines who marries
an alien retains her Philippine citizenship, unless
While there was, at one brief time, divergent by her act or omission she is deemed, under the
views on whether or not jus soli was a mode of law to have renounced her citizenship."
acquiring citizenship, the 1935 Constitution
brought to an end to any such link with common The 1987 Constitution generally adopted the
law, by adopting, once and for all, jus sanguinis provisions of the 1973 Constitution, except for
or blood relationship as being the basis of Filipino subsection (3) thereof that aimed to correct the
citizenship — irregular situation generated by the questionable
proviso in the 1935 Constitution.

95 | C O N S T I 2 _ A r t i c l e I V _ C I T I Z E N S H I P
Section 1, Article IV, 1987 Constitution now Allan F. Poe was stated to be twenty-five years
provides: old, unmarried, and a Filipino citizen, and Bessie
Kelley to be twenty-two years old, unmarried, and
"The following are citizens of the Philippines: an American citizen. The birth certificate of FPJ,
would disclose that he was born on 20 August
"(1)Those who are citizens of the Philippines at 1939 to Allan F. Poe, a Filipino, twenty-four years
the time of the adoption of this Constitution. old, married to Bessie Kelly, an American citizen,
twenty-one years old and married.
"(2)Those whose fathers or mothers are citizens
of the Philippines.

"(3)Those born before January 17, 1973 of Filipino Considering the reservations made by the parties
mothers, who elect Philippine citizenship upon on the veracity of some of the entries on the birth
reaching the age of majority; and certificate of respondent and the marriage
certificate of his parents, the only conclusions
"(4)Those who are naturalized in accordance with that could be drawn with some degree of
law." certainty from the documents would be that —

The Case Of FPJ 1.The parents of FPJ were Allan F. Poe and Bessie
Section 2, Article VII, of the 1987 Constitution Kelley;
expresses:
2.FPJ was born to them on 20 August 1939;
"No person may be elected President unless he is
a natural-born citizen of the Philippines, a 3.Allan F. Poe and Bessie Kelley were married to
registered voter, able to read and write, at least each other on 16 September, 1940;
forty years of age on the day of the election, and
a resident of the Philippines for at least ten years 4.The father of Allan F. Poe was Lorenzo Poe; and
immediately preceding such election."
5.At the time of his death on 11 September 1954,
The term "natural-born citizens," is defined to Lorenzo Poe was 84 years old.
include "those who are citizens of the Philippines
from birth without having to perform any act to Would the above facts be sufficient or insufficient
acquire or perfect their Philippine citizenship." 27 to establish the fact that FPJ is a natural-born
Filipino citizen? The marriage certificate of Allan F.
The date, month and year of birth of FPJ appeared Poe and Bessie Kelley, the birth certificate of FPJ,
to be 20 August 1939 during the regime of the and the death certificate of Lorenzo Pou are
1935 Constitution. Through its history, four documents of public record in the custody of a
modes of acquiring citizenship — naturalization, public officer. The documents have been
jus soli, res judicata and jus sanguinis 28 — had submitted in evidence by both contending parties
been in vogue. Only two, i.e., jus soli and jus during the proceedings before the COMELEC.
sanguinis, could qualify a person to being a
"natural-born" citizen of the Philippines. Jus soli, The birth certificate of FPJ was marked Exhibit "A"
per Roa vs. Collector of Customs 29 (1912), did for petitioner and Exhibit "3" for respondent. The
not last long. With the adoption of the 1935 marriage certificate of Allan F. Poe to Bessie
Constitution and the reversal of Roa in Tan Chong Kelley was submitted as Exhibit "21" for
vs. Secretary of Labor 30 (1947), jus sanguinis or respondent. The death certificate of Lorenzo Pou
blood relationship would now become the primary was submitted by respondent as his Exhibit "5."
basis of citizenship by birth. While the last two documents were submitted in
evidence for respondent, the admissibility
Documentary evidence adduced by petitioner thereof, particularly in reference to the facts
would tend to indicate that the earliest which they purported to show, i.e., the marriage
established direct ascendant of FPJ was his certificate in relation to the date of marriage of
paternal grandfather Lorenzo Pou, married to Allan F. Poe to Bessie Kelley and the death
Marta Reyes, the father of Allan F. Poe. While the certificate relative to the death of Lorenzo Pou on
record of birth of Lorenzo Pou had not been 11 September 1954 in San Carlos, Pangasinan,
presented in evidence, his death certificate, were all admitted by petitioner, who had utilized
however, identified him to be a Filipino, a resident those material statements in his argument. All
of San Carlos, Pangasinan, and 84 years old at three documents were certified true copies of the
the time of his death on 11 September 1954. The originals.
certificate of birth of the father of FPJ, Allan F.
Poe, showed that he was born on 17 May 1915 to Section 3, Rule 130, Rules of Court states that —
an Español father, Lorenzo Pou, and a mestiza
Español mother, Marta Reyes. Introduced by "Original document must be produced;
petitioner was an "uncertified" copy of a exceptions. — When the subject of inquiry is the
supposed certificate of the alleged marriage of contents of a document, no evidence shall be
Allan F. Poe and Paulita Gomez on 05 July 1936. admissible other than the original document
The marriage certificate of Allan F. Poe and Bessie itself, except in the following cases:
Kelley reflected the date of their marriage to be
on 16 September 1940. In the same certificate, xxx xxx xxx

96 | C O N S T I 2 _ A r t i c l e I V _ C I T I Z E N S H I P
compulsory acknowledgment was possible only if
"(d)When the original is a public record in the done during the lifetime of the putative parent;
custody of a public office or is recorded in a voluntary acknowledgment could only be had in a
public office." record of birth, a will, or a public document. 32
Complementary to the new code was Act No.
Being public documents, the death certificate of 3753 or the Civil Registry Law expressing in
Lorenzo Pou, the marriage certificate of Allan F. Section 5 thereof, that —
Poe and Bessie Kelly, and the birth certificate of
FPJ, constitute prima facie proof of their contents. "In case of an illegitimate child, the birth
Section 44, Rule 130, of the Rules of Court certificate shall be signed and sworn to jointly by
provides: the parents of the infant or only by the mother if
"Entries in official records. Entries in official the father refuses. In the latter case, it shall not
records made in the performance of his duty by a be permissible to state or reveal in the document
public officer of the Philippines, or by a person in the name of the father who refuses to
the performance of a duty specially enjoined by acknowledge the child, or to give therein any
law, are prima facie evidence of the facts therein information by which such father could be
stated." identified."

The trustworthiness of public documents and the In order that the birth certificate could then be
value given to the entries made therein could be utilized to prove voluntary acknowledgment of
grounded on 1) the sense of official duty in the filiation or paternity, the certificate was required
preparation of the statement made, 2) the to be signed or sworn to by the father. The failure
penalty which is usually affixed to a breach of of such requirement rendered the same useless
that duty, 3) the routine and disinterested origin as being an authoritative document of
of most such statements, and 4) the publicity of recognition. 33 In Mendoza vs. Mella, 34 the
record which makes more likely the prior Court ruled —
exposure of such errors as might have occurred. "Since Rodolfo was born in 1935, after the
31 registry law was enacted, the question here really
is whether or not his birth certificate (Exhibit 1),
The death certificate of Lorenzo Pou would which is merely a certified copy of the registry
indicate that he died on 11 September 1954, at record, may be relied upon as sufficient proof of
the age of 84 years, in San Carlos, Pangasinan. It his having been voluntarily recognized. No such
could thus be assumed that Lorenzo Pou was born reliance, in our judgment, may be placed upon it.
sometime in the year 1870 when the Philippines While it contains the names of both parents,
was still a colony of Spain. Petitioner would argue there is no showing that they signed the original,
that Lorenzo Pou was not in the Philippines during let alone swore to its contents as required in
the crucial period of from 1898 to 1902 Section 5 of Act No. 3753. For all that might have
considering that there was no existing record happened, it was not even they or either of them
about such fact in the Records Management and who furnished the data to be entered in the civil
Archives Office. Petitioner, however, likewise register. Petitioners say that in any event the
failed to show that Lorenzo Pou was at any other birth certificate is in the nature of a public
place during the same period. In his death document wherein voluntary recognition of a
certificate, the residence of Lorenzo Pou was natural child may also be made, according to the
stated to be San Carlos, Pangasinan. In the same Article 131. True enough, but in such a
absence of any evidence to the contrary, it case, there must be a clear statement in the
should be sound to conclude, or at least to document that the parent recognizes the child as
presume, that the place of residence of a person his or her own."
at the time of his death was also his residence
before death. It would be extremely doubtful if In the birth certificate of respondent FPJ,
the Records Management and Archives Office presented by both parties, nowhere in the
would have had complete records of all residents document was the signature of Allan F. Poe found.
of the Philippines from 1898 to 1902. There being no will apparently executed, or at
least shown to have been executed, by decedent
Proof of Paternity and Filiation Under Civil Law. Allan F. Poe, the only other proof of voluntary
Petitioner submits, in any case, that in recognition remained to be "some other public
establishing filiation (relationship or civil status of document." In Pareja vs. Pareja, 35 this Court
the child to the father [or mother]) or paternity defined what could constitute such a document
(relationship or civil status of the father to the as proof of voluntary acknowledgment:
child) of an illegitimate child, FPJ evidently being
an illegitimate son according to petitioner, the "Under the Spanish Civil Code there are two
mandatory rules under civil law must be used. classes of public documents, those executed by
private individuals which must be authenticated
Under the Civil Code of Spain, which was in force by notaries, and those issued by competent
in the Philippines from 08 December 1889 up public officials by reason of their office. The public
until the day prior to 30 August 1950 when the document pointed out in Article 131 as one of the
Civil Code of the Philippines took effect, means by which recognition may be made
acknowledgment was required to establish belongs to the first class."
filiation or paternity. Acknowledgment was either
judicial (compulsory) or voluntary. Judicial or Let us leave it at that for the moment.

97 | C O N S T I 2 _ A r t i c l e I V _ C I T I Z E N S H I P
"Art. 175.Illegitimate children may establish their
The 1950 Civil Code categorized the illegitimate filiation in the same way and on the
acknowledgment or recognition of illegitimate same evidence as legitimate children.
children into voluntary, legal or compulsory.
Voluntary recognition was required to be "The action must be brought within the same
expressedly made in a record of birth, a will, a period specified in Article 173, except when the
statement before a court of record or in any action is based on the second paragraph of
authentic writing. Legal acknowledgment took Article 172, in which case the action may be
place in favor of full blood brothers and sisters of brought during the lifetime of the alleged parent."
an illegitimate child who was recognized or
judicially declared as natural. Compulsory The provisions of the Family Code are
acknowledgment could be demanded generally in retroactively applied; Article 256 of the code
cases when the child had in his favor any reads:
evidence to prove filiation. Unlike an action to
claim legitimacy which would last during the "Art. 256.This Code shall have retroactive effect
lifetime of the child, and might pass exceptionally insofar as it does not prejudice or impair vested
to the heirs of the child, an action to claim or acquired rights in accordance with the Civil
acknowledgment, however, could only be brought Code or other laws."
during the lifetime of the presumed parent.
Thus, in Vda. De SyQuia vs. Court of Appeals, 36
Amicus Curiae Ruben F. Balane defined, during the Court has ruled:
the oral argument, "authentic writing," so as to "We hold that whether Jose was a voluntarily
be an authentic writing for purposes of voluntary recognized natural child should be decided under
recognition, simply as being a genuine or Article 278 of the Civil Code of the Philippines.
indubitable writing of the father. The term would Article 2260 of that Code provides that 'the
include a public instrument (one duly voluntary recognition of a natural child shall take
acknowledged before a notary public or other place according to this Code, even if the child
competent official) or a private writing admitted was born before the effectivity of this body of
by the father to be his. laws' or before August 30, 1950. Hence, Article
278 may be given retroactive effect."
The Family Code has further liberalized the rules;
Article 172, Article 173, and Article 175 provide: It should be apparent that the growing trend to
liberalize the acknowledgment of recognition of
"Art. 172.The filiation of legitimate children is illegitimate children is an attempt to break away
established by any of the following: from the traditional idea of keeping well apart
legitimate and non-legitimate relationships within
"(1)The record of birth appearing in the civil the family in favor of the greater interest and
register or a final judgment; or welfare of the child. The provisions are intended
to merely govern the private and personal affairs
"(2)An admission of legitimate filiation in a public of the family. There is little, if any, to indicate that
document or a private handwritten instrument the legitimate or illegitimate civil status of the
and signed by the parent concerned. individual would also affect his political rights or,
in general, his relationship to the State. While,
"In the absence of the foregoing evidence, the indeed, provisions on "citizenship" could be found
legitimate filiation shall be proved by: in the Civil Code, such provisions must be taken
in the context or private relations, the domain of
"(1)The open and continuous possession of the civil law; particularly —
status of a legitimate child; or
"Civil Law is that branch of law which has for its
"(2)Any other means allowed by the Rules of double purpose the organization of the family and
Court and special laws. the regulation of property. It has thus [been]
defined as the mass of precepts which determine
"Art. 173.The action to claim legitimacy may be and regulate the relations of assistance, authority
brought by the child during his or her lifetime and and obedience among member of a family, and
shall be transmitted to the heirs should the child those which exist among members of a society
die during minority or in a state of insanity. In for the protection of private interests." 37
these cases, the heirs shall have a period of five
years within which to institute the action. In Yañez de Barnuevo vs. Fuster, 38 the Court has
held:

"In accordance with Article 9 of the Civil Code of


"The action already commenced by the child shall Spain, . . . the laws relating to family rights and
survive notwithstanding the death of either or duties, or to the status, condition and legal
both the parties. capacity of persons, govern Spaniards although
they reside in a foreign country; that, in
"xxx xxx xxx consequence, 'all questions of a civil nature, such
as those dealing with the validity or nullity of the
matrimonial bond, the domicile of the husband
and wife, their support, as between them, the

98 | C O N S T I 2 _ A r t i c l e I V _ C I T I Z E N S H I P
separation of their properties, the rules governing "Act or Declaration about pedigree. The act or
property, marital authority, division of conjugal declaration of a person deceased, or unable to
property, the classification of their property, legal testify, in respect to the pedigree of another
causes for divorce, the extent of the latter, the person related to him by birth or marriage, may
authority to decree it, and, in general, the civil be received in evidence where it occurred before
effects of marriage and divorce upon the persons the controversy, and the relationship between the
and properties of the spouses, are questions that two persons is shown by evidence other than
are governed exclusively by the national law of such act or declaration. The word 'pedigree'
the husband and wife." includes relationship, family genealogy, birth,
marriage, death, the dates when and the places
The relevance of "citizenship" or "nationality" to where these facts occurred, and the names of the
Civil Law is best exemplified in Article 15 of the relatives. It embraces also facts of family history
Civil Code, stating that — intimately connected with pedigree."

"Laws relating to family rights and duties, or to For the above rule to apply, it would be necessary
the status, condition and legal capacity of that (a) the declarant is already dead or unable to
persons are binding upon citizens of the testify, (b) the pedigree of a person must be at
Philippines, even though living abroad" — issue, (c) the declarant must be a relative of the
person whose pedigree is in question, (d)
that explains the need to incorporate in the code declaration must be made before the controversy
a reiteration of the Constitutional provisions on has occurred, and (e) the relationship between
citizenship. Similarly, citizenship is significant in the declarant and the person whose pedigree is in
civil relationships found in different parts of the question must be shown by evidence other than
Civil Code, 39 such as on successional rights and such act or declaration.
family relations. 40 In adoption, for instance, an Thus, the duly notarized declaration made by
adopted child would be considered the child of his Ruby Kelley Mangahas, sister of Bessie Kelley Poe
adoptive parents and accorded the same rights submitted as Exhibit 20 before the COMELEC,
as their legitimate child but such legal fiction might be accepted to prove the facts of Allan F.
extended only to define his rights under civil law Poe, recognizing his own paternal relationship
41 and not his political status. with FPJ, i.e., living together with Bessie Kelly and
Civil law provisions point to an obvious bias his children (including respondent FPJ) in one
against illegitimacy. This discriminatory attitude house, and as one family —
may be traced to the Spanish family and property
laws, which, while defining proprietary and "I, Ruby Kelly Mangahas, of legal age and sound
successional rights of members of the family, mind, presently residing in Stockton, California,
provided distinctions in the rights of legitimate U.S.A., after being sworn in accordance with law
and illegitimate children. In the monarchial set-up do hereby declare that:
of old Spain, the distribution and inheritance of
titles and wealth were strictly according to "1.I am the sister of the late Bessie Kelly Poe.
bloodlines and the concern to keep these
bloodlines uncontaminated by foreign blood was "2.Bessie Kelley Poe was the wife of Fernando
paramount. Poe, Sr.

These distinctions between legitimacy and "3.Fernando and Bessie Poe had a son by the
illegitimacy were codified in the Spanish Civil name of Ronald Allan Poe, more popularly known
Code, and the invidious discrimination survived in the Philippines as 'Fernando Poe, Jr., or FPJ'.
when the Spanish Civil Code became the primary
source of our own Civil Code. Such distinction, "4.Ronald Allan Poe 'FPJ' was born on August 20,
however, remains and should remain only in the 1939 at St. Luke's Hospital, Magdalena Street,
sphere of civil law and not unduly impede or Manila.
impinge on the domain of political law.
"xxx xxx xxx
The proof of filiation or paternity for purposes of "7.Fernando Poe Sr., and my sister Bessie, met
determining his citizenship status should thus be and became engaged while they were students at
deemed independent from and not inextricably the University of the Philippines in 1936. I was
tied up with that prescribed for civil law purposes. also introduced to Fernando Poe Sr., by my sister
The Civil Code or Family Code provisions on proof that same year.
of filiation or paternity, although good law, do not
have preclusive effects on matters alien to "8.Fernando Poe, Sr., and my sister Bessie had
personal and family relations. The ordinary rules their first child in 1938.
on evidence could well and should govern. For
instance, the matter about pedigree is not "9.Fernando Poe, Sr., my sister Bessie and their
necessarily precluded from being applicable by first three children, Elizabeth, Ronald, Allan and
the Civil Code or Family Code provisions. Fernando II, and myself lived together with our
mother at our family's house on Dakota St. (now
Section 39, Rule 130, of the Rules of Court Jorge Bocobo St.), Malate until the liberation of
provides — Manila in 1945, except for some months between
1943-1944.

99 | C O N S T I 2 _ A r t i c l e I V _ C I T I Z E N S H I P
"10.Fernando Poe, Sr., and my sister, Bessie, were himself, consisting of a birth certificate of
blessed with four (4) more children after Ronald respondent and a marriage certificate of his
Allan Poe. parents showed that FPJ was born on 20 August
1939 to Filipino father and an American mother
"xxx xxx xxx who were married to each other a year later, or
"18.I am executing this Declaration to attest to on 16 September 1940. Birth to unmarried
the fact that my nephew, Ronald Allan Poe is a parents would make FPJ an illegitimate child.
natural born Filipino, and that he is the legitimate Petitioner contended that as an illegitimate child,
child of Fernando Poe, Sr. FPJ so followed the citizenship of his mother,
Bessie Kelly, an American citizen, basing his
"Done in City of Stockton, California, U.S.A., this stand on the ruling of this Court in Morano vs.
12th day of January 2004. Vivo, 43 citing Chiongbian vs. de Leon 44 and
Serra vs. Republic. 45
Ruby Kelly Mangahas
On the above score, the disqualification made by
Declarant amicus curiae Joaquin G. Bernas, SJ, is most
DNA Testing convincing; he states —
In case proof of filiation or paternity would be
unlikely to satisfactory establish or would be "We must analyze these cases and ask what the
difficult to obtain, DNA testing, which examines lis mota was in each of them. If the procurement
genetic codes obtained from body cells of the of the Court on jus sanguinis was on the lis mota,
illegitimate child and any physical residue of the the pronouncement would be a decision
long dead parent could be resorted to. A positive constituting doctrine under the rule of stare
match would clear up filiation or paternity. In decisis. But if the pronouncement was irrelevant
Tijing vs. Court of Appeals, 42 this Court has to the lis mota, the pronouncement would not be
acknowledged the strong weight of DNA testing a decision but a mere obiter dictum which did not
— establish doctrine. I therefore invite the Court to
look closely into these cases.
"Parentage will still be resolved using
conventional methods unless we adopt the "First, Morano vs. Vivio. The case was not about
modern and scientific ways available. Fortunately, an illegitimate child of a Filipino father. It was
we have now the facility and expertise in using about a stepson of a Filipino, a stepson who was
DNA test for identification and parentage testing. the child of a Chinese mother and a Chinese
The University of the Philippines Natural Science father. The issue was whether the stepson
Research Institute (UP-NSRI) DNA Analysis followed the naturalization of the stepfather.
Laboratory has now the capability to conduct Nothing about jus sanguinis there. The stepson
DNA typing using short tandem repeat (STR) did not have blood of the naturalized stepfather.
analysis. The analysis is based on the fact that
the DNA of a child/person has two (2) copies, one "Second, Chiongbian vs. de Leon. This case was
copy from the mother and the other from the not about the illegitimate son of a Filipino father.
father. The DNA from the mother, the alleged It was about a legitimate son of a father who had
father and the child are analyzed to establish become Filipino by election to public office before
parentage. Of course, being a novel scientific the 1935 Constitution pursuant to Article IV,
technique, the use of DNA test as evidence is still Section 1(2) of the 1935 Constitution. No one was
open to challenge. Eventually, as the appropriate illegitimate here.
case comes, courts should not hesitate to rule on
the admissibility of DNA evidence. For it was said, "Third, Serra vs. Republic. The case was not about
that courts should apply the results of science the illegitimate son of a Filipino father. Serra was
when completely obtained in aid of situations an illegitimate child of a Chinese father and a
presented, since to reject said result is to deny Filipino mother. The issue was whether one who
progress." was already a Filipino because of his mother who
still needed to be naturalized. There is nothing
there about invidious jus sanguinis.

Petitioner's Argument For Jurisprudential "Finally, Paa vs. Chan. 46 This is more
Conclusiveness complicated case. The case was about the
Petitioner would have it that even if Allan F. Poe citizenship of Quintin Chan who was the son of
were a Filipino citizen, he could not have Leoncio Chan. Quintin Chan claimed that his
transmitted his citizenship to respondent FPJ, the father, Leoncio, was the illegitimate son of a
latter being an illegitimate child. According to Chinese father and a Filipino mother. Quintin
petitioner, prior to his marriage to Bessie Kelly, therefore argued that he got his citizenship from
Allan F. Poe, on July 5, 1936, contracted marriage Leoncio, his father. But the Supreme Court said
with a certain Paulita Gomez, making his that there was no valid proof that Leoncio was in
subsequent marriage to Bessie Kelly bigamous fact the son of a Filipina mother. The Court
and respondent FPJ an illegitimate child. The therefore concluded that Leoncio was not Filipino.
veracity of the supposed certificate of marriage If Leoncio was not Filipino, neither was his son
between Allan F. Poe and Paulita Gomez could be Quintin. Quintin therefore was not only not a
most doubtful at best. But the documentary natural-born Filipino but was not even a Filipino.
evidence introduced by no less than respondent

100 | C O N S T I 2 _ A r t i c l e I V _ C I T I Z E N S H I P
"The Court should have stopped there. But Constitution, the fundamental law prevailing on
instead it followed with an obiter dictum. The the day, month and year of birth of respondent
Court said obiterthat even if Leoncio, Quintin's FPJ, can never be more explicit than it is.
father, were Filipino, Quintin would not be Filipino Providing neither conditions nor distinctions, the
because Quintin was illegitimate. This statement Constitution states that among the citizens of the
about Quintin, based on a contrary to fact Philippines are "those whose fathers are citizens
assumption, was absolutely necessary for the of the Philippines." There utterly is no cogent
case. . . . It was obiter dictum, pure and simple, justification to prescribe conditions or distinctions
simply repeating the obiter dictum in Morano vs. where there are clearly none provided.
Vivo.
In Sum —
"xxx xxx xxx (1)The Court, in the exercise of its power of
judicial review, possesses jurisdiction over the
"Aside from the fact that such a pronouncement petition in G.R. No. 161824, filed under Rule 64,
would have no textual foundation in the in relation to Rule 65, of the Revised Rules of Civil
Constitution, it would also violate the equal Procedure. G.R. No. 161824 assails the resolution
protection clause of the Constitution not once but of the COMELEC for alleged grave abuse of
twice. First, it would make an illegitimate discretion in dismissing, for lack of merit, the
distinction between a legitimate child and an petition in SPA No. 04-003 which has prayed for
illegitimate child, and second, it would make an the disqualification of respondent FPJ from
illegitimate distinction between the illegitimate running for the position of President in the 10th
child of a Filipino father and the illegitimate child May 2004 national elections on the contention
of a Filipino mother. that FPJ has committed material representation in
his certificate of candidacy by representing
"The doctrine on constitutionality allowable himself to be a natural-born citizen of the
distinctions was established long ago by People Philippines.
vs. Cayat. 47 I would grant that the distinction
between legitimate children and illegitimate (2)The Court must dismiss, for lack of jurisdiction
children rests on real differences. . . . But real and prematurity, the petitions in G.R. No. 161434
differences alone do not justify invidious and No. 161634 both having been directly
distinction. Real differences may justify elevated to this Court in the latter's capacity as
distinction for one purpose but not for another the only tribunal to resolve a presidential and
purpose. vice-presidential election contest under the
Constitution. Evidently, the primary jurisdiction of
". . . What is the relevance of legitimacy to the Court can directly be invoked only after, not
elective public service? What possible state before, the elections are held.
interest can there be for disqualifying an
illegitimate child from becoming a public officer. It (3)In ascertaining, in G.R. No. 161824, whether
was not the fault of the child that his parents had grave abuse of discretion has been committed by
illicit liaison. Why deprive the child of the fullness the COMELEC, it is necessary to take on the
of political rights for no fault of his own? To matter of whether or not respondent FPJ is a
disqualify an illegitimate child from holding an natural-born citizen, which, in turn, depended on
important public office is to punish him for the whether or not the father of respondent, Allan F.
indiscretion of his parents. There is neither justice Poe, would have himself been a Filipino citizen
nor rationality in that. And if there is neither and, in the affirmative, whether or not the alleged
justice nor rationality in the distinction, then the illegitimacy of respondent prevents him from
distinction transgresses the equal protection taking after the Filipino citizenship of his putative
clause and must be reprobated." father. Any conclusion on the Filipino citizenship
of Lorenzo Pou could only be drawn from the
The other amici curiae, Mr. Justice Vicente presumption that having died in 1954 at 84 years
Mendoza (a former member of this Court), old, Lorenzo would have been born sometime in
Professor Ruben Balane and Dean Martin the year 1870, when the Philippines was under
Magallona, at bottom, have expressed similar Spanish rule, and that San Carlos, Pangasinan, his
views. The thesis of petitioner, unfortunately place of residence upon his death in 1954, in the
hinging solely on pure obiter dicta, should indeed absence of any other evidence, could have well
fail. been his place of residence before death, such
that Lorenzo Pou would have benefited from the
Where jurisprudence regarded an illegitimate "en masse Filipinization" that the Philippine bill
child as taking after the citizenship of its mother, had effected in 1902. That citizenship (of Lorenzo
it did so for the benefit of the child. It was to Pou), if acquired, would thereby extend to his son,
ensure a Filipino nationality for the illegitimate Allan F. Poe, father of respondent FPJ. The 1935
child of an alien father in line with the assumption Constitution, during which regime respondent FPJ
that the mother had custody, would exercise has seen first light, confers citizenship to all
parental authority and had the duty to support persons whose fathers are Filipino citizens
her illegitimate child. It was to help the child, not regardless of whether such children are
to prejudice or discriminate against him. legitimate or illegitimate.

The fact of the matter — perhaps the most (4)But while the totality of the evidence may not
significant consideration — is that the 1935 establish conclusively that respondent FPJ is a

101 | C O N S T I 2 _ A r t i c l e I V _ C I T I Z E N S H I P
natural-born citizen of the Philippines, the
evidence on hand still would preponderate in his Azcuna, J., concurs in a separate opinion.
favor enough to hold that he cannot be held
guilty of having made a material Tinga, J., dissents as per his separate opinion.
misrepresentation in his certificate of candidacy
in violation of Section 78, in relation to Section
74, of the Omnibus Election Code. Petitioner has
utterly failed to substantiate his case before the
Court, notwithstanding the ample opportunity
given to the parties to present their position and
evidence, and to prove whether or not there has
been material misrepresentation, which, as so (Republic v. Lim, G.R. No. 153883, January
ruled in Romualdez-Marcos vs. COMELEC, 48 13, 2004)
must not only be material, but also deliberate
and willful. FIRST DIVISION

[G.R. No. 153883. January 13, 2004.]

WHEREFORE, the Court RESOLVES to DISMISS — REPUBLIC OF THE PHILIPPINES, petitioner, vs.
CHULE Y. LIM, respondent.
1.G.R. No. 161434, entitled "Maria Jeanette C.
Tecson and Felix B. Desiderio, Jr., Petitioners, DECISION
versus Commission on Elections, Ronald Allan
Kelley Poe (a.k.a. "Fernando Poe, Jr.,) and YNARES-SANTIAGO, J p:
Victorino X. Fornier, Respondents," and G.R. No.
161634, entitled "Zoilo Antonio Velez, Petitioner, This petition for review on certiorari under Rule
versus Ronald Allan Kelley Poe, a.k.a. Fernando 45 of the Rules of Court stemmed from a petition
Poe, Jr., Respondent," for want of jurisdiction. for correction of entries under Rule 108 of the
Rules of Court filed by respondent Chule Y. Lim
2.G.R. No. 161824, entitled "Victorino X. Fornier, with the Regional Trial Court of Lanao del Norte,
Petitioner, versus Hon. Commission on Elections Branch 4, docketed as Sp. Proc. No. 4933.
and Ronald Allan Kelley Poe, also known as
Fernando Poe, Jr.," for failure to show grave abuse In her petition, respondent claimed that she was
of discretion on the part of respondent born on October 29, 1954 in Buru-an, Iligan City.
Commission on Elections in dismissing the Her birth was registered in Kauswagan, Lanao del
petition in SPA No. 04-003. Norte but the Municipal Civil Registrar of
Kauswagan transferred her record of birth to
No Costs. ASCTac Iligan City. She alleged that both her Kauswagan
and Iligan City records of birth have four
SO ORDERED. erroneous entries, and prays that they be
corrected.
Davide, Jr., C.J., see separate opinion.
The trial court then issued an Order, 1 which
Puno, J., is on leave but was allowed to vote; see reads:
separate opinion.
WHEREFORE, finding the petition to be sufficient
Panganiban, J., is on official leave; allowed to vote in form and substance, let the hearing of this
but did not send his vote. case be set on December 27, 1999 before this
Court, Hall of Justice, Rosario Heights, Tubod,
Quisumbing, J., joins the dissent of Justices Tinga Iligan City at 8:30 o'clock in the afternoon at
and Morales; case should have been REMANDED. which date, place and time any interested person
may appear and show cause why the petition
Ynares-Santiago, J., concurs and also with J. should not be granted.
Puno's separate opinion.
Let this order be published in a newspaper of
Sandoval-Gutierrez, J., concurs, please see general circulation in the City of Iligan and the
separate opinion. Province of Lanao del Norte once a week for three
(3) consecutive weeks at the expense of the
Carpio, J., see separate opinion. petitioner.

Austria-Martinez, J., concurs, please see separate Furnish copies of this order the Office of the
opinion. Solicitor General at 134 Amorsolo St., Legaspi
Vill., Makati City and the Office of the Local Civil
Corona, J., joins the dissenting opinion of Justice Registrar of Iligan City at Quezon Ave., Pala-o,
Morales. Iligan City.

Carpio Morales, J., see dissenting opinion. SO ORDERED.

Callejo, Sr., J ., see concurring opinion. During the hearing, respondent testified thus:

102 | C O N S T I 2 _ A r t i c l e I V _ C I T I Z E N S H I P
The Republic of the Philippines appealed the
First, she claims that her surname "Yu" was decision to the Court of Appeals which affirmed
misspelled as "Yo". She has been using "Yu" in all the trial court's decision. 5
her school records and in her marriage certificate.
2 She presented a clearance from the National Hence, this petition on the following assigned
Bureau of Investigation (NBI) 3 to further show errors:
the consistency in her use of the surname "Yu".
I
Second, she claims that her father's name in her
birth record was written as "Yo Diu To (Co Tian)" THE COURT OF APPEALS ERRED IN ORDERING
when it should have been "Yu Dio To (Co Tian)." THE CORRECTION OF THE CITIZENSHIP OF
RESPONDENT CHULE Y. LIM FROM "CHINESE" TO
Third, her nationality was entered as Chinese "FILIPINO" DESPITE THE FACT THAT RESPONDENT
when it should have been Filipino considering that NEVER DEMONSTRATED ANY COMPLIANCE WITH
her father and mother never got married. Only THE LEGAL REQUIREMENTS FOR ELECTION OF
her deceased father was Chinese, while her CITIZENSHIP.
mother is Filipina. She claims that her being a
registered voter attests to the fact that she is a II
Filipino citizen.
THE COURT OF APPEALS ERRED IN ALLOWING
Finally, it was erroneously indicated in her birth RESPONDENT TO CONTINUE USING HER FATHER'S
certificate that she was a legitimate child when SURNAME DESPITE ITS FINDING THAT
she should have been described as illegitimate RESPONDENT IS AN ILLEGITIMATE CHILD. 6
considering that her parents were never married.
To digress, it is just as well that the Republic did
Placida Anto, respondent's mother, testified that not cite as error respondent's recourse to Rule
she is a Filipino citizen as her parents were both 108 of the Rules of Court to effect what
Filipinos from Camiguin. She added that she and indisputably are substantial corrections and
her daughter's father were never married changes in entries in the civil register. To clarify,
because the latter had a prior subsisting marriage Rule 108 of the Revised Rules of Court provides
contracted in China. the procedure for cancellation or correction of
entries in the civil registry. The proceedings under
In this connection, respondent presented a said rule may either be summary or adversary in
certification attested by officials of the local civil nature. If the correction sought to be made in the
registries of Iligan City and Kauswagan, Lanao del civil register is clerical, then the procedure to be
Norte that there is no record of marriage between adopted is summary. If the rectification affects
Placida Anto and Yu Dio To from 1948 to the the civil status, citizenship or nationality of a
present. party, it is deemed substantial, and the procedure
to be adopted is adversary. This is our ruling in
The Republic, through the City Prosecutor of Republic v. Valencia 7 where we held that even
Iligan City, did not present any evidence although substantial errors in a civil registry may be
it actively participated in the proceedings by corrected and the true facts established under
attending hearings and cross-examining Rule 108 provided the parties aggrieved by the
respondent and her witnesses. error avail themselves of the appropriate
adversary proceeding. An appropriate adversary
On February 22, 2000, the trial court granted suit or proceeding is one where the trial court has
respondent's petition and rendered judgment as conducted, proceedings where all relevant facts
follows: have been fully and properly developed, where
opposing counsel have been given opportunity to
WHEREFORE, the foregoing premises considered, demolish the opposite party's case, and where
to set the records of the petitioner straight and in the evidence has been thoroughly weighed and
their proper perspective, the petition is granted considered. 8
and the Civil Registrar of Iligan City is directed to
make the following corrections in the birth As likewise observed by the Court of Appeals, we
records of the petitioner, to wit: take it that the Republic's failure to cite this error
amounts to a recognition that this case properly
1. Her family name from "YO" to "YU"; falls under Rule 108 of the Revised Rules of Court
considering that the proceeding can be
2. Her father's name from "YO DIU TO (CO TIAN)" appropriately classified as adversarial.
to "YU DIOTO (CO TIAN)";
Instead, in its first assignment of error, the
3. Her status from "legitimate" to "illegitimate" by Republic avers that respondent did not comply
changing "YES" to "NO" in answer to the question with the constitutional requirement of electing
"LEGITIMATE?"; and, Filipino citizenship when she reached the age of
majority. It cites Article IV, Section 1(3) of the
4. Her citizenship from "Chinese" to "Filipino". 1935 Constitution, which provides that the
citizenship of a legitimate child born of a Filipino
SO ORDERED. 4 mother and an alien father followed the
citizenship of the father, unless, upon reaching

103 | C O N S T I 2 _ A r t i c l e I V _ C I T I Z E N S H I P
the age of majority, the child elected Philippine We agree with the Court of Appeals when it held:
citizenship. 9 Likewise, the Republic invokes the
provision in Section 1 of Commonwealth Act No. Firstly, Petitioner-appellee is now 47 years old. To
625, that legitimate children born of Filipino bar her at this time from using her father's
mothers may elect Philippine citizenship by surname which she has used for four decades
expressing such intention "in a statement to be without any known objection from anybody,
signed and sworn to by the party concerned would only sow confusion. Concededly, one of the
before any officer authorized to administer oaths, reasons allowed for changing one's name or
and shall be filed with the nearest civil registry. surname is to avoid confusion.
The said party shall accompany the aforesaid
statement with the oath of allegiance to the Secondly, under Sec. 1 of Commonwealth Act No.
Constitution and the Government of the 142, the law regulating the use of aliases, a
Philippines." 10 person is allowed to use a name "by which he has
been known since childhood."
Plainly, the above constitutional and statutory
requirements of electing Filipino citizenship apply
only to legitimate children. These do not apply in
the case of respondent who was concededly an Thirdly, the Supreme Court has already
illegitimate child, considering that her Chinese addressed the same issue. In Pabellar v. Rep. of
father and Filipino mother were never married. As the Phils., 16 we held:
such, she was not required to comply with said
constitutional and statutory requirements to Section 1 of Commonwealth Act No. 142, which
become a Filipino citizen. By being an illegitimate, regulates the use of aliases, allows a person to
child of a Filipino mother, respondent use a name "by which he has been known since
automatically became a Filipino upon birth. childhood" (Lim Hok Albano v. Republic, 104 Phil.
Stated differently, she is a Filipino since birth 795; People v. Uy Jui Pio, 102 Phil. 679; Republic
without having to elect Filipino citizenship when v. Tañada, infra). Even legitimate children cannot
she reached the age of majority. acCITS enjoin the illegitimate children of their father from
using his surname (De Valencia v. Rodriguez, 84
In Ching, Re: Application for Admission to the Bar, Phil. 222). 17
11 citing In re Florencio Mallare, 12 we held:
While judicial authority is required for a change of
Esteban Mallare, natural child of Ana Mallare, a name or surname, 18 there is no such
Filipina, is therefore himself a Filipino, and no requirement for the continued use of a surname
other act would be necessary to confer on him all which a person has already been using since
the rights and privileges attached to Philippine childhood. 19
citizenship (US. vs. Ong Tianse, 29 Phil. 332;
Santos Co vs. Government of the Philippine The doctrine that disallows such change of name
Islands, 42 Phil. 543; Serra vs. Republic, L-4223, as would give the false impression of family
May 12, 1952; Sy Quimsuan vs. Republic, L-4693, relationship remains valid but only to the extent
Feb. 16, 1953; Pitallano vs. Republic, L-5111, June that the proposed change of name would in great
28, 1954). Neither could any act be taken on the probability cause prejudice or future mischief to
erroneous belief that he is a non-Filipino divest the family whose surname it is that is involved or
him of the citizenship privileges to which he is to the community in general. 20 In this case, the
rightfully entitled. 13 Republic has not shown that the Yu family in
China would probably be prejudiced or be the
This notwithstanding, the records show that object of future mischief. In respondent's case,
respondent elected Filipino citizenship when she the change in the surname that she has been
reached the age of majority. She registered as a using for 40 years would even avoid confusion to
voter in Misamis Oriental when she was 18 years her community in general.
old. 14 The exercise of the right of suffrage and
the participation in election exercises constitute a WHEREFORE, in view of the foregoing, the instant
positive act of election of Philippine citizenship. petition for review is DENIED. The decision of the
15 Court of Appeals in CA-G.R. CV No. 68893 dated
May 29, 2002, is AFFIRMED. Accordingly, the Civil
In its second assignment of error, the Republic Registrar of Iligan City is DIRECTED to make the
assails the Court of Appeals' decision in allowing following corrections in the birth record of
respondent to use her father's surname despite respondent Chule Y. Lim, to wit:
its finding that she is illegitimate.
1. Her family name from "YO" to "YU";
The Republic's submission is misleading. The
Court of Appeals did not allow respondent to use 2. Her father's name from "YO DIU TO (CO TIAN)"
her father's surname. What it did allow was the to "YU DIOTO (CO TIAN)";
correction of her father's misspelled surname
which she has been using ever since she can 3. Her status from "legitimate" to "illegitimate" by
remember. In this regard, respondent does not changing "YES" to "NO" in answer to the question
need a court pronouncement for her to use her "LEGITIMATE?"; and,
father's surname.

104 | C O N S T I 2 _ A r t i c l e I V _ C I T I Z E N S H I P
4. Her citizenship from "Chinese" to "Filipino". Certificate of Repatriation by the Special
CAIaHS Committee on Naturalization, after he filed a
petition for repatriation pursuant to Republic Act
SO ORDERED. No. 8171. Thus, petitioner claimed that his
Filipino citizenship was already restored, and he
Davide, Jr., C.J., Panganiban, Carpio and Azcuna, was qualified to run as mayor in the May 10,
JJ., concur. 2004 elections. Petitioner sought the dismissal of
the petition. cEaTHD

On the date of the hearing, the parties were


required to submit their Memoranda within three
(Altarejos v. COMELEC, G.R. No. 163256, days. Private respondents filed their
November 10, 2004) Memorandum, while petitioner did not file one
within the required period. 4 Petitioner, however,
EN BANC filed a Reply Memorandum 5 subsequently.

[G.R. No. 163256. November 10, 2004.] Atty. Zacarias C. Zaragoza, Jr., regional election
director for Region V and hearing officer of this
CICERON P. ALTAREJOS, petitioner, vs. case, recommended that petitioner Altarejos be
COMMISSION ON ELECTIONS, JOSE ALMIÑE and disqualified from being a candidate for the
VERNON VERSOZA, respondents. position of mayor of San Jacinto, Masbate in the
May 10, 2004 national and local elections. He
DECISION found, thus:

AZCUNA, J p: xxx xxx xxx

This is a petition for certiorari, with prayer for the The provisions of law governing the qualifications
issuance of a temporary restraining order and/or and disqualifications of elective local officials are
a writ of prohibitory and mandatory injunction, to found in Sections 39 and 40 of Republic Act No.
set aside the Resolution promulgated by the 7160 otherwise known as The Local Government
Commission on Elections (COMELEC), First Code of 1991, which provide as follows:
Division, on March 22, 2004 disqualifying
petitioner Ciceron P. Altarejos from running as SEC. 39. Qualifications. — (a) An elective local
mayor of San Jacinto, Masbate, and another official must be a citizen of the Philippines; a
resolution of the COMELEC en banc promulgated registered voter in the barangay, municipality,
on May 7, 2004 denying petitioner's motion for city or province or, in the case of member of the
reconsideration. sangguniang panlalawigan, sangguniang
panlungsod, or sangguniang bayan, the district
The factual antecedents are as follows: where he intends to be elected; a resident therein
for at least one (1) year immediately preceding
Petitioner Altarejos was a candidate for mayor in the day of the election; and able to read and
the Municipality of San Jacinto, Masbate in the write Filipino or any other local language or
May 10, 2004 national and local elections. dialect.

On January 15, 2004, private respondents Jose xxx xxx xxx.


Almiñe Altiche and Vernon Versoza, registered (c) Candidates for the position of mayor or vice-
voters of San Jacinto, Masbate, filed with the mayor of independent component cities,
COMELEC, a petition to disqualify and to deny component cities or municipalities must be at
due course or cancel the certificate of candidacy least twenty-one (21) years of age on election
of petitioner on the ground that he is not a day.
Filipino citizen and that he made a false
representation in his certificate of candidacy that [SEC. 40. Disqualifications. — The following
"[he] was not a permanent resident of or persons are disqualified from running for any
immigrant to a foreign country." elective position:]

Private respondents alleged that based on a letter xxx xxx xxx.


1 from the Bureau of Immigration dated June 25, (d) Those with dual citizenship.
2001, petitioner was a holder of a permanent U.S.
resident visa, an Alien Certificate of Registration xxx xxx xxx.
No. E139507 issued on November 3, 1997, and (f) Permanent residents in a foreign country or
an Immigration Certificate of Residence No. those who have acquired the right to reside
320846 issued on November 3, 1997 by the abroad and continue to avail of the same right
Bureau of Immigration. 2 after the effectivity of this Code; . . .

On January 26, 2004, petitioner filed an Answer 3 Under the terms of the above quoted statutory
stating, among others, that he did not commit provisions, it is required that an elective local
false representation in his application for official must be a citizen of the Philippines, and
candidacy as mayor because as early as he must not have a dual citizenship; must not be
December 17, 1997, he was already issued a

105 | C O N S T I 2 _ A r t i c l e I V _ C I T I Z E N S H I P
a permanent resident in a foreign country or must perfected the reacquisition of Filipino citizenship.
not have acquired the right to reside abroad. Such false representation constitutes a material
misrepresentation as it relates to his qualification
In the present case, it has been established by as a candidate for public office, which could be a
clear and convincing evidence that respondent is valid ground for the cancellation of his certificate
a citizen of the United States of America. Such of candidacy under Section 78 of the Omnibus
fact is proven by his Alien Certificate of Election Code . . . 6
Registration (ACR) No. E139507 issued on 3
November 1997 and Immigration Certificate of In its Resolution promulgated on March 22, 2004,
Residence (ICR) with No. 320846 issued on 3 the COMELEC, First Division, adopted the findings
November 1997 by the Alien Registration and recommendation of Director Zaragoza. The
Division, Bureau of Immigration and Deportation. dispositive portion of said Resolution stated, thus:
This was further confirmed in a letter dated 25
June 2001 of then Commissioner ANDREA D. WHEREFORE, premises considered, respondent
DOMINGO of the Bureau of Immigration and CICERON PEREZ ALTAREJOS is hereby disqualified
Deportation. to run as Mayor of San Jacinto, Masbate.
Accordingly, his certificate of candidacy for the
Although respondent had petitioned for his position of Municipal Mayor of San Jacinto,
repatriation as a Filipino citizen under Republic Masbate is denied due course and cancelled and
Act No. 8171 on 17 December 1997, this did not his name deleted from the certified list of
restore to respondent his Filipino citizenship, candidates for the May 10, 2004 elections. 7
because Section 2 of the aforecited Republic Act
No. 8171 specifically provides that "repatriation On March 25, 2004, petitioner filed a motion for
shall be effected by taking the necessary oath of reconsideration and attached the following
allegiance to the Republic of the Philippines and documents to prove that he had completed all
registration in the proper civil registry and in the the requirements for repatriation which thus
Bureau of Immigration." SDAcaT entitled him to run for an elective office, viz:

It appears from the records of this case that (1) Oath of Allegiance dated December 17, 1997;
respondent failed to prove that he has fully
complied with requirements of the above-quoted (2) Identification Certificate No. 116543 issued by
Section 2 of Republic Act 8171 to perfect his the Bureau of Immigration on March 1, 2004;
repatriation and reacquire his Filipino citizenship.
Respondent has not submitted any document to (3) Certification from the City Civil Registration
prove that he has taken his oath of allegiance to Office, Makati City, that the Certificate of
the Republic of the Philippines and that he has Repatriation and Oath of Allegiance of petitioner
registered his fact of repatriation in the proper was received by said office and registered, with
civil registry and in the Bureau of Immigration. In the corresponding fee paid, on February 18,
fact, in a letter date 25 June 2001, Commissioner 2004;
ANDREA DOMINGO stated that RESPONDENT is
still a holder of visa under Section 13 (g) of the (4) A letter dated December 17, 1997 from the
Philippine Immigration Act of 1940 as amended, Special Committee on Naturalization to the
with an indefinite authorized stay in the Bureau on Immigration and Deportation that it
Philippines, implying that respondent did not was furnishing said office with the Oath of
register his supposed Certificate of Repatriation Allegiance and Certificate of Repatriation of
with the Bureau of Immigration otherwise his petitioner for the cancellation of petitioner's
Alien Visa would have already been cancelled. registration in said office as an alien, and the
The rule is that in case of doubt concerning the issuance to him of the corresponding
grant of citizenship, such doubt should be Identification Card as Filipino citizen;
resolved in favor of the State and against the
applicant (Cheng vs. Republic, L-16999, 22 June (5) A letter dated December 17, 1997 from the
1965). Special Committee on Naturalization to the Local
Registrar of San Jacinto, Masbate that it was
xxx xxx xxx sending petitioner's Oath of Allegiance and
Certificate of Repatriation for registration in their
Not having been able to prove that he has fully records and for petitioner's reacquisition of his
reacquired his Filipino citizenship after being former Philippine citizenship.
naturalized as a citizen of the United States, it is
clear that respondent is not qualified to be On May 7, 2004, the COMELEC en banc
candidate for the position of Mayor of San Jacinto, promulgated a resolution denying the motion for
Masbate, in the 10 May 2004 National and Local reconsideration, the dispositive portion of which
Elections, pursuant to the aforequoted Sections reads:
39 and 40 of the Local Government Code of 1991.
WHEREFORE, premises considered, the
As a further consequence of his not being a Commission (En Banc) RESOLVED as it hereby
Filipino citizen, respondent has also committed RESOLVES to DENY the Motion for
false representation in his certificate of candidacy Reconsideration for UTTER LACK OF MERIT and
by stating therein that he is a natural-born Filipino AFFIRMS the Resolution of the First Division. 8
citizen, when in fact, he has not yet even

106 | C O N S T I 2 _ A r t i c l e I V _ C I T I Z E N S H I P
The Comelec en banc held, thus: February 18, 2004. This time, she certifies that
Ciceron Perez Altarejos was registered under
The Comelec Rules of Procedure provides that Registry No. 1, Page 19, Book No. 1, Series of
insufficiency of evidence to justify the decision is 2004 and paid under OR nos. 88325/8833256
a ground for a motion for reconsideration (Rule dated February 18, 2004. (page 65 of the
19, Section 1). The evidence referred to in the records). Obviously, he was able to register in the
above provision and to be considered in the proper civil registry only on February 18, 2004.
Motion for Reconsideration are those which were
submitted during the hearing and attached to the The respondent was able to register with the
respective Memoranda of the parties which are Bureau of Immigration only on March 1, 2004 as
already part of the records of the case. In this evidenced by the Bureau of Immigration
regard, the evidence of the respondent were not Identification Certificate attached to the Motion
able to overcome the evidence of the petitioners. as Annex "3."
IcDHaT
This fact confirms the finding of the Commission
(First Division) that at the time respondent filed
his certificate of candidacy he is yet to complete
When the entire records of the case was the requirement under section two (2) of RA
forwarded to the Commission (First Division) the 8171.
respondent's only evidence was his Certificate of
Repatriation dated 17 December 1977 and As a consequence of not being a Filipino citizen,
marked as Annex 1 of his answer. This piece of he has committed false representation in his
evidence was not enough to controvert the certificate of candidacy. Such false representation
evidence of the petitioners which consist of the constitutes a material misrepresentation as it
letter of the then Bureau of Immigration relates to his qualification as a candidate. As such
Commissioner Andrea Domingo dated 25 June the certificate of candidacy may be cancelled on
2001 which stated that as of the even date such ground. (Ycain vs. Caneja, 18 Phil. 778) 9
respondent is a holder of permanent resident visa
(page 15 of the records) and the certification of On May 10, 2004, the election day itself,
Josephine C. Camata dated 28 January 2004 petitioner filed this petition praying that: (1) The
certifying, that the name of the respondent could petition be given due course and a temporary
not be found in the records of repatriation. (page restraining order and/or writ of preliminary
42 of the records) The questioned resolution, is injunction be issued ex parte restraining the
therefore, in order as the evidence submitted by respondents and all persons acting on their
the respondent were insufficient to rebut the behalf, from fully implementing the questioned
evidence of the petitioner. COMELEC Resolutions promulgated on March 22,
2004 and May 7, 2004; (2) a writ of preliminary
Now, the respondent, in his Motion for mandatory injunction be issued ordering the
Reconsideration, attempted to introduce to the COMELEC and all persons acting on its behalf to
record new pieces of evidence, which introduction allow petitioner to run as Mayor of San Jacinto,
is not anymore allowed in a Motion for Masbate in the May 10, 2004 elections, and to
Reconsideration. These are the following a) Annex count and canvass the votes cast in his favor and
"2" — Oath of Allegiance; b) Annex "3" — Bureau to proclaim him as the winning mayor of San
of Immigration Identification Certificate; c) Annex Jacinto, Masbate; and (3) after proper
"4" — Certification of the City Civil Registrar of proceedings, judgment be rendered declaring null
Makati City; d) Annex "5" — Letter addressed to and void and setting aside the COMELEC
the Local Civil Registrar of San Jacinto, Masbate Resolutions promulgated on March 22, 2004 and
by Aurora P. Cortes of Special Committee on May 7, 2004 and other related Orders of the
Naturalization; and e) Annex "6" — Letter COMELEC or its representatives which have the
addressed to the Bureau of Immigration and effect of illegally preventing petitioner from
Deportation by Aurora P. Cortes of Special running as Mayor of San Jacinto, Masbate. cCAIaD
Committee on Naturalization.
In its Comment, 10 the Office of the Solicitor
Assuming that the new evidence of the General stated that, based on the information
respondent are admitted, with more reason relayed to it by the COMELEC, petitioner's name,
should we cancel his certificate of candidacy for as a mayoralty candidate in San Jacinto, Masbate,
his act of [misrepresenting] himself as a Filipino was retained in the list of candidates voted upon
citizen when at the time he filed his certificate of by the electorate in the said municipality. Hence,
candidacy, he has not yet perfected the process the cancellation of petitioner's certificate of
of repatriation. He failed to comply with the candidacy was never implemented. The COMELEC
requirements under Section 2 of [Republic Act also informed the Office of the Solicitor General
No.] 8171 which provides that repatriation shall that petitioner's opponent, Dr. Emilio Aris V.
be effected by taking the necessary oath of Espinosa, was already proclaimed duly elected
allegiance to the Republic of the Philippines and Mayor of San Jacinto, Masbate.
registration in the proper civil registry and in the
Bureau of Immigration. The Office of the Solicitor General contends that
said supervening event has rendered the instant
The certification was issued by the same Ms. petition moot and academic, and it prayed for the
Josephine C. Camata, City Civil Registrar, dated dismissal of the petition.

107 | C O N S T I 2 _ A r t i c l e I V _ C I T I Z E N S H I P
the pertinent alien certificate of registration and
In his Reply, 11 petitioner opposed the dismissal issue the certificate of identification as Filipino
of his petition. He claims that the COMELEC citizen to the repatriated citizen.
resolutions disqualifying him from running as a
mayoralty candidate adversely affected his The law is clear that repatriation is effected "by
candidacy, since his supporters were made to taking the oath of allegiance to the Republic of
believe that his votes would not be counted. the Philippines and registration in the proper civil
Moreover, he stated that said COMELEC registry and in the Bureau of Immigration."
resolutions cast a doubt on his Philippine Hence, in addition to taking the Oath of
citizenship. Allegiance to the Republic of the Philippines, the
registration of the Certificate of Repatriation in
Petitioner points out that he took his Oath of the proper civil registry and the Bureau of
Allegiance to the Republic of the Philippines on Immigration is a prerequisite in effecting the
December 17, 1997. In view thereof, he ran and repatriation of a citizen. aSTcCE
was even elected as Mayor of San Jacinto,
Masbate during the 1998 elections. He argues In this case, petitioner took his Oath of Allegiance
that if there was delay in the registration of his on December 17, 1997, but his Certificate of
Certificate of Repatriation with the Bureau of Repatriation was registered with the Civil Registry
Immigration and with the proper civil registry, the of Makati City only after six years or on February
same was brought about by the inaction on the 18, 2004, and with the Bureau of Immigration on
part of said offices since the records of the March 1, 2004. Petitioner, therefore, completed
Special Committee on Naturalization show that all the requirements of repatriation only after he
his Certificate of Repatriation and Oath of filed his certificate of candidacy for a mayoralty
Allegiance have long been transmitted to said position, but before the elections.
offices.
When does the citizenship qualification of a
Petitioner also asserts that the subsequent candidate for an elective office apply?
registration of his Certificate of Repatriation with
the Bureau of Immigration and with the Civil In Frivaldo v. Commission on Elections, 15 the
Registry of Makati City prior to the May 10, 2004 Court ruled that the citizenship qualification must
elections has the effect of curing the defect, if be construed as "applying to the time of
any, in the reacquisition of his Filipino citizenship proclamation of the elected official and at the
as his repatriation retroacted to the date of his start of his term." The Court, through Justice
application for repatriation as held in Frivaldo v. Artemio V. Panganiban, discussed, thus:
Comelec.
Under Sec. 39 of the Local Government Code,
The pertinent issues raised are the following: (1) "(a)n elective local official must be:
Is the registration of petitioner's repatriation with
the proper civil registry and with the Bureau of * a citizen of the Philippines;
Immigration a prerequisite in effecting
repatriation; and (2) whether or not the COMELEC * a registered voter in the barangay, municipality,
en banc committed grave abuse of discretion city, or province . . . where he intends to be
amounting to excess or lack of jurisdiction in elected;
affirming the Resolution of the COMELEC, First
Division. * a resident therein for at least one (1) year
immediately preceding the day of the election;
As stated by the Office of the Solicitor General,
where the issues have become moot and * able to read and write Filipino or any other local
academic, there is no justiciable controversy, language or dialect."
thereby rendering the resolution of the same of
no practical use or value. 12 Nonetheless, courts * In addition, "candidates for the position of
will decide a question otherwise moot and governor . . . must be at least twenty-three (23)
academic if it is capable of repetition, yet evading years of age on election day."
review. 13
From the above, it will be noted that the law does
First Issue: Is the registration of petitioner's not specify any particular date or time when the
repatriation candidate must possess citizenship, unlike that
with the proper civil registry and with the Bureau for residence (which must consist of at least one
of year's residency immediately preceding the day
Immigration a prerequisite in effecting of election) and age (at least twenty three years
repatriation? of age on election day).
The provision of law applicable in this case is
Section 2 of Republic Act No. 8171, 14 thus:

SEC. 2. Repatriation shall be effected by taking Philippine citizenship is an indispensable


the necessary oath of allegiance to the Republic requirement for holding an elective public office,
of the Philippines and registration in the proper and the purpose of the citizenship qualification is
civil registry and in the Bureau of Immigration. none other than to ensure that no alien, i.e., no
The Bureau of Immigration shall thereupon cancel person owing allegiance to another nation, shall

108 | C O N S T I 2 _ A r t i c l e I V _ C I T I Z E N S H I P
govern our people and our country or a unit of xxx xxx xxx
territory thereof. Now, an official begins to govern
or to discharge his functions only upon his Another argument for retroactivity to the date of
proclamation and on the day the law mandates filing is that it would prevent prejudice to
his term of office to begin. Since Frivaldo re- applicants. If P.D. 725 were not to be given
assumed his citizenship on June 30, 1995 — the retroactive effect, and the Special Committee
very day the term of office of governor (and other decides not to act, i.e., to delay the processing of
elective officials) began — he was therefore applications for any substantial length of time,
already qualified to be proclaimed, to hold such then the former Filipinos who may be stateless,
office and to discharge the functions and as Frivaldo — having already renounced his
responsibilities thereof as of said date. In short, at American citizenship — was, may be prejudiced
that time, he was already qualified to govern his for causes outside their control. This should not
native Sorsogon. This is the liberal interpretation be. In case of doubt in the interpretation or
that should give spirit, life and meaning to our application of laws, it is to be presumed that the
law on qualifications consistent with the purpose law-making body intended right and justice to
for which such law was enacted. . . . Paraphrasing prevail. 17
this Court's ruling in Vasquez v. Giap and Li Seng
Giap & Sons, if the purpose of the citizenship Republic Act No. 8171 18 has impliedly repealed
requirement is to ensure that our people and Presidential Decree No. 725. They cover the same
country do not end up being governed by aliens, subject matter: Providing for the repatriation of
i.e., persons owing allegiance to another nation, Filipino women who have lost their Philippine
that aim or purpose would not be thwarted but citizenship by marriage to aliens and of natural-
instead achieved by construing the citizenship born Filipinos. The Court's ruling in Frivaldo v.
qualification as applying to the time of Commission on Elections that repatriation
proclamation of the elected official and at the retroacts to the date of filing of one's application
start of his term. 16 (Emphasis supplied.) for repatriation subsists for the same reasons
quoted above.
Moreover, in the case of Frivaldo v. Commission
on Elections, the Court ruled that "the Accordingly, petitioner's repatriation retroacted to
repatriation of Frivaldo RETROACTED to the date the date he filed his application in 1997.
of the filing of his application." In said case, the Petitioner was, therefore, qualified to run for a
repatriation of Frivaldo was by virtue of mayoralty position in the government in the May
Presidential Decree No. 725, which took effect on 10, 2004 elections. Apparently, the COMELEC was
June 5, 1975. The Court therein declared that cognizant of this fact since it did not implement
Presidential Decree No. 725 was a curative the assailed Resolutions disqualifying petitioner
statute, which is retroactive in nature. The to run as mayor of San Jacinto, Masbate.
retroactivity of Frivaldo's repatriation to the date
of filing of his application was justified by the Second Issue: Whether or not the COMELEC en
Court, thus: banc
gravely abused its discretion in affirming the
xxx xxx xxx Resolution of the COMELEC, First Division?
The Court cannot fault the COMELEC en banc for
. . . The reason for this is simply that if, as in this affirming the decision of the COMELEC, First
case, it was the intent of the legislative authority Division, considering that petitioner failed to
that the law should apply to past events — i.e., prove before the COMELEC that he had complied
situations and transactions existing even before with the requirements of repatriation. Petitioner
the law came into being — in order to benefit the submitted the necessary documents proving
greatest number of former Filipinos possible compliance with the requirements of repatriation
thereby enabling them to enjoy and exercise the only during his motion for reconsideration, when
constitutionally guaranteed right of citizenship, the COMELEC en banc could no longer consider
and such legislative intention is to be given the said evidence. As the COMELEC en banc correctly
fullest effect and expression, then there is all the stated:
more reason to have the law apply in a
retroactive or retrospective manner to situations, The Comelec Rules of Procedure provides that
events and transactions subsequent to the insufficiency of evidence to justify the decision is
passage of such law. That is, the repatriation a ground for a motion for reconsideration (Rule
granted to Frivaldo . . . can and should be made 19, Section 1). The evidence referred to in the
to take effect as of date of his application. As above provision and to be considered in the
earlier mentioned, there is nothing in the law that Motion for Reconsideration are those which were
would bar this or would show a contrary intention submitted during the hearing and attached to the
on the part of the legislative authority; and there respective Memoranda of the parties which are
is no showing that damage or prejudice to already part of the records of the case. In this
anyone, or anything unjust or injurious would regard, the evidence of the respondent were not
result from giving retroactivity to his repatriation. able to overcome the evidence of the petitioners.
Neither has Lee shown that there will result the 19
impairment of any contractual obligation,
disturbance of any vested right or breach of some It is, therefore, incumbent upon candidates for an
constitutional guaranty. IHCacT elective office, who are repatriated citizens, to be
ready with sufficient evidence of their repatriation

109 | C O N S T I 2 _ A r t i c l e I V _ C I T I Z E N S H I P
in case their Filipino citizenship is questioned to resident or an immigrant 4 of the United States of
prevent a repetition of this case. America; 5 and (5) that she is a Filipino citizen
when she is, in fact, an American citizen. 6
WHEREFORE, the petition seeking the nullification TaSEHC
of the Resolution of the COMELEC en banc of May
7, 2004, affirming the Resolution of its First In her Answer, petitioner countered that, while
Division dated March 22, 2004, is hereby DENIED. she is publicly known to be the wife of
No costs. Congressman Herminaldo I. Mandanas
(Congressman Mandanas), there is no valid and
SO ORDERED. binding marriage between them. According to
petitioner, although her marriage with
Davide, Jr., C .J ., Panganiban, Quisumbing, Congressman Mandanas was solemnized in a
Ynares-Santiago, Sandoval-Gutierrez, Carpio, religious rite, it did not comply with certain formal
Austria-Martinez, Carpio Morales, Callejo, Sr., requirements prescribed by the Family Code,
Chico-Nazario and Garcia, JJ ., concur. rendering it void ab initio. 7 Consequently,
petitioner argues that as she is not duty-bound to
Puno and Tinga, JJ ., are on official leave. live with Congressman Mandanas, then his
residence cannot be attributed to her. 8 As to her
Corona, J ., is on leave. date of birth, the Certificate of Live Birth issued
by the National Statistics Office shows that it was
(Reyes v. COMELEC, G.R. No. 207264, June on 3 July 1964. 9 Lastly, petitioner notes that the
25, 2013) allegation that she is a permanent resident
and/or a citizen of the United States of America is
EN BANC not supported by evidence. 10

[G.R. No. 207264. June 25, 2013.] During the course of the proceedings, on 8
February 2013, respondent filed a "Manifestation
REGINA ONGSIAKO REYES, petitioner, vs. with Motion to Admit Newly Discovered Evidence
COMMISSION ON ELECTIONS and JOSEPH and Amended List of Exhibits" 11 consisting of,
SOCORRO B. TAN, respondents. among others: (1) a copy of an article published
on the internet on 8 January 2013 entitled
RESOLUTION "Seeking and Finding the Truth about Regina O.
Reyes" with an Affidavit of Identification and
PEREZ, J p: Authenticity of Document executed by its author
Eliseo J. Obligacion, which provides a database
Before the Court is a Petition for Certiorari with record of the Bureau of Immigration indicating
Prayer for Temporary Restraining Order and/or that petitioner is an American citizen and a holder
Preliminary Injunction and/or Status Quo Ante of a U.S. passport; (2) a Certification of Travel
Order dated 7 June 2013 filed by petitioner Records of petitioner, issued by Simeon Sanchez,
Regina Ongsiako Reyes, assailing the Resolutions Acting Chief, Verification and Certification Unit of
dated 27 March 2013 and 14 May 2013 issued by the Bureau of Immigration which indicates that
public respondent Commission on Elections petitioner used a U.S. Passport in her various
(COMELEC) in SPA No. 13-053. The assailed travels abroad.
Resolutions ordered the cancellation of the
Certificate of Candidacy of petitioner for the On 27 March 2013, the COMELEC First Division
position of Representative of the lone district of issued a Resolution 12 cancelling petitioner's
Marinduque. COC, to wit:

On 31 October 2012, respondent Joseph Socorro WHEREFORE, in view of the foregoing, the instant
Tan, a registered voter and resident of the Petition is GRANTED. Accordingly, the Certificate
Municipality of Torrijos, Marinduque, filed before of Candidacy of respondent REGINA ONGSIAKO
the COMELEC an Amended Petition to Deny Due REYES is hereby CANCELLED. EHaCTA
Course or to Cancel the Certificate of Candidacy
(COC) of petitioner on the ground that it The COMELEC First Division found that, contrary
contained material misrepresentations, to the declarations that she made in her COC,
specifically: (1) that she is single when she is petitioner is not a citizen of the Philippines
married to Congressman Herminaldo I. Mandanas because of her failure to comply with the
of Batangas; 1 (2) that she is a resident of Brgy. requirements of Republic Act (R.A.) No. 9225 or
Lupac, Boac, Marinduque when she is a resident the Citizenship Retention and Re-acquisition Act
of Bauan, Batangas which is the residence of her of 2003, namely: (1) to take an oath of allegiance
husband, and at the same time, when she is also to the Republic of the Philippines; and (2) to make
a resident of 135 J.P. Rizal, Brgy. Milagrosa, a personal and sworn renunciation of her
Quezon City as admitted in the Directory of American citizenship before any public officer
Congressional Spouses of the House of authorized to administer an oath. In addition, the
Representatives; 2 (3) that her date of birth is 3 COMELEC First Division ruled that she did not
July 1964 when other documents show that her have the one-year residency requirement under
birthdate is either 8 July 1959 or 3 July 1960; 3 Section 6, Article VI of the 1987 Constitution. 13
(4) that she is not a permanent resident of Thus, she is ineligible to run for the position of
another country when she is a permanent

110 | C O N S T I 2 _ A r t i c l e I V _ C I T I Z E N S H I P
Representative for the lone district of petitioner the opportunity to question and
Marinduque. present controverting evidence, in violation of
Petitioner's right to due process of law.

Not agreeing with the Resolution of the COMELEC 33)Whether or not Respondent Comelec
First Division, petitioner filed a Motion for committed grave abuse of discretion amounting
Reconsideration 14 on 8 April 2013 claiming that to lack or excess of jurisdiction when it declared
she is a natural-born Filipino citizen and that she that Petitioner is not a Filipino citizen and did not
has not lost such status by simply obtaining and meet the residency requirement for the position
using an American passport. Additionally, of Member of the House of Representatives.
petitioner surmised that the COMELEC First
Division relied on the fact of her marriage to an 34)Whether or not Respondent Commission on
American citizen in concluding that she is a Elections committed grave abuse of discretion
naturalized American citizen. Petitioner averred, amounting to lack or excess of jurisdiction when,
however, that such marriage only resulted into by enforcing the provisions of Republic Act No.
dual citizenship, thus there is no need for her to 9225, it imposed additional qualifications to the
fulfill the twin requirements under R.A. No. 9225. qualifications of a Member of the House of
Still, petitioner attached an Affidavit of Representatives as enumerated in Section 6 of
Renunciation of Foreign Citizenship sworn to Article VI of the 1987 Constitution of the
before a Notary Public on 24 September 2012. As Philippines.
to her alleged lack of the one-year residency
requirement prescribed by the Constitution, she The petition must fail.
averred that, as she never became a naturalized
citizen, she never lost her domicile of origin, At the outset, it is observed that the issue of
which is Boac, Marinduque. jurisdiction of respondent COMELEC vis-a-vis that
of House of Representatives Electoral Tribunal
On 14 May 2013, the COMELEC En Banc, (HRET) appears to be a non-issue. Petitioner is
promulgated a Resolution 15 denying petitioner's taking an inconsistent, if not confusing, stance for
Motion for Reconsideration for lack of merit. while she seeks remedy before this Court, she is
asserting that it is the HRET which has jurisdiction
Four days thereafter or on 18 May 2013, over her. Thus, she posits that the issue on her
petitioner was proclaimed winner of the 13 May eligibility and qualifications to be a Member of
2013 Elections. the House of Representatives is best discussed in
another tribunal of competent jurisdiction. It
On 5 June 2013, the COMELEC En Banc issued a appears then that petitioner's recourse to this
Certificate of Finality 16 declaring the 14 May Court was made only in an attempt to enjoin the
2013 Resolution of the COMELEC En Banc final COMELEC from implementing its final and
and executory, considering that more than executory judgment in SPA No. 13-053.
twenty-one (21) days have elapsed from the date
of promulgation with no order issued by this Nevertheless, we pay due regard to the petition,
Court restraining its execution. 17 and consider each of the issues raised by
petitioner. The need to do so, and at once, was
On same day, petitioner took her oath of office 18 highlighted during the discussion En Banc on 25
before Feliciano R. Belmonte Jr., Speaker of the June 2013 where and when it was emphasized
House of Representatives. that the term of office of the Members of the
House of Representatives begins on the thirtieth
Petitioner has yet to assume office, the term of day of June next following their election.
which officially starts at noon of 30 June 2013.
According to petitioner, the COMELEC was ousted
In the present Petition for Certiorari with Prayer of its jurisdiction when she was duly proclaimed
for Temporary Restraining Order and/or 20 because pursuant to Section 17, Article VI of
Preliminary Injunction and/or Status Quo Ante the 1987 Constitution, the HRET has the exclusive
Order, petitioner raises the following issues: 19 jurisdiction to be the "sole judge of all contests
relating to the election, returns and
31)Whether or not Respondent Comelec is qualifications" of the Members of the House of
without jurisdiction over Petitioner who is a duly Representatives.
proclaimed winner and who has already taken her
oath of office for the position of Member of the Contrary to petitioner's claim, however, the
House of Representatives for the lone COMELEC retains jurisdiction for the following
congressional district of Marinduque. aEIADT reasons:

32)Whether or not Respondent Comelec First, the HRET does not acquire jurisdiction over
committed grave abuse of discretion amounting the issue of petitioner's qualifications, as well as
to lack or excess of jurisdiction when it took over the assailed COMELEC Resolutions, unless a
cognizance of Respondent Tan's alleged "newly- petition is duly filed with said tribunal. Petitioner
discovered evidence" without the same having has not averred that she has filed such action.
been testified on and offered and admitted in HcTIDC
evidence which became the basis for its
Resolution of the case without giving the

111 | C O N S T I 2 _ A r t i c l e I V _ C I T I Z E N S H I P
Second, the jurisdiction of the HRET begins only
after the candidate is considered a Member of the From the foregoing, it is then clear that to be
House of Representatives, as stated in Section considered a Member of the House of
17, Article VI of the 1987 Constitution: Representatives, there must be a concurrence of
the following requisites: (1) a valid proclamation,
Section 17.The Senate and the House of (2) a proper oath, and (3) assumption of office.
Representatives shall each have an Electoral
Tribunal which shall be the sole judge of all Indeed, in some cases, this Court has made the
contests relating to the election, returns, and pronouncement that once a proclamation has
qualifications of their respective Members. . . . been made, COMELEC's jurisdiction is already lost
and, thus, its jurisdiction over contests relating to
As held in Marcos v. COMELEC, 21 the HRET does elections, returns, and qualifications ends, and
not have jurisdiction over a candidate who is not the HRET's own jurisdiction begins. However, it
a member of the House of Representatives, to must be noted that in these cases, the doctrinal
wit: pronouncement was made in the context of a
proclaimed candidate who had not only taken an
As to the House of Representatives Electoral oath of office, but who had also assumed office.
Tribunal's supposed assumption of jurisdiction DIETcC
over the issue of petitioner's qualifications after
the May 8, 1995 elections, suffice it to say that For instance, in the case of Dimaporo v.
HRET's jurisdiction as the sole judge of all COMELEC, 27 the Court upheld the jurisdiction of
contests relating to the elections, returns and the HRET against that of the COMELEC only after
qualifications of members of Congress begins the candidate had been proclaimed, taken his
only after a candidate has become a member of oath of office before the Speaker of the House,
the House of Representatives. Petitioner not and assumed the duties of a Congressman on 26
being a member of the House of Representatives, September 2007, or after the start of his term on
it is obvious that the HRET at this point has no 30 June 2007, to wit:
jurisdiction over the question. (Emphasis
supplied.) On October 8, 2007, private respondent Belmonte
filed his comment in which he brought to Our
The next inquiry, then, is when is a candidate attention that on September 26, 2007, even
considered a Member of the House of before the issuance of the status quo ante order
Representatives? of the Court, he had already been proclaimed by
the PBOC as the duly elected Member of the
In Vinzons-Chato v. COMELEC, 22 citing Aggabao House of Representatives of the First
v. COMELEC 23 and Guerrero v. COMELEC, 24 the Congressional District of Lanao del Norte. On that
Court ruled that: very same day, he had taken his oath before
Speaker of the House Jose de Venecia, Jr. and
The Court has invariably held that once a winning assumed his duties accordingly.
candidate has been proclaimed, taken his oath,
and assumed office as a Member of the House of In light of this development, jurisdiction over this
Representatives, the COMELEC's jurisdiction over case has already been transferred to the House of
election contests relating to his election, returns, Representatives Electoral Tribunal (HRET).
and qualifications ends, and the HRET's own (Emphasis supplied.)
jurisdiction begins. (Emphasis supplied.) TaSEHD
Apparently, the earlier cases were decided after
This pronouncement was reiterated in the case of the questioned candidate had already assumed
Limkaichong v. COMELEC, 25 wherein the Court, office, and hence, was already considered a
referring to the jurisdiction of the COMELEC vis-a- Member of the House of Representatives, unlike
vis the HRET, held that: in the present case.

The Court has invariably held that once a winning Here, the petitioner cannot be considered a
candidate has been proclaimed, taken his oath, Member of the House of Representatives
and assumed office as a Member of the House of because, primarily, she has not yet assumed
Representatives, the COMELEC's jurisdiction over office. To repeat what has earlier been said, the
election contests relating to his election, returns, term of office of a Member of the House of
and qualifications ends, and the HRET's own Representatives begins only "at noon on the
jurisdiction begins. (Emphasis supplied.) thirtieth day of June next following their election."
28 Thus, until such time, the COMELEC retains
This was again affirmed in Gonzalez v. COMELEC, jurisdiction.
26 to wit:
In her attempt to comply with the second
After proclamation, taking of oath and requirement, petitioner attached a purported
assumption of office by Gonzalez, jurisdiction Oath of Office taken before Hon. Feliciano
over the matter of his qualifications, as well as Belmonte Jr. on 5 June 2013. However, this is not
questions regarding the conduct of election and the oath of office which confers membership to
contested returns — were transferred to the HRET the House of Representatives. ATHCDa
as the constitutional body created to pass upon
the same. (Emphasis supplied.)

112 | C O N S T I 2 _ A r t i c l e I V _ C I T I Z E N S H I P
Section 6, Rule II (Membership) of the Rules of the As to the issue of whether petitioner failed to
House of Representatives provides: prove her Filipino citizenship, as well as her one-
year residency in Marinduque, suffice it to say
Section 6.Oath or Affirmation of Members. — that the COMELEC committed no grave abuse of
Members shall take their oath or affirmation discretion in finding her ineligible for the position
either collectively or individually before the of Member of the House of Representatives.
Speaker in open session.
Petitioner alleges that the COMELEC gravely
Consequently, before there is a valid or official abused its discretion when it took cognizance of
taking of the oath it must be made (1) before the "newly-discovered evidence" without the same
Speaker of the House of Representatives, and (2) having been testified on and offered and
in open session. Here, although she made the admitted in evidence. She assails the admission
oath before Speaker Belmonte, there is no of the blog article of Eli Obligacion as hearsay
indication that it was made during plenary or in and the photocopy of the Certification from the
open session and, thus, it remains unclear Bureau of Immigration. She likewise contends
whether the required oath of office was indeed that there was a violation of her right to due
complied with. process of law because she was not given the
opportunity to question and present controverting
More importantly, we cannot disregard a fact evidence.
basic in this controversy — that before the
proclamation of petitioner on 18 May 2013, the Her contentions are incorrect.
COMELEC En Banc had already finally disposed of
the issue of petitioner's lack of Filipino citizenship It must be emphasized that the COMELEC is not
and residency via its Resolution dated 14 May bound to strictly adhere to the technical rules of
2013. After 14 May 2013, there was, before the procedure in the presentation of evidence. Under
COMELEC, no longer any pending case on Section 2 of Rule I, the COMELEC Rules of
petitioner's qualifications to run for the position of Procedure "shall be liberally construed in
Member of the House of Representative. We will order . . . to achieve just, expeditious and
inexcusably disregard this fact if we accept the inexpensive determination and disposition of
argument of the petitioner that the COMELEC was every action and proceeding brought before the
ousted of jurisdiction when she was proclaimed, Commission." In view of the fact that the
which was four days after the COMELEC En Banc proceedings in a petition to deny due course or to
decision. The Board of Canvasser which cancel certificate of candidacy are summary in
proclaimed petitioner cannot by such act be nature, then the "newly discovered evidence" was
allowed to render nugatory a decision of the properly admitted by respondent COMELEC.
COMELEC En Banc which affirmed a decision of
the COMELEC First Division. Furthermore, there was no denial of due process
in the case at bar as petitioner was given every
Indeed, the assailed Resolution of the COMELEC opportunity to argue her case before the
First Division which was promulgated on 27 March COMELEC. From 10 October 2012 when Tan's
2013, and the assailed Resolution of the petition was filed up to 27 March 2013 when the
COMELEC En Banc which was promulgated on 14 First Division rendered its resolution, petitioner
May 2013, became final and executory on 19 May had a period of five (5) months to adduce
2013 based on Section 3, Rule 37 of the evidence. Unfortunately, she did not avail herself
COMELEC Rules of Procedure which provides: of the opportunity given her.
SECIcT
Also, in administrative proceedings, procedural
Section 3.Decisions Final after five days. — due process only requires that the party be given
Decisions in pre-proclamation cases and petitions the opportunity or right to be heard. As held in
to deny due course to or cancel certificates of the case of Sahali v. COMELEC: 31 IDTHcA
candidacy, to declare nuisance candidate or to
disqualify a candidate, and to postpone or The petitioners should be reminded that due
suspend elections shall become final and process does not necessarily mean or require a
executory after the lapse of five (5) days from hearing, but simply an opportunity or right to be
their promulgation unless restrained by the heard. One may be heard, not solely by verbal
Supreme Court. presentation but also, and perhaps many times
more creditably and predictable than oral
To prevent the assailed Resolution dated 14 May argument, through pleadings. In administrative
2013 from becoming final and executory, proceedings moreover, technical rules of
petitioner should have availed herself of Section procedure and evidence are not strictly applied;
1, Rule 37 29 of the COMELEC Rules of Procedure administrative process cannot be fully equated
or Rule 64 30 of the Rules of Court by filing a with due process in its strict judicial sense.
petition before this Court within the 5-day period, Indeed, deprivation of due process cannot be
but she failed to do so. She would file the present successfully invoked where a party was given the
last hour petition on 10 June 2013. Hence, on 5 chance to be heard on his motion for
June 2013, respondent COMELEC rightly issued a reconsideration. (Emphasis supplied)
Certificate of Finality.

113 | C O N S T I 2 _ A r t i c l e I V _ C I T I Z E N S H I P
As to the ruling that petitioner is ineligible to run Citizenship dated 24 September 2012. 34
for office on the ground of citizenship, the Petitioner explains that she attached said
COMELEC First Division, discoursed as follows: Affidavit "if only to show her desire and zeal to
serve the people and to comply with rules, even
". . . for respondent to reacquire her Filipino as a superfluity." 35 We cannot, however,
citizenship and become eligible for public office, subscribe to petitioner's explanation. If petitioner
the law requires that she must have executed said Affidavit "if only to comply with the
accomplished the following acts: (1) take the oath rules," then it is an admission that R.A. No. 9225
of allegiance to the Republic of the Philippines applies to her. Petitioner cannot claim that she
before the Consul-General of the Philippine executed it to address the observations by the
Consulate in the USA; and (2) make a personal COMELEC as the assailed Resolutions were
and sworn renunciation of her American promulgated only in 2013, while the Affidavit was
citizenship before any public officer authorized to executed in September 2012. HDTcEI
administer an oath.
Moreover, in the present petition, petitioner
In the case at bar, there is no showing that added a footnote to her oath of office as
respondent complied with the aforesaid Provincial Administrator, to this effect: "This does
requirements. Early on in the proceeding, not mean that Petitioner did not, prior to her
respondent hammered on petitioner's lack of taking her oath of office as Provincial
proof regarding her American citizenship, Administrator, take her oath of allegiance for
contending that it is petitioner's burden to purposes of re-acquisition of natural-born Filipino
present a case. She, however, specifically denied status, which she reserves to present in the
that she has become either a permanent resident proper proceeding. The reference to the taking of
or naturalized citizen of the USA. oath of office is in order to make reference to
what is already part of the records and evidence
Due to petitioner's submission of newly- in the present case and to avoid injecting into the
discovered evidence thru a Manifestation dated records evidence on matters of fact that was not
February 7, 2013, however, establishing the fact previously passed upon by Respondent
that respondent is a holder of an American COMELEC." 36 This statement raises a lot of
passport which she continues to use until June 30, questions — Did petitioner execute an oath of
2012, petitioner was able to substantiate his allegiance for re-acquisition of natural-born
allegations. The burden now shifts to respondent Filipino status? If she did, why did she not present
to present substantial evidence to prove it at the earliest opportunity before the
otherwise. This, the respondent utterly failed to COMELEC? And is this an admission that she has
do, leading to the conclusion inevitable that indeed lost her natural-born Filipino status?
respondent falsely misrepresented in her COC
that she is a natural-born Filipino citizen. Unless To cover-up her apparent lack of an oath of
and until she can establish that she had availed allegiance as required by R.A. No. 9225,
of the privileges of RA 9225 by becoming a dual petitioner contends that, since she took her oath
Filipino-American citizen, and thereafter, made a of allegiance in connection with her appointment
valid sworn renunciation of her American as Provincial Administrator of Marinduque, she is
citizenship, she remains to be an American deemed to have reacquired her status as a
citizen and is, therefore, ineligible to run for and natural-born Filipino citizen.
hold any elective public office in the Philippines."
32 (Emphasis supplied.) cAHIaE This contention is misplaced. For one, this issue is
being presented for the first time before this
Let us look into the events that led to this Court, as it was never raised before the
petition: In moving for the cancellation of COMELEC. For another, said oath of allegiance
petitioner's COC, respondent submitted records of cannot be considered compliance with Sec. 3 of
the Bureau of Immigration showing that petitioner R.A. No. 9225 as certain requirements have to be
is a holder of a US passport, and that her status is met as prescribed by Memorandum Circular No.
that of a "balikbayan." At this point, the burden of AFF-04-01, otherwise known as the Rules
proof shifted to petitioner, imposing upon her the Governing Philippine Citizenship under R.A. No.
duty to prove that she is a natural-born Filipino 9225 and Memorandum Circular No. AFF-05-002
citizen and has not lost the same, or that she has (Revised Rules) and Administrative Order No. 91,
re-acquired such status in accordance with the Series of 2004 issued by the Bureau of
provisions of R.A. No. 9225. Aside from the bare Immigration. Thus, petitioner's oath of office as
allegation that she is a natural-born citizen, Provincial Administrator cannot be considered as
however, petitioner submitted no proof to support the oath of allegiance in compliance with R.A. No.
such contention. Neither did she submit any proof 9225.
as to the inapplicability of R.A. No. 9225 to her.
These circumstances, taken together, show that a
Notably, in her Motion for Reconsideration before doubt was clearly cast on petitioner's citizenship.
the COMELEC En Banc, petitioner admitted that Petitioner, however, failed to clear such doubt.
she is a holder of a US passport, but she averred
that she is only a dual Filipino-American citizen, As to the issue of residency, proceeding from the
thus the requirements of R.A. No. 9225 do not finding that petitioner has lost her natural-born
apply to her. 33 Still, attached to the said motion status, we quote with approval the ruling of the
is an Affidavit of Renunciation of Foreign

114 | C O N S T I 2 _ A r t i c l e I V _ C I T I Z E N S H I P
COMELEC First Division that petitioner cannot be has a specific meaning. It is the arbitrary or
considered a resident of Marinduque: despotic exercise of power due to passion,
prejudice or personal hostility; or the whimsical,
"Thus, a Filipino citizen who becomes naturalized arbitrary, or capricious exercise of power that
elsewhere effectively abandons his domicile of amounts to an evasion or refusal to perform a
origin. Upon re-acquisition of Filipino citizenship positive duty enjoined by law or to act at all in
pursuant to RA 9225, he must still show that he contemplation of law. For an act to be struck
chose to establish his domicile in the Philippines down as having been done with grave abuse of
through positive acts, and the period of his discretion, the abuse of discretion must be patent
residency shall be counted from the time he and gross. (Emphasis supplied.) SDIaCT
made it his domicile of choice.
Here, this Court finds that petitioner failed to
In this case, there is no showing whatsoever that adequately and substantially show that grave
[petitioner] had already re-acquired her Filipino abuse of discretion exists.
citizenship pursuant to RA 9225 so as to conclude
that she has regained her domicile in the Lastly, anent the proposition of petitioner that the
Philippines. There being no proof that [petitioner] act of the COMELEC in enforcing the provisions of
had renounced her American citizenship, it R.A. No. 9225, insofar as it adds to the
follows that she has not abandoned her domicile qualifications of Members of the House of
of choice in the USA. Representatives other than those enumerated in
the Constitution, is unconstitutional, We find the
The only proof presented by [petitioner] to show same meritless.
that she has met the one-year residency
requirement of the law and never abandoned her The COMELEC did not impose additional
domicile of origin in Boac, Marinduque is her qualifications on candidates for the House of
claim that she served as Provincial Administrator Representatives who have acquired foreign
of the province from January 18, 2011 to July 13, citizenship. It merely applied the qualifications
2011. But such fact alone is not sufficient to prescribed by Section 6, Article VI of the 1987
prove her one-year residency. For, [petitioner] has Constitution that the candidate must be a
never regained her domicile in Marinduque as she natural-born citizen of the Philippines and must
remains to be an American citizen. No amount of have one-year residency prior to the date of
her stay in the said locality can substitute the fact elections. Such being the case, the COMELEC did
that she has not abandoned her domicile of not err when it inquired into the compliance by
choice in the USA." 37 (Emphasis supplied.) petitioner of Sections 3 and 5 of R.A. No. 9225 to
ESAHca determine if she reacquired her status as a
natural-born Filipino citizen. It simply applied the
All in all, considering that the petition for denial constitutional provision and nothing more.
and cancellation of the COC is summary in
nature, the COMELEC is given much discretion in IN VIEW OF THE FOREGOING, the instant petition
the evaluation and admission of evidence is DISMISSED, finding no grave abuse of
pursuant to its principal objective of determining discretion on the part of the Commission on
of whether or not the COC should be cancelled. Elections. The 14 May 2013 Resolution of the
We held in Mastura v. COMELEC: 38 COMELEC En Banc affirming the 27 March 2013
Resolution of the COMELEC First Division is
The rule that factual findings of administrative upheld.
bodies will not be disturbed by courts of justice
except when there is absolutely no evidence or SO ORDERED. SHECcD
no substantial evidence in support of such Sereno, C.J., Leonardo-de Castro, Bersamin, Del
findings should be applied with greater force Castillo, Abad and Reyes, JJ., concur.
when it concerns the COMELEC, as the framers of Carpio, Villarama, Jr. and Leonen, JJ., join the
the Constitution intended to place the COMELEC dissent of J. Brion.
— created and explicitly made independent by Velasco, Jr. and Mendoza, JJ., took no part.
the Constitution itself — on a level higher than Brion, J., see dissent.
statutory administrative organs. The COMELEC Peralta, J., is on official leave.
has broad powers to ascertain the true results of Perlas-Bernabe, J., took no part due to voluntary
the election by means available to it. For the inhibition.
attainment of that end, it is not strictly bound by
the rules of evidence. (Republic v. Li Ching Chung, G.R. No.
197450, March 20, 2013)
Time and again, We emphasize that the "grave
abuse of discretion" which warrants this Court's THIRD DIVISION
exercise of certiorari jurisdiction has a well- [G.R. No. 197450. March 20, 2013.]
defined meaning. Guidance is found in Beluso v. REPUBLIC OF THE PHILIPPINES, petitioner, vs. LI
Commission on Elections 39 where the Court CHING CHUNG, a.k.a. BERNABE LUNA LI, a.k.a.
held: STEPHEN LEE KENG, respondent.

. . . A petition for certiorari will prosper only if DECISION


grave abuse of discretion is alleged and proved to MENDOZA, J p:
exist. "Grave abuse of discretion," under Rule 65,

115 | C O N S T I 2 _ A r t i c l e I V _ C I T I Z E N S H I P
This Petition for Review on Certiorari 1 under Rule newspaper of general circulation, on May 30,
45 of the 1997 Rules of Civil Procedure filed by 2008, 26 June 6, 2008 27 and June 13, 2008. 28
the Republic of the Philippines, represented by
the Office of the Solicitor General (OSG), Thereafter, respondent filed the Motion for Early
challenges the June 30, 2011 Decision 2 of the Setting 29 praying that the hearing be moved
Court of Appeals (CA) in CA-G.R. CV No. 93374, from April 3, 2009 to July 31, 2008 so he could
which affirmed the June 3, 2009 Decision 3 of the acquire real estate properties. The OSG filed its
Regional Trial Court, Branch 49, Manila (RTC), Opposition, 30 dated August 6, 2008, arguing
granting the petition for naturalization of that the said motion for early setting was a "clear
respondent Li Ching Chung (respondent). ATaDHC violation of Section 1, RA 530, which provides
that hearing on the petition should be held not
On August 22, 2007, respondent, otherwise earlier than six (6) months from the date of last
known as Bernabe Luna Li or Stephen Lee Keng, a publication of the notice." 31 The opposition was
Chinese national, filed his Declaration of Intention already late as the RTC, in its July 31, 2008 Order,
to Become a Citizen of the Philippines before the 32 denied respondent's motion and decreed that
OSG. 4 since the last publication in the newspaper of
general circulation was on June 13, 2008, the
On March 12, 2008 or almost seven months after earliest setting could only be scheduled six (6)
filing his declaration of intention, respondent filed months later or on December 15, 2008.
his Petition for Naturalization before the RTC,
docketed as Civil Case No. 08-118905. 5 On April On December 15, 2008, the OSG reiterated, in
5, 2008, respondent filed his Amended Petition open court, its opposition to the early setting of
for Naturalization, 6 wherein he alleged that he the hearing and other grounds that would merit
was born on November 29, 1963 in Fujian the dismissal of the petition. Accordingly, the RTC
Province, People's Republic of China, which ordered the suspension of the judicial
granted the same privilege of naturalization to proceedings until all the requirements of the
Filipinos; that he came to the Philippines on statute of limitation would be completed. 33
March 15, 1988 via Philippine Airlines Flight PR
311 landing at the Ninoy Aquino International The OSG filed a motion to dismiss, 34 but the RTC
Airport; that on November 19, 1989, he married denied the same in its Order, 35 dated March 10,
Cindy Sze Mei Ngar, a British national, with whom 2009, and reinstated the original hearing date on
he had four (4) children, all born in Manila; that April 3, 2009, as previously indicated in the
he had been continuously and permanently notice.
residing in the country since his arrival and is
currently a resident of Manila with prior residence Thereafter, respondent testified and presented
in Malabon; that he could speak and write in two character witnesses, Emelita V. Roleda and
English and Tagalog; that he was entitled to the Gaudencio Abalayan Manimtim, who personally
benefit of Section 3 of Commonwealth Act (CA) knew him since 1984 and 1998, respectively, to
No. 473 reducing to five (5) years the vouch that he was a person of good moral
requirement under Section 2 of ten years of character and had conducted himself in a proper
continuous residence, because he knew English and irreproachable manner during his period of
and Filipino having obtained his education from residency in the country.
St. Stephen's High School of Manila; and that he
had successfully established a trading general On June 3, 2009, the RTC granted respondent's
merchandise business operating under the name application for naturalization as a Filipino citizen.
of "VS Marketing Corporation." 7 As an 36 The decretal portion reads:
entrepreneur, he derives income more than
sufficient to be able to buy a condominium unit WHEREFORE, petitioner LI CHING CHUNG a.k.a.
and vehicles, send his children to private schools BERNABE LUNA LI a.k.a STEPHEN LEE KENG is
and adequately provide for his family. 8 hereby declared a Filipino citizen by
naturalization and admitted as such.
In support of his application, he attached his
barangay certificate, 9 police clearance, 10 alien However, pursuant to Section 1 of Republic Act
certification of registration, 11 immigration No. 530, this Decision shall not become executory
certificate of residence, 12 marriage contract, 13 until after two (2) years from its promulgation and
authenticated birth certificates of his children, 14 after the Court, on proper hearing, with the
affidavits of his character witnesses, 15 passport, attendance of the Solicitor General or his
16 2006 annual income tax return, 17 declaration representative, is satisfied, and so finds, that
of intention to become a citizen of the Philippines during the intervening time the applicant has: (1)
18 and a certification 19 from the Bureau of not left the Philippines; (2) has dedicated himself
Immigration with a list of his travel records from continuously to a lawful calling or profession; (3)
January 30, 1994. 20 has not been convicted of any offense or violation
of Government promulgated rules; (4) or
Consequently, the petition was set for initial committed any act prejudicial to the interest of
hearing on April 3, 2009 and its notice 21 was the nation or contrary to any Government
posted in a conspicuous place at the Manila City announced policies. SDHAcI
Hall and was published in the Official Gazette on
June 30, 2008, 22 July 7, 2008 23 and July 14, As soon as this decision shall have become
2008, 24 and in the Manila Times, 25 a executory, as provided under Section 1 of

116 | C O N S T I 2 _ A r t i c l e I V _ C I T I Z E N S H I P
Republic Act No. 530, the Clerk of Court of this Intention, was attached to the petition; and 3)
Branch is hereby directed to issue to the respondent's failure to comply with the
Petitioner a Naturalization Certificate, after the publication and posting requirements set under
Petitioner shall have subscribed to an Oath, in CA 473." 45 In particular, the OSG points out that
accordance with Section 12 of Commonwealth the publication and posting requirements were
Act No. 472, as amended. not strictly followed, specifically citing that: "(a)
the hearing of the petition on 15 December 2008
The Local Civil Registrar of the City of Manila is, was set ahead of the scheduled date of hearing
likewise directed to register the Naturalization on 3 April 2009; (b) the order moving the date of
Certificate in the proper Civil Registry. hearing (Order dated 31 July 2008) was not
published; and, (c) the petition was heard within
SO ORDERED. 37 six (6) months (15 December 2008) from the last
publication (on 14 July 2008)." 46
The OSG appealed the RTC decision to the CA. 38
The petition is meritorious.
On June 30, 2011, the CA affirmed the RTC
decision. 39 The CA held that although the Section 5 of CA No. 473, 47 as amended, 48
petition for naturalization was filed less than one expressly states:
(1) year from the time of the declaration of intent
before the OSG, this defect was not fatal. Section 5. Declaration of intention. — One year
Moreover, contrary to the allegation of the OSG prior to the filing of his petition for admission to
that respondent did not present his Certificate of Philippine citizenship, the applicant for Philippine
Arrival, the fact of his arrival could be easily citizenship shall file with the Bureau of Justice
confirmed from the Certification, dated August (now Office of the Solicitor General) a declaration
21, 2007, issued by the Bureau of Immigration, under oath that it is bona fide his intention to
and from the stamp in the passport of respondent become a citizen of the Philippines. Such
indicating his arrival on January 26, 1981. 40 The declaration shall set forth name, age, occupation,
CA further stated that "the Republic participated personal description, place of birth, last foreign
in every stage of the proceedings below. It was residence and allegiance, the date of arrival, the
accorded due process which it vigorously name of the vessel or aircraft, if any, in which he
exercised from beginning to end. Whatever came to the Philippines, and the place of
procedural defects, if at all they existed, did not residence in the Philippines at the time of making
taint the proceedings, let alone the Republic's the declaration. No declaration shall be valid until
meaningful exercise of its right to due process." lawful entry for permanent residence has been
41 established and a certificate showing the date,
place, and manner of his arrival has been issued.
Moreover, the CA noted that the OSG did not in The declarant must also state that he has
any way question respondent's qualifications and enrolled his minor children, if any, in any of the
his lack of disqualifications to be admitted as public schools or private schools recognized by
citizen of this country. Indeed, the CA was the Office of Private Education of the Philippines,
convinced that respondent was truly deserving of where Philippine history, government, and civics
this privilege. 42 are taught or prescribed as part of the school
curriculum, during the entire period of the
residence in the Philippines required of him prior
Hence, this petition. 43 to the hearing of his petition for naturalization as
Philippine citizen. Each declarant must furnish
To bolster its claim for the reversal of the assailed two photographs of himself. (Emphasis supplied)
ruling, the OSG advances this pivotal issue of: ADEHTS

. . . whether the respondent should be admitted As held in Tan v. Republic, 49 "the period of one
as a Filipino citizen despite his undisputed failure year required therein is the time fixed for the
to comply with the requirements provided for in State to make inquiries as to the qualifications of
CA No. 473, as amended — which are mandatory the applicant. If this period of time is not given to
and jurisdictional in character — particularly: (i) it, the State will have no sufficient opportunity to
the filing of his petition for naturalization within investigate the qualifications of the applicants
the one (1) year proscribed period from the date and gather evidence thereon. An applicant may
he filed his declaration of intention to become a then impose upon the courts, as the State would
Filipino citizen; (ii) the failure to attach to the have no opportunity to gather evidence that it
petition his certificate of arrival; and (iii) the may present to contradict whatever evidence
failure to comply with the publication and posting that the applicant may adduce on behalf of his
requirements prescribed by CA No. 473. 44 petition." The period is designed to give the
government ample time to screen and examine
The OSG argues that "the petition for the qualifications of an applicant and to measure
naturalization should not be granted in view of its the latter's good intention and sincerity of
patent jurisdictional infirmities, particularly purpose. 50 Stated otherwise, the waiting period
because: 1) it was filed within the one (1) year will unmask the true intentions of those who seek
proscribed period from the filing of declaration of Philippine citizenship for selfish reasons alone,
intention; 2) no certificate of arrival, which is such as, but not limited to, those who are merely
indispensable to the validity of the Declaration of interested in protecting their wealth, as

117 | C O N S T I 2 _ A r t i c l e I V _ C I T I Z E N S H I P
distinguished from those who have truly come to Consequently, the citation of the CA of the ruling
love the Philippines and its culture and who wish in Tam Tan v. Republic 54 is misplaced. In that
to become genuine partners in nation building. case, the Court did not excuse the non-
compliance with the one-year period, but
The law is explicit that the declaration of reiterated that the waiting period of one (1) year
intention must be filed one year prior to the filing is mandatory. In reversing the grant of
of the petition for naturalization. Republic v. Go naturalization to Tam Tan, the Court wrote:
Bon Lee 51 likewise decreed that substantial
compliance with the requirement is inadequate. The appeal is predicated on the fact that the
In that case, Go filed his declaration of intention petition for naturalization was filed (26 October
to become a citizen of the Philippines on May 23, 1950) before the lapse of one year from and after
1940. After eleven months, he filed his petition the filing of a verified declaration of his bona fide
for naturalization on April 18, 1941. In denying his intention to become a citizen (4 April 1950), in
petition, the Court wrote: violation of Section 5 of Commonwealth Act No.
473, as amended.
The language of the law on the matter being
express and explicit, it is beyond the province of The position of the Government is well taken,
the courts to take into account questions of because no petition for naturalization may be
expediency, good faith and other similar reasons filed and heard and hence no decree may be
in the construction of its provisions (De los Santos issued granting it under the provisions of
vs. Mallare, 87 Phil., 289; 48 Off. Gaz., 1787). Commonwealth Act No. 473, as amended, before
Were we to accept the view of the lower court on the expiration of one year from and after the date
this matter, there would be no good reason why a of the filing of a verified declaration of his bona
petition for naturalization cannot be filed one fide intention to become a citizen of the
week after or simultaneously with the filing of the Philippines. This is mandatory. 55 Failure to raise
required declaration of intention as long as the in the lower court the question of non-compliance
hearing is delayed to a date after the expiration therewith does not preclude the Government
of the period of one year. The ruling of the lower from raising it on appeal. 56 aHESCT
court amounts, in our opinion, to a substantial
change in the law, something which courts can Nevertheless, after the one-year period, the
not do, their duty being to apply the law and not applicant may renew his petition for
tamper with it. 52 naturalization and the evidence already taken or
heard may be offered anew without the necessity
The only exception to the mandatory filing of a of bringing to court the witnesses who had
declaration of intention is specifically stated in testified. And the Government may introduce
Section 6 of CA No. 473, to wit: evidence in support of its position. 57

Section 6. Persons exempt from requirement to The decree granting the petition for naturalization
make a declaration of intention. — Persons born is set aside, without costs.
in the Philippines and have received their primary
and secondary education in public schools or In naturalization proceedings, the burden of proof
those recognized by the Government and not is upon the applicant to show full and complete
limited to any race or nationality, and those who compliance with the requirements of the law. 58
have resided continuously in the Philippines for a The opportunity of a foreigner to become a
period of thirty years or more before filing their citizen by naturalization is a mere matter of
application, may be naturalized without having to grace, favor or privilege extended to him by the
make a declaration of intention upon complying State; the applicant does not possess any natural,
with the other requirements of this Act. To such inherent, existing or vested right to be admitted
requirements shall be added that which to Philippine citizenship. The only right that a
establishes that the applicant has given primary foreigner has, to be given the chance to become
and secondary education to all his children in the a Filipino citizen, is that which the statute confers
public schools or in private schools recognized by upon him; and to acquire such right, he must
the Government and not limited to any race or strictly comply with all the statutory conditions
nationality. The same shall be understood and requirements. 59 The absence of one
applicable with respect to the widow and minor jurisdictional requirement is fatal to the petition
children of an alien who has declared his as this necessarily results in the dismissal or
intention to become a citizen of the Philippines, severance of the naturalization process.
and dies before he is actually naturalized.
(Emphases supplied) Hence, all other issues need not be discussed
further as respondent failed to strictly follow the
Unquestionably, respondent does not fall into the requirement mandated by the statute.
category of such exempt individuals that would
excuse him from filing a declaration of intention It should be emphasized that "a naturalization
one year prior to the filing of a petition for proceeding is so infused with public interest that
naturalization. Contrary to the CA finding, it has been differently categorized and given
respondent's premature filing of his petition for special treatment. . . . [U]nlike in ordinary judicial
naturalization before the expiration of the one- contest, the granting of a petition for
year period is fatal. 53 naturalization does not preclude the reopening of
that case and giving the government another

118 | C O N S T I 2 _ A r t i c l e I V _ C I T I Z E N S H I P
opportunity to present new evidence. A decision the Naturalization Law. As such, his petition for
or order granting citizenship will not even naturalization must be denied without prejudice
constitute res judicata to any matter or reason to his right to re-file his application.
supporting a subsequent judgment cancelling the
certification of naturalization already granted, on WHEREFORE, the petition is GRANTED. The June
the ground that it had been illegally or 30, 2011 Decision of the Court of Appeals in CA-
fraudulently procured. For the same reason, G.R. CV No. 93374 is REVERSED and SET ASIDE.
issues even if not raised in the lower court may The petition for naturalization of respondent Li
be entertained on appeal. As the matters brought Ching Chung, otherwise known as Bernabe Luna
to the attention of this Court . . . involve facts Li or Stephen Lee Keng, docketed as Civil Case
contained in the disputed decision of the lower No. 08-118905 before the Regional Trial Court,
court and admitted by the parties in their Branch 49, Manila, is DISMISSED, without
pleadings, the present proceeding may be prejudice.
considered adequate for the purpose of
determining the correctness or incorrectness of SO ORDERED.
said decision, in the light of the law and extant
jurisprudence." 60 Velasco, Jr., Peralta, Abad and Leonen, JJ., concur.

Ultimately, respondent failed to prove full and


complete compliance with the requirements of

119 | C O N S T I 2 _ A r t i c l e I V _ C I T I Z E N S H I P

You might also like